LJ Form A, LJ Form G, LJ Form M, LJ Form E, LJ Form I, LJ form K, LJ form B, LJ FORM F, LJ FORM H, LJ FORM D, LJ FORM L, LJ FORM J

Ace your homework & exams now with Quizwiz!

A patient is undergoing a transport of 100 miles in a non-pressurized helicopter. The patient is breathing spontaneously and is receiving oxygen by non-rebreather mask. As the helicopter ascends above 5,000 ft, the patient begins to exhibit cyanosis and shortness of breath. The most likely reason for this is

decreased PAO2

A pulmonary rehabilitation patient is being monitored for compliance to a smoking cessation program. Which of the following values would be most helpful to evaluate? A. SPO2 B. COHb C. PaO2 D. metHb

B

Which of the following data represents subjective information? A. decreased tidal volume B. dyspnea C. scattered infiltrates on a chest radiography D. cyanosis in the lips

B

Which of the following parameters could be used to calculate tidal volume? A. PECO2 and VD/VT B. minute ventilation and respiratory rate C. PetCO2 and alveolar ventilation D. minute ventilation and alveolar ventilation

B

The best way to determine the accuracy of a vane respirometer is a

3-liter syringe

What is the static compliance in mL/cm H2O, given the following data on a patient receiving VC AC ventilation? Plateau pressure 15 cm H2O Peak pressure 25 cm H2O VT 500 mL PEEP 5 cm H2O

50

A negative effect of mechanical PEEP is reduced A. venous return B. FIO2 C. atelectasis D. A-aDO2

A

A respiratory therapist enters the patient's room to do a routine breathing treatment and finds the patient has no obvious chest movement. After calling for help, the first action of the therapist should be to A. Begin mask-valve ventilation B. Perform chest compressions C. Place the patient on a 100% nonrebreathing mask D. Check for a pulse

A

Which of the following antimicrobial medication is best suited to treat a blood-borne organism that is gram-positive.

Amoxicillin

Peak flow measurements are indicated in which of the following conditions, if present?

airway obstruction

Which of the following diagnostic procedures should the respiratory therapist recommend for a patient who presents in the emergency department (ED) with shallow, irregular respirations and paradoxical chest movement on the right side? A. bedside pulmonary function studies B. portable chest radiogram C. vital capacity (VC) and maximum inspiratory pressure (MIP) measurement D. maximum voluntary ventilation

The correct answer is : B Explanation : A patient with paradoxical chest movement is most likely suffering from fractured ribs. The source of the paradoxical movement may be confirmed by a chest radiograph.

A respiratory therapist is reviewing the medical record of a 69-year-old patient admitted 2 hours prior. The record indicates the patient has chronic hypercapnia, emphysema, and an 80 pack-year history of smoking. Admission documentation shows the patient reported with moderate dyspnea. Currently, the patient is sleeping but arouses with significant verbal stimulation. The therapist should FIRST recommend

arterial blood gas analysis

A respiratory therapist has provided Albuterol treatments via SVN to a patient in the emergency room for shortness of breath. The patient now has bleeding gums and epistaxis. What should the physician check? A. sinus X-ray B. allergic response to Albuterol C. arterial blood gas D. prothrombin time

The correct answer is : B Explanation : Bleeding from the mouth, nose, or gums is a rare but serious symptom of an allergic response to albuterol.

In a normally functioning chest tube drainage system, in which of the following chambers is bubbling likely to be observed? A. all chambers B. water seal C. drainage receptacle D. tubing

The correct answer is : B Explanation : Bubbling typically occurs in the water-seal compartment. Normally, bubbling is described as gentle. If bubbling is described as excessive, significant, or profuse, this usually indicates a leak somewhere between the waterseal compartment and the patient's lung. If the leak is not found in the tubing between the waterseal compartment and the patient, it is possible that the patient's lung tissue is perforated.

A respiratory therapist has completed a pre-scheduled ventilator/patient check on a 65-year-old male patient. Set tidal volume is 600 mL. Returned volumes for mechanical breaths are 591 mL. There are no detectable leaks within the circuit. What should the therapist recommend? A. Obtain a chest radiograph. B. Continue current therapy and document the returned volume. C. Replace the ventilator. D. Increase the set VT to 825 mL.

The correct answer is : B Explanation : During volume-cycled mechanical ventilation, returned tidal volume varies from preset tidal volume. The allowable variance is 10%. A variation greater than 10% requires troubleshooting by the respiratory therapist.

In which of the following would the respiratory therapist expect to see a decrease if inspiratory flow is reduced but the rate does not change? A. gas distribution B. expiratory time C. inspiratory time D. I:E ratio

The correct answer is : B Explanation : If rate is unchanged, and inspiratory flow is reduced, this would result in an increase in inspiratory time and a consequent decrease in expiratory time.

Which of the following indicate a patient is ready to wean from mechanical ventilation? A. MIP of -12 cmH2O B. VT of 6 mL/kg C. A-aDO2 of 380 mmHg D. RSBI 138

The correct answer is : B Explanation : In order to wean from the ventilator the MIP must be more than -28 cmH2O. Tidal volume must be at least 5 mL per kilogram and RSBI must be less than 106. The alveoli-arterial oxygen difference (A-a gradient) must be less than 300 mmHg. In this case only the tidal volume of 6 mL per kilogram and the RSBI of 90 indicate the patient is ready to wean from mechanical ventilatory support.

A faint, discontinuous popping sound auscultated at the end of a deep breath in the posterior basal lung sections would indicate A. CHF B. atelectasis. C. meconium aspiration. D. ARDS

The correct answer is : B Explanation : Late inspiratory crackles, or end-inspiratory crackles are commonly auscultated with a patient experiencing atelectasis. Crackles are also known as rales. Crackles sound like discontinuous popping or "rice crispies" upon auscultation

Which of the following would best augment spontaneous tidal volumes for a patient undergoing ventilator liberation attempts? A. APRV B. PS C. SBT D. PRVC

The correct answer is : B Explanation : Of the choices given, the use of PS, or pressure support, helps to increase or augment the size of spontaneous tidal volumes. This is especially helpful during any ventilator liberation strategy where the patient is in need of extra support.

Which of the following finding is most closely associated with increased airway resistance? A. altered P50 B. accessory muscle use C. increased PetCO2 D. reduced SpO2

The correct answer is : B Explanation : Of the options given, use of accessory muscles is most closely associated with an increase in airway resistance. This is especially true with patients who have asthma or other types of upper airway inflammation or bronchoconstriction.

Inhaled albuterol has which of the following most common side effects? A. systemic hypotension B. tremors C. nausea D. increased mPAP

The correct answer is : B Explanation : One of the most common side effects associated with inhaled albuterol is tremors.

Which of the following equipment is required to determine VD/VT ratio on a patient receiving mechanical ventilation? A. Fleisch pneumotachometer B. capnograph C. pressure differential pneumotachometer D. EZ® cap CO2 detector

The correct answer is : B Explanation : This question is largely about memorization. You must know what it takes to calculate of the VD/VT ratio. Of the equipment listed only a capnograph would be helpful because it relates end tidal CO2.

The best test determination for a forced vital capacity maneuver is done by which of the following calculations? A. TLC - FVC B. FEV1 + FVC C. SVC + FVC D. FEV1/FVC

The correct answer is : B Explanation : To determine the best trial or best patient effort for a forced vital capacity maneuver, the trial with the highest sum of the FEV1 and FVC is considered to be the patient's best effort.

Which of the following lung volume measurements is the largest? A. IC B. FRC C. TLC D. VC

The correct answer is : C Explanation : Of the volumes shown, total lung capacity is the largest. TLC includes all other volumes listed.

A patient with congestive heart failure, who is demonstrating moist crepitant rales upon auscultation, would benefit most from A. PEP therapy B. postural drainage and percussion C. administration of aminoglycocide medication D. Lasix

The correct answer is : D Explanation : A patient with congestive heart failure who is demonstrating moist crepitant rales is most likely in a state of fluid overload. The administration of Lasix is most appropriate in order to diurese the patient.

What type of procedure is the suctioning of blood-tinged secretions from a patient with an artificial airway? A. universal B. clean C. benign D. sterile

The correct answer is : D Explanation : All suctioning of a patient's airway is done with sterile technique, regardless of the type of secretions removed.

A respiratory therapist would expect what change to occur to the deadspace to tidal volume ratio when a patient is intubated and placed on a mechanical ventilator? A. increase at first, then gradually decrease B. decrease C. no change D. increase

The correct answer is : D Explanation : The initial value would be a low end-tidal CO2, followed by a steady and slow rise.

Into which interspace should a chest tube be inserted for the purpose of draining serosanguinous fluid from the pleural space and for the re-expansion of lung tissue?

5th interspace

A respiratory therapist has orders to perform a 12-lead ECG on a patient who has chest pain. When placing the chest leads, what best describes the proper placement for chest lead V4?

5th interspace, mid-clavicular line

Nitric oxide (NO) will decrease which of the following? A. pulmonary vascular resistance (PVR) B. ejection fraction C. arterial oxygen content D. systemic vascular resistance (SVR)

A

Which of the following would be a sufficient reason to discontinue an SBT (spontaneous breathing trial)?

blood pressure increases from 110/70 to 132/90 mmHg

Which of the following should be ordered for a post-surgical patient who has a history of contracting pneumonia following invasive surgery?

hyperinflation therapy

If a patient is prescribed Vancomycin, which of the following is most likely?

sputum culture shows gram negative organisms

Which of the following may be related to a fluid balance problem in a patient? A. ARDS B. Pneumothorax C. Renal disease D. Guillain-Barre Syndrome

The correct answer is : C Explanation : Renal or kidney disease can cause fluid-shifting in the body and may result in peripheral pitting edema and third-spacing.

The respiratory therapist obtains the following blood gas data on a patient breathing spontaneously on room air: pH 7.35 PaCO2 45 torr PaO2 50 torr HCO3- 27 mEq/L BE +2 mEq/L The therapist could accurately estimate the patient's SaO2 to be which of the following? A. 90% B. 75% C. 85% D. 80%

The correct answer is : D Explanation : A PaO2 of 50 mmHg most closely correlates with an oxygen saturation of about 80% on the oxygen dissociation curve.

A patient's radiological image is reported to have hyperlucency with diffuse dry crackles auscultated throughout the chest. These findings are most closely associated with A. Pneumonia B. Atelectasis C. pulmonary edema D. subcutaneous emphysema

The correct answer is : D Explanation : Auscultated dry crackles is the main sign of subcutaneous emphysema in the scenario. Increased radiolucency in the lung is also an indication that extra air exists in the chest, even though it is on the outside of the lung. Pulmonary edema would show fluffy infiltrates. Pneumonia is more localized and is associated with a reduced radiolucency.

A bronchoscopy is NOT used to diagnose A. malignant processes. B. pathogenic involvement. C. foreign body obstruction. D. atelectasis.

The correct answer is : D Explanation : Bronchoscopy is used to diagnose foreign body obstructions, active bleeding causes, pathogenic involvement (to identify organisms) and cancerous or malignant processes or areas. It may help decrease atelectasis if immobilized mucous plugs are blocking a portion of the lung from expanding, but it does not serve in the diagnosis of atelectasis.

An adult male is receiving volume-controlled ventilation and is on SIMV mode with a mandatory rate of 12/min, tidal volume of 550 mL, FIO2 of 0.60, and a PEEP of 10 cm H2O. Flow triggering is in use at 16 L/min baseline flow. During spontaneous expiration, the pressure remains higher than the set level of PEEP. The respiratory therapist should A. increase PEEP B. discontinue flow triggering C. perform an optimal PEEP study D. reduce the baseline flow of the flow triggering mechanism

The correct answer is : D Explanation : When the baseline flow of the flow triggering mechanism is excessive, the result will be that the pressure needle does not return to the baseline PEEP level at the end of each exhalation. This should be remedied by reducing the baseline flow.

Which of the following non-disposable equipment should be sterilized after use?

bronchoscope

A patient admitted with confirmed myasthenia gravis has a spontaneous VT of 350 mL and a VC of 1.9 L. Which of the following should be recommended?

continued monitoring of VC, VT, and MIP

Immediately following insertion of a tracheostomy tube for the provision of positive pressure mechanical ventilation, enough air should be added to the tracheostomy tube cuff to?

pressurize the cuff to 20 cm H2O

An adult post-op patient is unable to get any of the balls to rise on an incentive spirometer, in spite of reports of her best effort. The therapist should

switch to a volume-type incentive spirometer

A patient has the following arterial blood gas results: pH 7.35 PaCO2 59 torrPaO2 62 torrHCO3- 28 mEq/LBE + 4 mEq/L Which of the following can the respiratory therapist correctly conclude?

the patient likely has chronic obstructive pulmonary disease

A patient is four hours post operative for a partial lobectomy of the lingular lobe. The patient complains of shortness of breath and has rhonchi in the middle right lobe. The respiratory therapist should recommend which of the following?

vibratory PEP

What volumes can the respiratory therapist examine to calculate vital capacity? A. VT + IC B. IRV-RV C. ERV + VT + IRV D. TLC - FRC

C

Which of the following parameters could be used to calculate tidal volume? A. minute ventilation and respiratory rate B. minute ventilation and alveolar ventilation C. PetCO2 and alveolar ventilation D. PECO2 and VD/VT

The correct answer is : A Explanation : If you take a minute ventilation and divide it by the respiratory rate you get tidal volume.

What would the respiratory therapist exclude in an education plan for a patient going home after abdominal surgery? A. peak-flow monitoring B. incentive spirometry C. incision splinting D. effective cough technique

The correct answer is : A Explanation : Peak-flow monitoring is for asthma patients, not post-operative care.

A home care respiratory therapist is providing instruction on proper cleaning technique of a hand-held nebulizer. Which of the following should be included in the instruction? A. wash with warm, mild soapy water, air dry B. wash with acetic acid, dry with lint free towel C. soak in Cidex, rinse with acetic acid, air dry D. soak in alcohol, air dry

The correct answer is : A Explanation : Small volume nebulizer equipment should be washed with mild detergent in warm water, briefly soaked in a acetic acid solution (vinegar) and then air-dried.

Which of the following is needed to calculate alveolar oxygen tension? A. FIO2, PB, PH2O B. PAO2, PaO2 C. A-aDO2, PAO2 D. FIO2 and RQ

The correct answer is : A Explanation : To answer this question one must know the formula for the alveolar air equation. Water vapor pressure, barometric pressure, FIO2, and PaO2 are all included in the formula.

After two aerosolized Albuterol treatments by small volume jet nebulizer, the patient's breath sounds have changed from diminished to profuse wheezing bilaterally. Respiratory rate has decreased from 32 to 26. The respiratory therapist should A. continue with bronchodilator therapy B. intubate the patient C. discontinue current therapy D. administer 80/20% Heliox gas mixture

The correct answer is : A Explanation : When breath sounds change from absent or diminished to a more audible sound, such as wheezing, it can be concluded that there is more air flow in the bronchials. This indicates the patient has improved from bronchodilator therapy. The therapy should be continued.

The following flow-volume loop is obtained from a 45-year-old male with vocal cord cancer. Which of the following interpretations is most accurate? A. foreign body aspiration B. fixed airway obstruction C. asthma D. COPD

The correct answer is : B Explanation : This flow volume loop is round in appearance, indicating a problem with getting air in and out. This is associated with a fixed upper airway obstruction.

Which of the following exams provides the best diagnostic information for an obstructive defect in a COPD patient? A. FEF200-1200 B. Fev1/FVC% C. DLCO D. SVC

The correct answer is : B Explanation : To determine if the patient has an obstructive defect, as would be seen with a COPD patient, the best pulmonary function test would be the Fev1/FVC%. The second best test would be the Fev1.0 by itself. In order to determine if a patient is obstructive you must examine the flows. Slow vital capacity is a volume and therefore would be helpful in determining if the patient is restrictive. DLCO helps determine the diffusion capacity of the alveoli and will not differentiate between obstruction and restriction.

The following flow-time ventilator graphic data is observed. The therapist should suggest A. inverse I:E ratio ventilation B. increasing I:E ratio C. increasing inspiratory flow rate D. pressure control ventilation

The correct answer is : C Explanation : The development of autoPEEP, which is shown in this graphic, is treated by allowing more time for expiration. This is most commonly is done by increasing the flow, thereby decreasing inspiratory time and lengthening expiratory time. Although not offered here, a decrease in mandatory rate is another method for decreasing autoPEEP.

A patient who is being mechanically ventilated in the intensive care unit had a complete pulmonary function test one-month prior. SVC was 75% of predicted. FRC was 70% of predicted. The patient is now experiencing an increase in A-a gradient. The use of mechanical PEEP in this patient would A. decrease MAP. B. decrease oxygenation. C. increase FRC. D. not affect lung capacities.

The correct answer is : C Explanation : The use of mechanical PEEP would increase functional residual capacity (FRC).

A patient in the intensive care unit (ICU) is suspected for pulmonary embolism. Which of the following exams would help investigate for this suspicion?

capnography

Which of the following chest radiographic findings would most likely be observed on a patient who has a long-standing condition of congestive heart failure, secondary to obesity?

cardiomegaly

Which of the following would NOT increase if inspiratory flow is increased on a time-cycled ventilator and pressure is not limited? A. Tidal volume B. Total rate C. Expiratory time D. Inspiratory pressure

B

A patient has a sleep study (PSG) which shows an AHI of 50 and an average oxygen desaturation of 10%. Which of the following should the respiratory therapist recommend?

CPAP/BIPAP nocturnal titration study

Which of the following must be done to transition a patient with a fenestrated tracheostomy tube, who is receiving positive pressure ventilation, to a speaking configuration? A. sever the pilot valve line B. change the tracheostomy tube C. deflate the cuff, install the cap D. insert the inner cannula

C

Which part of a pulmonary function testing machine is being evaluated when performing quality control with a 3-liter syringe? A. internal electronic components B. mouthpiece and circuit C. pnemotachometer D. body box

C

The respiratory therapist notes a biological tape indicator inside an equipment sterilization package indicates the presence of viable spores. The therapist should A. send the mask back to sterilization B. wipe the mask down with disinfectant before use C. open the package and inspect the mask prior to use D. discard the mask

A

A patient with suspected drug overdose is being evaluated by the respiratory therapist for adequacy of ventilation. In addition to assessing respiratory rate and rhythm, spontaneous tidal volume is evaluated. Spontaneous tidal volume should minimally be at which value for the patient to be considered normal?

5 mL/kg

How many milligrams of drug are in 3.0 mL of medication whose strength is specified to be 2.5%?

75 mg

A respiratory therapist is determining a VD/VT ratio. Which of the following is needed to complete this analysis?

PetCO2 and PaCO2

A 24-year-old Caucasian male has suffered a spontaneous pneumothorax on the right side. Upon inspection, which of the following would most likely be observed?

accessory muscle use

Which of the following oxygen modalities would be appropriate for a 220-lb (100-kg) male with diabetic ketoacidosis who requires 40% supplemental oxygen?

air-entrainment mask

The following volume-pressure graphic is observed on a patient who is receiving VC A/C ventilation. The respiratory therapist should

change the scale volume axis of the graph

A patient with a 90-pack-year smoking history is receiving volume-controlled ventilation in the SIMV mode. Sedation has been discontinued to facilitate weaning. The patient seems increasingly anxious. Which of the following would be most beneficial to the patient?

apply a nicotine dermal patch to the patient

Prior to performing pulmonary function testing on a several patients, a respiratory therapist is conducting quality control on the pneumotachometer. After injecting a 3.0 L syringe through the pneumotachometer, the therapist notes a measurement of 2.86 L. Based on this result the therapist should

commence testing on patients.

A patient with decreasing pulmonary compliance is receiving volume-controlled ventilation with an I:E ratio of 1:2. A chest radiograph shows increased atelectasis. Blood gases show signs of refractory hypoxemia. Which of the following is an appropriate change for this patient?

decrease inspiratory flow rate

Predicted Observed TLC (liters) 4.50 3.10FRC (liters) 2.45 1.70SVC (liters) 3.20 2.00FEV1 (liters) 2.15 1.98FEF50 (liters/sec) 4.00 3.85 Which of the following represents the most appropriate interpretation of the preceding spirometry results?

sarcoidosis

A female patient has left-side pneumonia brought on by a gram-positive bacterial infection. How shall the patient be positioned to achieve maximum pulmonary drainage?

lying on her right side

The following data is available for a patient receiving volume-controlled mechanical ventilation. 10 am 2 pm Ppeak (cm H2O) 25 36Pplat (cm H2O) 15 26Set VT (L) 0.6 0.6PEEP (cm H2O) 5 5 The respiratory therapist should:

order a chest radiograph

A home-care plan is being created for a COPD patient who will be discharged home. Which of the following would be most helpful in assessing a patient's ability to conduct activities of daily living (ADLS) relative to the oxygen needs of the patient?

oxygen titration with exercise

A respiratory therapist is caring for a patient with asthma. Bronchodilator therapy has been initiated and some relief is observed. Which of the following should the therapist recommend to further assess the degree of improvement?

peak flow

A patient is orally intubated with a 7.0-mm ET tube. Immediately following the procedure, the respiratory therapist will FIRST do which of the following to check for proper positioning of the airway?

observe the color of the CO2 detector

Which of the following equations would help determine cardiac output?

Fick

A patient has congestive heart failure and fluid overload. Which of the following agents will help to decrease the pre-load pressure of the right atrium in the heart?

Lasix (furosemide)

The patient has a PetCO2 that has been decreasing over the last several hours while PaCO2 roughly remains unchanged. Vd/Vt has been climbing and is currently at 38%. Which of the following can be concluded?

Physiological dead space is increasing.

Which of the following can be used to estimate the level of Hemoglobin?

RBC

An 18-year-old patient with asthma is receiving a second dose of aerosolized albuterol. Following the first treatment, the patient demonstrates more audible wheezing. Ten minutes after the second treatment, the patient's breath sounds are markedly diminished. The respiratory therapist should recommend

Start continuous nebulized bronchodilator therapy at 10 mg/hr

The respiratory therapist notes in the medical record of a 65-year-old male that the patient is ordered to receive bronchodilator therapy with Albuterol. The therapist also notes the patient is receiving beta-blocker medication. The therapist should recommend

Switch from Albuterol to ipratropium bromide (Atrovent)

Which part of a pulmonary function testing machine is being evaluated when performing quality control with a 3-liter syringe? A. spirometer B. internal electronic components C. body box D. mouthpiece and circuit

The correct answer is : A Explanation : A 3-liter super syringe is used to calibrate the spirometer for a pulmonary function testing machine.

Which of the following should NOT be included in the initial assessment of a comatose patient in the intensive care unit? A. Symptoms B. Vital signs C. Breath sounds D. General appearance

The correct answer is : A Explanation : A comatose patient is not cable of self-reporting problems. Therefore, subjective information, or symptoms cannot be determined. Signs, however, which are objective, can be determined by the caregiver, independent of the patient's ability to communicate.

Which of the following laboratory examinations would be helpful in further assessing a patient with diabetic ketoacidosis? A. glucose level B. creatinine C. PD50 D. P50

The correct answer is : A Explanation : A patient with diabetic ketoacidosis primarily has a problem with the blood glucose level. Further assessment therefore can be done by examining the glucose level.

A respiratory therapist is determining the proper sterilization technique to clean a pneumatically driven IPPB machine. The therapist should choose which of the following methods? A. Irradiation B. Cidex (alkaline gluteraldehyde) C. Sonacide (acid gluteraldehyde) D. Ethylene oxide (ETO)

The correct answer is : D Explanation : Because an IPPB machine cannot be placed in a liquid for sterilization, gas sterilization is most appropriate. This can be accomplished with ethylene oxide or ETO.

A patient who maintains a fenestrated tracheostomy tube would like to speak. Immediately after configuring the tracheostomy tube for speech, the patient is unable to ventilate. Which of the following is most likely?

The cuff is inflated.

Prior to the removal of chest tubes, which of the following should be done?

clamp the chest tube for 24 hours

A normally health track runner is hospitalized with right side pneumonia. The patient has chills and a temperature of 38.9 deg C (102 deg F). Which of the following will mostly likely provide sufficient aid to the patient?

coached coughing and deep breathing exercises

A 5-year old female patient is brought to the clinic in moderate respiratory distress. The mother believes the child has a small toy in her lungs. If this suspicion is true, what finding might the respiratory therapist expect to observe?

monophonic wheezing upon auscultation

A patient's chest radiograph reveals an enlarged heart whose shadow occupies two thirds of the chest cavity from side-to-side. With which of the following findings would this finding be most closely associated?

morbid obesity

A patient in the neurological intensive care unit (NICU) is being monitored for intracranial pressure (ICP). The average pressure over the last several hours is 15 mm Hg. In the last few minutes, ICP has risen to 25 mm Hg. This is most likely due to

over-stimulation of the patient.

A 24-year-old patient is receiving mechanical ventilation. The following pressure - volume graph is available. What can accurately be stated about this information? (graph kinda looks like a fishtail)

this breath was patient-triggered

A patient receiving oxygen therapy at 40% has the following arterial blood gas results: pH 7.41 PaCO2 39 torr PaO2 349 torr HCO3- 24 mEq/L The respiratory therapist detects an error in the results. Which of the following could be the cause of the error? A. Damaged Severinghaus electrode B. Improperly functioning Clark electrode C. Oxygen toxicity D. Dysfunctional membrane on the Sanz electrode

B

A respiratory therapist is helping a patient with COPD prepare to be discharged home. The therapist is reviewing alternative methods of performing activities of daily living (ADLs) with the patient. Which of the following are considered ADLs? 1-Mowing the lawn 2-Shaving 3-Grocery shopping 4-Walking around the neighborhood

2 and 3 only

To determine potential post-operative risk of a patient preparing for surgery, the respiratory therapist should recommend which of the following? A. measurement of inspiratory capacity B. pulmonary diffusion studies C. pulmonary stress test D. cardiac stress test

A

A patient intubated with a 7.0 mm endotracheal tube requires suctioning. Which of the following suction catheters would be most effective? A. 14 Fr catheter B. 16 Fr coude-tipped catheter C. 18 Fr catheter D. 10 Fr catheter

D

A radiology report of a chest x-ray indicates the oral endotracheal tube is level with the carina. The respiratory therapist should A. withdraw the ET tube by 3 cm B. withdraw the ET tube by 1 cm C. advance the ET tube by 2 cm D. withdraw the ET tube by 6 cm

The correct answer is : A Explanation : The endotracheal tube should be 2 - 5 cm above the carina. Because the endotracheal tube is at or level with the carina the tube should be pulled back between 2 - 5 cm.

Which of the following is the most significant complication of bronchoscopy?

laryngospasms

Which of the following could the respiratory therapist use to determine a patient's rapid shallow breathing index?

vane respirometer

While performing oxygen rounds, the respiratory therapist visits a patient who is ordered to receive 5 L/min by nasal cannula with a bubble humidifier. The therapist finds the oxygen flow meter is reading 0 L/min, even though the knob is not in the off position, and there is no flow to the patient. Which of the following may cause this? A. low water level in the humidifier B. clogged down-tube C. loose humidifier bottle D. large hole in the cannula

B

The head of 41-gestational week infant has just been delivered through the birth canal. The respiratory therapist notices meconium staining about the infant's face. Which of the following actions should the therapist recommend next A. begin chest compressions B. provide vigorous manual resuscitation C. perform deep tracheal suctioning D. place the infant on mechanical ventilation

C

The preferred method of delivery for humidification for a pediatric patient during a transport lasting twelve hours is a (an) A. heated humidifier B. bubble humidifier C. HME D. cascade humidifier

C

During the weaning process of a 70 year-old patient, the respiratory therapist notes the following ventilator settings and clinical information: Mode SIMV Mandatory rate 8 Total rate 28 VT 500 mL VT(spont) 220 mL FIO2 0.4 PEEP 5 cm H2O The respiratory therapist should do which of the following? A. increase rate B. increase set VT C. discontinue PEEP D. add pressure support

D

How many minutes will a full E cylinder oxygen tank last when running at 5 L/min? A. 90 minutes B. 616 minutes C. 1320 minutes D. 123 minutes

D

The respiratory therapist measures the volume when a patient maximally exhales after passively exhaling a normal tidal volume. Which of the following volumes has the therapist observed? A. FRC B. VC C. TLC D. ERV

D

A respiratory therapist is administering epinephrine directly down the ET tube of a patient in full cardiopulmonary arrest. What is the most important procedural consideration?

The normal dose should be doubled.

For a patient receiving volume-controlled mechanical ventilation, the lower inflection point on a pressure-volume loop can best be described as:

amount of pressure required to keep the alveoli and small airways open

The legal guardian of a patient with asthma calls into the clinic and reports that her child is in the red zone on the asthma action plan and asks for direction. The respiratory therapist should instruct the guardian to

call 911 for an ambulance.

Which of the following would be most effective at resolving a humidity deficit for a patient who is intubated with an 8.0-mm endotracheal tube?

concha heated humidifier

A 68-year-old near drowning victim is triggering a high-pressure alarm on a VC ventilator. The following ventilatory data is available: 8 pm 8 am 8 pm 8 am VT (mL) 550 550 550 550 PEEP (cm H2O) 5 5 5 5 Px (plat) (cm H2O) 18 23 27 32 Px (peak) (cm H2O) 28 33 37 42 SpO2 (%) 98 96 94 93 For which of the following reasons has the high-pressure alarm most likely been triggered?

decreased static compliance

Which of the following must be done to transition a patient with a fenestrated tracheostomy tube, who is receiving positive pressure ventilation, to a speaking configuration?

deflate the cuff, install the cap

A patient who requires continuous mechanical ventilatory support is suspected for having a perforation in the lung tissue of the left lower base. A chest radiograph confirms this suspicion. Which form of ventilatory support would minimize the chance of further perforation?

high-frequency jet ventilation (HFJV)

Immediately following a puncture of the right radial artery, the respiratory therapist notices a purplish swelling over the puncture site. This is most likely caused from which of the following?

inadequate time holding pressure on the site

Which of the following is an important strategy when caring for a patient with adult respiratory distress syndrome?

keep FIO2 below 0.60 when possible

While performing postural drainage with the head-of-bed down 30 degrees, a patient with bronchiectasis begins to cough uncontrollably and exhibits PVCs on the ECG monitor. The therapist should

place the patient in Flower's position

A left-to-right shunt, due to a leaking septum wall in the heart, is best determined by an increase in mixed venous oxygen saturation. From what location would the respiratory therapist recommend blood extraction for analysis to determine mixed venous oxygen saturation?

pulmonary artery

The mother of a patient with asthma calls into the clinic to get advice regarding her child's ventilatory difficulty. She reports the child is breathing fast and deeply and is having trouble talking between breaths despite the delivery of 3 doses of albuterol. She also indicates she has a call in the child's doctor but has not yet received a return call. The respiratory therapist should recommend

reporting to the emergency room immediately.

Following surgery, a patient is slow to emerge from sedation and requires continued ventilatory support through an 8.0-mm ET tube. The patient will be transferred to the intensive care unit overnight for close observation. The respiratory therapist notices sputum in the HME device attached to the end of the ET tube. The therapist should

switch to heated humidification.

An adult patient experiences an adverse cardiac and blood pressure reaction to 0.25 mL Albuterol when administered by small volume nebulizer. The respiratory therapist should

switch to unit dose Atrovent (ipratropium bromide).

A 12-year-old patient is in the clinic with difficulty breathing. Audible wheezing without a stethoscope is present. Which of the following would be helpful in further assessing the patient's current ventilatory status?

bedside peak flow measurement

A 6-year-old male is brought to the emergency room with uncontrollable coughing. Auscultation reveals unilateral wheezing. SpO2 is 88%. Which of the following would be most helpful at resolving the underlying problem?

bronchoscope

A patient develops rhonchi during postural drainage and percussion. SpO2 drops to 92% during a related coughing episode. After bringing the patient's head-of-bed up to allow for recuperation, the therapist should recommend

continuance of therapy as ordered.

A respiratory therapist responds to a low tidal volume alarm. A quick glance reveals the tidal volume is set to 650 mL and the exhaled tidal volume is 610 mL. The therapist should make which of the following adjustments?

decrease low tidal volume alarm

The respiratory therapist has discovered the development of autoPEEP on a patient due to decreased dynamic compliance. Which of the following should the respiratory therapist do?

decrease mandatory rate

A 75-kg (165-lb) patient in the intensive care unit (ICU) has gradually experienced a decrease in pulmonary compliance. Consequently, the ordering physician has ordered a change in mechanical ventilation settings from volume-controlled to pressure-controlled with the following settings: PIP 35 cm H2O PEEP 5 cm H2O FIO2 0.60 Rate 20/min Return VT 760 mL SpO2 0.91HR 110/min The respiratory therapist should recommend

decreasing PIP

The high return-volume alarm is sounding on a 68-kg (150 lb) IBW patient who is receiving PC ventilation in the SIMV mode. Current settings and ventilatory data are: PIP 35 cm H2O PEEP 5 cm H2O FIO2 0.50 Rate 18/min I:E ratio 1:2.4 Return VT 768 mL SPO2 0.99 The respiratory therapists should recommend

decreasing PIP

A patient's medical record indicates the physician recently documented auscultation of a cardiac murmur. For which of the following possible examination results should the respiratory therapist search in the record?

echocardiogram

Which of the following circumstances, if occurring during a cardiopulmonary exercise test, would indicate that the test should be halted?

heart rate does not increase with an increase in workload

A patient has returned to the sleep clinic after using nasal mask CPAP for 2 days. The patient complains of a small amount of continuous air escaping from a hole in the plastic mask. The respiratory therapist should instruct the patient to

ignore the escaping air because it is built into the mask.

A respiratory therapist has provided Xopenex treatments via hand-held nebulizer to a patient in the emergency room for shortness of breath. The patient now has bleeding gums and epistaxis. What should the physician check?

incompatibility to Xopenex

Which of the following is most indicative of a pulmonary embolism?

increased PAP, decreased PCWP

A physician suspects a patient has a pleural effusion. What should the respiratory therapist recommend to further investigate the suspicion?

lateral decubitus radiograph

Which of the following alarms would be most important for a patient receiving positive pressure ventilation by a pressure-cycled ventilator?

low volume

A therapist palpates a popping sensation in the upper chest and neck within hours after the insertion of an 8.0-mm standard tracheostomy tube. Which of the following is the most likely cause?

mal-positioned trach tube

A respiratory therapist observes a chest radiographic assessment that shows hyperlucency with no vascular markings on the right, and tracheal deviation to the left. There are no distinctive spaces between the vertebrae. The therapist should

obtain another chest-radiograph

A patient has the following chest radiograph. The radiology report suggests the presence of a concave superior interface. Which diagnosis is most likely?

pleural effusion

A patient is brought to the emergency room (ER) after being retrieved from a motor vehicle accident where he was the driver. The respiratory therapist notices paradoxical chest movement on the right side, a deviated trachea to the left, and the presence of tachypnea. Which of the following should the therapist suspect?

pneumothorax

Which of the following is an important strategy when caring for a patient with COPD receiving volume-controlled mechanical ventilation?

prolonged expiratory time

A respiratory therapist notices the fluid collection chamber on a disposable chest tube drainage system is full. The respiratory therapist should

replace the entire apparatus.

A patient who is receiving VC A/C ventilation has developed thick, mucoid secretions over the last 8 hours. The servo-controlled humidifier is set to it maximum temperature. The digital temperature reading is 31 deg C (88 deg F). The therapist should

replace the servo-controlled humidifier

Which of the following could cause a false exhaled CO2 reading on a capnograph placed in a ventilator circuit?

soiled infrared detector

A patient in the emergency room expectorates thick, yellow sputum. A CBC shows the following: RBC 6.0 mill/cu mm Hb 17 g/dLHCT 64 %WBC 22,000 cu mm The patient could benefit most from which of the following?

sputum culture and sensitivity

While setting up for oxygen administration with an air/oxygen blender and a heated large volume nebulizer for an infant who has orders for 30% oxygen via oxyhood, the respiratory therapist notices a loud continuous sound emanating from the blender. The most likely cause is

the air hose is not seated properly into the 50 PSI source

A patient who has an 80-pack-year history of smoking and is chronically obstructive has the following arterial blood gas results on room air at standard temperature and pressure (STP): pH 7.35 PaCO2 69 torrPaO2 148 torrHCO3- 31 mEq/LBE + 5 mEq/LCOHb 20% The respiratory therapist can accurately assume which of the following?

the blood gas analysis is erroroneous

Which of the following statements would be helpful in determining a patient's location of pain?

"Tell me or point to the location of any pain you are feeling"

When performing suctioning through an endotracheal tube on a patient receiving volume-controlled mechanical ventilatory support in the A/C mode, what is the maximum time that the application of suctioning should not exceed?

15 seconds

A physician asks the respiratory therapist to calculate the patient's Vd/Vt ratio. Which of the following equipment should be utilized to obtain the appropriate values? 1. CO-Ox 2. capnograph 3. blood gas analyzer 4. pneumotachometer

2 and 3 only

Which of the following conditions could cause a tracheal shift to the right? 1. atelectasis in the left 2. sputum consolidation on the right 3. pneumothorax on the right 4. lobectomy on the right.

2 and 4 only

A patient who receives oxygen by E cylinder compressed gas is planning an 8-hour trip in a car. The prescribed flow rate of oxygen is 2 L/min. With how many full E cylinder tanks should the patient travel with to have sufficient oxygen for the entire trip?

2 tanks

What is the oxygen index for a patient who has the following clinical and laboratory data? ABG pH 7.35 PaCO2 45 torr PaO2 75 torr HCO3- 25 mEq/L BE 0 mEq/L Vent settings Mode VC, A/C Rate 14/min VT 500 mL FIO2 0.60 PEEP 8 cm H2O mPAW 25 cm H2O C.I. 3.0 L/min/m2

20.... OI = (mPAW x O2%) / PaO2

The standard dosing frequency for inhaled tiotropium (Spiriva) is every?

24hrs

In the presence of a normal pH, a PaO2 of 75 mm Hg would correlate to an SpO2 of approximately

90%

A patient with a history of Status Asthmaticus arrives in the emergency room (ER) after having received multiple bronchodilator treatments. Paramedics report breath sounds have not improved. The physician orders supplemental oxygen. The respiratory therapist will recommend which of the following? A. nonrebreathing mask B. nasal cannula C. Venturi mask D. mechanical ventilator

A

A respiratory therapist should evaluate which of the following first to evaluate the patient's ventilatory status? A. chest movement B. diagnostic chest percussion C. arterial blood gas analysis D. breath sounds

A

Heated humidification would be most important for a patient using A. an endotracheal tube B. a 6 L/min nasal cannula C. a Venturi mask set at FIO2 0.5 D. a face tent with an air entrainment device set at 50%

A

In preparation for a patient who will be receiving oxygen at 3 L/min at home, the respiratory therapist should recommend which of the following devices for primary oxygen delivery? A. oxygen concentrator B. E cylinder C. H cylinder D. bulk liquid oxygen conversion system

A

The purpose of a holding chamber, used in conjunction with a metered dose inhaler, is to

facilitate delivery of optimally-sized particles

Which of the following is most appropriate for handling biohazardous material?

Discard by using red bags and containers marked biohazard.

Which of the following would be the most effective, appropriate method for resolving atelectasis in a spontaneously breathing, post operative patient who is under the influence of sedation and will not respond to verbal stimuli?

IPPB

When creating a home-care plan for a 67-year-old male patient who is being discharged from the hospital after episodic acute-on-chronic hypercapnia, the respiratory therapist should investigate which of the following about the patient and/or his environment? A. current activities of daily living B. current COHb level C. record of immunization shots received in the last 3 years D. history of hospitalization in the last 4 years

The correct answer is : A Explanation : Of the options listed the assessment of a patient's current activities of daily living, which include such activities as bathing, cooking, toileting, and others, should be assessed. Every effort should be made to help the patient accomplish ADLs in a way that does not tax the patient's ventilatory abilities too much. Assessment of ADLs is a key preparation for a homecare patient who will be discharged. If they are unable to accomplish those ADLs, they may need some professional assistance which should be planned for during the discharge process.

Arterial blood gases on a patient in the cardiac intensive care unit are as follows: pH 7.31 PaCO2 50 mmHg PaO2 81 mmHg HCO3- 24 mEq/L BE 0 mEq/L Which of the following represents an accurate interpretation of these results?

The correct answer is : A Explanation : A CO2 of 50 mmHg is above its normal range and therefore presents acidosis. A quick glance at the pH shows no compensation. Therefore respiratory acidosis is present. Further, the patient's PO2 is normal so hypoxemia is not present. The correct interpretation is respiratory acidosis with no hypoxemia.

Which of the following assessment criteria all relate to APGAR scores? A. grimace, pulse, appearance, color B. respiratory rate, breath sounds, heart auscultation, cough C. SpO2, grunting, nasal flaring, retractions D. cord blood gas, glucose, coombs test, billirubin

The correct answer is : A Explanation : APGAR (appearance, pulse, grimace, activity, and respiratory effort) is done on every newborn after 1-minute and after 5-minutes of life.

A 32-year old female, suspected of drug overdose, is on mechanical ventilation on the following settings: Mode SIMV Mandatory rate 16 Tidal volume 450 mL FIO2 0.40 Pressure limit 60 cm H2O I:E 1:1.5 The physician asks the respiratory therapist to make an adjustment to the ventilator to change the I:E ratio to 1:2. Which of the following will the therapist increase to comply with the request? A. flow B. sensitivity C. base flow D. mandatory rate

The correct answer is : A Explanation : To increase expiratory time, inspiratory time should be decreased. This is done by manipulating the flow control on a ventilator, specifically by increasing the flow.

Which of the following should the respiratory therapist do or observe to rapidly estimate the adequacy of oxygenation? A. color B. perform arterial blood gas analysis C. sensorium D. chest rise

The correct answer is : A Explanation : To obtain information rapidly and quickly, the method of assessment must be quick to perform. The quickest methods include those that are visual in nature and do not require waiting for test results or interpretations. Of the options offered, the best method to determine the adequacy of oxygenation rapidly is to observe color. Observation of chest rise would assess ventilation and performing an arterial blood gas is too time-consuming to be considered a rapid assessment.

Which of the following methods of communication should the respiratory therapist use to assess symptomatic pain in a 6-year-old patient? A. ask the patient to describe the intensity of the pain B. use of pictograms C. ask the parent or guardian D. closed-ended questioning

The correct answer is : B Explanation : A 6-year-old patient may not be able to communicate the type, quality, and degree of their symptoms due to their age. Therefore, the use of pictograms will help the patient communicate his or her feelings and health status.

A 14-year-old child is admitted to the emergency room with a medical history of asthma. The patient is currently febrile, breath sounds are decreased with fine crackles at the bases, and a chest radiograph reveals plate-like infiltrates in the bases bilaterally. Which of the following therapies should the respiratory therapist recommend? A. Intal (cromolyn sodium) B. Serevent (salmeterol) MDI C. Atropine sulfate, IV D. Mucomyst (10% in 4 cc of NS)

The correct answer is : B Explanation : A Serevent MDI will be most helpful to this patient to promote deeper breathing and resolution of atelectasis, which is manifested by crackles at the basis and the presence of plate-like infiltrates.

Examination of a 12-lead ECG reveals normal P-R interval with the P wave pointing up and the T wave pointing down. Based on this information the respiratory therapist should recommend A. administering a bolus of KCL B. increasing FIO2 C. administering normal saline by IV D. diuresing the patient with Lasix (furosemide)

The correct answer is : B Explanation : A T wave on an ECG waveform that is pointing in a direction that is opposite from the direction of the P-wave, suggests the presence of cardiac ischemia and should be addressed by increasing supplemental oxygen administration to the patient.

In order for a patient with a fenestrated tracheostomy tube to feel the least amount of airway resistance while breathing spontaneously, what should the respiratory therapist recommend? A. deflate cuff, remove inner cannula, cap tube B. deflate cuff, remove inner cannula, remove cap C. inflate cuff, insert inner cannula, remove cap D. inflate cuff, remove inner cannula, cap tube

The correct answer is : B Explanation : A fenestrated tracheostomy tube has an inner cannula with a hole to facilitate talking and weaning. It also has an inner cannula for resuscitation, or mechanical ventilation. The cuff is like that of an endotracheal tube, low-pressure, high-volume. And finally, a cap is used to close the tube for speech therapy. When the cap is used, the inner cannula should be removed and the cuff should be deflated.

A respiratory therapist should recommend which of the following medications to arrest bacterial growth on a patient who has gram positive pathogenic organisms in the sputum and blood? A. Rifampin B. Amoxicillin C. Ethambutol D. Steptomycin

The correct answer is : B Explanation : A gram-positive bacteria can usually be killed with penicillin-type antibiotics. In this case, amoxicillin is the only penicillin-type antibiotic offered as an option.

Which of the following should the respiratory therapist suspect if a neonate's transcutaneous oxygen reading shows a sudden rise in PO2? A. probe membrane requires replacement B. disconnected probe C. capillary vessel temperature is insufficient D. monitor is out of calibration

The correct answer is : B Explanation : A transcutaneous oxygen probe reads oxygen levels through minute profusion of gases through the skin. If the probe becomes detached, the likely result is an apparent increase in oxygenation because the probe is reading oxygen more plentifully in the ambient air.

A patient with Status Asthmaticus has an airway resistance (Raw) of 3.2 cmH2O/L/sec. The patient's respiratory rate is 28/min and tidal volume is gradually falling. Saturation is 94% on 3 L/min. Which of the following would be most beneficial to the patient? A. increase oxygen to 4 L/min B. helium/oxygen mixture 70%/30% C. helium/oxygen mixture 80%/20% D. mechanical ventilation with pressure support

The correct answer is : B Explanation : Asthma is often associated with high airway resistance. When unable to lower airway resistance through bronchodilation, such as is seen with status asthmaticus, the use of helium-oxygen mixtures may be helpful. 70-30% Heliox therapy is appropriate because the 30% oxygen is closest to the 3 L/min. oxygen the patient is receiving by nasal cannula.

Which of the following should be monitored for a patient diagnosed with Guillain-Barre Syndrome who is breathing spontaneously? A. PaO2, P50, Fev1 B. MEP, SVC, Fev1/FVC% C. VC, VT, MIP D. MVV, FEF200-1200

The correct answer is : C Explanation : A patient with Guillain-Barré has muscular paralysis that prevents ventilation. As ventilation slowly degrades, the patient should be monitored for the point that he or she can no longer sustain life. Parameters should be monitored, include vital capacity, maximum inspiratory pressure, and tidal volume.

Which of the following laboratory examinations would be helpful in further assessing a patient with diabetic ketoacidosis? A. creatinine B. respiratory quotient C. glucose level D. urine specific gravity

The correct answer is : C Explanation : A patient with diabetic ketoacidosis primarily has a problem with the blood glucose level. Further assessment therefore can be done by examining the glucose level.

A 50-year-old male presents in the emergency department (ED) complaining of frequent vomiting. Arterial blood gas results on room air show: pH 7.54 PaCO2 41 torr PaO2 96 torr HCO3- 30 mEq/L BE +6 mEq/L Which of the following would account for this blood gas anomaly? A. Cl- 110 mEq/L B. K+ 4.9 mEq/L C. K+ 3.4 mEq/L D. Hyperventilation

The correct answer is : C Explanation : Carbon dioxide in this blood gas shows adequate ventilation. However, a pH of 7.54 is an indication of alkalosis. Because the alkalosis cannot be attributed to an elevated CO2, it must be caused metabolically. Observation of the bicarb shows an elevation. Of the options offered the most likely cause of this elevation is the potassium of 3.4 mEq/L.

The following ECG tracing is recorded for a 60-year-old female who is being monitored in the emergency department with a chief complaint of chest pain. Based on this rhythm, the respiratory therapist will note which of the following in the medical record? A. the patient is currently having a mild heart attack B. the patient has sinus tachycardia C. cardiac ischemia is present D. infarcted cardiac tissue is present

The correct answer is : C Explanation : Chest pain, as well as an ECG that demonstrates inverted T waves, is suggestive of myocardial ischemia.

A postoperative patient is receiving volume-controlled ventilation following abdominal surgery. Auscultation of the chest reveals vesicular breath sounds in the upper lobes, but fine crackles in the bases. This is most consistent with which of the following conditions? A. sputum in the mid-sized airways B. tuberculosis C. atelectasis D. pleural friction rub

The correct answer is : C Explanation : Fine crackles at the bases is associated with atelectasis. This is not to be confused with moist crepitant rales, which is associated with fluid overload and CHF. In addition to the fine crackles, the fact that the patient has undergone surgery is further supportive evidence that the patient has atelectasis.

Hypervolemia would first be manifested in which of the following values? A. mPAP B. SVR C. CVP D. wedge pressure

The correct answer is : C Explanation : Hypervolemia is shown hemodynamically by an increase in all hemodynamic values including CVP, PAP, PCWP, and cardiac output. Among these values CVP is the first and most significant indicator when fluid levels in the body are high or low. Remember, CVP may be known by other names such as, right atrial pressure, right side preload, right ventricular filling pressure, and right ventricular end-diastolic pressure.

A patient complaining of shortness of breath with exercise has the following pre/post bronchodilator results. Which is the most meaningful indicator that a bronchodilator is effective? A. Patient reports the maneuver is easier after the bronchodilator B. 10% improvement in FEV1/FVC%. C. 15% improvement in FEV1 D. 9% improvement in FVC

The correct answer is : C Explanation : Improvement with bronchodilator therapy is considered significant if flows increase by 12% or more, or if FEV1 increases by at least 200 mL. In this example we see a 15% improvement in FEV1, well beyond the 12% threshold.

While assisting the physician with a thoracentesis procedure, the patient suddenly becomes short of breath. Respiratory rate has increased to 32/min, heart rate is 120/min, and oxygen saturation had dropped to 86%. The respiratory therapist should FIRST do which of the following? A. obtain a chest radiograph B. perform tracheal palpation C. place the patient on a non-rebreathing mask D. insert chest tube on the side of the thoracentesis

The correct answer is : C Explanation : One of the risks associated with the thoracentesis procedure is accidental puncture of the lung. When this happens, a pneumothorax may occur. In response to this the most appropriate option offered is to place the patient on a non-rebreathing mask which is close to 100% oxygen.

For which of the following conditions is PEP therapy most beneficial? A. ARDS B. Postoperative recovery C. pneumonia D. myasthenia gravis

The correct answer is : C Explanation : PEP therapy is beneficial at removing secretions. Of the options given, only pneumonia is a disease where secretions are a primary concern.

A patient presents in the emergency department (ED) with shallow, irregular respirations. Which of the following diagnostic procedures would best help rule out a pleural effusion? A. PA chest radiograph B. ventilation/perfusion (V/Q scan) C. series of oblique chest radiographs D. pulmonary angiogram

The correct answer is : C Explanation : Pleural effusions may be diagnosed through obtaining a lateral decubitus x-ray or by obtaining a series of x-rays from oblique angles and different positions.

A respiratory therapist is given the charge to sterilize a bronchoscope. Which of the following should the respiratory therapist use to accomplish sterilization? A. Pasteurization B. Irradiation followed by ethylene oxide C. Alkaline gluteraldehyde (Cidex) D. Steam autoclave

The correct answer is : C Explanation : The best agent to use when sterilizing a bronchoscope is alkaline glutaraldehyde (Cidex).

A patient in the intensive care unit has an arterial catheter in place. The respiratory therapist notices the blood pressure from the arterial line is 95/70 mmHg. Blood pressure taken by a sphygmomanometer reads 110/78 mmHg. The patient is asymptomatic. The therapist should A. record the arterial line blood pressure B. calibrate the blood pressure cuff C. record the sphygmomanometer blood pressure D. discard the blood pressure cuff Skipped

The correct answer is : C Explanation : When blood pressure by sphygmomanometer (cuff) differs from the blood pressure obtained from an indwelling arterial line, the blood pressure taken by cuff is considered more accurate and reliable. Problems that may occur in the yard line include a clots in the line or bubbles in the transducer dome.

A respiratory therapist is examining a chest radiograph. Which of the following would the therapist examine to determine if the patient was rotated during the radiographic procedure? A. spinal process B. heart size C. comparative level of the apecies D. mediastinal shift

The correct answer is : C Explanation : When examining her chest radiograph, the respiratory therapist should look at a few key landmarks to ensure the quality of the x-ray. The apecies should be level and the spinal process should be midline. One should also be able to see the space between the vertebrae clearly. If one apex is higher than the other, the patient is likely rotated incorrectly (the patient's body was turned during the x-ray (the x-ray was not taken from a straight-on perspective)

A respiratory therapist is conducting a bedside pulmonary function test. The patient has the following results: 1st effort 2nd effort 3rd effort FVC (L) 4.3 4.4 4.45 What should the respiratory therapist recommend? A. Discard the results and calibrate the spirometer. B. Report the average of the three efforts in the medical record. C. Report the best effort in the medical record. D. Continue testing the FVC.

The correct answer is : D Explanation : Because each effort increases in volume, one can conclude that the patient is improving their understanding and skill related to the FVC maneuver. Therefore, the respiratory therapist should continue testing in order to achieve the patient's best result.

A patient has a balloon-tipped, flow-directed catheter in place in the pulmonary artery. Which pressure can the respiratory therapist expect to obtain from the distal lumen? A. MAP B. SVR C. CVP D. PAP

The correct answer is : D Explanation : Because the distal lumen of a pulmonary artery catheter is in the pulmonary artery, the pressure monitored through that lumen is called the pulmonary artery pressure or PAP.

The respiratory therapist is preparing for defibrillation of a patient with ventricular tachycardia. A pulse is not palpable. Prior to handing the physician the paddles, the therapist will ensure which of the following? A. the machine is set to 50 joules B. the machine is set to 480 joules C. synchronization is active D. synchronization is deactivated

The correct answer is : D Explanation : Defibrillation is the treatment for ventricular tachycardia without a pulse. When performing this type of defibrillation the synchronization of the machine must be deactivated. Cardioversion is performed with the synchronization active but defibrillation the synchronization must be deactivated.

A physician performing bronchoscopy does NOT need the ability to provide which of the following down the multi-lumen ports? A. lidocaine B. epinephrine C. saline D. albuterol

The correct answer is : D Explanation : During bronchoscopy, lidocaine, saline, or epinephrine may be instilled down the scope itself. Gauze may be used during the procedure to wipe the scope clean when withdrawn, but gauze is never placed into the lung.

The respiratory therapist notes a fall in pulse from 80/min to 60/min with a proportional drop in blood pressure during a suction procedure. Which of the following would be the most likely cause of these observations? A. Copious secretions B. Transient hypoxemia C. Suction pressure is too high D. Vagal reflex

The correct answer is : D Explanation : During suctioning, a patient's heart rate may fall. This is a normal physiological response associated with the vagal reflex, or the vagal nerve response. This response will also lower blood pressure.

A patient in the emergency room demonstrates intercostal use and sternal retractions. Which of the following most likely represents the patient's condition? A. ARDS B. flail chest C. carbon monoxide poisoning D. upper airway obstruction

The correct answer is : D Explanation : Intercostal use and retractions is associated with an obstruction in the upper airway.

Which of the following is a cardiac enzyme that can be used to detect the presence of a recent myocardial infarction?

Troponin

A patient has been receiving mechanical ventilatory support for one week, following a pulmonary contusion. The physician would like to consider weaning. Current ventilator settings and other data reveal: Mode SIMV f 4/min VT(set) 500 mL PEEP 5 cm H2O FIO2 0.40 VC 1.8 L VT (spont) 525 mL MIP -38 cm H2O The patient follows commands intermittently and, at times, seems somewhat disoriented to time, person, and place. The respiratory therapist should recommend

continuing mechanical ventilatory support.

To facilitate drainage of secretions consolidated in the bases of the lungs, the respiratory therapist should anticipate placing the patient in which of the following positions?

head-of-bed down 30 degrees

Which of the following can be instructed to a COPD patient to help reduce FRC?

pursed-lip breathing

A respiratory therapist is preparing for a cardioversion procedure on a patient with atrial tachycardia. Which of the following should NOT be part of that preparation?

set the defibrillator's synchronization to OFF

Immediately after placing an indwelling arterial catheter, the respiratory therapist checks the blood pressure through the transducer and notices it varies widely with the blood pressure taken by arm cuff. Most likely, the therapist needs to

zero the transducer.

Approximately how many hours will an H cylinder of oxygen at 1800 PSI last for a patient who is receiving oxygen by nasal cannula flowing at 3 L/min?

31 hrs

Which is the proper order of steps to prepare a patient with a fenestrated tracheostomy tube for speech? A. deflate cuff, remove inner cannula, cap the tube B. insert inner cannula, inflate the cuff, cap the tube C. insert inner cannula, deflate cuff, cap the tube D. cap the tube, remove inner cannula, deflate cuff

A

Which of the following may be related to a fluid balance problem in a patient? A. Congestive heart failure (CHF) B. Elevated CVP C. Pleurisy D. Pulmonary tuberculosis

A

Which of the following test results would be helpful in determining whether a patient should receive Bi-level therapy with supplemental oxygen? A. PetCO2 B. DLCO C. MVV D. SBN2

A

A 9-year-old asthmatic patient has previously responded well to Xopenex via small-volume nebulizer. The patient is intubated and has been placed on mechanical ventilation for acute respiratory failure. The physician orders Xopenex to be delivered via SVN through the ventilator circuit. Which of the following is an important consideration for effective medication delivery? A. particle deposition will be increased B. an alpha I response is increased C. Placement of the nebulizer can result in variation of medication delivery D. tachycardia is more likely

C

Which of the following could be used to determine the level of carbon monoxide poisoning for the purpose of definitive diagnosis and treatment?

Hemoximetry

A 68-year-old male is scheduled for abdominal surgery. A preoperative spirometry exam reveals an FEV1 of 82% of predicted and an FEV1/FVC of 84%. The results would best indicate the patient has

a low risk for postoperative complications

A patient receiving high-frequency mechanical ventilation has the following ABGs: PH 7.51 PaCO2 30 tor rPaO2 82 torr HCO3- 23 mEq/L Which of the following should be recommended?

a reduction in oscillatory amplitude

A respiratory therapist is asked to determine a patient's RSBI value. Which of the follow devices would be helpful in this determination?

a vane respirometer

The respiratory therapist working in a clinic receives a call from a patient who has asthma and is only able to achieve 40% of their normal baseline peak flow measurement. The therapist should advise the patient to

call the doctor immediately

The respiratory therapist observes an ECG wave form on a patient that is consistent with atrial tachycardia. The patient is complaining of chest pain, dizziness, and nausea. The respiratory therapist should recommend

cardioversion

Which of the following should NOT be included in the goals of a pulmonary rehabilitation program for a COPD patient?

return the patient normal life

An external air compressor is being used to provide pressure to a transport ventilator. The respiratory therapist notices a frayed electrical cord on the compressor. Which of the following should be done?

The compressor should be removed from service.

A 14-year old patient with asthma is in the emergency department (ED) with shortness of breath. Two treatments with Albuterol have been given, but the patient is still wheezing and peak flow measurements have not changed. The respiratory therapist should recommend? A. ordering a chest radiograph B. drawing arterial blood for gas analysis C. administering ipratropium bromide (Atrovent) D. administering cromolyn sodium (Intal)

C

A patient with suspected myasthenia gravis in the emergency room should receive which of the following to confirm a diagnosis?

Edrophonium Bromide

A patient who has chest tubes in place for the treatment of a pneumothorax does not breathe deeply despite active coaching by the respiratory therapist. The patient indicates he is unable to do so. The therapist should assess

Pain Level

During the administration of 2.5 mg nebulized albuterol, a patient complains of nausea and palpations. Heart rate has increased from 98 to 128/min. After terminating the treatment and reporting to the appropriate health care team members, which of the following should the respiratory therapist recommend for the next treatment?

Switch to 0.63 mg Xopenex

Why should a respiratory therapist FIRST open the airway of a patient who is apneic?

The patient may be experiencing upper airway occlusion

The results of a V/Q scan shows poor perfusion with adequate ventilation. A chest radiograph shows a wedge-shaped infiltrate over the right lung field. The patient most likely has

a pulmonary embolism

An air-oxygen blender is being used to provide 40% oxygen to a newborn. Suddenly the blender begins to emit a high-pitched sound. The respiratory therapist should

ensure the blender is adequately attached to 50 psi source gas.

A respiratory therapist is observing a patient in the intensive care unit (ICU) during an exacerbation of congestive heart failure. The doctor orders close monitoring of the hemodynamic status of the patient. The therapist should recommend

insertion of a pulmonary artery catheter.

Which of the following would result in an increase in pulmonary capillary wedge pressure?

mitral valve stenosis

An infant who is 35 weeks of gestation requires supplemental oxygen at 40%. Which of the following modalities is most appropriate?

oxygen hood with an air/oxygen blender

While performing nasotracheal suctioning, the respiratory therapist notices excessive blood on the outside of the suction catheter upon removal, despite the use of water-soluble lubricant. The therapist should consider

using a nasal pharyngeal airway (a trumpet).

How many milliliters of a medication whose strength is 5% is needed to deliver 25 mg of that medication?

0.50 mL....This is done by multiplying the drug strength by 10. mg/mL = 5% x 10 = 50 mL = 25 mg / 50 mg/mL = 0.50 mL

A patient with COPD is on mechanical ventilation on the following settings: Mode VC SIMV FIO2 0.45 VT 0.4 L Mandatory rate 12/min PEEP 4 cm H2O I:E 1:2 To minimize the development of autoPEEP, which of the following could be changed? 1. decrease in inspiratory time 2. increase in expiratory time 3. increase in inspiratory flow rate 4. decrease in mandatory rate

1, 2, 3, and 4

A patient is on 70%/30% heliox therapy by nonrebreathing mask. If flowing through an oxygen flow meter, what is the actual flow of the mixture if the flow meter indicates 10 L/min?

16 L/min

A patient is receiving 40% aerosol with the oxygen flow meter running at 8 L/min. What is the total flow of gas to the patient?

32 L/min

After assisting the repositioning of a tracheostomy patient, the respiratory therapist palpates a crackling sensation about the neck and clavicle area. The therapist should conclude the tracheostomy tube is A. incorrectly positioned B. in the trachea C. too small D. too large

A

After performing bedside pulmonary function testing with a computer and spirometer, the respiratory therapist examines the results and compares them to some predicted values for the patient. When doing so the therapist is comparing the patient results to A. a national database B. other patients with the same diagnosis C. statistical averages based on other tests in the computer D. baseline values for this patient

A

A 39-year-old unconscious male is brought to the emergency room after a suspected suicide attempt. He was found in his garage with the car running and the exhaust vented to the interior portion of the vehicle. Which of the following should be examined immediately? A. SpO2 B. COHb C. homocysteine level D. MetHb

B

A left-to-right shunt, due to a leaking septum wall in the heart, is best determined by an increase in mixed venous oxygen saturation. From what location would the respiratory therapist recommend blood extraction for analysis to determine mixed venous oxygen saturation? A. aorta B. pulmonary artery C. vena cava D. pulmonary vein

B

A neonate who was born at 38 weeks of gestation is receiving oxygen therapy by oxyhood and a heated aerosol device at FIO2 1.0. Arterial blood gases show pH 7.41 PaO2 98 torr PaCO2 40 torr HCO3- 24 mEq/L BE -1 mEq/L The respiratory therapist will recommend which of the following? A. Maintaining current therapy B. Decreasing FIO2 to 0.90 C. Placing infant on a mechanical ventilator D. Decreasing FIO2 to 0.50

B

An adult patient in the emergency department is febrile with a temperature of 40 deg C (104 deg F) taken rectally. Arterial blood gas analysis is done and the following results are reported with the patient breathing room air. The results are not corrected for temperature. pH 7.35 PaCO2 46 torr PaO2 80 torr HCO3- 26 mEq/L BE +1 mEq/L The respiratory therapist should recommend A. A cooling blanket B. Supplemental oxygen by nasal cannula at 5 L/min C. Non-invasive positive pressure ventilatory support (NIPPV) D. Ventilatory assistance with Bi-level

B

An infant delivered 2 hours prior is consistently cyanotic regardless of high levels of oxygenation administration. The physician would like to rule out patent ductus arteriosis. Which of the following tests should the respiratory therapist recommend A. Perform transillumination B. Compare PO2 values in the umbilical and brachial arteries C. Pass the feeding tube down both nares D. Insert a pulmonary artery catheter

B

When evaluating a patient's pulmonary condition, the therapist should include which of the following in the assessment? A. language skills and social status B. occupation and smoking history C. marital status and disease history D. insurance and disease history

B

While performing a test discharge of a metered dose inhaler (MDI) prior to administering to a patient, the respiratory therapist is unable to visually confirm the discharge of any particles or aerosol. The therapist should A. Administer the dose to the patient B. Discard the canister, obtain a new MDI C. Shake the canister and administer the dose to the patient D. Utilize a chamber and administer the dose to the patient

B

A patient in the cardiac intensive care unit has marked congestion and is expectorating large amounts of pink, frothy secretions. Oxygen saturation by pulse oximetry is 85%. This finding is most consistent with A. mitral valve stenosis B. dehydration C. pulmonary embolism D. CHF

D

A patient is in the emergency room (ER) with paradoxical chest movement on the right side. The trachea is deviated from mid-line. Which of the following should the respiratory therapist suspect? A. pleural effusion B. pulmonary embolism C. pulmonary edema D. fractured ribs and a pneumothorax

D

A respiratory therapist has achieved the following weaning parameters on a post-operative patient who experienced a total knee replacement. The SpO2 is 97% on 35% oxygen. Spont VT 480 mL VC 1600 mL VE 7.0 L MIP -4 cm H2O The next best action would be to A. reduce FIO2 to 0.30 B. return to full mechanical ventilation C. evaluate the MEF D. check the pressure manometer for leaks

D

A patient in the intensive care unit is receiving mechanical ventilation by PC SIMV. Capnography reveals increasing PECO2 while SpO2 remains steady. Which of the following can be concluded?

Dead space ventilation is decreasing

A patient with myasthenia gravis is weaning from volume-controlled ventilation and is currently on the following settings Mode SIMV Mandatory rate 4Total rate 12FIO2 0.40PEEP none Additional data shows VD/VT 0.50 A-aDO2 100 mm HgRSBI 95MIP -35 cm H2O Blood gases pH 7.36 PaCO2 42 torrPaO2 128 torrHCO3- 24 mEq/LBE -1 mEq/L Based on this information, the respiratory therapist should recommend which of the following?

Discontinue mechanical ventilation; they show adequate independent ventilatory ability

For a patient receiving mechanical ventilation, what ventilator control would be most helpful at improving oxygenation and reducing collapsed alveoli?

PEEP

The chest radiograph of a post-operative abdominal surgery patient shows pneumonia in the lower right lobe. The respiratory therapist should recommend which of the following?

PEP therapy

A physician suspects a patient is transitioning into ARDS. Which of the following calculations and or data would be helpful in evaluating the patient for this suspicion?

PF ratio and oxygen index

A respiratory therapist is analyzing the blood gas results of a capillary blood sample taken from an infant. Which of the following would not be considered a reliable or accurate value?

PO2

A patient receiving mechanical ventilation has received Halcion in the last twenty minutes. The physician orders the institution of spontaeneous breathing trials to commence immediately. Which of the following medications would be most helpful to the patient?

Romazicon (Flumazenil)

Which of the following pulmonary function tests would be most helpful in determining the presence of restrictive lung disease?

SVC

A patient is receiving heliox therapy at 70% / 30% mixture by nonrebreathing mask. The oxygen flow meter is set to 10 L/min. The patient has a minute ventilation of 18 L/min. What can the respiratory therapist conclude? A. The total flow of the gas mixture is not meeting the demands of the patient B. The set flow of gas at the oxygen flow meter should be reduced to 8 L/min C. The patient should be switched to 100% oxygen D. An air flow meter should be used in place of the oxygen flow meter

The correct answer is : A Explanation : A Heliox mixture of 70%/30% that flows through an oxygen flow meter at 10 L per minute is actually producing a total flow of 16 L per minute. The patient has a minute ventilation of 18 L per minute and therefore the total flow of gas mixture to the patient is not meeting the patient's inspiratory demand.

A decreasing A-aDO2 is most closely related to which of the following? A. decreasing atelectasis B. Increased pulmonary shunting C. refractory hypoxemia D. increasing venous admixture

The correct answer is : A Explanation : A decrease in the oxygen tension gap between the alveoli and the artery (A-a gradient) indicates the patient is improving. The best interpretation, of the options given, is decreasing atelectasis. We do not acutally know that it is decreasing atelectasis specifically, but this option is the only one that would result in improvement of the patient and that can affect the alveolar-arterial oxygen gradient. An increase in venous admixture and an increase in pulmonary shunting would result in an increased A-aDO2 which is opposite of what is actually happening.

Which of the following can accurately be stated about a widening alveolar-arterial oxygen gradient? A. increased intrapulmonary shunting B. decreased venous admixture C. improving V/Q mismatch D. onset of oxygen toxicity

The correct answer is : A Explanation : A normal alveolar arterial oxygen gradient is below 65 mmHg. When that gradient rises to 200, 300, 400, or more, then you have what is called shunting. If the A-a gradient is less than 300 it's technically called a ventilation perfusion (VQ) mismatch. But generally speaking, as the gradient widens it can properly be said that there is an increase in pulmonary shunting.

A patient with ARDS and asthma could benefit from which of the following medications? A. Tobramycin and albuterol B. exogenous surfactant C. Spiriva and decadron D. cromolyn sodium

The correct answer is : A Explanation : A patient with adult respiratory distress syndrome could benefit from bronchodilators to open up the airway and surfactant therapy to decrease the surface tension of the alveoli.

Which of the following is an important strategy when caring for a patient with adult respiratory distress syndrome? A. keep mean airway pressures as low as possible B. ventilate in Control mode with paralytics C. keep FIO2 higher than 0.60 D. promote AutoPEEP

The correct answer is : A Explanation : A patient with adult respiratory distress syndrome has decreased pulmonary compliance. This places the patient at risk for barotrauma because of the high ventilatory pressures required during mechanical ventilation. Therefore, keeping mean airway pressures as low as possible is of primary concern.

Which of the following would be an appropriate recommendation for a patient with markedly decreased static compliance? A. permissive hypoxemia B. SIMV VC ventilation C. high inspiratory flow rates D. prolonged expiratory time

The correct answer is : A Explanation : A patient with decreased static compliance, such as ARDS and pulmonary fibrosis, requires increased inspiratory time to allow more time for gases to distribute throughout the lungs during inhalation. The question, however, is asking which of the following is appropriate for this type of patient. Decreasing inspiratory time, or a decrease in I:E ratio is the best answer.

A pulmonary rehabilitation patient is having difficulty complying with a smoking cessation program. A physical and psychological screen reveals a heavy physiological dependence and a low psychological dependence on smoking. Which of the following is appropriate? A. nicotine replacement therapy B. psychological counseling C. Versed D. Valium

The correct answer is : A Explanation : A physiological dependence on nicotine can be overcome slowly by having the patient use nicotine replacement therapy. This may include chewing gum with nicotine or transdermal preparations.

Which of the following may NOT be used with a face mask? A. peak flow meter B. NPPV C. Wright's respirometer D. CPAP

The correct answer is : A Explanation : A respiratory therapist can achieve the goals of CPAP, NPPV, and even a Wright's respirometer with the use of a facemask. However a peak flow meter must be positioned between the teeth of the patient with the lips sealed, and nose plugged in order to achieve accurate peak-flow measurements.

For which of the following is the respiratory therapist evaluating if examining phase IV of a single-breath nitrogen test (SBN2)? A. Closing volume B. Mixed deadspace and alveolar gases C. Pure deadspace gases D. Evenness of gas distribution

The correct answer is : A Explanation : A single-breath nitrogen elimination test (SBN2) is used to determine the adequacy of distribution of gases and the evenness of that distribution. A patient breathes out a single breath and the nitrogen is analyzed in four phases. The first phase is pure dead space. The second phase is a mixture of dead space and alveolar gas. The third phase represents pure alveolar gas and is the stage where gas distribution is truly assessed. Phase 4 is called closing volume.

A tandem aerosol device is used A. to deliver sufficient flowrates through large-volume nebulizers. B. for patients requiring FIO2 of 0.50 or greater. C. on patients with croup D. whenever the flowrate is set to 15 L/min or above.

The correct answer is : A Explanation : A tandem aerosol device is used when only one device is incapable of producing enough total gas flow to meet or exceed the patient's inspiratory demand.

A chest radiograph of an abdominal post-operative patient shows abnormal elevation of the left hemidiaphragm. Which of the following conditions explains the observation? A. atelectasis in the left lower lobe B. hemothorax C. pneumothorax on the left D. herniation of the left hemidiaphragm

The correct answer is : A Explanation : Abnormal elevation of the left hemidiaphragm is an indication that the lung on that side is smaller for some reason. This could be due to a partial pneumothorax or profound atelectasis. Oftentimes, atelectasis can develop as a result of surgery. Therefore, the raised hemidiaphragm, combined with the postoperative status of the patient, indicate the most likely problem is atelectasis in the left lower lobe.

A 48-year-old patient has just been orally intubated prior to being placed on volume-controlled ventilation. The respiratory therapist should do which of the following to best confirm proper placement of the endotracheal tube? A. obtain a chest radiograph B. examine tube markings near the lips C. assess chest rise symmetry D. auscultate breath sounds

The correct answer is : A Explanation : Although there are many ways to determine the location of an endotracheal tube inside the patient's airway, there is only one way to confirm its location - a chest x-ray or radiograph. Other methods include auscultation of breath sounds and observation of chest rise and symmetry. While these methods are quick and helpful they do not provide CONFIRMING evidence of the ET tube's location.

A premature neonate with respiratory distress receives a dose of exogenous surfactant. Three hours following the administration, distress returns. FIO2 has required titration from 0.40 to 0.70. The respiratory therapist should recommend which of the following FIRST? A. chest radiography B. repeat surfactant administration C. ECMO D. mechanical ventilation with NAVA

The correct answer is : A Explanation : Because improvement from exogenous surfactant does not persist, the infant may have a more significant pulmonary problem. This should be investigated with chest radiography.

A patient is receiving non-invasive positive pressure ventilation with an IPAP of 25 and an EPAP of 10 cmH2O. FIO2 is 0.3. The following blood gas data on these settings is available: pH 7.35 PaCO2 45 mm Hg PaO2 72 mm Hg HCO3- 24 mEq/L BE 0 mEq/L Which of the following changes is most appropriate? A. increase FIO2 B. increase EPAP, decrease IPAP C. increase IPAP D. increase EPAP

The correct answer is : A Explanation : Because this patient is hypoxic the most appropriate response is to increase FIO2. This is especially true because FIO2 is considerably low and may be increased.

The following chest CT image is being observed by the respiratory therapist. The therapist should recommend A. Thoracentesis B. Postural draining and percussion of the right chest C. Needle decompression of the left chest D. Oblique chest radiograph

The correct answer is : A Explanation : Close observation of this radiological image reveals the lung is partially collapsed. Because of the partial collapse, is not likely that air has entered that space and therefore this is not a pneumothorax. Rather, this exam shows an increase in fluid in the pleural area, decompressing the lung tissue. This is known as a hemothorax. The most appropriate action is a thoracentesis.

A patient is undergoing full cardiac arrest. The patient is intubated with an 7.5 ET tube. The physician and nurse are having difficulty obtaining intravenous access. After several minutes of chest compressions, the ECG waveform monitor shows asystole in two leads when chest compressions are paused. The respiratory therapist should recommend which of the following? A. administer epinephrine down the ET tube B. administer intramuscular Atropine C. establish an intraosseous catheter, administer Isuprel D. perform a precordial thump

The correct answer is : A Explanation : During full cardiac arrest, in the presence of asystole, it is appropriate to administer the medication epinephrine. This medication may be administered directly down the endotracheal tube if there is no intravenous access.

The respiratory therapist should gather which of the following equipment in preparation for the administration of heliox therapy? A. non-rebreathing mask B. air/oxygen blender C. air flow meter D. face tent

The correct answer is : A Explanation : Helium-oxygen therapy requires a non-rebreathing mask for delivery.

Following the insertion of a tracheostomy tube, the patient is found to have diffuse crackles upon auscultation secondary to subcutaneous emphysema. Which of the following radiographic findings would be expected with this condition? A. Hyperlucency in the soft tissues B. Scattered patchy infiltrates C. Tracheal shift from midline D. Diffuse pulmonary hyperlucency

The correct answer is : A Explanation : Hyperlucency, seen on a chest x-ray is darker in color. Air is radiolucent. Therefore, air located in the soft tissue, as seen with subcutaneous emphysema would result in a hyperlucent X-ray over soft tissue areas. Subcutaneous emphysema by itself will not shift the trachea from midline. Scattered patchy infiltrates are associated with ARDS, not subcutaneous emphysema.

The respiratory therapist analyzes the blood from a patient rescued from an area lake. The patient had been ice fishing when the ice broke and he plunged into the water. Rescuers indicate it took about 5 minutes to get the patient out. The patient remains hypothermic. Blood gas analysis reveals pH 7.50 PaCO2 28 torr PaO2 65 torr The respiratory therapist should suspect erroneous readings in which of the following? A. PaO2 would read erroneously low B. both PaO2 and PaCO2 would read erroneously high C. pH would be erroneously low D. PaCO2 would be erroneously high

The correct answer is : A Explanation : If the patient is hypothermic and blood gas results are not corrected for temperature, the reported results from the analyzer will be erroneously low.

A patient is in the intensive care unit following the repair of a hip fracture. Within 2 hours after surgery, the patient is hypotensive and is increasingly short of breath. Heart rate has risen from 90 to 110 bpm. Which of the following test results would be most helpful at this time? A. Hb and HCT B. Arterial blood gases C. Creatinine D. COHb

The correct answer is : A Explanation : Immediately following surgery a patient that is hypotensive is most likely hemorrhaging from a leaky vessel from the operation. To confirm the loss of blood an Hb and HCT may be done to examine hemoglobin level. If low, the patient should be sent back to surgery to explore for a leak but not before blood is adminisitered, if the is an option.

In a patient with restrictive disease, which of the following would the respiratory therapist expect to be higher than predicted? A. peak flow B. tidal volume C. inspiratory capacity D. expiratory reserve volume

The correct answer is : A Explanation : In restrictive disease patterns, lung volumes and capacities are reduced. However, because of the decreased elasticity of the lung, oftentimes peak flow rates increase above predicted values.

A written teaching plan for a pediatric patient with chronic asthma should include A. PEFR monitoring and proper MDI use B. proper titration of bronchodilator dosage C. determination of oxygen saturation D. titration of oxygen

The correct answer is : A Explanation : Monitoring one's peak flow, using a metered-dose inhaler correctly, and learning to detect early symptoms of oncoming bronchoconstriction are all important aspects of a teaching plan for a pediatric asthma patient. A patient should never be encouraged or instructed to adjust their prescribed frequency for any type of therapy by the respiratory therapist.

Following administration of an aerosol treatment with 2.0 mL of 20% Acetylcysteine (Mucomyst), the patient demonstrates expiratory wheezing bilaterally. Which of the following represents the best action? A. administer Albuterol sulfate by small volume nebulizer B. administer Narcan (naloxone) C. decrease the volume of mucolytic administration to 1.0 cc D. decrease the strength of future treatments to 10%

The correct answer is : A Explanation : Mucomyst is used for thinning secretions. Unfortunately, it can also cause bronchoconstriction. When this happens, administration of a bronchodilator is appropriate. Often, administering a bronchodilator with the Mucomyst is also indicated.

Refractory hypoxemia is associated with which of the following diseases? A. ARDS B. Pleural effusion C. Rocky Mountain Spotted Fever D. Chronic bronchitis

The correct answer is : A Explanation : Of the options listed, the disease that is most closely associated with refractory hypoxemia is ARDS. Refractory hypoxemia is a condition where arterial oxygenation does not increase in spite of elevated FIO2.

Which of the following lung volume measurements is normally the largest? A. Vital Capacity B. Functional residual capacity C. Inspiratory reserve volume D. Residual capacity

The correct answer is : A Explanation : Of the volumes offered, vital capacity would be the largest.

A patient in a current myasthenic crisis is being monitored every 2 hours. The most recent evaluation shows: MIP -29 cmH2O, VC 1.35 L, Vt 410 mL, respiratory rate of 24. The respiratory therapist should recommend A. Continue to monitor the patient closely B. Begin incentive spirometry C. Intubate and initiate mechanical ventilatory support D. Administer Tensilon

The correct answer is : A Explanation : Patients with myasthenia gravis have a somewhat slow onset of paralysis of ventilatory muscles. The patient should be monitored by examining MIP, VC, and VT. when VC falls below 1 L, the patient should receive mechanical ventilatory support. Prior to that, the patient should be routinely observed and monitored closely.

A patient recovering from a stab-wound to the chest and a consequent pneumothorax is on a chest tube drainage system with 3 chambers. The patient is also receiving volume-controlled ventilation and has the following data noted in the medical record 8 am 11 am 2 pm Plateau pressure (Pplat) 20 cm H2O 18 cm H2O 10 cm H2O PIP 38 cm H2O 28 cm H2O 16 cm H2O PEEP 5 cm H2O 5 cm H2O 5 cm H2O Set VT 500 mL 500 mL 500 mL Return VT 496 mL 452 mL 280 mL Significant bubbling is noted in the water-seal chamber. The respiratory therapist should recommend A. sending the patient to surgery B. implementing an inspiratory retard C. switching to pressure controlled ventilation D. replacing the chest tube drainage system

The correct answer is : A Explanation : Plateau pressures are decreasing over a short period of time. This is not likely do to changes in pulmonary compliance. Additionally, the low return volume and increased bubbling in the water seal chamber of the chest tube drainage system are consistent with a loss of volume through the lung - a perforation in the lung tissue. The only way to correct perforation in lung tissue is to send the patient to surgery.

Which of the following conditions would benefit most from polysomnography? A. obesity with increased day-time sleepiness B. congestive heart failure C. COPD D. suspected increased airway resistance

The correct answer is : A Explanation : Polysomnography is another word for sleep study. An obese patient who has daytime sleepiness would benefit most from polysomnography. Keep in mind that Polysomnography is the study of the problem - not the solution. But it must be done for diagnostic purposes and in order to find the adequate CPAP setting.

To determine potential post-operative risk of a patient preparing for surgery, the respiratory therapist should recommend which of the following tests? A. basic spirometry B. pulmonary diffusion studies C. cardiac stress test D. pulmonary stress test

The correct answer is : A Explanation : Potential postoperative risk may be assessed preoperatively through basic spirometry.

A physician is concerned about escalating pulmonary artery pressures in a patient with ARDS and is considering use of nitric oxide therapy. Which of the following would help to further assess pulmonary artery pressure? A. placement of a quadruple-lumen pulmonary artery catheter B. 64-slice computed tomography (CT scan) C. echocardiography with M-mode D. V/Q scan with contrast

The correct answer is : A Explanation : Pulmonary artery pressures must be measured invasively with a pulmonary artery catheter. None of the other options here will suffice.

A patient has a balloon-tipped, flow-directed catheter in place in the pulmonary artery. Which pressure can the respiratory therapist expect to obtain from the proximal lumen? A. CVP B. MAP C. PAP D. PCWP

The correct answer is : A Explanation : The proximal lumen of a pulmonary artery catheter is situated inside or right before the right atrium. The value measured here is called CVP.

A chest radiograph indicates the presence of plate-like infiltrates. This finding is most closely associated with which of the following? A. atelectasis B. mycoplasma pneumonia C. mesothelioma D. CHF

The correct answer is : A Explanation : The radiological description of platelike infiltrates is most closely associated with atelectasis.

A 62-year-old male has ventricular tachycardia. The patient is not alert and will not respond to verbal stimuli. A pulse is palpable. The respiratory therapist will respond by A. performing cardioversion B. defibrillating the patient C. administering Isuprel (Isoproterenol) D. administering methyl prednisone, IV

The correct answer is : A Explanation : The treatment for ventricular tachycardia without a pulse is defibrillation at 300-360 W. Cardioversion, however, is used with atrial arrhythmias and ventricular tachycardia where there is a pulse present.

A patient is receiving manual ventilatory support with a bag-valve connected to a size 8.0 mm ET tube. While transporting the patient from CT scan back to the intensive care unit, the respiratory therapist notices oxygen saturation has dropped to 88% and that the bag-valve is easier to squeeze than normal. Suspecting mal-positioning of the ET tube, the respiratory therapist should first A. examine chest rise and auscultate breath sounds B. obtain an arterial blood gas C. perform diagnostic chest percussion D. call for a chest radiograph

The correct answer is : A Explanation : There are several options when assessing the proper location of an endotracheal tube. These options range from quick to thorough. The quickest way is to assess the tube location visually. This may be done by observing symmetry of chest rise or marking on the tube relative to the lip line. The next quickest way is to auscultate breath sounds. To CONFIRM location of the endotracheal tube, a chest x-ray is required but is time-consuming and should not be the first action taken to assess the endotracheal tube position.

The following data is observed on a patient who is 20 minutes post coronary artery bypass graft (CABG): 10:05 10:10 10:15 SVO2 (%) 69 67 52 SpO2 (%) 98 97 97 Which of the following can the respiratory therapist conclude? A. cardiac output is decreasing B. tissue oxygen consumption is decreasing C. pulmonary shunting is decreasing D. C(a-v)O2 is narrowing

The correct answer is : A Explanation : This data shows a steady SpO2 with a decreasing venous oxygen saturation. From this information, it can be concluded that oxygen tissue consumption is increasing because there is less oxygen remaining after the blood passes over the tissues. An increase in the gap between arterial saturation and venous saturation is most commonly caused by a decrease in cardiac output.

A 70-kg (154-lb) patient is being weaned from mechanical ventilation. The initial T-piece trial data is reported below. The respiratory therapist should Before trial 30 minutes on T-piece VT (spont) 440 mL 390 mL MIP -32 cmH2O -30 cmH2O VC 2.5 L 2.5 L HR 82 110 RR 16 22 SpO2 (%) 92 89 A. return to mechanical ventilatory support B. extend current trial to 1 hour C. permanently discontinue mechanical ventilation D. administer Doxapram, IV

The correct answer is : A Explanation : This patient is obviously weaning from mechanical ventilation. T-Piece trials show the patient is doing well on most parameters but a closer look shows that the heart rate has increased more than 20 above baseline. This is an indication to discontinue weaning at this time.

A 90 kg (198 lb) adult male is receiving volume-controlled ventilation following abdominal surgery. Current ventilator settings and corresponding arterial blood gas results are: Mode SIMV Mandatory rate 15 Total rate 15 Tidal volume 550 mL PEEP 5 cm H2O FIO2 0.45 pH 7.50 PaCO2 30 torr PaO2 118 torr HCO3- 25 mEq/L BE +1 mEq/L The respiratory therapist should recommend A. decreasing the mandatory rate B. reducing PEEP to 2 cm H2O C. switching to assist/control mode D. decreasing FIO2 to 0.40

The correct answer is : A Explanation : This patient, according to arterial blood gas results, is hyperventilating with a PaCO2 of 30 mmHg. To correct this, rate, tidal volume, or dead space may be adjusted. Because PaCO2 is off target by more than 4 mmHg, the most appropriate action is to decrease mandatory rate.

A respiratory therapy supervisor is responsible for making suggestions in the initial development of a smoking cessation program. Which of the following would be part of an effective smoking cessation program? A. nicotine replacement therapy B. daily completion of an asthma action plan C. removing pulmonary irritants from the house D. daily COHb analysis

The correct answer is : A Explanation : To stop smoking, both physiological and emotional barriers must be addressed. The patient may be helped best through nicotine replacement therapy and emotional support. Routine monitoring of the patient's progress is also helpful. Removing irritants from the house will not help the patient stop smoking.

A newborn with IRDS is being mechanically ventilated in the time-cycled, pressure-limited SIMV mode. The physician would like to increase minute ventilation. In order to accomplish this request, the respiratory therapist would make what modification? A. Increase I-time B. Change to a volume-cycled mode C. Decrease flowrate D. Increase tidal volume

The correct answer is : A Explanation : Unlike volume ventilation, time-cycled pressure-limited mechanical ventilation will deliver more volume per breath when inspiratory time is increased. Because the breath will be delivered for a longer period of time at a constant flow rate, ventilation will increase. It is not appropriate to change to a volume-cycled mode on a newborn. Tidal volume is not a direct control in this mode and therefore is an incorrect answer. Decreasing the flow rate would result in a reduction in ventilation.

On a time-cycled, pressure-limited ventilator, modifications to the ventilator settings occurred that resulted in an I:E of 1:6 becoming an I:E of 1:4. Which of the following is a possible setting change that could have produced this new I:E? A. an increase in I-Time B. development of autoPEEP C. Increase in PEEP D. Decrease in the flowrate

The correct answer is : A Explanation : Unlike volume-cycled mechanical ventilation, flow rate does not affect the I:E ratio in time-cycled mechanical ventilation. Only direct changes to the inspiratory time (I-Time) or to the total respiratory rate will affect the I:E ratio.

Which of the following best describes the maneuver most suited to evaluate a patient's vocal cord paralysis? A. maximum inhalation followed by a complete exhalation followed by maximum inhalation B. pant for 6-8 seconds followed by a maximum inhalation and hold C. maximum inhalation followed by maximum exhalation, repeated for 12-15 seconds D. maximum inhalation with a 3-5 second breath hold, followed by a quick exhalation

The correct answer is : A Explanation : Vocal cord paralysis and other causes of upper airway obstruction is best diagnosed through a pulmonary function test, specifically a flow-volume loop. The answer that most closely resembles a flow-volume loop is "maximum inhalation followed by complete exultation followed by maximum inhalation". These are the actual instructions given to the patient for the maneuver.

While transferring a ventilator-dependent patient from bed to gurney, the respiratory therapist notices sudden resistance when manually ventilating a patient through a standard tracheostomy tube. The therapist is unable to pass a suction catheter. The therapist should NEXT A. replace the tracheostomy tube. B. ensure adequate cuff inflation. C. provide ventilation by bag and mask. D. call for help.

The correct answer is : A Explanation : When a standard tracheostomy tube becomes clogged, the tracheostomy tube must be removed and replaced to ensure ventilation.

While administering aerosolized bronchodilator with IPPB, the therapist notices the pressure manometer has a significant negative deflection with each breath. The patient appears to be showing signs of fatigue. The respiratory therapist should A. increase sensitivity B. increase pressure C. decrease pressure D. increase inspiratory flow rate

The correct answer is : A Explanation : When delivering an IPPB treatment, the respiratory therapist should monitor the pressure manometer to see how much effort the patient must produce to trigger the machine to cycle into inhalation. If a significant negative deflection of the needle is observed, the machine is not as sensitive as it needs to be. Increasing sensitivity is appropriate.

Which of the following would a respiratory therapy supervisor NOT include when considering appropriate staffing levels for a shift? A. Patient meal times B. Patient locations C. Frequency of therapy D. Type of therapy ordered

The correct answer is : A Explanation : When planning staffing for patient care, the location of the patient (the distance staff must travel to get to the patient), frequency of therapy, and the type of therapy ordered are all important considerations. While patient mealtimes may require extra coordination of time, they should not affect staffing.

An oxygen-dependent COPD patient is brought to the emergency room on 4 L/min oxygen by nasal cannula. From the paramedic documentation it is observed that respiratory rate has changed from 30 to 18 bpm. Heart rate has decreased from 120 to 90. Saturation is 92% and the patient's color has also improved. The therapist should do which of the following? A. decrease flow to 1 lpm B. maintain current therapy C. switch to a partial rebreathing mask D. discontinue oxygen therapy

The correct answer is : A Explanation : When the COPD patient receives more than 2 L/min. oxygen by nasal cannula, or more than 28% by any other means, it is possible to suppress their hypoxic drive and reduce their total ventilation. In this question the patient has a decreased respiratory rate from 30 to 18. The solution is to lower the supplemental oxygen. An oxygen flow rate of 1 to 2 L/min. by nasal cannula is most appropriate.

A patient receiving oxygen at home via a molecular sieve device complains that he is not feeling any oxygen coming from the cannula. The respiratory therapist should A. have the patient utilize portable oxygen until the oxygen concentrator is examined B. inform the patient that this finding is normal C. instruct the patient to do a flow check/calibration of the concentrator D. instruct the patient to call the manufacturer of the concentrator

The correct answer is : A Explanation : When the patient complains of not receiving enough oxygen or when the function of oxygen delivery device is a question, the first best option is to ensure adequate oxygenation of the patient by switching to another modality that is known to be functioning properly.

A pulmonary function study reveals a forced vital capacity (FVC) that is greater than a slow vital capacity (SVC). To which of the following can this be attributed? A. poor patient effort on the SVC maneuver B. partial diaphragmatic paralysis C. chronic air-trapping D. excessive effort on the FVC maneuver

The correct answer is : A Explanation : When the patient exhales slowly they are able to breathe out more air than if they were to exhale quickly. For this reason, and FVC should always be less than an SVC. If FVC is noted to be higher than the SVC, this is clear evidence that the patient's effort during the SVC maneuver was poor or insufficient.

Which of the following can be obtained from an unconscious, unresponsive patient? A. Tidal volume B. Peak expiratory flow C. Forced vital capacity D. Slow vital capacity

The correct answer is : A Explanation : When the patient is unconscious or unresponsive, and possibly comatose, assessments involving cooperation are not possible. In this question, a tidal volume may be assessed because cooperation is not required, even if a mask is required to obtain the measurement. However, obtaining a slow vital capacity, expiratory flowrate, or a forced vital capacity measurement requires the patient to follow specific directions to perform the maneuver.

The respiratory therapist notices the PtcO2 value on a transcutaneous monitor has suddenly jumped within the last minute. The therapist should suspect a problem with A. Adherence of the electrode to the skin B. Insufficient heating of the electrode C. Development of skin burns D. Overheating of the electrode

The correct answer is : A Explanation : When the transcutaneous PO2 value suddenly increases, the most likely cause is an electrode that is become dislodged from the skin and is reading the oxygen tension of the ambient air.

The oxygen percentage measured in an oxygen hood is erratic when measured with a recently calibrated galvanic fuel cell oxygen analyzer. The most likely remedy is which of the following? A. increase flow to the hood B. obtain small oxygen hood C. recalibrate the analyzer D. replace the fuel cell

The correct answer is : A Explanation : With devices such as oxygen hoods, where the space must be filled completely to obtain consistent oxygen percentage, adequate flow is important. If erratic oxygen percentage measurements are observed, the likely problem is inadequate flow to the hood.

A patient is found to have a rhythm on the ECG monitor consistent with asystole. Which of the following is the best initial action of the respiratory therapist? A. Defibrillate at 50 joules, synchronized B. Confirm in a second lead C. Begin chest compressions D. Defibrillate at 360 joules, unsynchronized

The correct answer is : B Explanation : This rhythm is called asystole. When observed, prior to starting chest compressions, the rhythm should be confirmed in his second lead.

After attempting to pass a catheter down an oral ET tube, the respiratory therapist feels strongly that the endotracheal tube cuff is herniated over the end of the ET tube. The bag-valve remains difficult to squeeze and oxygen saturation is 86% and falling. The therapist should do which of the following? A. switch to a high pressure demand valve B. remove the ET tube and insert a new one C. perform nasal intubation with the oral tube left in place D. call the physician and report the finding

The correct answer is : B Explanation : A herniated endotracheal tube cuff indicates th ET tube is defective and therefore should be replaced.

Which of the following can be obtained from an unconscious, unresponsive patient to determine the adequacy of ventilation? A. vital capacity B. Tidal volume C. MIP D. MEP

The correct answer is : B Explanation : A patient who is unconscious, unresponsive, and possibly comatose is unable to follow directions. Therefore, assessments of ventilation can only include those evaluations that do not require patient cooperation. Of those listed, only tidal volume, respiratory rate and pattern, and arterial blood gas results may be done without the cooperation of the patient.

A 32 year-old female patient reports to the emergency room complaining of a sudden onset of coughing. She indicates the coughing started when she was eating. Which of the following would most definitively rule out a cancerous mass in the lungs: A. lateral neck radiograph B. spiral CT scan C. BUN D. V/Q scan

The correct answer is : B Explanation : A sudden onset of coughing while the patient was eating is most likely associated with foreign body aspiration (food in the main stem bronchus). However, nonproductive coughing could also be caused by a cancerous mass in the upper airway. To rule this out a procedure that shows the upper airway with three dimensionality is most appropriate. A ventilation perfusion scan and a BUN blood test would not do this. It may be tempting to get a lateral neck x-ray, but the results are not three dimensional and therefore would not be as confirming in nature. The most confirming examination would be a spiral CT scan.

Which of the following devices may be used to create aqueous materials of know gas values which are helpful at ensuring the quality control of a blood gas analyzer? A. multiple wave-length blood testing B. tonometer C. Sanz electrode D. capnograph

The correct answer is : B Explanation : A tonometer is a device used to create aqueous materials for blood gas machines to ensure the precision and accuracy of blood gas analyzers.

A patient with asthma shows a small improvement in peak expiratory flow rates from 45% to 49% of predicted after 1 hour of continuous bronchodilator therapy with 10 mg/hr albuterol. Which of the following should the therapist consider NEXT? A. ECMO B. heliox therapy C. hyperbaric therapy D. 1.25 mg Xopenex q 20 min X 3 doses

The correct answer is : B Explanation : After several attempts have been made to bronchodilate a patient with asthma, the use of Heliox can help to immediately lower airway resistance while the source of the bronchoconstriction can be addressed by other means.

Pulmonary function results of a 22 year old patient who began smoking 1 year prior shows an airway resistance (Raw) of 1.7 cm H2O/L/sec. Which of the following represents an accurate interpretation of the measurement? A. chronic asthma B. normal C. acute asthma D. neuromuscular disease

The correct answer is : B Explanation : An airway resistance of 1.7 cmH2O/L/sec is considered normal. Elevated airway resistance is associated with asthma.

After the oral intubation of a neonate who is 38 weeks of gestation, the respiratory therapist observes air leaking around the ET tube with each squeeze of the bag-valve. The respiratory therapist should A. add air to the cuff B. intubate the infant with a larger ET tube C. obtain an arterial blood gas D. monitor the infant's oxygen saturation

The correct answer is : B Explanation : An infant who has gas leaking around the ET tube with each positive pressure ventilation is most likely in need of a larger ET tube. These tubes do not have cuffs and therefore, the seal is controlled by the size of the tube.

During positive-pressure mask ventilation on a newborn, cricoid pressure may be used to A. check for a pulse. B. limit gas entry into the stomach. C. prevent aspiration of meconium D. decrease the neonate's heart rate.

The correct answer is : B Explanation : By applying proper cricoid pressure, the rigid rings of the trachea are indirectly pressed down over the esophagus, which blocks air from entering the stomach during mask ventilation.

A patient is orally intubated and receiving mechanical ventilation. The respiratory therapist notes the PetCO2 monitor is persistently reading 60 mmHg. Which of the following can the therapist conclude? A. the patient is being hyperventilated B. minute ventilation should be increased C. the infrared device is not working properly D. the CO2 detector is covered with condensate

The correct answer is : B Explanation : End-tidal CO2 is correlated to arterial CO2. An end-tidal CO2 reading of 60 mmHg correlates with an arterial CO2 of 70 mmHg. This is clearly hypoventilation and should be addressed by increasing minute ventilation.

A patient in the emergency room complains of frequent vomiting over the last 24 hours. A 12-lead ECG shows a normal P-R interval and flattened T waves. Which of the following is the most likely explanation for the ECG waveform? A. first degree heart block B. hypokalemia C. congestive heart failure D. cardiac ischemia

The correct answer is : B Explanation : Fluid loss from a patient will likely change electrolyte status. An ECG that shows flattened T waves is associated with hypokalemia. This is the expected change that occurs to those patients who have fluid loss or whose electrolyte levels are not normal.

A respiratory therapist is setting up home oxygen by oxygen concentrator for a home care patient. Which of the following is the most appropriate safety consideration for the patient? A. poison control hotline phone number B. adequate electrical circuit load C. proper length of the oxygen tubing D. oxygen concentrator recalibration procedure

The correct answer is : B Explanation : Home care of a pulmonary patient has much to do with ensuring safety in the home. Electrical devices, such as oxygen concentrators, should be installed with care and with the patient's safety in mind. In this case, it is most important to ensure adequate electrical load for the oxygen concentrator.

Which of the following describes how Mucomyst (Acetylcysteine) works? A. changes the pH of sputum B. disrupts di-sulfide bonds C. causes bronchodilation D. increases mucociliary activity

The correct answer is : B Explanation : Mucomyst (Acetylcysteine) is a medication used to decrease the thickness of secretions. It works by disrupting the di-sulfide bonds in the mucus.

Which of the following humidification devices can provide 100% humidity at 37 deg C? A. large volume nebulizer B. heated wick humidifier C. babbington nebulizer D. bubble humidifier

The correct answer is : B Explanation : Of the devices listed, only a heated wick-type humidifier can provide 100% humidity and make up the entire humidity deficit for a patient.

A 70-year-old patient has the following pulmonary function results: Predicted Observed FVC (L) 5.80 3.20 FEV1/FVC (%) 70 72 FEV1 (L) 4.21 3.44 TLC (L) 6.90 4.69 SVC (L) 6.00 3.84 The respiratory therapist should anticipate a treatment plan for A. mixed restrictive and obstructive pulmonary defect B. mild restrictive lung disease C. moderate obstructive lung disease process D. mild obstructive pulmonary defect

The correct answer is : B Explanation : Pulmonary function results show that the flow is above 70% which is considered normal. Therefore, the patient is not considered obstructive. However, SVC, which is the key indicator of restrictiveness, is less than 80% of predicted and indicates the patient is restrictive. To determine what 80% of predicted is for this patient, the SVC predicted volume of 6.0 L can be multiplied by eight to determine the 80% mark. 6x8 is 48. Putting the decimal back in place reveals an 80% mark of 4.8 L. Since this patient has an SVC of 3.84 L, the patient is considered to be restrictive.

Predicted Observed TLC (liters) 4.50 4.30 FRC (liters) 2.45 2.30 SVC (liters) 3.30 3.10 FEV1 (liters) 2.15 1.65 FEF50 (liters/sec) 4.00 3.09 Which of the following represents the most appropriate interpretation of the preceding spirometry results? A. pulmonary fibrosis B. cystic fibrosis C. scoliosis D. tracheomalacia

The correct answer is : B Explanation : Pulmonary function values show low flows as shown by an FEV1 which is less than 80% of predicted. This means the patient is obstructive. Volumes are normal as shown by the slow vital capacity, and indicates the patient is not restrictive. We must pick the option that is an obstructive disease. In this case, only cystic fibrosis is obstructive.

A patient with ARDS is receiving mechanical ventilatory support. An end-tidal CO2 monitor shows a PetCO2 of 59 torr. The therapist should A. obtain an arterial blood gas B. increase minute ventilation C. continue current therapy and ventilator settings D. recalibrate the capnography

The correct answer is : B Explanation : Remember that the end-tidal CO2 data from a capnometer is usually 10 mmHg less than the corresponding arterial CO2 level. Thus, an end-tidal CO2 of 59 mmHg is consistent with an arterial CO2 of about 69 mmHg. The best solution to correct this problem is to increase ventilation.

The addition of a 2.0 second inspiratory plateau facilitates the determination of which of the following physiological parameters? A. laminar and turbulent airflow properties B. static compliance C. airway resistance D. pulmonary secretions

The correct answer is : B Explanation : The addition of an inspiratory hold or plateau helps to determine what is called the plateau pressure, which is used primarily to calculate and evaluate static compliance.

A patient has the following arterial blood gas results and ventilatory parameters: pH 7.12 PaCO2 30 mm Hg PaO2 80 mm Hg HCO3- 9 mEq/L BE +15 mEq/L RR 34 VT (spont) 600 mL Which of the following would provide helpful diagnostic information? A. venous level of HCO3- B. blood glucose level C. BUN D. creatinine

The correct answer is : B Explanation : The blood glucose level should be evaluated in his case in order to determine the cause of the severe metabolic acidosis, which is likely to be diabetic ketoacidosis.

Which of the following would be the best medication to use with a patient who will require oral intubation in an emergent setting? A. Flurazepam B. Anectine C. Valium D. Versed

The correct answer is : B Explanation : The drugs most commonly used to facilitate intubation are Anectine (succinylcholine), Pavulon (pancuronuim bromide), Curare (d-tubocurarine) and Vecuronium. Valium and Versed are anxiety/pain relievers. Flurazepam is used to treat insomnia.

The following pressure-volume ventilator graphic is observed on a patient receiving VC A/C ventilation. The therapist should recommend A. decrease the minute ventilation B. decrease the tidal volume C. decrease in rate D. decrease the pressure limit

The correct answer is : B Explanation : The image shown here is a ventilator graphic that shows the relationship between the volume and the pressure. The presence of the small bird beak indicates that there is some level of pulmonary overdistension. This can be remedied by decreasing the tidal volume.

When selecting a suction catheter to be used in an oral endotracheal tube, the respiratory therapist should select a catheter whose diameter should not exceed what fraction of the internal diameter of the endotracheal tube? A. 3/4 B. 1/2 C. 2/3 D. 1/4

The correct answer is : B Explanation : The outer diameter of a suction catheter should not exceed one half of the inner diameter of an endotracheal tube.

A patient who weighs 70 kg (154 lb) has a minute ventilation requirement of 15 L/min to maintain a PaCO2 of 43 torr. Which of the following can explain the ventilatory requirements? A. pulmonary emphysema B. febrile conditions C. decreased PVR D. decreased dead space ventilation

The correct answer is : B Explanation : The patient is febrile and likely has an infection, which can cause an increase in minute ventilation. Intrapulmonary shunting will also cause the patient to increase minute ventilatory requirements.

The respiratory therapist observes a gradual increase in peak inspiratory pressure over the last 24 hours on a patient who is receiving mechanical ventilation. Additionally, the patient has become febrile and is producing purulent pulmonary secretions. The therapist should recommend which of the following? A. exploratory bronchoscopy B. sputum gram stain C. bronchodilator therapy D. lung tissue biopsy

The correct answer is : B Explanation : The presence of fever and purulent pulmonary secretions suggests the possibility of an infection. A CBC would confirm the presence of an infection but a sputum gram stain would actually identify the class of offending pathogenic organism (gram-positive or gram-negative) Although not offered here, a more appropriate exam would be a sputum C&S which would provide specific organism information and suggest the most appropriate antimicrobial therapy. If a sputum gram stain is done and the class of organism is identified, a general class of antibiotics may be started until more specific pathogenic organism identification is done.

Which of the following is an important strategy when caring for a patient with adult respiratory distress syndrome? A. keep FIO2 higher than 0.60 B. keep FIO2 below 0.60 when possible C. promote AutoPEEP D. ventilate in Control mode with paralytics

The correct answer is : B Explanation : The primary strategy for a patients with adult respiratory distress syndrome who are on a ventilator is to keep the FIO2 as low as possible. Generally below 0.6 is most appropriate. This, however is a tough battle because hypoxemia is the cardinal attribute of adult respiratory distress syndrome. In order to compensate and keep hypoxemia at a minimum, PEEP is also used.

What is the standard dosing frequency of inhaled Atrovent (Ipratroprium bromide)? A. q 4-6 hrs B. q 6-8 hrs C. q 2-4 hrs D. q 8-12 hrs

The correct answer is : B Explanation : The proper dosing frequency for inhaled Atrovent is every 6 to 8 hours.

Which of the following patients or conditions would benefit most from PEP therapy? A. myasthenia gravis B. chronic bronchitis C. ARDS D. Guillain-Barre' syndrome

The correct answer is : B Explanation : The purpose of PEP therapy is to help mobilize secretions. Of the diseases presented only chronic bronchitis has a significant problem with excess secretions and is therefore more suitable for PEP therapy.

A patient with an ideal body weight of 80-kg (176-lb) is being evaluated for cessation of mechanical ventilation. The following weaning parameters and clinical data are available: Vital Capacity 1.2 L RSBI 50 Spont VT 400 mL Resp Rate 20/min Which of the parameters is the most reliable indicator that the patient is ready for cold cessation of mechanical ventilation? A. Spont VT B. RSBI C. VC D. RR

The correct answer is : B Explanation : The rapid shallow breathing index is the best indicator that the aptient is ready for weaning in this scenario. RSBI = RR/VT. A result less than 106 is acceptable. 50 is well below and suggests the patient should be able to successfully wean off mechanical ventilation.

A written teaching plan for asthma patients using peak flow meters should include A. the unit of measure for the flow meter. B. the meaning of red, yellow and green zone results. C. avoidance of placing one's teeth around the mouthpiece. D. the importance of trending over baseline results.

The correct answer is : B Explanation : The respiratory therapist may determine that there is value in the three options that are incorrect in this scenario, but it is important to consider the best answer, which, in this case, is a thorough understanding of the color zones. Understanding the zones will help an asthma patient effectively manage their condition over time.

A respiratory therapist palpates a crackling sensation over the clavicle area when evaluating a patient receiving IPPB treatments via tracheostomy tube. This is most likely caused by A. fractured clavicle B. subcutaneous emphysema C. secretions in the upper airway D. air embolism

The correct answer is : B Explanation : The term for crackling sensations palpated over the clavicle and the neck on a patient who has a tracheostomy tube is called subcutaneous emphysema. This condition is usually caused from an improperly positioned tracheostomy tube.

The expired air over one minute that participates in active gas exchange is one's A. minute ventilation. B. alveolar ventilation. C. respiratory quotient. D. anatomical gas exchange.

The correct answer is : B Explanation : The total expired air over 1 minute is known as minute ventilation. Not all of this air participates in gas exchange due to anatomical dead space. Once anatomical deadspace is removed, the remaining ventilation is known as alveolar minute ventilation or simply alveolar ventilation.

The respiratory therapist measures the volume of a single exhalation on a patient who is breathing at normal rate and rhythm. Which of the following volumes has the therapist observed? A. inspiratory reserve B. tidal C. functional residual D. expiratory reserve

The correct answer is : B Explanation : The volume that is being observed when a patient is breathing normally is called tidal volume.

A respiratory therapist notes the word cachectic used in the medical documentation to describe a patient. This is most consistent with A. obesity B. a wasted apperance C. obtunded D. non-attentiveness

The correct answer is : B Explanation : The word "cachectic" refers to a wasted appearance, skinny, and poor skin turgor. A person with AIDS or pulmonary tuberculosis may demonstrate this.

During direct laryngoscopy, a respiratory therapist notes a patient has a Mallampati score of 4. The therapist should recommend A. surgical tracheostomy B. use of a video assist device C. use of an airway exchange catheter D. use of a bougie device

The correct answer is : B Explanation : There are many methods to facilitate intubation when the intubation procedure appears that aspect of it may be difficult. A Mallampati scoring system is a method to rate the intubation difficulty of the patient. It is based upon the ability to visualize key anatomical landmarks in the oropharynx. Specifically, the system looks at the ability to visualize the uvula, the hard palate, the soft palate, and other landmarks. The Mallampati scoring system is a four-point system with a '1' being a non-complicated intubation and a '4' exhibiting maximum complexity. In this case the Mallampati score of '4' indicates that the patient may be a complex intubation. In such a case the use of a video assist device is appropriate. Other options include bronchoscopy, use of a bougie device, and other direct visualization devices. Direct laryngoscopy should be avoided.

A female patient is intubated with a 7.0 mm endotracheal tube. The tube is noted to be positioned at 26 cm at the teeth. Breath sounds are not bilateral. Which of the following will best confirm placement of the artificial airway? A. chest rise B. chest radiograph C. breath sounds D. capnography

The correct answer is : B Explanation : There are several ways to assess proper endotracheal tube placement, such as auscultation of breathsounds, observance of symetric chest rise, or end-tidal CO2 evaluation. However, this scenario asks for confirmation of proper artificial airway placement, which is accomplished with a chest X-ray.

A patient with ARDS receiving volume-controlled ventilation has the following arterial blood results on the settings below: Mode Assist/control Mandatory rate 14 VT 400 mL FIO2 0.70 PEEP 20 cm H2O pH 7.29 PaCO2 50 torr PaO2 69 torr HCO3- 23 mEq/L BE -1 mEq/L Which of the following changes is most appropriate? A. increase PEEP to 22 cmH2O B. increase rate to 16 C. add 100 mL deadspace D. increase FIO2 to 0.8

The correct answer is : B Explanation : This ARDS patient is both under ventilating and under oxygenating. Of these problems under ventilation should be addressed first. The most appropriate method of reducing CO2 that is offered among these options is an increase in rate. Adjusting PEEP or FIO2 will address oxygenation. Adding deadspace will cause a change in CO2 in the wrong direction (increase).

A patient with CHF has the following input/output history Yesterday Today IN 1900 mL 2100 mL OUT 1000 mL 1100 mL Which of the following clinical findings is most likely to be observed? A. friction rub B. moist crepitant rales C. decreased egophony D. bronchial wheezing

The correct answer is : B Explanation : This CHF patient is obviously retaining fluid because he is receiving more fluid in that he is putting out. The clinical finding most associated with fluid retention is moist crepitant rales. Fine rales are associated with atelectasis but moist rales is associated with fluid retention, especially congestive heart failure (CHF).

A patient is receiving volume-controlled ventilation in the assist/control mode. The following data are available: Plateau Pressure Peak pressure Exhaled VT 6 PM 16 cm H2O 22 cm H2O 561 mL 9 PM 22 cm H2O 28 cm H2O 548 mL 1 PM 27 cm H2O 33 cm H2O 550 mL The respiratory therapist would categorize the most significant problem noted from this data as A. increasing pulmonary compliance B. decreasing pulmonary compliance C. decreasing airway resistance D. increasing airway resistance

The correct answer is : B Explanation : This data shows an increase in peak airway pressures. However, plateau pressures are increasing to the same degree, indicating that the source of the increasing peak airway pressures is due to a decrease in pulmonary compliance, or a stiffening of the lung.

The following data is available for a patient receiving mechanical ventilation: 2 PM 4 PM Peak airway pressure 24 cm H2O 30 cm H2O Static airway pressure 12 cm H2O 12 cm H2O Delivered tidal volume 550 mL 550 mL Inspiratory flow 40 L/min 40 L/min PEEP 5 cm H2O 5 cm H2O Which of the following may explain these results A. decreasing pulmonary compliance B. excess water in the ventilator circuit C. leak in the ventilator circuit D. deflated endotracheal tube cuff

The correct answer is : B Explanation : This data shows that pulmonary compliance is remaining steady with static airway pressures that are not significantly changing. However, peak airway pressures are increasing. Increasing peak airway pressures while static pressures remain constant is an indication of either secretions in the airway or another artificial blockage of the airway. Of the options offered, only excess water in the ventilator circuit can explain the decrease in dynamic compliance.

A patient with combined central and obstructive sleep apnea is being titrated for initial settings on Bi-Level therapy with oxygen bleed-in at 2 L/min. IPAP is 16 cmH2O and an EPAP is 8 cmH2O. Set back-up rate is 12/min. The following blood gas data with a spontaneous rate of 12/min is available: pH 7.33 PaCO2 48 mm Hg PaO2 80 mm Hg HCO3- 24 mEq/L BE 0 mEq/L Which of the following changes is most appropriate? A. decrease EPAP to 6 cmH2O B. increase IPAP to 18 cm H2O C. increase backup rate to 14/min D. increase IPAP and EPAP by 2 cm H2O

The correct answer is : B Explanation : This patient has elevated CO2, indicating under ventilation, but adequate oxygenation. Ventilation is increased by increasing the distance between the inspiratory and expiratory pressures. In this case simply raising IPAP alone is sufficient.

A 38-year-old male is breathing spontaneously at a respiratory rate of 30/min with moderate accessory muscle use. Arterial blood gas results and other clinical data are listed : pH 7.32 PaCO2 48 torr PaO2 74 torr HCO3- 25 mEq/L BE -1 mEq/L The respiratory therapist should recommend A. high frequency chest wall oscillation (HFCWO) B. NIV C. expiratory flow resistor therapy D. CPAP

The correct answer is : B Explanation : This patient is demonstrating respiratory acidosis. Although the patient is under ventilating, acute respiratory failure is not yet shown. When pH falls below 7.25, acute respiratory failure is present and the patient is in need of full mechanical ventilatory support. In this case, the patient is showing beginning signs of under ventilation, as shown by an increasing CO2. Additionally, the respiratory rate is climbing, which indicates a significant increase in work of breathing. The most appropriate way to combat this is to use NIV therapy (BiPAP, NIPPV).

During weaning from mechanical ventilation, a patient is on the following settings and has the following clinical data: Mode SIMV Mandatory rate 4 Total rate 32 VT (set) 450 mL VT (spont) 200 mL FIO2 0.40 PEEP 5 cm H2O PS 5 cm H2O pH 7.34 PaCO2 46 mm Hg PaO2 79 mm Hg HCO3- 25 mEq/L BE -1 mEq/L Which of the following can be increased to improve the patient's ventilation and decrease the work of breathing? A. VT B. pressure support C. FIO2 D. PEEP

The correct answer is : B Explanation : This patient is obviously weaning from mechanical ventilatory support. The SIMV mode, in addition to a low mandatory rate, are associated with weaning. The total rate of the patient however, is excessive and the spontaneous tidal volume is only 200 mL, which is far less than the 5 mL per kilogram needed to sustain life. To help this patient continue weaning, pressure support is most helpful. Pressure support helps the patient take larger spontaneous tidal volumes, which will promote a decrease in total rate and will ultimately result in a decrease in work of breathing.

A patient who weighs 75 kg (165-lbs) is receiving mechanical ventilatory support after ingestional error of barbiturates. Recent settings and clinical data include: Mode SIMV Mandatory rate 4 VT 500 mL FIO2 0.40 PEEP 5 cm H2O Total rate 12 VT (spont) 290 mL MIP -30 cm H2O The therapist should recommend: A. switch to CPAP B. continue mechanical ventilatory support C. decrease rate to 2 D. remove from mechanical ventilation

The correct answer is : B Explanation : This patient is obviously weaning from the ventilator. All parameters look appropriate except for spontaneous tidal volume. Tidal volume must be at least 5 mL per kilogram. This patient weighs 75 kg and therefore must have a minimum tidal volume of 375 mL. The patient is only able to sustain 290 mL making them ineligible for further weaning.

A 70-lb (32-kg) adult patient suffering from dwarfism is being mechanically ventilated with an adult circuit on the following settings and arterial blood gas values: Mode SIMV Mandatory rate 18 VT 150 mL FIO2 0.50 PEEP 8 cm H2O pH 7.32 PaCO2 48 torr PaO2 78 torr HCO3- 26 mEq/L BE +2 mEq/L Based on these values, the respiratory therapist should do which of the following? A. increase FIO2 B. increase VT C. change to a pediatric circuit D. increase respiratory rate

The correct answer is : B Explanation : This patient is under ventilating and is hypoxic. Of these two problems addressing ventilation is most important. Based on the patient's ideal body weight an increase in tidal volume is the most appropriate. This is because a tidal volume of 150 mL is slightly below the patient's minimum tidal volume range of 192-256 mL.

Which of the following ventilator circuit change frequencies is important to minimize nosocomial infections among patients receiving mechanical ventilation? A. monthly B. every 7 days C. when soiled D. daily

The correct answer is : B Explanation : This question is about infection control and requires you to memorize that ventilator circuits must be changed every seven days. It may occur differently in your hospital, but you must memorize seven days for the exam.

A patient shows signs of hypoxemia while on a heated aerosol set at 40% with the flow rate set to 8 L/min. The patient's minute ventilation is 38.0 L. Which of the following changes should be made? A. increase FIO2 to 0.5, maintain flow rate B. increase flow rate to 10 L/min C. increase to FIO2 to 1.0 with a flow rate to 15 L/min D. decrease FIO2 to 28%, maintain flow rate

The correct answer is : B Explanation : This question requires one to recognize that the flow rate is not meeting the patient's ventilatory demand. It requires a little math to determine how much total flow the patient is receiving. In this case the ratio for 40% is 3:1. If you add 3+1, you get 4. Then, if you multiply 4 times 8 L/min. you get a total flow of 32 L per minute. The patient's reported minute ventilation is 38 L per minute. This would cause the patient to entrain extra air and would ultimately lower FIO2, resulting in an increase in hypoxemia.

Arterial Venous pH 7.40 7.38 PCO2 40 torr 42 torr PO2 85 torr 48 torr HCO3- 24 mEq/L 24 mEq/L BE 0 mEq/L 0 mEq/L SAT 96% 80% Hb 14 g/dL 14 g/dL A patient receiving mechanical ventilator support in the intensive care unit has the follow blood gas results: The respiratory therapist should record which of the following C(a-v)O2 values in the patient's medical record? A. 6.2 vol% B. 3.1 vol% C. 1.6 vol% D. 5.1 vol%

The correct answer is : B Explanation : To answer this question, one must complete two calculations. First, CaO2 and CvO2 must be determined. Once CvO2 is subtracted from CaO2, 3.1 vol% is the closest answer.

A patient is receiving mechanical ventilator support with an adult VC ventilator. The following data is available: pH 7.41 PB 747 torr PaCO2 40 torr PaO2 70 torr HCO3- 24 mEq/L BE +0 mEq/L FIO2 0.60 Mode Assist/control Rate 14 VT 550 mL Pressure (peak) 24 cm H2O PEEP 5 cm H2O The respiratory therapist should report which of the following values as an accurate A-aDO2? A. 350 torr B. 300 torr C. 225 torr D. 490 torr

The correct answer is : B Explanation : To determine the alveolar-arterial oxygen difference or gradient, one must first calculate the alveolar oxygen tension by using the alveolar air equation or by a shortcut. To determine PAO2, first take the oxygen percentage and multiply it by 7. 60% x 7 = 420. Next, subtract the CO2 + 10. 420 - 50 = 370 mmHg. Finally, if you subtract the PaO2 you get 300 mmHg, which represents the A-aDO2.

The following data is available for a patient receiving volume-controlled mechanical ventilation: Mode Assist/control VT 500 mL Mandatory rate 16 FIO2 0.5 PEEP 5 cm H2O PIP 28 cm H2O PetCO2 28 torr pH 7.42 PaCO2 38 torr PaO2 98 torr PvO2 76 torr HCO3- 22 mEq/L BE -2 mEq/L SaO2 98% SvO2 78% Hb 15 g/dL The respiratory therapist should record which of the following values as an accurate CaO2? A. 5.0 vol% B. 19.4 vol% C. 25.1 vol% D. 15.2 vol%

The correct answer is : B Explanation : To determine the arterial oxygen content, the hemoglobin must be multiplied by the saturation and also multiplied times a factor of 1.34. In this case 15 x .98 x 1.34 is nearly 20 vol%. The closest answer is 19.4 vol%

What is the minute alveolar ventilation (L/min) of a 68-kg (150-lb) IBW patient who has a tidal volume of 500 mL and a respiratory rate of 14? A. 4.9 B. 6.0 C. 2.1 D. 7.0

The correct answer is : B Explanation : To determine the minute ventilation of the patient, one should multiply the tidal volume by the rate. In this case, tidal volume is expressed in milliliters. Because minute ventilation should be expressed as L/min, tidal volume must be translated to liters. To subtract deadspace from the tidal volume, one must estimate deadspace by looking at the ideal body weight in lbs. In this case, 150 lbs is about 150 mL of deadspace. Thus alveolar tidal volume (the amount of volume that reaches the alveoli) is 350 mL, or 0.35 L. This number should then be multiplied by the RR (14) to get the minute alveolar ventilation. (0.35 x 14 = 4.9)

A patient receiving mechanical ventilatory support via volume-controlled ventilation is experiencing a decrease in pulmonary compliance. Peak airway pressure has risen to 42 cm H2O in the previous week. Which of the following can the respiratory therapist do to improve gas exchange? A. Add PEEP B. Decrease inspiratory flow rate C. Increase mandatory rate D. Increase inspiratory flow rate

The correct answer is : B Explanation : To improve gas exchange, a patient benefits most from a slow and prolonged inspiratory phase. Providing volume slowly has a tendency to result in greater gas distribution throughout the lungs and results in improved gas exchange.

A respiratory therapist is performing endotracheal suctioning of a patient with thick, tenacious secretions. Suctioning has been difficult and somewhat ineffective. Which of the following would be most helpful at increasing the effectiveness of suctioning? A. increasing suction pressure B. increasing suction catheter size C. increasing suction catheter length D. increasing suction duration

The correct answer is : B Explanation : To increase the effectiveness of suctioning, increasing suction catheter size, increasing suction pressure, and increasing duration are three methods that may be used. The most effective method is to increase the suction catheter size. However, caution must be used. The suction catheter size may not exceed half the diameter of the endotracheal tube. If the patient is already using a maximum suction catheter size than an increase in suction pressure is the next choice.

A 32-week gestational age infant is receiving supplemental oxygen by oxyhood with an oxygen blender set at 40% and a heated aerosol. The oxygen analyzer fuel cell is placed near the infant's mouth and FIO2 is determined to be 35%. The physician would like to confirm the patient's oxygenation status. The respiratory therapist should evaluate A. COHb B. Pulse oximetery C. PetCO2 D. Capillary blood gas

The correct answer is : B Explanation : To monitor the infant's oxygenation status, there are several options, including arterial blood gas analysis, transcutaneous PO2 monitoring, and pulse oximetry. Capillary blood gases are helpful in monitoring for pH and CO2 but are erroneous when it comes to measuring PO2. The PO2 results on the capillary sample should not be trusted.

While performing endotracheal suctioning of a patient, the respiratory therapist observes PVCs on the monitor. How might the therapist modify this therapy to prevent PVCs from occurring during future suctioning? A. apply intermittent suctioning technique B. Decrease suction duration C. Change the suction pressure D. Instill lidocaine, IV

The correct answer is : B Explanation : To prevent the development of PVCs during endotracheal suctioning, the respiratory therapist should decrease suction duration, which is a primary cause of PVCs, and hyper oxygenate the patient prior to suctioning. Both of these actions will prevent the onset of acute hypoxemia that sometimes occurs with prolonged suctioning.

A patient is receiving volume-controlled mechanical ventilation. Which of the following adjustments are appropriate to reduce autoPEEP? A. decrease expiratory time B. increase inspiratory flow rate C. increase PEEP D. increase tidal volume

The correct answer is : B Explanation : To reduce autoPEEP, expiratory time must be increased. This may be done by decreasing inspiratory time, which is accomplished by increasing inspiratory flow rate.

Several hours after oral endotracheal extubation of an 8.0-mm tube, an adult patient begins to demonstrate inspiratory stridor. Which of the following would be most helpful to the patient? A. Atropine sulfate B. racemic epinephrine C. heated aerosol by mask D. Albuterol

The correct answer is : B Explanation : When a patient is extubated and has swelling of the upper airway tissues as demonstrated by stridor, racemic epinephrine is an appropriate medication to reduce swelling quickly. Another, less effective option, is to provide a cool mist. In extreme cases, such as marked stridor, reintubation or an emergency tracheotomy may be indicated.

A homecare patient indicates that she is not getting enough air from her transtracheal oxygen catheter. The therapist should instruct the patient to A. remove the catheter. B. use a nasal cannula. C. increase the flow of oxygen to the catheter. D. flush the catheter with saline.

The correct answer is : B Explanation : When a patient reports difficulty getting air or oxygen through one oxygen delivery device, before troubleshooting, the most appropriate first action is to ensure adequate ventilation by changing to a different, reliable device. In this case, switching to a nasal cannula is most appropriate.

A patient vomits and aspirates during postural drainage and percussion while in the appropriate position to drain the left lower lobe. The respiratory therapist's immediate reaction should be to A. implement incentive spirometry B. place the patient in the right lower lobe drainage position C. suction the patient's oropharynx D. place the patient in the upper right lobe drainage position

The correct answer is : B Explanation : When somebody vomits during postural drainage and percussion, when draining a specific side, the immediate response should be to drain the opposite side.

A 38-week gestational age infant is to receive oxygen therapy by a blender set at 100% with a heated large-volume nebulizer. To ensure the delivery of the ordered FIO2, the respiratory therapist should properly set the nebulizer FIO2 to A. 21% B. 100% C. 0% D. 50%

The correct answer is : B Explanation : When using an air-oxygen blender, oxygen percentage is predetermined prior to entering the large volume nebulizer device. When this happens, the large volume nebulizer should be set at 100% in order to avoid entraining additional room air and lowering FIO2.

Arterial blood gases on a patient in the cardiac intensive care unit are as follows: pH 7.31 PaCO2 35 mm Hg PaO2 100 mm Hg HCO3- 18 mEq/L BE -6 mEq/L Which of the following is an appropriate action? A. decrease FIO2 B. administer sodium bicarbonate C. increase FIO2 D. increase minute volume

The correct answer is : B Explanation : When we look at a patient's blood gas we usually start by looking at the ventilation aspect. In this case the PaCO2, which is 35 mmHg, shows normal ventilation and therefore should have normal acid-base relationship. However, when we observe the acid-base relationship we see that the pH is 7.31 and therefore is acidodic. If the source of the acidosis is not respiratory, then it must be metabolic. A quick glance at the rest of the blood gas shows the HCO3- is low. Therefore, the administration of sodium bicarbonate is most appropriate to treat metabolic acidosis.

A patient has undergone overnight oximetry. Data shows several episodes of oxygen desaturation throughout the night. The respiratory therapist would expect orders for which of the following kind of testing? A. V/Q scan B. 24-hour Holter monitoring C. polysomnography D. pulmonary function testing

The correct answer is : C Explanation : The use of overnight oximetry is often used to detect oxygen desaturation during the night that may be associated with central or obstructive sleep apnea. Because data indicates some periodic desaturation, the practitioner should suspect possible sleep apnea and should order diagnostic testing that will confirm or rule out the suspicion. This kind of sleep testing is also called polysomnography.

A respiratory therapist notes a blunted costophrenic angle in the lower right lung periphery when viewing a patient's PA chest radiograph. The patient is demonstrating ventilatory distress after being involved in a bicycle accident at high speed. What should the therapist recommend to further investigate the problem in the lower right lung periphery? A. BAL (bronchoalveolar lavage) B. ventilation/perfusion scan C. lateral decubitus chest radiograph D. AP chest radiograph

The correct answer is : C Explanation : When suspecting a pleural effusion, the key radiographic tests is a lateral decubitus chest radiograph. A bronchoalveolar lavage is used to clean out the lungs and a ventilation perfusion scan will help determine blood flow adequacy and gas flow adequacy in the pulmonary vasculature and alveoli, respectively, and is not relevant in this case. Merely changing to an AP x-ray will not help diagnose a pleural effusion.

A patient is experiencing shortness of breath. The radiological AP view of the chest shows a blunt costophrenic angle in the left lower lobe. This would indicate A. atelectasis. B. pneumonia. C. pleural effusion. D. pulmonary embolus.

The correct answer is : C Explanation : A chest X-ray reveals a pleural effusion if we see descriptors such as "blunt or obliterated costophrenic angles", or "concave superior interface".

Which of the following are associated with a fenestrated tracheostomy tube? A. button B. foam cuff C. cap D. silver

The correct answer is : C Explanation : A fenestrated tracheostomy tube has an inner cannula with a hole to facilitate talking and weaning. It also has an inner cannula for resuscitation, or mechanical ventilation. The cuff is like that of an endotracheal tube, low-pressure, high-volume. And finally, a cap is used to close the tube for speech therapy. When the cap is used, the inner cannula should be removed and the cuff should be deflated.

The respiratory therapist reads a patient's medical record and notes documented unilateral friction rub on the left. Which of the following may be associated with this finding? A. pulmonary edema B. pulmonary embolism C. pulmonary tuberculosis D. mycoplasma pneumonia

The correct answer is : C Explanation : A friction rub, sometimes called a pleural friction rub, is most associated with pulmonary tuberculosis.

An alert patient with supraventricular tachycardia is to undergo cardioversion. The respiratory therapist will do which of the following to prepare for the procedure? A. Obtain an arterial blood gas B. Obtain an arterial blood gas C. Prepare a bag-valve-mask D. Make ready a preparation of Atropine

The correct answer is : C Explanation : A patient who undergoes cardioversion often receives sedation prior to receiving synchronized defibrillation. The defibrillation procedure, as well as the as the sedative medications, can cause suppression or loss of the ventilatory drive. Therefore, prior to the procedure, assuring the ability to deliver emergency oxygen and to provide manual resuscitation is paramount.

Which phase of the nitrogen elimination (SBN2) test should the therapist primarily look at to find a mixture of deadspace and alveolar gas? A. Phase III B. Phase IV C. Phase II D. Phase I

The correct answer is : C Explanation : A single breath nitrogen elimination test (SBN2) I is useful in determining the evenness of gas distribution in the lungs. The results come in four phases as the patient exhales a single breath. Phase I is the exhalation of pure deadspace gas. Phase II consists of some deadspace and some alveolar gas. Phase III consists of pure alveolar gas and is the phase that indicates the evenness of distribution. Phase IV is called "closing volume".

A spiral CT scan would best evaluate which of the following? A. atelectasis B. sarcoidosis C. pulmonary embolism D. pneumonia

The correct answer is : C Explanation : A spiral CT scan is a way of examining the lungs in a three-dimensional fashion. So, it would be most useful for evaluating for a pulmonary embolism. Pneumonia, atelectasis, and sarcoidosis do not have three-dimensional qualities and therefore do not need to be observed through a spiral CT scan.

A chest radiograph on a patient shows a "wedge" pattern. This is most suggestive of A. pulmonary contusion B. pleural effusion C. pulmonary embolism D. pulmonary shunting

The correct answer is : C Explanation : A wedge-shaped pattern on a chest radiograph is associated with a pulmonary embolism. When a clot forms in the pulmonary vasculature all vessels beyond do not receive blood. Since the pulmonary vasculature is somewhat like a tree the clotting of one vessel leads to a wedge-shape developing on the chest radiograph.

The physician requests a recommendation for the delivery of surfactant therapy for a 32-week gestational neonate weighing 1800 grams with IRDS. The respiratory therapist should recommend which of the following? A. Provide one dose of Exosurf, turning the patient from side to side B. Provide 2 doses of Survanta using 2.5 mL/kg C. Intubate with a size 3.0mm endotracheal tube for the delivery of surfactant D. Review the APGAR scores prior to therapy

The correct answer is : C Explanation : APGAR scores would not be helpful for a premature neonate who needs surfactant. The options, which include Exosurf and Survanta, do not contain appropriate dosages or recommended delivery methods. Therefore, the only appropriate answer is to intubate with a properly sized endotracheal tube for an 1800 gram infant.

In response to an Asthma action plan, the patient has attempted to contact their physician after determining peak flow measurement is less than 50% of the patient's usual baseline value. The physician is not responding to the call. According to NAEP guidelines, the patient should NEXT A. take a short-term bronchodilator, check peak flow in 1 hour B. take a corticosteroid inhaler and check again in 20 minutes C. report to the hospital or call an ambulance D. take a short-term bronchodilator and contact a different physician

The correct answer is : C Explanation : According the national asthma guidelines, a self monitored peak flow of 50% of baseline is an indication to contact one's physician. However, if one's physician is not available, the patient should report to the hospital or to the emergency room or call an ambulance.

Which of the following is the best indicator of the adequacy of alveolar ventilation? A. color B. respiratory rate C. PaCO2 from an arterial blood gas analysis D. tidal volume

The correct answer is : C Explanation : Adequate alveolar ventilation is best manifest by the exhalation of CO2 observed on an arterial blood gas. Tidal volume, respiratory rate, or color are not good indicators of the adequacy of alveolar ventilation.

A patient complains of shortness of breath during a nebulizer treatment is hypertonic saline. The respiratory therapist should do which of the following? A. Switch to normal saline B. Add Albuterol to the nebulizer treatment C. Discontinue therapy and notify the physician D. Switch to hypotonic saline

The correct answer is : C Explanation : Adverse reactions during any therapy should be responded to initially by discontinuing therapy and notifying the physician.

A 32-year-old 81-kg (178-lb) adult male is receiving mechanical ventilatory support. Settings are as follows: Mode Assist/control Mandatory rate 10 Total rate 18 Tidal volume 450 mL FIO2 0.6 PEEP 5 cm H2O The patient is cyanotic and anxious. The respiratory therapist will recommend which of the following? A. increase FIO2 B. decrease mandatory rate C. increase tidal volume D. increase inspiratory flow rate

The correct answer is : C Explanation : All of the answers offered indicate a change in the ventilator settings. However, the question shows no evidence of arterial blood gas analysis. This is because a close examination of the ventilator settings will show that there is something wrong. The patient weighs 81 kg which would suggest a minimum tidal volume of 500 mL. (6 x 81 kg = 486 mL). The patient's set tidal volume is only 450 mL. This should be corrected.

A COPD patient who receives 2 lpm continuous oxygen therapy by nasal cannula is exercising in conjunction with a monitored pulmonary rehabilitation program. The patient has begun breathing quickly and deeply. To ensure consistent arterial oxygenation, the respiratory therapist should A. decrease oxygen flow to 1 L/min B. use a partial rebreathing mask C. increase oxygen flow rate D. use a nonrebreathing mask

The correct answer is : C Explanation : Although COPD patients should rarely have more than 2 L/min. continuous oxygen, when they exercise and breathe more deeply, they inadvertently lower their FIO2, especially when on a nasal cannula. In such a case, it is appropriate to increase the flow rate temporarily for the duration of the exercise.

A COPD patient who receives 2 lpm continuous oxygen therapy by nasal cannula is exercising in conjunction with a monitored pulmonary rehabilitation program. The patient has begun breathing quickly and deeply. To ensure consistent arterial oxygenation, the respiratory therapist should A. use a nonrebreathing mask B. use a partial rebreathing mask C. increase oxygen flow rate D. decrease oxygen flow to 1 L/min

The correct answer is : C Explanation : Although COPD patients should rarely have more than 2 L/min. continuous oxygen, when they exercise and breathe more deeply, they inadvertently lower their FIO2, especially when on a nasal cannula. In such a case, it is appropriate to increase the flow rate temporarily for the duration of the exercise.

A patient with diabetes has significant respiratory distress. Arterial blood gases reveal a pH of 7.10, PaCO2 of 29 torr, and a PaO2 of 81 torr. The patient has a spontaneous tidal volume of 700 mL and a respiratory rate of 30/min. The patient is receiving 100% oxygen by an air entrainment device with the flow at 15 L/min. Which of the following will help the patient and is the most appropriate action? A. increase flow rate to 25 L/min B. switch to a Venturi mask at FIO2 0.5 with a flow set at 12 L/min C. employ a tandem air-entrainment device D. sedate the patient

The correct answer is : C Explanation : Although this patient is showing obvious signs of ventilatory failure the options given do not allow us to address that problem. The options given relate to a correction of the oxygenation status of the patient. If you look closely you will see that the total gas flow to the patient at 100% at 15 L per minute is insufficient to keep up with the patient's inspiratory demand. Some simple math, multiplying 600 mL times a rate of 30 per minute is equal to a minute ventilation of 18 L/min. But, the patient is receiving only 15 L/min. To correct this problem and meet or exceed the patient's inspiratory demand, the best option is to utilize a tandem device, or a side-by-side large volume nebulizer. This will double the total flow to the patient to be 30 L/min, which will exceed the patient's inspiratory demand.

A 4-year-old male weighing 27-kg (60-lbs) is receiving volume-cycled mechanical ventilation with a microprocessor ventilator using an adult circuit and a cuffed size 5.0-mm ET tube. Tubing compliance and airway resistance was measured and delivered tidal volumes are corrected. An arterial blood gas reveals the need for an increase in VE of 1.6 L/min. What is the optimal way to accomplish this? A. subtract 10 ml deadspace B. change to a pediatric circuit C. increase respiratory rate D. replace ET tube with a 4.5 uncuffed tube

The correct answer is : C Explanation : An increase in VE, or minute ventilation, may be done by increasing tidal volume or increasing respiratory rate. Of the options offered, increasing respiratory rate is the most appropriate. The other options will not result in an increase in minute ventilation.

After 20 minutes into a spontaneous breathing trial (SBT) a mechanically ventilated female patient becomes anxious. Her baseline heart rate has increased by 20 /min. What should the respiratory therapist do? A. Calculate the RSBI B. Increase the FIO2 and continue to monitor C. Return to full mechanical ventilation D. Measure MIP

The correct answer is : C Explanation : An increasing heart rate by at least 20 bpm defines the failure of a weaning trial. Further calculations or measurements are not appropriate as the patient needs mechanical ventilation prior to further deterioration.

The following is observed on a newborn 1 minute after delivery: Color pink extremities, blue trunk Reflex irritability grimace Muscle tone weak flexion Heart rate 98/min Respiratory effort weak cry What is the one-minute APGAR score A. 3 B. 4 C. 5 D. 6

The correct answer is : C Explanation : Apgar is a ten-point scoring system that is used to determine the condition of a newborn immediately after birth. Each of these criteria demonstrates a middle response, or a one-point response. Color demonstrates a mixed result and therefore receives one point. A grimace is a very slight reflex and therefore also receives one point. If muscle tone is flaccid the patient receives zero points, but in this case there is some weak flexion allowing for one point. Heart rate is present but is not over 100 and therefore is worth one point. The infant is crying but not vigorously and is therefore worth 1 point. Together the Apgar score is five.

A 65-year-old patient with increased work of breathing is on bi-level therapy at an IPAP of 16 cmH2O and EPAP of 8 cmH2O, FIO2 0.55. The following blood gases are reported on these settings: pH 7.31 PaCO2 49 torr PaO2 110 torr HCO3- 24 mEq/L BE 0 mEq/L The therapist should A. increase IPAP B. increase IPAP and EPAP C. decrease EPAP D. increase EPAP and IPAP

The correct answer is : C Explanation : Arterial blood gases reveal the patient is under ventilating as manifested by high CO2. Additionally, the patient is over oxygenating. Normally, to increase ventilation we raise IPAP, thereby increasing the distance between the inspiratory and expiratory pressure. However, in this case, we can simply lower the EPAP which will decrease oxygenation but will also increase ventilation because the distance between the inspiratory and expiratory pressures is increased.

A 65-year-old patient with increased work of breathing is on bilevel therapy at an IPAP of 16 cmH2O and EPAP of 8 cmH2O. The following blood gases are reported on these settings: pH 7.39 PaCO2 40 torr PaO2 66 torr HCO3- 24 mEq/L BE 0 mEq/L FIO2 0.60 Which of the following adjustments is most appropriate? A. decrease EPAP to 6 cm H2O and IPAP to 14 cm H2O B. decrease EPAP to 6 cm H2O C. increase IPAP to 18 cm H2O and EPAP to 10 cm H2O D. increase EPAP to 10 cm H2O

The correct answer is : C Explanation : Arterial blood gases show this patient is ventilating adequately but is hypoxic. An increase in EPAP is indicated. However, increasing EPAP alone, without making other changes, would inadvertently decrease ventilation. Ventilation is determined by the distance between the expiratory pressure and the inspiratory pressure. If you raise EPAP by 2 you should also raise IPAP by the same amount.

A patient undergoes a change in PEEP from 15 to 20 cm H2O. Which of the following would indicate an adverse reaction to the change in PEEP? A. decrease in C(a-v)O2 B. elevated systolic BP, decreased diastolic BP C. elevated PCWP, decreased C.I. D. elevated CVP, decreased PAP, normal C.O.

The correct answer is : C Explanation : As PEEP is increased, the patient is likely to experience decreased venous return and degradation in hemodynamic pressures. Of the options listed, an elevated PCWP in conjunction with a decreased cardiac index (which is another way of looking at cardiac output), is the most likely outcome. Additionally, an elevated pulmonary capillary wedge pressure in the presence of decreased cardiac output is indicative of our left heart problem. Although the actual source of the problem may not be the left cardiac muscle, excessive PEEP takes on this appearance.

During the surgical placement of an 8.0-mm tracheostomy tube, the tube is immediately ejected by a strong patient cough before sutures are placed. The tube lands several inches away onto the sterile field. What should be done NEXT? A. Obtain a new, sterile trach tube of the same size. B. Discard the sterile field and start the procedure over. C. Replace the ejected tube into the surgical opening. D. Obtain a larger diameter trach tube.

The correct answer is : C Explanation : As long as the ejected, sterile tracheostomy tube remains in the sterile field when dropped, it may be retrieved and used. If a sterile piece of equipment lands anywhere other than the sterile field it must be replaced or re-sterilized.

Which of the following oxygen-delivery devices will most likely lead to an increase in infections? A. pulse-dose B. reservoir cannula C. transtracheal catheter D. nasal cannula

The correct answer is : C Explanation : Because a transtracheal oxygen catheter is an invasive, internal device, the likelihood of infection is greater than any other device listed among the options.

The most appropriate method of sterilizing used, disposable needles is which of the following? A. steam autoclave B. Cidex (alkaline gluteraldehyde) C. incineration D. pasteurization

The correct answer is : C Explanation : Because disposable needles will never be used again, the optimal sterilization method is incineration. Incineration should not be used for equipment that may be used again because it completely destroys the equipment. In the case of disposable needles, sterilization that also destroys the item is acceptable.

Levy-Jennings charts associated with the quality control history of a blood gas analyzer shows one point outside the 4 SD range. Subsequent data points are all within range. The respiratory therapist should A. do not use the blood gas analyzer B. call the medical director C. classify the data point as an outlier and continue analysis on blood samples D. replace the blood gas analyzer

The correct answer is : C Explanation : Blood gas machines must occasionally have quality control material run through them to prove they are accurate and precise. The data that is produced is plotted on a chart and monitored to ensure the results stay inside a predetermined range. The range is a statistical number that is equivalent to two standard deviations above the mean and two standard deviations below the mean for a total range of a 4-SD. When points are outside this range the machine is said to be "out-of-control" and may not be used to report patient samples.

Which of the following conditions would be most helped by therapy intended for secretion mobilization? A. hemothorax B. pulmonary edema C. bronchiectasis D. bronchiolitis

The correct answer is : C Explanation : Bronchiectasis is a condition which may exist along with several other disease states. While there are multiple types of bronchiectasis, the common outcome is a considerably high amount of sputum produced by the lung. Secretion mobilization therapies should be selected for a patient with bronchiectasis.

A bronchoscopy is used to diagnose A. atelectasis. B. emphysema C. malignant processes. D. idiopathic chronic bronchitis

The correct answer is : C Explanation : Bronchoscopy is used to diagnose foreign body obstructions, active bleeding causes, pathogenic involvement (to identify organisms) and cancerous or malignant processes or areas. It may help decrease atelectasis if immobilized mucous plugs are blocking a portion of the lung from expanding, but it does not serve in the diagnosis of atelectasis.

A respiratory therapist is asked whether patient instructions were effective for a patient being sent home with supplemental oxygen. How would this best be accomplished? A. Assure that the home health technician will repeat the instructions. B. Call the patient in one week to see if there are any complications. C. Ask the patient to repeat the instructions back to the therapist. D. Assure the patient signs the medical record for receipt of instructions.

The correct answer is : C Explanation : By having the patient repeat the instruction back, one can assure that the instructions are understood. Avoid closed-ended questions when determining whether a patient understands what they are to do.

After a 6-hour drive a couple arrives in the emergency room complaining of headaches and nausea. The physician on duty confirms carbon monoxide poisoning. Which of the following should be include the care of the patients? A. Acetaminophen B. Partial rebreathing mask C. Hyperbaric oxygen therapy D. inhaled CO2

The correct answer is : C Explanation : Carbon monoxide poisoning is primarily treated by giving the patient the maximum oxygen tension possible. To start, 100% oxygen is indicated. However, the oxygen tension may be vastly improved by placing the patient in a hyperbaric chamber. When the barometric pressure is increased, the oxygen tension of the ambient air inside the hyperbaric chamber may be increased significantly, drastically reducing the half-life of carbon monoxide on the hemoglobin.

Which of the following conditions would benefit most from cardioversion? A. frequent PVCs B. asystole C. ventricular tachycardia with a pulse D. ventricular fibrillation

The correct answer is : C Explanation : Cardioversion is appropriate for patients with ventricular tachycardia who have a pulse. If the patient has no pulse then defibrillation should be used. Cardioversion is with the machine in synchronized mode, or the synchronization is set to Active. Defibrillation is done with the machine's synchronization deactivated.

A patient admitted to the emergency room for chest pain is diaphoretic with cold extremities. These data are most consistent with which of the following A. pneumothroax B. pulmonary embolism C. myocardial infarction D. pulmonary tuberculosis

The correct answer is : C Explanation : Chest pain, diaphoresis, and cold extremities is associated with myocardial infarction. Other terminology that would describe this includes cold, wet, clammy skin. The next most appropriate action, although not asked in this question, would be to administer oxygen and to obtain an ECG.

A patient is receiving oxygen therapy via nasal cannula and a bubble humidifier. The respiratory therapist is called to address a continous whistling sound coming from the humidifier. Which of the following should the therapist suspect is the problem? A. the humidifier reservoir is empty B. the diffuser in the humidifier has fallen off C. the nasal cannula is kinked D. the down-tube in the humidifier is clogged

The correct answer is : C Explanation : Continuous whistling sound that comes from a bubble humidifier, which is often used with a nasal cannula, is usually due to a clogged or kinked nasal cannula. When the cannula is kinked, back pressure is applied to the bubble humidifier and ultimately a pop-off valve releases the pressure, causing a whistling sound.

How can proper application of cricoid pressure facilitate a difficult intubation? A. increases oxygenated blood flow to the brain B. eliminates the need for visualization of the vocal cords C. facilitates the view of the vocal cords D. protects of the trachea from oral contents

The correct answer is : C Explanation : Cricoid pressure may be helpful in protecting the airway from gastric contents and bringing the vocal cords into view during a difficult intubation.

A morbidly obese patient who is trached with a fenestrated tracheostomy tube is experiencing frequent dislodging with minor movement or cough. What should the respiratory therapist suggest? A. Inflate the cuff. B. Tie or suture the trach into place more securely. C. Increase the length of the tracheostomy tube. D. Replace with a Bivona-type tracheostomy tube with a foam cuff.

The correct answer is : C Explanation : Due to the increased neck circumference of a morbidly obese patient, special tracheostomy tubes that have long proximal extensions allow for the increased distance from the interior tracheal wall to the opening of the stoma at the skin.

A homebound oxygen-dependent patient receives 2 L/min oxygen by molecular sieve device. An electrical storm has resulted in the loss of electricity. She has 2 full E-cylinders. How many hours will her oxygen last? A. 6 B. 8 C. 10 D. 12

The correct answer is : C Explanation : Duration per E cylinder tank = 2200 x 0.28 = 616 L/ 616 L / 2 L/min = 308 minutes. 308 minutes / 60 = 5.1 hours per tank. 2 tanks will last about 10.2 hours, or 10 hours.

A physician requests the presence of a respiratory therapist for an intubation to deliver Exosurf to a newborn. Which of the following clinical finding is most likely? A. Normal radiolucency on chest radiograph B. esophageal atresia C. L/S ratio less than 2:1 D. hypopharyngeal swelling

The correct answer is : C Explanation : Exosurf, a brand of surfactant, is indicated for infant respiratory distress syndrome (IRDS). A newborn suffering from IRDS would not respond very well to oxygen, would likely have a problematic L/S ratio due to prematurity, and have a high respiratory rate. X-ray would reveal a honeycomb or ground glass pattern.

Fine crackles are auscultated at the bases of both lung fields. This is most consistent with the presence of A. tuberculosis B. pulmonary secretions C. atelectasis D. pleural friction rub

The correct answer is : C Explanation : Fine crackles heard about the bases of the pulmonary tree are most closely associated with atelectasis.

What is the typical frequency for Flovent MDI? A. PRN B. Q6 hours C. BID D. TID

The correct answer is : C Explanation : Flovent (fluticasone) is a corticosteroid used generally to prevent and relieve inflammation of airway walls. Inflammation is one of the components of asthma so corticosteroids are key in treatment. The frequency is twice per day or b.i.d.

Which of the following may be done to increase the effectiveness of gas distribution for a patient with ARDS who is receiving volume-controlled ventilation? A. Increase inspiratory flow rate B. Lengthen expiratory time C. Implement an inverse I:E ratio D. Decrease inspiratory time

The correct answer is : C Explanation : For a patient with ARDS, a key strategy is to prolong inspiratory time allowing the gases to permeate distal parts of the lung more effectively. To accomplish this, inspiratory flow rate is drastically decreased, sometimes to the point that inspiratory time is greater than expiratory time. When this happens, this is known as an inverse I:E ratio.

A respiratory therapist is instructing the family of a patient who will be completely ventilator-dependent at home. Which of the following are appropriate to include in that instruction? A. basic ventilator setting changes B. titration of FIO2 C. recognizing signs of infection D. setting ventilator alarms

The correct answer is : C Explanation : For the family of the patient who is ventilator-dependent at home, providing instruction on recognizing signs of infection, providing CPR, and performing bronchial hygiene are all appropriate. The patient will not be expected to make ventilator changes because this should be done by a qualified professional and only at the order of a physician.

During a cardiac stress test, the respiratory therapist notes no increase in heart rate or blood pressure after the incline of the treadmill has been increased. The patient appears to be pale in color. Which of the following can be concluded? A. the treadmill should be inclined further B. the treadmill be declined by 5% C. the patient's maximum exercise level has been exceeded D. the patient's minimum exercise level has been exceeded

The correct answer is : C Explanation : Heart rate and blood pressure should increase during cardiac stress testing when workload is increased. Workload may be increased by inclining the treadmill. In this case, as workload is increased, the patient's blood pressure and heart rate have not responded. And, the patient is showing clinical signs of distress. This is evidence that the patient's maximum workload has increased above the patient's ability. This is also defined as the patient's maximum exercise level.

An 8-year-old pediatric patient with a 6.0 mm endotracheal tube requiring endotracheal suctioning is experiencing bradycardia during the procedure. Suction pressure is set to -100 mm Hg. A respiratory therapist is suctioning for approximately 15 seconds using a 9 Fr catheter and is able to clear the airway effectively. To remedy the problem, the therapist should A. decrease the suction pressure. B. increase the pressure to -110 mm Hg and decrease duration to 5 seconds. C. decrease suction duration time. D. decrease catheter size.

The correct answer is : C Explanation : If suctioning is not adequate, correct the problem in this order: 1. check connections, change collection bottle if full 2. ensure suction pressure is in the right range 3. increase to the maximum size catheter within range 4. increase pressure within range 5. increase suction time

Which of the following clinical information can be obtained from a neonatal assessment? A. symptoms B. dyspnea C. signs D. subjective information

The correct answer is : C Explanation : Neonatal patients have no ability to report their own condition and therefore subjective information is not attainable. Of the options listed only "signs" and "color" may be determined objectively.

Which of the following would be most helpful at quantifying exposure to pulmonary irritants? A. single breath nitrogen elimination test B. tine test C. pack-years history of smoking D. occupation

The correct answer is : C Explanation : Of the options given, the assessment of pack-years history of smoking would provide significant information regarding the patient's exposure to pulmonary irritants.

A patient with ARDS has a ruptured oral ET tube cuff, necessitating tube replacement. Which of the following techniques will facilitate the transition to a new ET tube with minimal patient stress? A. intubation video assist device B. intubation with a bronchoscope C. use of an AEC D. use of a gum elastic device

The correct answer is : C Explanation : Provision of mechanical ventilatory support without interruption is critical on a patient with ARDS. Therefore, a requirement to replace the endotracheal tube should be met with great caution. Of the options given, use of an AEC (airway exchange catheter) will provide the least stressful, fastest way to exchange the tube.

Pulmonary function is done on a 68-year old male with a 20-pack-year history of smoking. The following pulmonary function data is recorded: Percent of Pred Actual Fev1/FVC % 62% FEF200-1200 78% FEF25-75 60% SVC 85% FVC 83% DLCO 22 CO/min/mm Hg Which of the following most likely represents the patient's condition? A. sarcoidosis B. asbestosis C. chronic bronchitis D. emphysema

The correct answer is : C Explanation : Pulmonary function test data indicates the patient has an obstructed defect. This is manifest by a reduced FEV1/FVC%. Anything less than 75% is considered abnormal. The SVC is normal and therefore indicates the patient has no restrictive defect. So, we must pick an answer that is an obstructive disease. When we look at the answers we see there are two options - emphysema and chronic bronchitis. To know which one it is we must look back at the pulmonary function testing data and observe the DLCO. Only emphysema shows a poor DLCO. In this case the DLCO is greater than 20 CO/min/mmHg and is therefore normal. This means that emphysema can not be the correct option leaving only chronic bronchitis as the only possible correct choice.

A patient is breathing spontaneously at a rate of 20 per minute and an average tidal volume of 500 mL. Which of the following represents the patient's rapid-shallow breathing index (RSBI): A. 25 B. 0.04 C. 40 D. 0.025

The correct answer is : C Explanation : Rapid shallow breathing index (RSBI) is calculated by dividing the respiratory rate by the tidal volume in Liters. RSBI = 20/0.5L = 40.

A patient who has a tracheostomy is complaining of dried secretions and difficulty expectorating sputum from the tracheostomy tube. Which of the following will be most helpful to the patient? A. Chest physiotherapy B. Administer Acetylcysteine (Mucomyst) C. Ensure inspired gas temperature is 35 deg C and humidified D. PEP therapy

The correct answer is : C Explanation : Secretions are best hydrated with heated humidity. This is especially true for patients who are orally or nasally intubated. In order to deliver proper humidification, inspired gases must be at temperatures that are close to normal body temperature (37 deg C).

An 75-kg (165-lb), 5-ft 11-in. female patient with pneumonia is receiving mechanical ventilator support by a Servo adult ventilator on the following settings with corresponding arterial blood gas values: Mode A/C Mandatory rate 16 VT 400 mL FIO2 0.5 pH 7.32 PaCO2 48 mmHg PaO2 72 mm Hg HCO3- 24 mEq/L BE 0 mEq/L Which of the following actions is most appropriate? A. increase FIO2 B. increase PEEP C. increase tidal volume D. increase mandatory rate

The correct answer is : C Explanation : The blood gas reveals the patient is not ventilating enough. An increase in mandatory rate seems to be appropriate, but a closer look at the ventilator settings shows a different problem. The patient's ideal body weight is about 75 kg. This means tidal volume must be at least 450 mL. The ventilator is set to only 400 mL. So, the first step would be to adjust the tidal volume to the appropriate range. This could, by itself, correct the hypoventilation.

A firefighter who was intubated after inhalation of heated gases is receiving VC A/C ventilation and is being monitored by a quadruple-lumen pulmonary artery catheter. The following recent information is recorded on the patient's medical record: CVP 15 cm H2O mPAP 21 mm Hg PCWP 8 mm Hg C.I. 2.5 L/min/m2 The therapist should be most concerned about A. right heart failure B. reduced cardiac output C. pulmonary hypertension D. fluid overload

The correct answer is : C Explanation : The hemodynamic data here shows an elevated CVP, an elevated mean pulmonary artery pressure value, and normal PCWP and cardiac index values. This is consistent with pulmonary hypertension which should be of primary concern in this case.

For which of the following reasons would a respiratory care department create and maintain patient care protocols? A. decrease healthcare costs B. increase respiratory care field scope C. better patient outcomes D. improved work efficiency

The correct answer is : C Explanation : The key advantage to patient-care protocols is standardized care and better patient outcomes.

A local college basketball team collapsed during a pre-game warm up after leaving a newly-constructed dressing room with an improperly-vented furnace. Multiple wavelength spectrophotometer readings suggest CO poisoning. To confirm this a respiratory therapist would evaluate A. Arterial blood gases B. hyperbaric chamber tolerance C. COHb D. MetHb

The correct answer is : C Explanation : The key word in the question here is to "confirm" carbon monoxide poisoning. This is done with a co-oximeter. The result is called a COHb (carboxyhemoglobin) level. One might be tempted to choose arterial blood gas, which may or may not include COHb. For this reason COHb is the best answer.

The respiratory therapist is performing an initial assessment on a new client in the pulmonary clinic. The patient has a 90 pack-year history of smoking, an FEV1 of 42% of predicted, and a DLCO of 15 mL CO/min/mm Hg. The following arterial blood gas data is available pH 7.37 PaCO2 56 torr PaO2 43 torr HCO3- 32 mEq/L BE + 6 mEq/L Which of the following is the most appropriate NEXT step? A. admit the patient a pulmonary rehabilitation program B. schedule the patient for sleep studies C. initiate supplemental oxygen D. perform a chemical cardiac stress test

The correct answer is : C Explanation : The patient's history and pulmonary function result (the FEV1 of 42%), in addition to an excessive smoking history, suggests chronic obstructive pulmonary disease. This suspicion is further corroborated by noting a normalized pH in the presence of hypercapnia (excessive CO2). Additionally, the patient is very hypoxic and thus requires supplemental oxygen.

During endotracheal suctioning, a patient becomes agitated, markedly hypertensive, and demonstrates PVCs on the cardiac monitor. The therapist should prepare to A. administer aerosolized lidocaine 4% B. administer a bolus of intravenous fluids C. provide maximum supplemental oxygen D. discontinue suctioning, report to the physician

The correct answer is : C Explanation : The presence of PVCs during any procedure, especially suctioning, would suggest that the patient has become hypoxic during the procedure and could benefit from maximum supplemental oxygen. This should be provided both before, during, and after the procedure to ensure arterial oxygenation and avoid hypoxemia.

During endotracheal suctioning, blue-tinted secretions are suctioned. The respiratory therapist should FIRST recommend A. instilling ethylene blue down the ET tube and manually ventilate B. performing a bronchoscopy C. discontinuance of tube feeding, assess ET tube cuff pressure D. monitoring the patient for fever

The correct answer is : C Explanation : The presence of secretions that are tinted in non-normal colors, such as blue or pink, is suggestive of potential aspiration of feeding solution into the lungs. Tube feeding is often purposefully died a specific color so that it may be discovered if accidentally aspirated into the lungs. When this is found, tube feeding should be discontinued immediately and the patient should be checked for possible aspiration.

A patient receiving volume-controlled ventilation has a balloon-tipped pulmonary artery catheter in place. The respiratory therapist notices the PA waveform is ascending and descending with inflection points at 25 and 2 mmHg. Based on this information, the therapist should recommend A. rotating the catheter B. deflating the catheter balloon C. advancing the catheter D. monitoring the patient closely

The correct answer is : C Explanation : The pulmonary artery catheter waveform that has a high inflection point of 25 and a low inflection point of 0-2 mmHg is an indication that the tip of the catheter is in the right ventricle of the heart. The proper placement of this catheter is in the pulmonary artery. Therefore, advancing the catheter is indicated. This is done by inflating the balloon and allowing the catheter to sail into a proper position in the pulmonary artery.

A chest radiograph on a neonate indicates a ground glass appearance. This finding is most closely associated with which of the following? A. tetralogy of fallot B. perihilar pneumonia C. IRDS D. meconium aspiration

The correct answer is : C Explanation : The radiological description of a neonate's chest x-ray that indicates a "ground glass appearance" is associated with IRDS, or infant respiratory distress syndrome. Another similar description is "reticulogranular pattern". The question is asking for the poper treatment. Because we know this is IRDS the proper treatment is surfactant therapy.

A respiratory therapist is called STAT to the general floor where a patient with a tracheal button in place is unable to breathe. The respiratory therapist should A. remove the cap and insert the inner cannula. B. remove the inner cannula and deflate the cuff. C. attempt to manually ventilate. D. remove the button and put a tracheostomy tube in through the stoma.

The correct answer is : C Explanation : The respiratory therapist should not confuse a tracheal button with a cap, which would be utilized with a fenestrated tracheostomy tube. A button is inserted into the stoma as a placeholder. Therefore, this patient should be treated as any other patient who cannot breathe by attempting to manually ventilate after opening the airway.

The following arterial blood gas values are reported for a patient who is weaning from mechanical ventilation. There is no notable change in the patient's condition. 0800 hrs 1000 hrs pH 7.42 7.38 PaCO2 37 torr 32 torr PaO2 80 torr 41 torr HCO3- 26 mEq/L 26 mEq/L FIO2 0.40 0.40 Based upon this data, the respiratory therapist should A. increase the FIO2 to 1.0. B. extubate the patient. C. repeat the arterial draw. D. increase the FIO2 to 0.50.

The correct answer is : C Explanation : The respiratory therapist should question all laboratory results to assure that they match the clinical scenario prior to reporting them. In this example, the patient's condition has not changed with the PaO2 of 41 torr. The sample might be a venous sample, and therefore should be redrawn.

A 70-kg (154-lb) COPD patient who was started on mechanical ventilation due to a severe pulmonary infection has the following weaning parameters & clinical data three days after initiation of mechanical ventilation: MIP -25 cmH20 Spont VT 360 mL RR 27 WBC 8,000 per cu mm Hb 16 gm/dL Based on the calculated RSBI, the respiratory therapist should recommend A. extubating and providing Bi-level ventilation via mask. B. changing to Pressure/control. C. discontinuing mechanical ventilation. D. returning to full mechanical ventilation.

The correct answer is : C Explanation : The underlying problem of pulmonary infection seems to be resolved as the WBC is 8,000 per cu mm. The rapid shallow breathing index, RR/VT (L) is 75, well below the threshold of 106. Therefore, the best option is to discontinue mechanical ventilation.

A respiratory therapist is doing pulmonary function studies on a patient. The upper inflection point of a flow-volume loop is considered which of the following pulmonary function values? A. inspiratory capacity B. expiratory reserve volume C. peak expiratory flow rate D. functional residual capacity

The correct answer is : C Explanation : The very top (inflection point) of a flow volume loop reveals peak expiratory flow rate.

The respiratory therapist measures the volume exhaled after a patient completes a maximal inhalation. Which of the following volumes has the therapist observed? A. IRV B. IC C. VC D. FRC

The correct answer is : C Explanation : The volume being observed when a patient exhales maximally after a maximal inhalation is called vital capacity or VC.

While transporting at patient from the emergency department (ED) to the intensive care unit (ICU), the respiratory therapist suspects the oral endotracheal tube has changed position. The quickest way to assess the ET tube position would be to A. obtain a chest radiograph B. visualize diaphragmatic excursion C. auscultate breath sounds D. analyze end-tidal CO2

The correct answer is : C Explanation : There are many ways to determine the location or position of the ET tube. The question is asking for the quickest way. Of the options offered, auscultation of breath sounds would provide the most immediate pertinent data. If an option such as examination of equal and bilateral chest rise were offered, that option would be even better because it is visual in nature and therefore quicker.

Which of the following should be examined to conclusively determine proper placement of an oral endotracheal tube? A. lateral neck radiograph B. breath sounds C. chest radiograph D. ET tube marking at the teeth

The correct answer is : C Explanation : There are various ways to determine the proper positioning of an endotracheal tube. Examining ET tube markings at the teeth and listening to breath sounds are both quick ways to roughly determine the position of the tube. However, neither of those methods are conclusive. The only conclusive way to determine the proper position of the ET tube is to do a chest x-ray. Keep in mind the question asks for a "conclusive" method. If the question asked which should be done first or immediately, the best answer would be to visualize the ET tube markings at the teeth and the next best answer would be to check for bilateral breath sounds.

The following data is observed on a patient who is 10 minutes post coronary artery bypass graft (CABG): 5 min after 15 min after 25 min after PaO2 (mm Hg) 92 90 93 PvO2 (mm Hg) 65 60 52 Which of the following can the respiratory therapist conclude? A. CvO2 is increasing B. tissues are becoming oxygen deprived C. cardiac index is decreasing D. A-aDO2 is decreasing

The correct answer is : C Explanation : This data shows a steady arterial oxygenation level with a decreasing venous oxygenation level. When the gap between arterial and venous oxygen levels start to increase, this is consistent with an increase in tissue oxygen consumption and an increase in the C(a-v)O2. This is related to a decrease in cardiac output. Of the options given, a decrease in cardiac index is another way of indicating a decrease in cardiac output.

A 60-year-old female receiving oxygen therapy by a nonrebreathing mask has the follow arterial blood gas results: pH 7.47 PaCO2 32 torr PaO2 50 torr HCO3- 24 mEq/L BE 1 mEq/L The respiratory therapist should document which of the following conditions in the patient's medical record? A. intrapulmonary shunt secondary to hyperventilation B. idiopathic ventilatory failure C. refractory hypoxemia with mild hypocapnia D. compensated respiratory alkalosis

The correct answer is : C Explanation : This patient has profound hypoxemia in spite of maximum FIO2. This is known as refractory hypoxemia. Additionally, hypocapnia (reduced CO2) is present.

The following pulmonary function test results are reported for a 57-year-old male patient with a smoking history of 50-pack years. % of predicted Actual Value FVC 56 SVC 68 Fev1.0/FVC% 67% Fev1.0 62 FEF 200-1200 68 FEF25-75 68 DLCO 18 Based on this information the patient has A. asthma B. restrictive defect C. emphysema D. severe diffusion defect

The correct answer is : C Explanation : This patient is demonstrating an obstructive defect as shown by an Fev1/FVC of 67%, which is less than the 75% required to be normal. Additionally, DLCO is also less than 80% of predicted indicating mild diffusion impairment. These two conditions, together, are associated with pulmonary emphysema.

An 83 kg (183 lb) male, postoperative abdominal surgery, is receiving mechanical ventilatory support on the following settings: FIO2 0.40 Mode SIMV Mandatory rate 14 Total rate 15 Tidal volume (VT) 550 mL Inspiratory flow 50 L/min Corresponding blood gases show: pH 7.25 PaCO2 55 torr PaO2 69 torr HCO3- 26 mEq/L BE +1 mEq/L Which of the following should the respiratory therapist increase? A. inspiratory flow B. FIO2 C. mandatory rate D. tidal volume

The correct answer is : C Explanation : This patient is hyperventilating. Because the patient is receiving mechanical ventilatory support, the first thing that should be checked is the appropriateness of the ventilator settings, especially the tidal volume. If the set tidal volume is found outside the range it should be corrected first prior to any other change. In this case, the set tidal volume is appropriate for the patient's height and weight and therefore an increase in mandatory rate is most appropriate to blow off excess CO2.

A patient with ARDS receiving volume-controlled ventilation has the following arterial blood results on the settings below: Mode Assist/control Mandatory rate 22 VT 350 mL FIO2 0.70 PEEP 22 cm H2O C.I. 2.2 L/min/m2 Heart rate 102 pH 7.35 PaCO2 45 torr PaO2 58 torr HCO3- 25 mEq/L BE +1 mEq/L Which of the following changes is most appropriate? A. decrease FIO2 to 0.6 B. increase FIO2 to 0.8 C. increase PEEP to 25 cm H2O D. decrease PEEP to 20 cm H2O

The correct answer is : C Explanation : This patient is hypoxic. To correct this problem either PEEP or FIO2 must be increased. Because the patient is already on 70% the next logical step is to increase PEEP. Hemodynamic data, namely a normal cardiac output, supports this change.

A patient is brought by paramedics to the emergency room after being found unconscious in a garage with a car running. A note was found on the scene. Which of the following devices will be most helpful in immediately assessing the patient? A. single wavelength pulse oximetry B. arterial blood gas analyzer C. obtain a reading from a multiple wavelength spectrophotometer D. blood gas analyzer with CO-OX ability

The correct answer is : C Explanation : This patient is likely to have an increased carbon monoxide level. To assess the degree of carbon monoxide poisoning, COHb would be an appropriate exam. Although this is an option, there is a better option. Unlike a normal pulse oximeter, a multiple wavelength spectrophotometer can assess carbon monoxide in a similar fashion as a pulse oximeter - by finger probe. This is a superior option because it is quicker, less invasive, less painful, and probably less expensive.

A volume-pressure ventilator graphic is observed on a patient who is receiving VC A/C ventilation on the following settings: FIO2 0.4 Mandatory rate 16/min Total rate 20/min VT 450 mL Flow 50 L/min PEEP 5 cm H2O The patient seems anxious and SpO2 is fluctuating between 89-94%. The therapist should suggest A. increasing VT B. decreasing sensitivity C. utilizing flow triggering D. increasing inspiratory flow rate

The correct answer is : C Explanation : This pressure-volume loop demonstrates that the patient is required to inhale significantly before the ventilator is triggered, thereby producing significant negative pressures to actuate a ventilator breath. This increase in work of breathing can cause the patient to deteriorate. One way of dealing with this is to increase the sensitivity so that the ventilator actuates more quickly with a negative deflection in pressure. However, this is not an option in the question. Flow triggering is another more sensitive way to detect patient effort and actuate a ventilator breath.

A patient receiving mechanical ventilatory support has the following capnographic waveform obtained from an infrared device. Which of the following is most consistent with this data? A. hypoventilation B. defective infrared sensor C. deadspace disease D. condensate on the infrared sensor

The correct answer is : C Explanation : This saw-tooth pattern seen on the capnograph is associated with dead space disease. Notice the low end-tidal CO2 and the rapid rate.

A patient with 90 pack-year smoking history is participating in a smoking cessation program. The respiratory therapist responsible for monitoring the patient should evaluate which of the following to determine the patient's compliance to the program? A. blood level of nicotine B. RHb C. multiple wave-length spectrophotometry D. pulse oximetry

The correct answer is : C Explanation : To determine if a patient is compliant with a smoking cessation program, COHb may be evaluated. If elevated or equal to the baseline, it can be assumed the patient is noncompliant with the program. In lieu of a blood gas which reveals the COHb, a special type of pulse oximeter can be used. A normal pulse oximeter can measure only oxygen. This is called a single wavelength spectrophotometer. A multiple wavelength spectrophotometer is capable of measuring carbon monoxide and other gases in the blood without performing an arterial puncture.

Which of the following devices would be used to obtain a PCWP measurement? A. central venous catheter B. indwelling arterial catheter C. pulmonary artery catheter D. double lumen catheter

The correct answer is : C Explanation : To measure pulmonary capillary wedge pressure, a pulmonary artery catheter must be in place.

In a normal lung, which of the following lung volumes would the respiratory therapist expect to be larger than the rest? A. inspiratory capacity B. residual volume C. vital capacity D. expiratory reserve volume

The correct answer is : C Explanation : Vital capacity is defined as the total amount of air the patient can either exhale or inhale. Of the volumes or capacities listed in this scenario, it is the largest for a patient with normal lungs.

Which of the following would be most helpful in evaluating partial paralysis of the vocal cords? A. infrared end-tidal CO2 detector B. forced vital capacity C. flow volume loop D. ventilatory response to CO2

The correct answer is : C Explanation : Vocal cord paralysis and other fixed upper airway obstructions are best diagnosed by doing a flow-volume loop pulmonary function test. If present, the flow-volume loop will be described as "round" in shape. This is because a fixed obstruction causes an equal amount of resistance on both inhalation and exhalation.

While performing nasal tracheal suctioning on a patient with mycoplasma pneumonia, the respiratory therapist notices PVCs on the monitor. The first action of the therapist should be to A. decrease the suction pressure B. recommend administration of Lidocaine for future suctioning of the patient C. discontinue suctioning D. monitor the patient carefully

The correct answer is : C Explanation : When PVCs develop during the suction procedure, this indicates acute hypoxemia that is most likely caused from the procedure. The first action should be to stop the procedure and allow the patient to recover. Future PVCs may be prevented by decreasing suction duration and by hyper oxygenating the patient.

A blood gas analyzer fails again to produce acceptable results on the pH value when running quality control material for the second time. The respiratory therapist should A. perform a one-point calibration B. monitor future results C. remove the machine from service D. perform proficiency testing

The correct answer is : C Explanation : When a blood gas machine fails to produce accurate results on one of its analytes, the respiratory therapist may respond by: removing the machine from service, notifying the supervisor, or rerunning the control. In this case, removing the machine from service is the only suitable answer offered.

A home care patient, who receives supplemental oxygen by concentrator at 2 L/min, indicates that not enough oxygen is coming through the nasal cannula even though the machine is on. The therapist should tell the patient to A. discontinue oxygen therapy until a technician arrives B. increase the flow rate C. connect the cannula to an E cylinder D. perform a flow calibration check

The correct answer is : C Explanation : When a home care patient complains of difficulty getting enough oxygen from their oxygen concentrator, the first step is to ensure they are receiving adequate oxygen by another source. This should be done before troubleshooting or asking the patient to help determine the source of the problem.

A patient receiving supplemental oxygen at home with a molecular sieve device reports appropriate flow coming from her cannula, but does not believe she is receiving enough oxygen. The therapist should advise the patient to A. change out the oxygen tubing. B. increase the flowrate by 1 L/min. C. change to E-cylinder oxygen and evaluate. D. call 911.

The correct answer is : C Explanation : When a patient reports difficulty getting air or oxygen through one oxygen delivery device, before troubleshooting, the most appropriate first action is to ensure adequate ventilation by changing to a different, reliable device. In this case switching to an E cylinder is most appropriate while the oxygen concentrator is assessed.

When analyzing the FIO2 for an infant in an oxygen hood receiving oxygen therapy with a blender set at 50%, the respiratory therapist notes an oxygen concentration of 35% near the patient's mouth. The jet nebulizer entrainment setting is set to 50%. To correct the problem, the therapist should A. obtain a smaller oxyhood B. adjust blender setting to 60% C. change the nebulizer entrainment port to 100% D. increase total flow to the oxyhood

The correct answer is : C Explanation : When administering oxygen by oxygen hood with a blender and a nebulizer, the oxygen control on the nebulizer should be set to 100%. This will prevent additional entrainment of room air which will cause a decrease in FIO2.

Endotracheal suctioning should be immediately discontinued upon which of the following findings? A. patient complaint B. thick secretions C. bradycardia D. coughing

The correct answer is : C Explanation : When the patient becomes hypoxic, their first response is bradycardia follow quickly by tachycardia Therefore, bradycardia is an important and direct indicator to stop suctioning when observed. Coughing, complaints and discomfort of the patient are common and normal.

While transporting a ventilator-dependent patient to the cardiac catheterization laboratory, the respiratory therapist notices the cotton tape securing the oral endotracheal tube has come loose. The patient's respiratory rate is suddenly increased and cyanosis is noted. Markings on the ET tube are 24 at the teeth. The therapist will next do which of the following? A. withdraw the ET tube by 5 cm B. obtain a chest radiograph C. auscultate breath sounds D. advance the ET tube by 2 cm

The correct answer is : C Explanation : When trying to quickly determine the location of the endotracheal tube, the action that must be taken first is that which is the quickest. Of the options offered, auscultation of breath sounds is the quickest.

Just prior to providing a patient with manual ventilation, following oral intubation, the respiratory therapist assesses the function of a self-inflating resuscitator bag. The therapist notices the bag is easy to squeeze when attached to the patient. It is also evident that there is no chest rise with each squeeze. Which of the following could be the cause of the problem? A. bag is too small B. insufficient flow C. detached reservoir D. faulty inlet valve

The correct answer is : D Explanation : On a self-inflating resuscitator bag, there is most likely a faulty inlet valve if the bag is easy to squeeze once it is connected to the patient. In normal circumstances, when the bag is connected to the patient, pulmonary resistance should be felt through the resuscitator device. If no resistance is felt, it is likely that the gas is being ejected through a one-way valve that is responsible for allowing gases to enter the bag. This would suggest a faulty inlet valve that is allowing gases to move both directions.

A patient, who is orally intubated with an 8.0 ET tube, has been receiving mechanical ventilation. Recent radiological evidence indicates a bronchopleural fistula is present. Which of the following could the respiratory therapist recommend that may be helpful to the patient? A. chest tubes B. thoracentesis C. bronchography D. independent lung ventilation

The correct answer is : D Explanation : A bronchopleural fistula is a perforation in the lung tissue allowing air to enter the pleural space. When this is present, too much positive pressure can expand this perforation, causing more damage and increasing loss of volume. In a case like this, independent lung ventilation will allow normal ventilation to the normal lung and reduced ventilation and pressure to the affected lung.

It is suspected that a patient is bleeding from a site within the pulmonary tree. Which of the following would be most appropriate to locate the potential bleeding site? A. percutaneous biopsy B. CT scan of the chest C. V/Q scan D. bronchoscopy

The correct answer is : D Explanation : A bronchoscopy is the best procedure to use when a specific bleeding site needs to be located within the pulmonary tree. Although the bronchoscope is unable to penetrate the distal portions of the tree, most bleeding will occur higher up and can be visualized through the bronchoscope. Additionally, once the bleeding site is observed, measures can be taken with the bronchoscope to stop or minimize bleeding. Some of these measures include instillation of epinephrine, compression of the site with the scope, or placement of a Fogarty catheter, which is a long-term tamponade of the bleeding site.

A patient has a fenestrated tracheostomy tube configured to allow speech. In preparation for a positive pressure breathing treatment, the respiratory therapist should do which of the following? A. inflate the cuff, insert the inner cannula, remove the cap B. remove the cap, deflate the cuff and replace the inner cannula C. remove the inner cannula, deflate the cuff, place the cap D. remove the cap, inflate the cuff, insert the inner cannula

The correct answer is : D Explanation : A fenestrated tracheostomy tube has two potential configurations. The first configuration allows the patient to speak. The second configuration allows the patient to receive positive pressure ventilation. When transitioning from a speaking configuration to a positive pressure ventilation configuration, several things must be done in the right order to ensure the patient can breathe through the change. In this case, the first step is to remove the cap, then inflate the cuff, and finally, insert the inner cannula. Doing this in any different order could present a time where the patient is unable to breathe freely.

A galvanic fuel cell oxygen analyzer may read erroneously high under which of the following conditions? A. when liquid gets on the membrane B. when the analyzer batteries are depleted C. during a sudden increase in the partial pressure of oxygen D. when a volume-controlled ventilator at high inspiratory pressures

The correct answer is : D Explanation : A galvanic fuel-cell oxygen analyzer may read erroneously when ambient pressures change significantly, such as when a patient is receiving high inspiratory pressure or when a patient changes altitude quickly.

What effect does decreasing the I:E ratio through pursed lips have for a spontaneously breathing asthmatic patient? A. decreases oxygen consumption B. increases FRC C. increases IC D. decreases air trapping

The correct answer is : D Explanation : A patient with active asthma suffers from the inability to exhale gas from the lung. This results in air-trapping. A solution is pursed-lips breathing, along with an extension of expiratory time. One may also decrease I-time or increase E-time to resolve air-trapping created by asthma.

A patient has been diagnosed with central sleep apnea. Which of the following would be helpful and most appropriate in the further treatment of the patient? A. full nocturnal ventilatory support B. weight loss C. surgical remove of soft tissue D. bi-level therapy with a rate

The correct answer is : D Explanation : A patient with central sleep apnea either needs mechanical ventilatory support or medication that stimulates ventilation. Dopram is such a medication. Mechanical ventilatory support can be achieved with bilevel therapy in conjunction with a rate.

A patient with myasthenia gravis who is on mechanical ventilatory support is being monitored every 2 hours. Recent findings show that the maximum inspiratory pressure (MIP) has changed from -30 cm H2O to -18 cm H2O in the last 2 hours. The respiratory therapist should recommend A. FEV1.0 sec B. FEV1.0/FVC ratio C. SpO2 D. measure tidal volume and vital capacity

The correct answer is : D Explanation : A patient with myasthenia gravis will suffer a slow degradation of ventilatory muscles as paralysis spreads down the body. The point at which the patient is in need of mechanical ventilatory support is identified primarily by observation of three measurements. These include MIP, VC, and VT. In this question, MIP has fallen significantly. Vital capacity should be monitored closely as well. As soon as VC falls below 1.0 L, the patient is in need of mechanical ventilatory support.

A respiratory therapist is preparing a smoking cessation program to be used in conjunction with pulmonary rehabilitation. The therapist should include which of the following methods of support in the plan? A. Acetylzolamide (Diamox) B. Daily telephone reminders not to smoke C. electronic vapor cigarrettes D. Counseling for associated emotional challenges

The correct answer is : D Explanation : A person trying to stop smoking must conquer both physiological and psychological barriers. Appropriate methods of support include emotional counseling and nicotine replacement therapy.

A patient is found unconscious by a respiratory therapist. Pulse is palpated and found to be 38/min. An agonal breathing pattern is also noted. The therapist should immediately A. place the patient on a non-rebreathing mask at 15 L/min. B. administer epinephrine. C. perform oral endotracheal intubation. D. begin CPR

The correct answer is : D Explanation : A pulse of less than 40 per minute is an indication to begin cardiopulmonary resuscitation or BLS.

Which of the following interventions should be started FIRST for a patient experiencing status asthmaticus? A. pursed-lip breathing training B. arterial blood gas C. rescue inhalers D. oxygen

The correct answer is : D Explanation : According to the National Institute of Health for guidelines on the treatment of asthma the first treatment should be to administer oxygen.

A patient with a history of chronic bronchitis is post-operative for an exploratory laparotomy. Review of the medical record suggests the physician has concern about difficulty clearing secretions after the procedure. To assist the patient with mobilization and removal of secretions, the respiratory therapist should recommend A. MDI therapy with an anti-inflammatory corticosteroid B. administer cromolyn sodium C. administer tetracycline D. instruction on deep breathing and coughing techniques

The correct answer is : D Explanation : After surgery a primary concern is secretion removal. The easiest way to increase secretion removal is to teach people how to breathe deeply and cough. An MDI may be tempting but it will not remove secretions. The other options are not related.

Prior to obtaining an MIP value with a pressure manometer, the respiratory therapist notes the needle is pointing at a positive pressure of 7 cm H2O prior to the maneuver. During the MIP maneuver, the needle reaches -20 cmH2O. The therapist should A. record and MIP of -20 cmH2O B. repeat the maneuver five more times C. record an MIP of -13 cmH2O D. record an MIP of -27 cmH2O

The correct answer is : D Explanation : Although -20 cmH2O is observed on the pressure manometer, the real pressure being produced is -27 cmH2O. This is because the manometer is not properly calibrated to zero. Therefore, an adjustment must be accounted for when taking a measurement. You must add the number that the needle is pointing to prior to the maneuver to the number achieved by the patient during the maneuver to get an accurate measurement.

After observing symmetrical chest rise of a patient who was just orally intubated, the respiratory therapist should do which of the following to confirm proper placement of the endotracheal tube? A. obtain an arterial blood gas B. attach a color-changing CO2 detector to the end of the ET tube C. auscultate breath sounds D. order a chest radiograph

The correct answer is : D Explanation : Although there are multiple ways to determine proper positioning of the endotracheal tube, to confirm proper placement, a chest x-ray is imperative. Other methods are quick and helpful because they provide immediate assessment but they are not CONFIRMING in nature. These methods include auscultation of breath sounds and observation of chest rise and symmetry, among others.

Which of the following would NOT be included in the education process for a patient receiving instruction related to treatment of their asthma? A. evaluation B. assessment C. planning D. enforcement

The correct answer is : D Explanation : Appropriate elements of an education plan include assessment, planning, and evaluation. Enforcement is not included.

A patient with Adult Respiratory Distress syndrome (ARDS) is receiving volume-controlled ventilation. As PEEP is increased and pulmonary shunting is decreased from 70% to 50%, the respiratory therapist should expect to see which of the following trend in PETCO2? A. sudden fluctuation followed by a decrease B. no change C. decrease D. increase

The correct answer is : D Explanation : As PEEP is increased and as pulmonary shunting reduces, it is likely that an increase in the effectiveness of ventilation will be observed. End-tidal CO2 will increase as ventilation increases.

A patient with Guillain-Barre Syndrome is being monitored for adequate spontaneous ventilation. Which of the following would indicate most clearly that mechanical ventilator support is required? A. VT of 500 mL B. MIP of -28 cmH2O C. PaO2 of 60 mmHg D. VC of 0.9 L

The correct answer is : D Explanation : As a patient with Guillain-Barré syndrome slowly degrades, the point at which ventilatory support is needed is when the tidal volume falls below 1 L.

A patient receiving mechanical ventilation is experiencing an increase in autoPEEP. Which of the following should the respiratory therapist increase to lower autoPEEP? A. pressure support B. mandatory rate C. PEEP D. flow rate

The correct answer is : D Explanation : AutoPEEP is caused when a mechanical ventilator delivers the next breath before the patient has exhaled completely. This air-trapping causes over distention of the alveoli and results in intrinsic PEEP. To minimize this, greater time for expiration must be allowed. This can be done by increasing flow rate, which will decrease inspiratory time and prolonged expiratory time.

The following graphic from an orally intubated patient receiving mechanical ventilation is most likely caused by A. PEP therapy B. fixed upper airway obstruction C. vocal cord paralysis D. condensate in the tubing

The correct answer is : D Explanation : Condensate in the tubing will show up as a fluttering expiratory flow.

Evaluation of maximum expiratory pressure (MEP) is most useful in evaluating which of the following? A. fixed upper airway obstruction B. inspiratory muscle strength C. presence of restrictive defect D. forcefulness of cough

The correct answer is : D Explanation : Maxim expiratory pressure is an important measurement when determining how forcefully a patient can cough, though it can also be used to determine if the patient is able to sustain ventilation. However, MIP is more relevant when considering a patient's ability to sustain ventilation on his own.

Under which of the following circumstances should a respiratory therapist consider stopping a PEP therapy treatment of a child? A. Cold B. Bronchitis C. Lingular lobe pneumonia D. Middle ear infection

The correct answer is : D Explanation : PEP therapy is contraindicated by sinusitis, epistaxis, and a middle ear infection.

An adult is receiving NT suctioning and experiences a decrease in SpO2 from 98% to 80% during the procedure. The respiratory should A. check for breathing. B. manually ventilate with bag/mask. C. provide 100% oxygen. D. stop the suction procedure.

The correct answer is : D Explanation : The respiratory therapist should immediately stop suctioning if any signs of distress are present cardiac, desaturation, or otherwise).

A patient receiving oxygen via a pulse-dose oxygen system complains of difficulty breathing and indicates she believes the cannula is not working properly. The respiratory therapist should A. instruct the patient call the manufacturer B. troubleshoot the pulse-dose regulator C. replace the oxygen tank D. instruct the patient to switch to her oxygen concentrator with a regular nasal cannula

The correct answer is : D Explanation : When the function of an oxygen delivery device is in question, the first best option is to switch to a different mode of oxygen delivery to ensure that the patient is receiving oxygen. After the patient's oxygenation is secure, troubleshooting of the problem may begin.

A respiratory therapist auscultates expiratory wheezing on a patient with COPD who has a 75 pack-year history of smoking. The therapist should recommend

Unit dose of Albuterol by small volume nebulizer.

A patient who is receiving mechanical ventilation has the following ventilatory data: 2 pm 2 am 2 pm 2 am VT (mL) 650 650 650 650 PEEP (cm H2O) 7 7 7 7 Pressure (plat) 28 30 32 36 SpO2 (%) 97 95 94 94 If no changes are made on the ventilator, which of the following alarms could be expected if this trend continues?

a high-pressure

Which of the following finding is most closely associated with increased airway resistance?

accessory muscle use

While performing routine chest physiotherapy on a patient, the respiratory therapist observes a sudden change in the patient's condition. A quick assessment reveals: HR 62/min RR 20/min, shallow SpO2 87% and falling on 4 L/min NC BP 80/55 mm Hg The therapist should immediately

activate the rapid response team

A 7-year old female patient is receiving PEP therapy by mouthpiece. In spite of repeated coaching, the child does not keep the mouthpiece in her mouth. The therapist should respond by doing which of the following?

add a mask to the PEP device

A 38-year-old male presents in the emergency department (ED) complaining of frequent vomiting. The following laboratory data is available: Arterial blood gases pH 7.55 PaCO2 42 torrPaO2 85 torrHCO3- 31 mEq/LBE +7 mEq/LFIO2 0.21K+ 3.0 mEq/LCl- 95 mEq/LNa+ 135 mEq/L Which of the following should the respiratory therapist recommend?

administer KCL

Which of the following airway clearance therapies should be administered first in the order of therapy?

aerosolized Acetylcysteine (Mucomyst)

What is the respiratory therapist observing if monitoring the total amount of gas exhaled in 60 seconds after excluding dead space?

alveolar minute ventilation

A patient complains of shortness of breath only upon exertion. There is no significant smoking history or family history of lung disease. What should the respiratory therapist recommend?

cardiopulmonary stress testing

A patient in the neurological intensive care unit (NICU) is dependent on mechanical ventilation following evidence of significant brain-stem herniation. A cerebral brain perfusion study shows large areas without perfusion. An EEG shows virtually no brain activity. Ventilator settings and ABGs are as follows: FIO2 1.0 VT 450 mL Rate(set) 18/min Rate(total) 18/minMode VC A/C PEEP 0 cm H2O pH 7.42 PaCO2 39 torr PaO2 92 torr HCO3- 24 mEq/L BE 0 mEq/L Which of the following should be recommend by the respiratory therapist?

cessation of mechanical ventilation.

After performing minimum occluding volume technique with a 65-kg (143-lb) patient who is orally intubated with a 7.0-mm ET tube, the respiratory therapist should NEXT

check ET tube cuff pressure

Which of the following emergency airway clearance techniques should be used on a female who is 7 months pregnant and who has complete airway obstruction by aspirated food?

chest thrusts

A local flu epidemic has flooded the area's emergency services with more patient's than can be handled by local facilities. All emergency services in the area are at or above capacity. Which of the follow would be an appropriate strategy for a respiratory therapist to suggest in an ER when additional patients are expected to arrive while the ER is at capacity?

cohorting of patients

A 70-kg (154-lb) female patient is receiving mechanical ventilation on the following settings: Mode assist/control Mandatory rate 14Total rate 14VT 500 mLFIO2 0.55PEEP 12 cmH2O Arterial blood gases reveal pH 7.49 PaCO2 32 mmHgPaO2 109 mmHgHCO3- 24 mEq/LBE 0 mEq/L The respiratory therapist should

decrease PEEP

A decreasing A-aDO2 is mostly closely related to which of the following?

decreasing pulmonary shunting

A patient in the emergency room (ER) has multiple fractured ribs bilaterally after a motor vehicle accident. ABGs show pH 7.51 PaCO2 29 torr PaO2 102 torr HCO3- 24 mEq/L BE +3 mEq/L FIO2 0.21 The respiratory therapist should

determine the level of pain.

A COPD patient that requires intubation and VC SIMV ventilation has a Mallampati classification of 1. Which of the following methods and/or equipment should the respiratory therapist expect to utilize for oral intubation?

direct laryngoscopy

While performing endotracheal suctioning on a patient in the intensive care unit (ICU), the respiratory therapist notices the following ECG wave form on the monitor: The therapist should immediately:

discontinue suctioning, provide 100% oxygen.

A patient with congestive heart failure is transferred to the intensive care unit (ICU) following an exacerbation resulting in acute pulmonary edema. A pulmonary artery catheter is placed. The following data is reported: mPAP 22 mm Hg CVP 9 mm Hg PCWP 12 mm Hg C.I. 4.2 L/min/m2 BP 122/88 mm Hg HR 110/min The respiratory therapist should recommend

diuresis

A patient's chest radiograph shows consolidation in the right middle and lower lobes. What additional finding is likely?

dull to percussion over the affected areas

A neonatal patient exhibits a heart murmur upon auscultation one hour after birth. The respiratory therapist is having difficulty maintaining adequate oxygenation saturation in spite of increasing FIO2. Which of the following should the respiratory therapist recommend?

echocardiography

A patient with COPD develops rhonchi immediately after postural drainage and percussion. The respiratory therapist should

encourage the patient to cough

While preparing to perform oral intubation by direct laryngoscopy to a patient who is experiencing a complete cardiopulmonary collapse, the respiratory therapist notices the laryngoscope light does not illuminate when the blade is snapped into place. The therapist should FIRST

ensure the bulb is tightened

A home care patient calls into the clinic and complains that his oxygen concentrator is not working even though the machine is plugged in and the switch is in the 'on'. Position. After ensuring the patient is receiving oxygen from a reliable alternate source, the therapist should advise the patient to

ensure the circuit breaker is in the on position

After several moments of cardiac compressions with a patient suffering from a complete cardiopulmonary collapse, the patient's cardiac rhythm is consistent with asystole when compressions are momentarily halted. Which of the following medications would be most helpful to the patient:

epinephrine

A respiratory therapist is alerted by a low-volume alarm on a VC ventilator. The inspiratory pressure manometer is rising to a peak of 7 cm H2O during each inspiration. The therapist should

examine the function of the exhalation valve

Which of the following descriptions most typifies the sound a therapist might here on a patient with a pleural friction rub?

grating sound

Which of the following would be most helpful in reducing the risk for nosocomial infection in a hospital?

handwashing with soap and water

A 52-year-old female with metastasized cervical cancer is admitted to the hospital for observation. Within 48 hours, cancerous processes caused a deterioration in the patient's ventilation, which necessitated the need for mechanical ventilatory support. Current data: Vent settings Mode VC, A/C FIO2 0.60 PEEP 5 cm H2O Rate 16/min VT 450 mL ABGs pH 7.41 PaCO2 39 torr PaO2 75 torr P(barometric) 726 mm Hg The respiratory therapist should conclude that the patient

has venous admixture.

Which of the following would be most helpful for a patient with fixed upper airway obstruction?

heliox therapy

Which of the following is a strategy that may help reduce or prevent autoPEEP in a patient with ARDS?

high flow rates

A patient in the emergency department (ED) is breathing spontaneously after inhalation of kerosene. The following physiologic data is available: ABG pH 7.31 PaCO2 49 torr PaO2 74 torr HCO3- 23 mEq/L BE -2 mEq/L HR 110/min SpO2 92% The patient is lethargic but responds appropriately to commands. The respiratory therapist should recommend

implementing non-invasive ventilation.

A pressure-volume loop ventilator graphic shows no rise in pressure for the first 200 mL of delivered volume. The therapist should

increase PEEP

What would occur on a time-cycled ventilator with a fixed rate if the inspiratory flow rate were increased?

increase in tidal volume

A patient is receiving CPAP on a Hamilton adult ventilator at the following settings: CPAP 5 cm H2O PS 5 cm H2OFIO2 0.40 Additional data shows: Respiratory rate 30/min VT(spont) 220 mLSpO2 90% The respiratory therapist should recommend

increase pressure support to 10 cm H2O

An adult male is being suctioned though an 8.0-mm ET tube with a 12 Fr. suction catheter at a pressure of 90 mmHg. Secretion removal has been ineffective. Which of the following modifications should be recommended?

increase suction pressure to 110 mmHg

A time-cycled, pressure-limited ventilator is failing to reach the set pressure with each breath while ventilating a newborn on the following settings: PIP 24 cm H2O PEEP 4 cm H2O (f) 35/min FIO2 0.30 The respiratory therapist should

increase the flow.

After increasing the mandatory rate on a volume-controlled ventilator, an alarm begins to sound. The respiratory therapist notices that the delivered tidal volume is only 412 mL. The respiratory therapist should most likey need to

increase the peak flow rate.

Which of the following best describes the function of digitalis?

increases strength of cardiac contractility

In providing a patient with education and direction for the performance of positive expiratory pressure (PEP) therapy, the respiratory therapist will include which of the following directives?

inhale deeply and exhale actively

A respiratory therapist has intubated an adult patient who is receiving mechanical ventilation. The therapist intends to adjust ET tube cuff pressure using he minimal leak technique. The first step is to

inject enough air to create a seal

While receiving ventilatory support by a non-invasive ventilator, a patient vomits into the full inflatable mask. Although not known, it is believed that the patient may have aspirated. The following arterial blood gases are determined 20 minutes after the event while receiving ventilatory support on IPAP 25 cm H2O, EPAP 5 cm H2O: ABGs pH 7.38 PaCO2 42 torr PaO2 81 torr HCO3- 24 mEq/L BE +1 mEq/L The respiratory therapist should

institute invasive mechanical ventilation.

A 5-year-old child is brought to the emergency room (ER) with a fever and difficulty breathing. The patient is drooling but is making no vocal sounds. Which of the following procedures should the respiratory therapist recommend?

lateral neck radiograph

A respiratory therapist is adjusting the ET tube cuff pressure on a patient receiving positive pressure ventilatory support. The therapist is using the minimum leak technique to adjust the volume in the cuff. After insertion of air into the cuff, the therapist auscultates the neck and notices a slight passage of air around the cuff at the top of each mechanical inspiration. The therapist should

maintain current cuff pressure.

Which of the following is a central goal of pulmonary rehabilitation on a patient diagnosed with advanced stage COPD?

maintenance of ADLs

A pulse oximeter reading is significantly less than oxygen saturation by blood gas. Which of the following could be the cause?

marked hypotension

A patient is receiving volume-controlled ventilation following bariatric surgery for obesity. Which of the following medications should the respiratory therapist recommend to ensure the patient's comfort and assist in ventilator management?

morphine sulfate

Which of the following exams can be used to quantify pulmonary residual volume (RV) on a patient?

nitrogen washout

A patient who has bedside screening spirometry has flow-volume loops that are short and wide in appearance. This is associated with a(n)

obstructive pulmonary defect.

A patient who is receiving VC A/C ventilation has been experiencing climbing peak inspiratory and mean airway pressures due to decreasing pulmonary compliance. Additionally, greater FIO2 is required to maintain adequate PaO2 levels. To further investigate the effectiveness of oxygen administration, the therapist should evaluate which of the following?

oxygen index

An ECG is performed on a patient in the emergency room (ER) who is complaining of chest pain. The respiratory therapist notices flipped T waves on the ECG tracing. Which of the following best remedies this finding?

oxygen therapy

Use of overnight oximetry at home is intended to determine the need for which of the following?

polysomnography

A respiratory therapist is caring for a 68 year old female who has been receiving mechanical ventilation via tracheostomy tube for 6 weeks. After adjusting the cuff pressure using the minimum seal technique, the therapist measures the cuff pressure and notes it is 35 cm H2O. Which of the following could explain this finding?

presence of a transesophageal fistula

A chest radiograph indicates the presence of fluffy infiltrates. This finding is most closely associated with which of the following?

pulmonary edema

For a patient receiving mechanical ventilation, with which of the following is plateau pressure most closely associated?

pulmonary static compliance

A respiratory therapist is providing instruction to a home-care patient regarding different methods of increasing the effectiveness of airway clearance. The therapist may suggest all of the following techniques EXCEPT

pursed-lip breathing

A patient who has mild stridor following endotracheal extubation would benefit most from which of the following?

racemic epinephrine

Immediately after extubation, a patient demonstrates moderate inspiratory and expiratory stridor. The respiratory therapist should prepare to administer

racemic epinephrine

A patient has the following 12-lead ECG tracing after being found emaciated, hypothermic, and unconscious in an abandoned building. The ECG shows flattened 'T' waves. The respiratory therapist should suggest checking which of the following, based on the ECG tracing?

serum electrolyte levels

A 14-year-old, 66-kg (145-lb) female patient with a history of asthma is in the emergency room (ER) in response to an exacerbation of asthmatic symptoms over the last 2 hours. While en route the patient received 2 small volume nebulizer treatments with 0.5 mL albuterol. Paramedics report no appreciable responses to the bronchodilator. The respiratory therapist should recommend

starting continuous albuterol therapy at 10 mg/hr.

A respiratory therapist is called to the emergency waiting room to offer advice on the cleanup method of liquid oxygen that was ejected from a faulty portable liquid oxygen device. The therapist will instruct which of the following?

stay clear of the area for a few minutes, nothing else is required

An adult patient with asthma is receiving Albuterol by small volume nebulizer Q.I.D. at a dosage of 0.5 mL. The patient complains of dizziness, tingling in his fingers, and anxiety with each treatment. The therapist should

switch to Xopenex 0.63 mg

A patient is receiving oxygen by nasal cannula at 4 L/min. After 24 hours of use with a properly functioning bubble humidifier, the patient complains of nosebleeds. The therapist would do which of the following to modify therapy?

switch to an air-entrainment mask at FIO2 35%

Which of the following is an example of an advanced directive?

a living will

Which of the following instructions should be given to a patient in preparation for incentive spirometry with a volume-type device?

"inhale as deeply as possible"

Which of the following sputum characteristics would be helpful in assessing the nature and status of pulmonary congestion on a patient? 1. color 2. amount 3. odor 4. viscosity

1, 2, 3 and 4

Utilization of a speaking valve on a patient with a long-standing tracheostomy tube serves which of the following purposes? 1. increases diaphragmatic muscle strength 2. increases cough explosiveness 3. helps maintain patient dignity 4. facilitates better communication of patient needs

3 and 4 only

For a 77-kg (170-lb) patient with ARDS, which of the following target tidal volume settings would be appropriate when initiating mechanical ventilation?

350 mL

Which of the following is required for an air/oxygen proportionator (blender) to function properly?

50 PSI sources gases

Which of the following PaO2 ranges is acceptable for a patient with ARDS according to the ARDSnet protocol?

55 to 80 mm Hg

A patient has the following data: Hb 9 g/dL PvO2 35 torrSvO2 65%CaO2 15 vol% What is the patient's approximate C(a-v)O2?

8.1 vol%

In preparation for pulmonary function testing, the respiratory therapist interviews the patient to investigate use of tobacco products. The patient reports smoking 5 packs a day for 10 years and 1.5 packs a day for 20 years. What is the pack-year history of smoking for this patient?

80

A 16-year-old child is admitted to the emergency room with a medical history of asthma. The physician has ordered a rescue bronchodilator to address bronchoconstriction. Which of the following medications should the respiratory therapist recommend to address issues with airway inflammation? A. Prednisone B. Pirbuterol C. Xopenex D. Atrovent

A

A 40-year-old female presents in the emergency room after being rescued from a building fire where she was found unconscious. Which of the following assessments would be most helpful in the evaluation of the patient? A. hemoximetry B. single-wavelength pulse oximetery C. measurement of PaO2 D. beside pulmonary function testing

A

A 5-year old patient with acute epiglottitis has just received a tracheotomy and is returned to the emergency department for monitoring. The patient is now breathing through a tracheostomy tube. Which of the following is most important at this time? A. provide heated humidity B. provide cool aerosol therapy C. mechanical ventilatory support D. keep the tracheostomy tube cuff inflated

A

A patient breathing spontaneously through a 7.5 mm endotracheal tube self-extubates. The ET tube is found lying on the floor with the cuff still inflated. Which is the most important physical sign for which the respiratory therapist should monitor over the next few hours? A. stridor B. unilateral wheezing C. inability to vocalize D. vocal cord paralysis

A

A patient complains of tremors following 2 puffs of Albuterol by metered dose inhaler. The tremors happen within 30 minutes after every treatment and subside within 20 minutes. The respiratory therapist should recommend A. instructing the patient this is a normal side affect B. use of a chamber C. informing the patient this is an adverse reaction D. stop using the MDI

A

A patient is admitted to the emergency room (ER) after being found unconscious in a closed garage with an automobile running. A note was also found at the scene. Which of the following therapies would be most helpful to the patient? A. hyperbaric oxygen therapy B. mechanical ventilation with high levels of PEEP C. oxygen therapy with a non-rebreathing mask D. oxygen therapy with a partial-rebreathing mask

A

A patient is receiving bi-level therapy to decrease work of breathing. Current settings and blood gas results are: IPAP 18 cm H2O EPAP 10 cm H2O FIO2 0.21 pH 7.32 PaCO2 48 torr PaO2 110 torr HCO3- 24 mEq/L BE 0 mEq/L The respiratory therapist should make which of the following changes? A. decrease EPAP to 8 cmH2O B. increase both IPAP and EPAP by 2 cmH2O C. increase IPAP to 22 cmH2O D. increase EPAP to 12 cmH2O

A

A patient with a history of Congestive Heart failure (CHF) is experiencing pulmonary edema. Oxygen saturation is 87% and the patient has marked congestion of the lungs. Currently, the patient is receiving supplemental oxygen by nonrebreathing mask. Which of the following is the most appropriate action? A. intubate the patient, perform endotracheal suctioning B. spray alcohol into the patient's oropharynx C. administer aerosolized bronchodilator medication D. administer heated aerosol therapy by ultrasonic nebulizer

A

A patient diagnosed with myasthenia gravis is receiving mechanical ventilatory support by an adult volume-ventilator. To determine if the disease has run its course, which of the following would be helpful information? A. MEP, LOC, Fev1 B. VT (spont), VC, MIP C. EMG, EEG, CSF D. FVC, TLC, VT

B

A patient with bronchiectasis has thick and tenacious secretions. The respiratory therapist is unable to adequately suction the patient with a 10 Fr catheter with the suction pressure set at 110 mmHg. Which of the following would be most helpful? A. administer 2.0 cc of 10% Mucomyst (Acetylcysteine) B. administer Atropine sulfate by aerosol C. administer prednisone, IV D. decrease suction pressure to 100 mmHg

A

A physician has just inserted a balloon-tipped flow directed pulmonary artery catheter. When the chest radiographic report returns, the respiratory therapist will expect to see which of the following descriptions if the line is properly placed? A. in the pulmonary artery B. over the right ventricle C. mid-lung space over the right lung field D. in the superior vena cava

A

A respiratory therapist is determining a VD/VT ratio. Which of the following is needed to complete this analysis? A. PetCO2 and PaCO2 B. VT and VA C. FIO2 and PaO2 D. PECO2 and PaO2

A

A respiratory therapy supervisor is responsible for making suggestions in the initial development of a smoking cessation program. Which of the following would be part of an effective smoking cessation program? A. Nicotine replacement therapy B. removing pulmonary irritants from the house C. daily COHb analysis D. daily completion of an asthma action plan

A

During the flow volume loop maneuver, the therapist notes a nearly round loop with virtually no inflection points. Which of the following may represent the patient's condition? A. laryngeal cancer B. obstructive pulmonary defect C. laryngotracheobronchitis D. restrictive pulmonary defect

A

In preparation for use of an oxygen concentrator in a patient's home, the respiratory therapist should evaluate which of the following environmental conditions? A. electrical load limits B. square footage of the home C. presence and function of CO detector in the home D. patient's ability to perform simple maintinence

A

The low ventilator alarm is sounding on a 28-year-old patient with myasthenia gravis. The respiratory therapist notes the oral endotracheal tube cuff is failing to hold pressure. The ET tube is positioned at 22 cm at the teeth. The therapist should next A. replace the endotracheal tube B. send to surgery for placement of a tracheostomy C. decrease the low tidal volume alarm D. increase the tidal volume

A

The measurement of a patient's FRC is 50% higher when measured by body box compared to the results when measured by helium dilution method. Which of the following is the likely reason for the difference in this patient? A. increased non-ventilated lung space B. poor patient effort during the helium dilution C. small leak during the helium dilution maneuver D. presence or choanale atresia

A

The respiratory therapist is asked to estimate the alveolar minute ventilation on a spontaneously breathing 68 kg (150 lb) female who is receiving oxygen therapy by air-entrainment mask at FIO2 0.50. The following data is available Exhaled VT 450 mL mPAP 15 torr PaCO2 40 torr PaO2 70 torr Respiratory rate 12/min The therapist should report an alveolar minute ventilation of: A. 3.6 L/min B. 4.6 L/min C. 5.4 L/min D. 12.0 L/min

A

When considering appropriate staffing levels and proper care of patients, a respiratory therapy supervisor would include which of the following A. Individual skills of staff members B. previously missed therapy C. staff member preference D. frequency of declined therapy

A

Which alarm is most critical for a post-operative mechanically ventilated patient who has a machine rate of 4/min? A. low minute volume B. low pressure C. high pressure D. high respiratory rate

A

Which of the following Carboxyhemoglobin levels is consistent with that of a regular, frequent smoker? A. 6.5% B. 20% C. 1.5% D. 2.9%

A

Which of the following would NOT be an indication for implementing mechanical ventilatory support? A. VT of 6 mL/kg B. impending ventilatory failure C. VC of 8 mL/kg D. acute ventilatory failure

A

While in the emergency room, the respiratory therapist receives a patient who was working as a firefighter when the burning building collapsed. He was rescued and brought to the ER. The therapist observes paradoxical chest movement in the right chest and hears marked stridor of the upper airway. The therapist should next do which of the following? A. intubate the patient with a bronchoscope B. administer cool aerosol by mask C. administer IPPB treatment D. immobilize the fractured ribs with a chest wrap

A

A patient is being weaned from mechanical ventilation after being placed on the ventilator 3 days prior due to complications from a pulmonary infection. Which of the following would be most helpful in determining the status of the infection prior to weaning? A. temperature B. CBC C. ABG D. evaluate color of sputum

A patient is being weaned from mechanical ventilation after being placed on the ventilator 3 days prior due to complications from a pulmonary infection. Which of the following would be most helpful in determining the status of the infection prior to weaning? A. temperature B. CBC C. ABG D. evaluate color of sputum

A physician orders a chest radiograph on a comatose patient in the intensive care unit to confirm proper ET tube placement. Which of the following should be ordered?

AP radiograph

Which of the following drugs can be administered through an endotracheal tube for the purpose of treating bradycardia?

Atropine

A 65-year-old male patient has been ventilator dependent for several weeks. Weaning attempts have failed. The patient will be transferring to a long-term care facility where daily weaning trials will commence. Which of the following artificial airways would be most appropriate for the patient? A. nasal tracheal tube B. cuffed tracheostomy tube C. oral endotracheal tube D. nasal pharyngeal airway

B

A patient in the emergency room (ER) is showing inverted T waves on the 12-lead ECG tracing. The physician is interested in decreasing the work of the heart. The respiratory therapist should suggest A. administration of Crytodigin (digitalis) B. oxygen by nasal cannula at 5 L/min C. oxygen by nasal cannula at 2 L/min D. adminsitration of Nitroclycerin (Isordil)

B

A patient is in acute ventilatory failure. The physician is preparing to perform oral intubation of the patient in preparation for mechanical ventilatory support. Prior to intubation, the physician delivers Anectine (succinylcholine chloride). What evidence should the therapist watch for to indicate the medication has taken enough effect to perform intubation? A. cessation of respirations B. muscle twitching about the face and neck C. dilated pupils D. patient's inability to count backwards

B

A patient is in the cardiac intensive care unit. Hemodynamic monitoring has been instituted with a Swan-Ganz catheter. From where should the respiratory therapist obtain a measurement of central venous pressure (CVP)? A. distal lumen of the PA catheter B. proximal lumen of the PA catheter C. jugular vein D. subclavian vein

B

A patient is receiving volume-controlled ventilation. A chest tube drainage system is in place. The low-volume ventilator alarm is sounding continuously and the respiratory therapist notices profuse bubbling in the water seal chamber. Which of the following should the therapist suspect? A. faulty chest drainage system B. perforated lung C. excess suction pressure from the wall D. excessive PEEP

B

A patient is receiving volume-controlled ventilation. The ventilator settings and corresponding clinical data are as follows: Mode Assist/control Mandatory rate 16 Total rate 16 FIO2 1.0 PEEP 25 cm H2O Pulse 118/min Blood pressure 138/74 mm Hg (stable) PAP 14 mmHg C.O. 5.8 L/min pH 7.49 PaCO2 31 torr PaO2 254 torr HCO3- 25 mEq/L BE +1 mEq/L The respiratory therapist should recommend A. Sedate the patient B. Decrease FIO2 C. Decrease mandatory rate D. Decrease PEEP

B

A patient receiving volume-controlled ventilation has an oxygen saturation of 87% within moments after the low pressure ventilator alarm begins to sound. After providing manual ventilation with a bag valve, the respiratory therapist observes the bag is easier than normal to squeeze and that oxygen saturation continues to fall. Which of the following could be the cause of this observation? A. pneumothorax B. partial extubation C. ET tube cuff herniation D. excess secretions in the ET tube

B

A patient remains hypoxic even when receiving supplemental oxygen at FIO2 0.45 by air-entrainment device. The respiratory therapist should recommend which of the following? A. mechanical ventilation B. increase in FIO2 C. CPAP D. reduction in FIO2

B

A patient with a history of congestive heart failure has been transferred to the cardiac intensive care unit with fulminating pulmonary edema, crepitations in the left lung field, and +3 pitting peripheral edema. A non-rebreathing mask is in place on the patient and the oxygen flow rate is set to 15 L/min. Which of the following would best resolve the patient's condition? A. administer Albuterol sulfate B. diurese the patient C. administer lactated ringers, IV D. administer aerosolized alcohol

B

A patient with pneumonia has thick secretions and is complaining of difficulty coughing and expectorating sputum. Which of the following should the therapist recommend as the most effective method of hydrating secretions? A. Cool aerosol treatments B. Oral consumptions of water C. Heated aerosol treatments D. Ultrasonic nebulizer treatments

B

In which of the following positions can the respiratory therapist expect to place a patient who will receive chest physiotherapy and postural drainage to address secretions found in the upper lobes, anterior segments of the lungs? A. Trendenlenburg - 30 degrees B. Supine position C. Prone position D. Trendelenburg - 15 degrees

B

The respiratory therapist notices a patient in the intensive care unit has a blood pressure of 110/80 mm Hg by arterial line and a blood pressure 120/85 when taken by sphygmomanometer with a cuff on the right arm. The therapist should A. replace the art line B. document the pressure taken by cuff C. document the pressure by arterial line D. replace the blood pressure cuff

B

The respiratory therapist reads a patient's medical record and notes a unilateral wheeze on the left is documented. Which of the following may be associated with this finding? A. bronchoconstriction B. bronchial carcinoma C. asthma D. vocal cord paralysis

B

The respiratory therapist reviews the results of a patient diagnosed with mixed restrictive and obstructive pulmonary defects. Which of the following data is NOT associated with this diagnosis? A. SVC of 68% of predicted B. FEV1/FVC% of 78% C. FEV1 of 75% of predicted D. FEF 200-1200 of 74% of predicted

B

What is the best method for a therapist to determine if a patient is compliant with a smoking cessation program? A. ask the patient's peers B. evaluate COHb C. ask the patient D. ask the patient's spouse

B

Which of the following ABG results would the respiratory therapist expect to see for a patient who is experiencing an acute on chronic episode with known COPD? A. pH 7.33 PaCO2 55 mm Hg PaO2 52 mm Hg HCO3- 30 mEq/L B. pH 7.48 PaCO2 50 mm Hg PaO2 51 mm Hg HCO3- 34 mEq/L C. pH 7.28 PaCO2 62 mm Hg PaO2 49 mm Hg HCO3- 33 mEq/L D. pH 7.51 PaCO2 35 mm Hg PaO2 60 mm Hg HCO3- 29 mEq/L

B

Which of the following methods would be most helpful to a patient weaning from mechanical ventilation who has a documented restrictive pulmonary defect and has been having difficulty in recent weaning attempts? A. pressure control ventilation B. pressure support C. permissive hypercapnia D. T-piece trials of 24 hour periods several times a week

B

Which of the following should the respiratory therapist evaluate to determine if the patient's perfusion is adequate? A. arterial blood gas analysis B. urine output C. pulse oximetery D. blood urea nitrogen level (BUN)

B

A 4-year-old patient is demonstrating ventilatory difficulty with use of intercostal accessory muscles. The patient has a low grade fever and a loud, barky cough. Which of the following mostly likely represents the patient's condition? A. Bacterial infection B. Meningitis C. Laryngotracheobronchitis (croup) D. Acute epiglottitis

C

A patient is receiving volume-controlled ventilation in the Assist/control mode at the following settings Mandatory rate 16 VT 550 mL FIO2 0.6 PEEP 22 cm H2O Recent arterial blood gases show pH 7.35 PaCO2 44 torr PaO2 68 torr HCO3- 26 mEq/L BE 0 mEq/L In response to the blood gas, PEEP is increased from 22 to 25 cm H2O. Immediately following the increase, cardiac index drops from 2.8 to 1.9. The therapist should now A. Perform an optimal PEEP study B. Increase FIO2 to 0.70 C. Decrease PEEP to 22 cm H2O D. Increase FIO2 to 1.0

C

A patient receiving volume-controlled ventilation has a pressure volume loop that shows the inspiratory cycle begins before exhalation is complete. Which of the following is an appropriate response to this situation? A. decrease expiratory time B. decrease inspiratory flow rate C. increase inspiratory flow rate D. increase I:E ratio

C

A patient receiving volume-controlled ventilation is coughing uncontrollably. The high-pressure alarm is sounding continuously. The respiratory therapist should first A. sedate the patient B. suction the patient C. provide the patient with manual ventilatory support D. silence the ventilator alarm, calm the patient's anxiety

C

A physician suspects a patient has a pleural effusion. Which of the following radiography procedures is helpful in diagnosing the problem? A. apical lordotic radiograph B. PA chest radiograph C. lateral decubitus radiograph D. AP chest radiograph

C

A pulmonary rehabilitation patient is being monitored for compliance to a smoking cessation program. Which of the following values would be most helpful to evaluate? A. SPO2 B. PaO2 C. multiple wave-length spectrophotometry D. metHb

C

A radiological report is placed in the medical record that describes the results of a chest radiograph. The report indicates a batwing pattern. This finding is consistent with A. pneumonia B. atelectasis C. pulmonary edema D. pleural effusion

C

A respiratory therapist is initiating mechanical ventilation at the following settings and preferences: Mode Assist/control Mandatory rate 20 VT 500 mL FIO2 0.5 PEEP 5 cm H2O I:E 1:2 Which of following is the minimum inspiratory flow setting? A. 80 L/min B. 65 L/min C. 40 L/min D. 28 L/min

C

A respiratory therapy department supervisor is seeking ideas for quality improvement in patient care. Which of the following would be appropriate? A. study of acuity index B. number of treatments given per therapist C. effectiveness of an oxygen titration protocol D. department staffing efficiency

C

After analyzing arterial blood on a patient with a history of arterial blood gas analysis, the respiratory therapist concludes the results are not possible. They are very different than the patient's historical blood gas results. Quality control records show no evidence of any trends, shifts, or out-of-control situations in the last month. The therapist should A. Inform the physician of the suspect blood gas B. Ask the medical director for direction in this situation C. Repeat the analysis with the same blood sample and same analyzer D. Perform a two-point calibration on the analyzers

C

After three weeks participating in a smoking cessation program, the patient's COHb is 12%. Which of the following can the respiratory therapist conclude? A. The patient should be dropped from the smoking cessation program B. The patient has been successful at quitting C. The patient is still smoking D. Nicotine replacement therapy medication should be decreased

C

An adult female complains of double vision, dysphasia, and fatigue. Symptoms improve with rest. The respiratory therapist notes the patient is drooling and is having difficulty enunciating words. Which of the following should the therapist recommend? A. Spinal tap B. Lateral neck x-ray C. Tensilon test D. Acute epiglottitis

C

For a patient who is receiving mechanical ventilation, which of the following would cause the most rise in arterial carbon dioxide? A. addition of an MDI interface B. addition of tubing to the inspiratory side of the circuit C. addition of a heat-moisture exchanger (HME) and tubing D. addition of tubing to the expiratory side of the circuit

C

For which of the following vagal reflex response should the respiratory therapist monitor in a patient that is receiving nasotracheal suctioning: A. hypertension B. third degree heart block C. bradycardia D. increased systemic vascular resistance

C

The radiological report of a chest radiogram indicates a batwing pattern. This description is most consistent with which of the following? A. ARDS B. atelectasis C. pulmonary edema D. pleural effusion

C

The respiratory therapist analyzes the oxygen percentage coming from the end of the aerosol tubing on a large-volume, nebulizer. The analysis shows a higher oxygen percentage than the set FIO2 on the air-entrainment device. Which of the following could be the reason for this disparity? A. depleted reservoir B. clogged down tube in the nebulizer C. kinked aerosol tubing D. leak in the aerosol tubing

C

Which of the following is the least important to consider when drawing an arterial blood sample? A. patient temperature B. needle size C. supplemental oxygen delivery mode D. FIO2

C

Which of the following may be done to increase the effectiveness of gas distribution for a patient with ARDS who is receiving volume-controlled ventilation in the assist-control mode? A. Switch to SIMV mode B. Administer surfactant C. Use pressure-controlled ventilation D. Decrease the I:E ratio

C

Which of the following methods should the respiratory therapist recommend for a patient receiving mechanical ventilation who requires chest physiotherapy? A. autogenic drainage B. pressure control ventilation C. high-frequency chest wall compression D. huff coughing

C

A patient reports to the emergency room (ER) following an accident while playing football. Paramedics on the scene report a sudden on-set of tachypnea. The following chest radiograph is taken. What is the most likely diagnosis?

pneumothorax

While instructing a patient on the use of a volume-type incentive spirometer, the therapist observes the patient exhaling forcefully in the spirometer, but no volume is registering on the spirometer. The therapist should A. encourage the patient to exhale more forcefully B. switch to a flow-type incentive spirometer C. instruct the patient to inhale through the device D. obtain a new volume-type spirometer

C

While receiving nasal CPAP delivered by an infant ventilator, the patient is retracting and creating a fluctuating baseline pressure on the monometer. What should the respiratory therapist do? A. Suction the patient B. Adjust the threshold resistor C. Increase the flowrate D. Increase the size of the fixed orifice resistor

C

While performing overnight CPAP/Bi-level titration on a patient previously diagnosed with obstructive sleep apnea, the respiratory therapist reviews the patient's medical record and notes an AHI of 65, observed during diagnostic polysomnography. Currently, the patient is receiving nasal mask CPAP at 9 cm H2O. The corresponding AHI is 22. The therapist should conclude that

CPAP pressure in insufficient

A newborn infant experiences apnea when breast feeding, but recovers quickly when breast feeding is discontinued. Which of the following mostly likely explains the condition?

Choanale atresia

Which of the following is an important action when drawing an arterial blood gas sample?

Choose a radial site with adequate ulnar circulation

A 12-year-old patient diagnosed with bronchiectasis is receiving postural drainage and percussion to drain the basal segments of the right lung. While draining with the head of the bed down 30 degrees, the patient begins to exhibit signs of distress including frequent PVCs and a decrease in oxygen saturation. After discontinuing the therapy, the respiratory therapist should recommend A. Provide percussion with the bed flat B. Provide percussion with the head of bed up in semi-fowlers position C. Placing head of bed down only 15 degrees D. Administering therapy with an external percussive device (Vest)

D

A 38-year-old male patient presents to the emergency room with difficulty swallowing and double vision. He reports a history of Myasthenia Gravis at a prior time, but was never ventilator-dependent. He now has the following clinical data: Vital Capacity Tidal volume MIP 0.9 L 350 mL -22 cm H2O What should the respiratory therapist recommend? A. Atropine B. NRB mask C. manual ventilation D. Tensilon challenge

D

A 5-year-old child is brought to the emergency room (ER) with a fever and difficulty breathing. The patient is drooling but is making no vocal sounds. Which of the following procedures should the respiratory therapist recommend? A. bronchoscopy B. CBC C. inspection of the pharyngeal area with a tongue depressor D. lateral neck radiograph

D

A burn trauma patient who is also suspect for carbon monoxide poisoning is receiving oxygen therapy by a nonrebreathing mask. The respiratory therapist observes that the reservoir bag collapses completely with each inspiration. The therapist should A. remove the one-way valve in the reservoir B. continue current therapy as this is normal C. obtain a larger reservoir D. increase flow to the reservoir bag

D

A during a routine visit to a patient's home, a home care therapist notes unsanitary conditions including the presence of cockroaches, moldy shower door and spoiled food on the kitchen counter. Which of the following clinical findings is most likely? A. malnourishment B. polycythemia C. sarcoidosis D. recurring infections

D

A motorcycle accident victim is in the emergency room with multiple wounds including a contusion to the head and torn deep muscle tissue in the right leg. The patient is bleeding massively. The respiratory therapist should first A. monitor respiratory rate q 10 minutes B. recommend blood type and crossmatch evaluation in preparation for blood administration C. recommend a CT scan of the head D. place the patient on oxygen at FIO2 1.0

D

A patient in ICU has a balloon-tipped pulmonary artery catheter in place. The following hemodynamic data is observed CVP 10 torr mPAP 19 torr PWP 24 torr MAP 112 torr Arterial blood gas analysis reveals: pH 7.38 PaCO2 43 torr PaO2 70 torr HCO3- 21 mEq/L BE -3 mEq/L Which of the following should the respiratory therapist recommend? A. Crystodigin (digitalis) B. Fluid administration C. Antibiotic therapy D. Diuresis

D

A patient receiving heliox therapy at 80% / 20% mixture by nonrebreathing mask has a total flow of 18 L/min to the mask. The oxygen flow meter that is adapted to fit on the heliox tank is indicating a flow rate of 10 L/min. The therapist would best explain the difference to the physician by saying A. The heliox mixture has greater viscosity B. The oxygen flow meter is not compensated for back pressure C. The helium expands once delivered into the ambient pressure environment D. Helium is less dense and therefore reads a lower flow by an oxygen flow meter

D

A patient receiving oxygen therapy at home complains the oxygen concentrator is not working and that the flow feels insufficient. The respiratory therapist should A. instruct the patient to check the flow with a calibration tube B. tell the patient to call the manufacturer C. tell the patient everything is normal D. instruct the patient use back-up E cylinders until the concentrator can be examined

D

A patient receiving volume-controlled ventilation who was admitted 3 days prior for drug overdose is beginning to awake from a barbiturate-induced coma. There is no history of pulmonary disease in the patient's medical record. As the patient becomes alert, the respiratory therapist should recommend A. sedate the patient with Ativan B. perform pulmonary function testing C. interview the patient for family medical history of drug abuse D. begin following the ventilator weaning protocol

D

The respiratory therapist notes the following results of an arterial blood gas with a patient breathing air: pH 7.42 PaCO2 38 torr PaO2 124 torr HCO3- 24 mEq/L BE 0 mEq/L Which of the following could explain the reason for these results? A. blood has coagulated B. hyperventilation C. normal V/Q matching D. air bubbles in the sample

D

A patient who weighs 70 kg (154 lb) is receiving volume-controlled ventilation at the following settings: Mode SIMV Mandatory rate 10 Total rate 28 Set tidal volume 500 mL Spontaneous VT 520 mL FIO2 0.45 PEEP 5 cm H2O Which of the following can be done to decrease the work of breathing? A. increase tidal volume to 700 cc B. decrease tidal volume to 400 cc C. increase PEEP to 10 cm H2O D. increase mandatory rate to 14

D

A patient with COPD is receiving oxygen therapy at 2 L/min by nasal cannula. During a routine check, the respiratory therapist finds the patient unresponsive. The ECG waveform on the monitor is consistent with ventricular tachycardia. The therapist should immediately A. Begin chest compressions B. Perform cardioversion C. Defibrillate at 360 joules with synchronization set to ON D. Place the patient on a nonrebreathing oxygen mask and check for a pulse

D

A patient, who is on a 3-chamber chest-tube drainage system, is receiving volume controlled ventilation in the assist/control mode. The low return volume alarm begins to sound. Which of the following should the respiratory therapist troubleshoot to determine the cause of the alarm? A. secretions in the airway B. excess condensate in the circuit C. loose circuit connection D. bubbling in the water seal chamber

D

A respiratory therapist is having difficulty obtaining an oxygen saturation reading from a pulse oximeter. Which of the following should the therapist evaluate to determine the cause of the problem? A. C(a-v)O2 B. PaO2 C. CVP D. Presence of finger nail polish

D

A respiratory therapist is preparing a patient who will be transferred home and will be ventilator dependent during the night. Which of the following devices would be most helpful to ensure adequate hydration of the patient's airway during the night? A. Heated wire ventilator circuit with water traps B. Cascade humidifier C. Large volume nebulizer D. Heat moisture exchanger (HME)

D

A respiratory therapist notes endotracheal suctioning is ineffective as the patient's secretions have become thick and tenacious. The patient is intubated with an 8.0 mm endotracheal tube and is being suctioned at a pressure of 100 mm Hg with a 10 Fr closed system suction catheter. Which of the following modifications in therapy would be most beneficial to the patient? A. Have the patient orally consume water B. Diurese the patient C. Increase suction pressure to 110 mm Hg D. Use a 12 Fr suction catheter

D

A victim of a multiple trauma motorcycle accident is receiving mechanical ventilation and has chest tubes inserted in the right lung. The respiratory therapist notices gentle bubbling in the water-seal bottle and a small amount of fluid dripping into the fluid collection bottle. The following data is also available: Mode Assist/control Mandatory rate 18 Total rate 18 VT (set) 550 mL VT(exhaled) 534 mL Peak pressure 28 cm H2O Plateau pressure 18 cm H2O The therapist should A. suction the patient B. clamp the chest tube(s) near the patient C. troubleshoot the chest tube drainage system D. continue current therapy

D

After confirming the blood gas analyzer is "in control", the respiratory supervisor introduces arterial blood from a patient breathing room air and obtains the following results: pH 7.39 PaCO2 41 torr PaO2 210 torr HCO3- 24 mEq/L BE -2 mEq/L Which of the following should the respiratory therapist investigate? A. Fluid level of the reference electrode B. Membrane integrity of the Severinghaus electrode C. Membrane integrity of the Sanz electrode D. Function of the Clark electrode

D

An infant born at 41 weeks of gestation is now 2 days old. The respiratory therapist notices the infant appears jaundice in color. Vital signs are normal. The medical record over the last 24 hours indicates occasional apneic periods lasting 10-20 seconds. Which of the following disorders could account for these observations? A. maternal history of barbiturate abuse B. pulmonary immaturity C. hormone imbalance D. problem with bilirubin level

D

At patient in pulseless ventricular tachycardia is receiving chest compressions while being orally intubated. Immediately after ET tube insertion, the respiratory therapist checks breath sounds and notices vesicular sounds bilaterally. The therapist should next do which of the following: A. remove the ET tube and attempt nasal intubation B. withdraw the ET tube by 2 cm, observe chest rise C. advance the ET tube and auscultate the chest D. recommend a chest radiograph

D

In preparation for a smoking cessation class, the therapist should include information on which of the following agents? A. Aminophylline B. Sub-lingual Lidocaine spray C. Pronestyl D. nicotine patch

D

The following arterial blood gas results are recorded for a patient during cardiopulmonary resuscitation: pH 7.14 PaCO2 47 torr PaO2 180 torr HCO3- 14 mEq/L BE -10 mEq/L FIO2 1.0 The respiratory therapist should recommend A. decrease manual ventilation rate B. administer Verapamil C. increase manual ventilation rate D. sodium bicarbonate administration

D

The high pressure alarm is sounding on a patient who is receiving long-term mechanical ventilation. After manually ventilating the patient with a bag-valve, the respiratory therapist notices a large amount of thick sputum is lodged in the heat moisture exchanger (HME). The therapist should A. Replace the HME device with a new one B. Suction the HME device to remove secretions C. Clean the HME by soaking it in normal saline followed by a rinse D. Replace the HME with a heated humidifier

D

The physician has asked the respiratory therapist to help determine oxygen consumption by the tissues. The therapist should suggest evaluation of which of the following? A. CvO2 B. CaO2 C. PaO2 D. C(a-v)O2

D

The respiratory therapist is determining the proper size of an oropharyngeal airway on a patient who is orally intubated. Which of the following methods will be helpful in properly estimating the appropriate size of the airway? A. determine the patient's ideal body weight in kilograms B. examine the distance between the earlobe and nasal septum C. determine the age and sex of the patient D. examine the distance between the angle of the jaw and tip of the chin

D

The respiratory therapist is performing a preoperational test of a ventilator. The therapist notices the return volume is 200 mL less than the set tidal volume. The therapist should A. tag the ventilator as non-functional and isolate it from functional equipment B. notify the medical director and seek advice C. accept the results and place the ventilator into service D. check for a leak in the circuit or the circuit's connections

D

The respiratory therapist responds to an adult patient on a ventilator whose low pressure alarm is sounding. The therapist determines the 6.0 mm ET tube is in proper position according to the markings. Gas escaping around the ET tube is audible. The therapist should A. add air to the cuff B. switch to a cuffless ET tube C. schedule the patient for a tracheotomy D. switch to a larger ET tube

D

What should a respiratory therapist monitor during the cardioversion of a patient who is alert, conscious, and calm? A. temperature B. pupillary reaction C. carotid pulse D. airway patency

D

Which of the following calculations can be used to determine FRC? A. VC - RV B. RV + ERV + VT C. TLC - ERV - RV D. TLC - IRV - VT

D

Which of the following calculations can be used to determine TLC? A. FRC + VT + IC B. RV + ERV + VT + IC C. RV + ERV + VT D. IC + FRC

D

Which of the following resistors is not associated with pressurizing the circuit for a patient receiving CPAP therapy? A. water-column B. weighted C. spring-loaded D. fixed orifice

D

While doing routine rounds on patients in the medical-surgical unit, the respiratory therapist discovers a patient is non responsive and has no visible chest movement. The first action of the therapist should be to A. leave and get help B. check for a pulse C. begin chest compressions D. administer manual ventilations with a bag/valve Correct

D

While performing a routine ventilator a check on a patient with a balloon-tipped flow-directed pulmonary artery catheter in place, the respiratory therapist notices the inflection points on the waveform indicate the tip of the pulmonary artery catheter is improperly placed in the right ventricle. To correct this problem, the therapist should recommend A. recording the pulmonary artery pressure as shown on the waveform B. removing the catheter and inserting a new one C. twisting the catheter until pulmonary artery pressures are observed D. inflating the balloon and advancing the catheter

D

A 32-week-gestional age infant is in respiratory distress following a cesarean section birth. A chest x-ray shows a reticulogranular pattern. Which of the following would be most helpful to the patient?

Exosurf

A physician suspects a patient has an obstructive pulmonary defect. Which of the follow tests would be most helpful at determining this?

Fev1.0/FVC%

Hemoximetry can be used to determine which of the following levels?

Hb

A COPD patient who is receiving 1 L/min by nasal cannula becomes short of breath after ambulating to the bathroom and back. The patient indicates she is having difficulty catching her breath more than usual. Her respiratory rate is 22/min. The respiratory therapist should do which of the following to help the patient return to her normal breathing status?

Increase oxygen flow rate to 3 lpm.

An 80-kg (176 lb) male is receiving volume-controlled ventilation after undergoing a partial lobectomy of the left lung. Pertinent data is below: pH 7.32 PaCO2 49 torrPaO2 74 torrHCO3- 24 mEq/LBE -2 mEq/L FIO2 0.50 VT 600 mLMode Assist/controlMandatory rate 12Total rate 12PEEP 5 cm H2O Which of the following should the respiratory therapist recommend?

Increase rate to 14

Which of the following electrolyte values is not within range? Na+ 135 mEq/L HCO3- 23 mEq/L K+ 5.5 mEq/L Cl- 101 mEq/L

K+

A respiratory therapist is monitoring a patient's maximal inhalation and exhalation repeatedly over a period of 15 seconds. The therapist is attempting to observe which of the following?

MVV

A patient is about to go to surgery for nasal reconstruction and will be orally intubated for the procedure. The respiratory therapist determines that the patient is likely to be very difficult to intubate based on an anatomical assessment of the upper oropharynx. Maximum intubation difficulty would be associated with a(n)

Mallampati classification of 4.

A 32-year old, 73-kg (161-lb), 178-cm (5 ft, 10 in) male patient is receiving VC ventilatory support on the following settings with the following ABG results: Mode SIMV VT 400 mL(f) 12/min PEEP 5 cm H2O FIO2 0.45 ABGs pH 7.29 PaCO2 50 mm HgPaO2 81 torr HCO3- 24 mEq/L BE -5 mEq/L The respiratory therapist should make which of the following observations about the patient's condition?

Minute ventilation is too low.

A gram-positive organism is identified in the blood of patient who has bacterial pneumonia and who is penicillin-resistant. Which of the following antimicrobials would be most suitable for treatment?

Oxacillin

Which of the following data would provide the most information about arterial oxygenation relative to administered supplemental oxygen?

P/F ratio

Which of the following can be used to determine fetal lung maturity

PC level (phosphatydlchloride)

Which of the following will help a patent to speak while intubated with a tracheostomy tube?

Passy-Muir valve

During endotracheal suctioning of a patient with a closed-system suction catheter (BallardÒ), the respiratory therapist observes the following ECG rhythm on the monitor. The therapist could do which of the following to prevent the rhythm in the future?

Reduce applied suction duration.

A patient with COPD who has been receiving mechanical ventilation for 3 weeks has been weaned down to the following settings: FIO2 0.30 Mandatory rate 8Mode SIMVVT(set) 400 mLVT(spont) 320 mLPEEP 3 cm H2OPS 4 cm H2O On these settings, the following laboratory data is observed: pH 7.36PaO2 65 torrPaCO2 70 torrHCO3- 30 mEq/LBE -1 mEq/L The respiratory therapist should recommend

SBT

A respiratory therapist is assisting the anesthesiologist in the placement of an 8.0-mm tracheostomy tube on a patient who has ARDS and is receiving mechanical ventilation in the PC mode. During a troublesome insertion, the therapist observes the development of tachycardia, a marked reduction in the return volume, and absent breath sounds over the right lung field. The therapist should suspect A. right-sided pneumothorax B. a pleural effusion is developing on the right C. massive bilateral atelectasis D. the tracheostomy tube is too large

The correct answer is : A Explanation : One of the possible negative outcomes that can occur with the insertion of a tracheostomy tube is a pneumothorax. Data indicating absent breath sounds over the right lung field as well as a reduction in return volume suggests that a pneumothorax may have occurred on the right. A chest radiograph would be required to confirm this.

A 60-year-old patient with COPD is brought to the emergency department in ventilatory distress. The patient is current receiving supplemental oxygen at 50% via Venturi mask. The respiratory therapist notes the patient is difficult to arouse and has short, shallow respirations. Which of the following should the therapist do? A. Obtain arterial blood for a blood gas analysis B. Obtain a A-P chest radiograph C. Manually ventilate the patient D. Calculate C(a-v)DO2

The correct answer is : A Explanation : A COPD patient should not receive more than 1-2 L/min oxygen by nasal cannula or no more than 28% oxygen by any other device. Over oxygenating a COPD patient may result in suppression of the hypoxic drive and will be noticed by a decrease in the depth and rate of respiration. The most appropriate action would be to lower the oxygen percentage. However, this option is not offered. The best option, therefore, would be to obtain an arterial blood gas, which would prove that the patient's PaO2 is too high.

A bronchopleural fistula on a patient receiving mechanical ventilation with a chest tube drainage system in place would be most likely be indicated by which of the following observations? A. excess bubbling in the water seal chamber B. occasional low-exhaled volume alarm C. I:E ratio alarms D. high pressure ventilator alarm

The correct answer is : A Explanation : A bronchopleural fistula is essentially a leak from the airway in the lungs to the pleural space. This condition can result in two things - one) a low-pressure or low-volume alarm, 2) excess bubbling in the water seal chamber of the chest tube drainage system. When you look at the answers, a leak would not cause a high pressure alarm.

A patient has two different views of a chest x-ray in the medical record. One is an anterior/posterior view and the other is a lateral view. Both findings show blunt costophrenic angles. The most likely diagnosis associated with these findings is A. pleural effusion. B. COPD. C. pulmonary edema. D. double pneumothoraces.

The correct answer is : A Explanation : A chest X-ray reveals a pleural effusion if one sees descriptors such as "blunt or obliterated costophrenic angles", or "concave superior interface".

The medical record of a patient indicates a chest radiogram shows a concave superior interface border. This description is most consistent with which of the following? A. pleural effusion B. ARDS C. COPD and chronic air-trapping D. pulmonary edema

The correct answer is : A Explanation : A chest x-ray that shows a "concave superior interface border" is associated with pleural effusion.

The following patient data is observed: Blood gas and CO-oximeter results pH 7.35 PaCO2 27 torr PaO2 40 torr Hemoglobin 14.0 g/dL Oxyhemoglobin 96% HCO3- 13 mEq/L Carboxyhemoglobin 2% Methemoglobin 2% SaO2 76% BE -11 mEq/L The respiratory therapist should A. perform a calibration of the instrument and repeat the analysis B. report SaO2 as 76% C. administer sodium bicarbonate D. administer potassium chloride, IV

The correct answer is : A Explanation : A close examination of the blood gas reveals some contrasting information. Oxyhemoglobin is 96% while the O2 sat is only 76%. These data indicate a need to recalibrate the instrument and repeat the analysis.

A fire fighter is brought to the emergency department after being found down in a room of a burning building. He was found with his protective mask off. Which of the following therapies may be helpful to the patient? A. hyperbaric chamber oxygen therapy B. volume-replacement therapy C. IV methylprednisolone D. bronchodilator therapy

The correct answer is : A Explanation : A firefighter who is found down in a burning building without a mask has likely inhaled smoke and carbon monoxide. The treatment for this is maximum oxygen therapy and hyperbaric oxygen therapy.

A patient presents to the emergency room with clear lung sounds, but a high-pitched noise is present during inspiration in the upper airway. This finding is most closely associated with A. stridor. B. bronchospasm. C. wheezing. D. a fixed upper airway obstruction.

The correct answer is : A Explanation : A high-pitched noise in the upper airway is also known as stridor. Stridor is related to inflammation in the upper airways resulting in a narrowing passageway for air movement. In some cases this is life-threatening.

Excessive bubbling is noted in the water seal chamber of a chest tube drainage system. Which of the following could be the cause? A. leak in the drainage tubing coming from the patient B. water seal fluid level is too high C. wall pressure is too low D. water seal fluid level is too low

The correct answer is : A Explanation : A leak in the drainage tubing coming from a patient's chest will cause the entrainment of additional air into the system which will show up as excessive bubbling in the water-seal compartment. Normally gentle bubbling should be observed. When excessive bubbling is present, a leak most likely exists somewhere between the waterseal compartment and the patient's lung tissue and may be caused from a perforation or hole in the lung tissue.

Which of the following medications is classified as a decongestant ? A. racemic epinephrine B. cromolyn sodium (Intal) C. Tiotropium (Spiriva) D. ipratropium bromide

The correct answer is : A Explanation : Although not well known, racemic epinephrine is classified as a decongestant.

Which of the following requires a quality control assessment?

point-of-care blood gas analyzer

Arterial blood gases on a patient in the emergency department are as follows: pH 7.29 PaCO2 51 mmHg PaO2 70 mmHg HCO3- 25 mEq/L BE +4 mEq/L Which of the following represents an accurate interpretation of these results? A. uncompensated respiratory acidosis with mild hypoxemia B. compensated metabolic alkalosis C. uncompensated metabolic acidosis with moderate hypoxemia D. ketoacidosis with hypoxemia

The correct answer is : A Explanation : A look at the CO2 reveals an elevation. When we glance at the pH we see that there is no compensation, thus making the problem acute uncompensated respiratory acidosis. Additionally, we see the patient is mildly hypoxic. This, mild hypoxemia and respiratory acidosis is the correct interpretation for this patient.

The following pulmonary function test results are reported for a 60-year-old male patient who weighs 65-kg (143-lb) and is 5-ft, 6-in (168 cm) tall. % of predicted Actual Value FVC 82 SVC 91 Fev1.0/FVC% 58% Fev1.0 62 FEF 200-1200 79 FEF25-75 60 DLCO 88 Based on this information the patient has A. mild obstructive defect B. mixed obstructive and restrictive defect C. moderate restrictive defect D. pulmonary emphysema

The correct answer is : A Explanation : A mild obstructive defect is suggested by a Fev1/FVC% of 58% and a Fev1 of 62% of predicted.

The respiratory therapist should evaluate which of the following to develop a teaching plan for a patient who will be using a metered dose inhaler (MDI) at home? A. manual dexterity and language skills B. disease history and current diagnosis C. visual acuity and color blindedness D. highest achieved grade level and GPA

The correct answer is : A Explanation : A patient taking nebulizer treatments should be able to demonstrate manual dexterity, or the ability to hold and manipulate the nebulizer device. It is also important that they demonstrate appropriate language skills so they may understand the instruction provided them regarding the use of the device. Their ability to see, or visual acuity, and disease history is not important or critical in learning how to use a nebulizer device.

Which of the following should be excluded in an evaluation of a patient's ability to learn to self-administer medication by metered dose inhaler (MDI)? A. disease history B. manual dexterity C. language skill D. level of dementia

The correct answer is : A Explanation : A patient that is taking nebulizer treatments needs to be able to demonstrate manual dexterity, and the ability to hold and manipulate the nebulizer. During the instruction, appropriate language skills and mental capabilities are also important. The patient should also have the mental ability to learn. What is NOT important is the patient's disease history. This will have no effect on their ability to self administer nebulizer treatments.

A patient is receiving non-invasive positive pressure ventilation with an IPAP of 20 and an EPAP of 10 cm H2O. Which of the following changes would remedy the patient's snoring while asleep? A. increase EPAP B. decrease IPAP C. decrease EPAP D. increase IPAP

The correct answer is : A Explanation : A patient who has signs of obstructive sleep apnea, such as snoring, would benefit most from increased expiratory pressure. This continuous pressure combats obstruction caused by relaxation of upper airway tissues.

A patient in the cardiac intensive care unit is receiving dopamine HCl. Hypotension persists in spite of the administration of 2 units of blood and a quick infusion of additional blood expanding fluid in the past 20 minutes. The therapist should recommend A. inserting an indwelling arterial catheter B. placing the patient in Trendelenburg position C. placing the patient in reverse Trendelenburg position D. administering additional fluid

The correct answer is : A Explanation : A patient who is having drastic changes in blood pressure due to depleting blood volume would do well to have an indwelling arterial catheter. This line will help to administer blood more quickly and will allow a live monitoring of blood pressure.

A patient receiving volume-controlled ventilation is beginning to awake from sedation after abdominal surgery. As the patient becomes alert, the respiratory therapist should recommend A. weaning B. checking end-tidal CO2 C. ambulation of the patient D. monitoring closely

The correct answer is : A Explanation : A patient who is receiving mechanical ventilatory support as a result of surgery, and who has no problem with the pulmonary system, is likely able to wean from the ventilator once they awaken from sedation. Weaning is the most appropriate option here but it would also be appropriate to simply remove the patient from mechanical ventilatory support.

A respiratory therapist is preparing to intubate and place a patient on a mechanical ventilator. The patient will require humidity therapy in conjunction with ventilatory support. Which of the following should the therapist prepare in order to accomplish this? A. Cascade or Wick-type humidifier B. bubble humidifier C. ultrasonic nebulizer D. spinning-disc humidifier

The correct answer is : A Explanation : A patient who is receiving mechanical ventilatory support through an endotracheal tube is unable to naturally provide humidification to inspired gases because the natural humidification processes of the body are bypassed. In this case, a device that is capable of making up the entire humidity deficit is required. Of the devices listed only a Cascade or Wick-type humidifier can accomplish this.

During a routine ventilator check the respiratory therapist notices the patient is extremely restless and is thrashing about in bed. Suddenly the low return volume alarm sounds and oxygen saturation drops to 87%. The first action of the respiratory therapist should be to A. check endotracheal tube markings and position B. order a chest radiograph, STAT C. increase PEEP D. sedate the patient

The correct answer is : A Explanation : A patient who is trashing about in bed may inadvertently change the position of the endotracheal tube. Because this patient is demonstrating some level of respiratory distress after thrashing about in bed, proper position of the endotracheal tube should be determined. This should only be done with a method that is quick. Of the options offered, visually inspecting the markings on the endotracheal tube will provide the quickest data.

A patient with COPD is found with shallow respirations and is disoriented to time and place. The patient is receiving oxygen by Venturi mask set at 45%. SpO2 is 99%. The therapist should first do which of the following? A. decrease FIO2 to 0.28 B. switch to a nonrebreathing mask set at 12 L/min C. institute mechanical ventilation D. switch to a nasal cannula at 5 L/min

The correct answer is : A Explanation : A patient with COPD should not receive more than 28% oxygen, or more than 1-2 liters per minute by nasal cannula. Excessive oxygen delivery can result in suppression of the ventilatory drive, which can lead to disorientation, hypoventilation, and ventilatory failure. Oxygen saturation should be kept at or below 94%.

A patient receiving volume-controlled ventilation in the assist/control mode has a peak inspiratory pressure of 55 cm H2O. Both dynamic and static compliance have decreased over the last few days. The patient will benefit most from which of the following? A. pressure control ventilation (PCV) B. Bi-level therapy C. pressure-cycled ventilation D. pressure support

The correct answer is : A Explanation : A peak pressure over 50 cmH2O is an indication for pressure control ventilation. This is true only if the cause of the peak pressures are not a result of temporary conditions such as secretions in the airway or a kinked ET tube. The problem states that the patient's dynamic and static compliance has been decreasing over the last few days, indicating pulmonary changes. This data suggests that the source of the high airway pressure is not caused by a temporary condition.

The respiratory therapist is having difficulty getting the Polorgraphic (Ohio) oxygen analyzer to calibrate to 100% oxygen. The analyzer consistently reads 70%. Which of the following may be a helpful remedy to the problem? A. Change the batteries B. Tape a note to the analyzer indicating a 30% offset C. Change the fuel cell D. Send the analyzer to the manufacture for repair

The correct answer is : A Explanation : A polargraphic (Ohio) oxygen analyzer requires batteries, computer, and an electrolyte solution in the probe to function properly. When the analyzer fails to calibrate, the most likely cause is depleted batteries. Because this is something the respiratory therapist can remedy, sending the analyzer in for repair is not indicated. A fuel cell change is not indicated because only a galvanic-type oxygen analyzer has a fuel cell.

Which of the following would NOT be an indication for implementing mechanical ventilatory support? A. MIP -28 cm H2O B. tidal volume < 4 mL/kg C. respiratory rate of 4 per minute D. no chest movement

The correct answer is : A Explanation : A respiratory rate of 4/min or less, a tidal volume of less than 5 mL per kilogram, and no chest movement are all indications the patient is in need of mechanical ventilatory support. An MIP of -28 cmH2O is consistent with independent ventilatory ability.

Which if the following is appropriate regarding suctioning? A. Increase suction duration within limits for thick secretions B. use a suctioning catheter diameter size less than 5/6th of the ETT diameter C. apply intermittent suctioning during withdrawal of the catheter D. ventilator patients should be suctioned every two hours

The correct answer is : A Explanation : A suction catheter should not exceed 1/2 the internal diameter of an artificial airway. Suctioning should be done as needed and not on a routine schedule for mechanically ventilated patients. It is appropriate to increase the suction duration to manage thick tenacious secretions. It is also appropriate to decrease the suction duration for a patient who is experiencing serious side effects related to suctioning.

Over the course of three weeks, the respiratory therapist notes a documented change in the color of sputum on a patient receiving mechanical ventilatory support. The sputum has changed from white or clear to light green and now has a sweet odor. The respiratory therapist should recommend which of the following? A. Antibiotic therapy B. wetting agent therapy C. antiviral therapy D. antiprotozoan therapy

The correct answer is : A Explanation : A sweet odor noted in sputum is associated with Pseudomonas, which is a bacterial infection in the lungs. Antibiotic therapy is be most appropriate.

You are called to the emergency room to provide ventilation with a manual resuscitator for a patient receiving CPR. The patient is intubated and the endotracheal tube position has been confirmed. It is difficult to squeeze the bag. A possible causes is A. low pulmonary compliance. B. a deflated ET tube cuff C. insufficient oxygen flow to the bag D. right maintstem bronchus

The correct answer is : A Explanation : A trained respiratory therapist is able to assess the appropriate effort required to manually ventilate a patient by squeezing a resuscitator bag. There are numerous causes of increased difficulty in squeezing the bag, including low pulmonary compliance.

In response to an Asthma action plan, the patient has attempted to contact their physician after determining peak flow measurement is less than 50% of the patient's usual baseline value. The physician is not responding to the call. According to NAEP guidelines, the patient should NEXT A. report to the hospital or call an ambulance B. take a corticosteroid inhaler and check again in 20 minutes C. take a short-term bronchodilator, check peak flow in 1 hour D. take a short-term bronchodilator and contact a different physician

The correct answer is : A Explanation : According the national asthma guidelines, a self monitored peak flow of 50% of baseline is an indication to contact one's physician. However, if one's physician is not available, the patient should report to the hospital or to the emergency room or call an ambulance.

According to the ARDSnet protocol, tidal volume on a mechanically ventilated patient should be kept between A. 4-6 mL/kg B. 2-4 mL/kg C. 8-10 mL/kg D. 6-8 mL/kg

The correct answer is : A Explanation : According to the ARDSnet protocol, appropriate tidal volume is between 4-6 mL/kg.

Which of the following would result in an increase in anatomical airway resistance? A. pulmonary secretions B. autoPEEP C. water build up in the ventilator circuit D. an undersized endotracheal tube

The correct answer is : A Explanation : Airway resistance is increased in the presence of pulmonary secretions and bronchoconstriction. The other examples listed are not anatomically related.

A patient is being monitored with a Swan-Ganz catheter while receiving volume-controlled ventilation. CVP is 1 torr, PAP is 19/6 mmHg, CO is 3.8 L. The respiratory therapist should recommend A. lactated ringers B. positive inatropic cardiac medication C. administration of Verapamil D. Lasix (furosemide)

The correct answer is : A Explanation : All hemodynamic values are below normal. This is indicative of hypovolemia. To correct this dehydration the patient needs fluid. Of the answers offered, administration of lactated ringers is the best option.

A COPD patient who receives 2 lpm continuous oxygen therapy by nasal cannula is exercising in conjunction with a monitored pulmonary rehabilitation program. The patient has begun breathing quickly and deeply. To ensure consistent arterial oxygenation, the respiratory therapist should A. increase oxygen flow rate B. use a nonrebreathing mask C. use a partial rebreathing mask D. decrease oxygen flow to 1 L/min

The correct answer is : A Explanation : Although COPD patients should rarely have more than 2 L/min. continuous oxygen, when they exercise and breathe more deeply, they inadvertently lower their FIO2, especially when on a nasal cannula. In such a case, it is appropriate to increase the flow rate temporarily for the duration of the exercise.

In order to confirm proper placement of an oral endotracheal tube immediately after intubation, the respiratory therapist should do which of the following? A. Order a chest radiogram B. Auscultate breath sounds C. Observe tube markings on the ET tube relative to the patient's teeth D. Observe chest rise and symmetry

The correct answer is : A Explanation : Although one may determine the proper positioning of an endotracheal tube by examining the symmetry a chest movement and by auscultating breath sounds, it is only through a chest x-ray that confirmation of the position of the endotracheal tube may be achieved.

In which of the following situations should an HME be replaced with heated humidity? A. presence of thick secretions B. patient is hyperthermic C. patient has asthma D. afebrile conditions

The correct answer is : A Explanation : An HME should be changed if the patient has thick secretions or is hypothermic. If the patient is hypothermic he or she cannot breathe out enough heat into the exchanger to capture and rebreathe. In that case the device should not be used.

One hour after performing quality control on a blood gas analyzer, a respiratory therapist obtains arterial blood from a patient breathing supplemental oxygen at 2 L/min by nasal cannula. The following blood gas results are available: pH 7.41 PaCO2 39 torr PaO2 190 torr HCO3- 25 mEq/L BE -1 mEq/L The respiratory therapist should A. perform a two-point calibration and repeat the analysis B. replace the Sanz electrode, repeat the analysis C. recommend discontinuing supplemental oxygen based on these results D. report the blood gas results

The correct answer is : A Explanation : An estimation of the alveolar oxygen tension reveals that on 2 L/min the maximum oxygen tension in the alveolo is about 147 mmHg. However, the oxygen tension in the blood is high at 190 mmHg, according to the blood gas results. This would be impossible. Arterial oxygen tension cannot exceed the maximum possible oxygen tension in the alveoli. The blood gas instrument should be recalibrated and the analysis should be repeated.

A 75-kg (165-lb) patient with mycoplasma pneumonia is receiving mechanical ventilator support by a PB 840 ventilator on the following settings with corresponding arterial blood gas values: Mode assist/control Mandatory rate 10/min VT 525 mL FIO2 0.6 pH 7.29 PaCO2 51 mmHg PaO2 78 mmHg HCO3- 23 mEq/L BE -1 mEq/L

The correct answer is : A Explanation : An examination of the arterial blood gas reveals that this patient is under-ventilating and is under-oxygenating. In such a case, ventilation should be addressed prior to oxygenation. Since carbon dioxide is too high and is off by more than 4 mmHg from the target, the most suitable action is to increase the rate. Increasing the tidal volume may also be helpful but the patient's current tidal volume is already near the top of the range. Additionally, adjustment of tidal volume should be reserved for changes of CO2 that are less than 4 mmHg.

Immediately after obtaining blood from an indwelling radial artery catheter, the respiratory therapist notices arterial line pressure dampening on the monitor. Which of the following could be the cause of the dampening? A. air bubbles in the line B. blood clotting in the arterial line C. inappropriate transducer level D. hypotension

The correct answer is : A Explanation : An indwelling arterial catheter, also known as an ART line, can become clotted with blood or develop air bubbles in the transducer. Both of these problems would result in pressure dampening.

The resulting flow-volume loop of a patient with pulmonary obstruction following a bronchodilator shows an inflection point on the positive side of the graph that is 25% taller than the loop seen prior to the bronchodilator trial. This would indicate that A. patient would benefit from bronchodilators B. patient may have taken a bronchodilator prior to the test C. study should be repeated D. vital capacity increased by 25% with a bronchodilator

The correct answer is : A Explanation : An inflection point is a significant change in direction seen in various types of graphic depictions. In this case the inflection point represents the top or "peak" of the flow volume loop. This point is peak flow. An increase in peak flow by 25% is well above the 12% threshold that is considered significant, indicating bronchodilator therapy.

A respiratory therapist reviews the medical record of a patient and notes the HbCO is 10%. This data was collected 1 hour prior and the patient is asymptomatic. From these results, the respiratory therapist can conclude the patient most likely A. is a heavy smoker B. attempted to commit suicide C. has pulmonary fibrosis D. has an exhaust leak into the cabin of the vehicle

The correct answer is : A Explanation : And HbCO of 10% is much higher than normal and indicates the patient likely smokes tobacco.

A COPD patient is receiving oxygen at 2 L/min by a pulse-dose oxygen delivery device. The patient complains of shortness of breath and feels she is not getting enough oxygen. The home care respiratory therapist should A. switch to a continuous oxygen regulator and normal nasal cannula B. increase the flow rate C. encourage the patient to relax and call her physician D. switch to a Venturi mask at 40%

The correct answer is : A Explanation : Any time a patient is having difficulty or complains that they are not receiving enough oxygen through their oxygen delivery device, the first step is to ensure that they are receiving enough oxygen by changing or replacing the device immediately. In this case the patient is using a pulse dose oxygen delivery device. On these devices there is an option to switch to continuous flow. That is the best option until the problem can be determined and solved. Another good option would be to have the patient use a standard oxygen cylinder while the respiratory therapist troubleshoots and solves the problem.

A COPD patient is receiving oxygen at 2 L/min by a pulse-dose oxygen delivery device. The patient complains of shortness of breath and feels she is not getting enough oxygen. The home care respiratory therapist should A. switch the pulse-dose device to continuous mode B. encourage the patient to relax C. increase the flow rate D. switch to a Venturi mask at 40%

The correct answer is : A Explanation : Any time a patient is having difficulty or complains that they are not receiving enough oxygen through their oxygen delivery device, the first step is to ensure that they are receiving enough oxygen by changing or replacing the device immediately. In this case, the patient is using a pulse dose oxygen delivery device. On these devices there is an option to switch to continuous flow. That is the best option until the problem can be determined and solve. Another good option would be to have the patient use a standard oxygen cylinder while the respiratory therapist troubleshoots and resolves the problem.

A patient has undergone overnight oximetry. Data shows several episodes of oxygen desaturation throughout the night. The respiratory therapist would expect orders for which of the following kind of testing?

polysomnography

A neonate delivered 3 minutes ago has an APGAR score of 2. The therapist should FIRST do which of the following? A. begin cardiac compressions B. obtain umbilical cord blood gas values C. insert a chest tube D. provide routine care

The correct answer is : A Explanation : Apgar scores of 4 to 6 indicate the administration of supplemental oxygen and general stimulation of the infant. Apgar scores higher than six require only routine care of the infant. Apgar scores of 0-3 indicate the need for CPR.

An 80 kg (176 lb) male patient is admitted to the cardiac care unit after sustained complications following cardiac catheterization and angioplasty of several coronary arteries. The patient is receiving oxygen at 100% via bag-valve mask. There are no spontaneous respirations. SpO2 is 97%. Which of the following would be appropriate initial ventilator settings? A. VC, A/C, rate 14, FIO2 1.0, VT 550 mL, PEEP 5 cm H2O B. PC, rate 10, IP 40 cm H2O, FIO2 0.5, I-time 1.0 secL, no PEEP C. VC, SIMV, rate 12, FIO2 0.6, VT 650 mL, PEEP 5 cmH2O D. VC, SIMV, rate 10, FIO2 1.0, VT 400 mL, PEEP 5 cm H2O

The correct answer is : A Explanation : Appropriate tidal volume for this patient is between 6-10 mL/kg of ideal body weight. Rate should be set between 10-20. Because the patient is receiving 100% oxygen, the initial ventilator oxygen setting should also be 100%. Finally, PEEP of 4- 6 cm H2O is appropriate.

A 32-year-old patient with increased work of breathing is on bi-level therapy at an IPAP of 16 cmH2O and EPAP of 8 cmH2O. The following blood gases are reported on these settings: FIO2 0.21 pH 7.32 PaCO2 48 torr PaO2 81 torr HCO3- 24 mEq/L BE 0 mEq/L Which action is most appropriate? A. increase IPAP B. increase EPAP C. increase IPAP and EPAP D. decrease EPAP

The correct answer is : A Explanation : Arterial blood gases show an increase in CO2 which means the patient is not ventilating enough. The patient does appear to be oxygenating adequately. The appropriate change in the non-invasive positive pressure ventilator is an increase in IPAP.

A 60 kg (132 lb) postoperative cholecystectomy patient is receiving mechanical ventilatory support on the following settings: Mode SIMV Mandatory rate 10 Total rate 12 FIO2 0.40 Tidal volume (VT) 450 mL Inspiratory flow 50 L/min Corresponding blood gases show: pH 7.48 PaCO2 32 torr PaO2 92 torr HCO3- 24 mEq/L BE +1 mEq/L Which of the following should the respiratory therapist decrease? A. tidal volume B. FIO2 C. inspiratory flow D. mandatory rate

The correct answer is : A Explanation : Arterial blood gases show this patient is hyperventilating just slightly. To correct hyperventilation, rate may be increased, tidal volume may be decreased, or dead space may be added. In this case, because CO2 is off by only 3 torr compared to the target range, decreasing tidal volume is most appropriate. This is also the most appropriate option because the current tidal volume is at the top of the range for the patient and the patient would benefit from having a tidal volume closer to the center of the range.

A patient who has been receiving negative pressure ventilation at home is admitted to the intensive care unit, intubated, and placed on positive pressure ventilation. What should be monitored closely as a result of the positive chest pressure? A. urinary output B. glucose C. electrolytes D. BUN

The correct answer is : A Explanation : As positive pressure ventilation increases, venous return may be decreased. This may result in a decrease in urinary output. Therefore monitoring urinary output is appropriate when concerned about the negative effects of positive pressure ventilation.

What will occur if a gas that is saturated with 100% relative humidity with no humidity deficit at 33.4 deg C (92.1 deg F) cools to 25.1 deg C (77.2 deg F)? A. relative humidity will remain 100%, rainout will occur B. relative humidity will drop by about 25% C. a humidity deficit will develop D. evaporation will decrease the humidity deficit

The correct answer is : A Explanation : As the temperature of any gas decreases, its ability to hold water also decreases. Therefore, a gas that has a relative humidity of 100% that then undergoes a temperature decrease must eject the excessive water. This is called condensate or rainout. This phenomenon can be seen commonly in the morning when there is dew found on plants and objects.

Which of the following would result in a decrease in AutoPEEP? A. decrease in mandatory rate B. decrease in inspiratory flow rate C. decrease in expiratory time D. increase in inspiratory time

The correct answer is : A Explanation : AutoPEEP is caused when the expiratory time is insufficient. In other words, the ventilator begins the inspiratory phase before the patient has exhaled completely. There are two controls that will reduce autoPEEP. The first is to increase flow rate. This will decrease inspiratory time thus allowing more time for exhalation and prevent air trapping. Another way to allow more time for expiration is to decrease the rate. In this case this is the best option because increasing the inspiratory flow is not offered.

A 38-year-old male patient is admitted to the ICU with several broken ribs and paradoxical chest movement on the right. Arterial blood gas results show: pH 7.23 PaCO2 61 torr PaO2 54 torr HCO3- 26 mEq/L BE 0 mEq/L FIO2 0.60 The physician orders the institution of mechanical ventilation. The patient is (6 ft 2 in) 188 cm tall and weighs 79 kg (172 lb). The respiratory therapist will recommend which of the following initial settings? A. Rate 12, Vt 550 mL, FIO2 0.60, PEEP 5 cm H2O B. Rate 14, Vt 600 mL, FIO2 0.80, PEEP 5 cm H2O C. Rate 12, Vt 850 mL, FIO2 0.40, no PEEP D. Rate 16, Vt 450 mL, FIO2 0.70, PEEP 10 cm H2O

The correct answer is : A Explanation : Based on the patient's height and weight, the most appropriate tidal volume is between 500 and 800 mL. Appropriate rate is between 10-12 per minute. Appropriate FIO2 is between 30-60%. PEEP should start between 4-6 cm H2O.

Quality control data for an arterial blood gas analyzer is plotted on a graph that has a range of 4 standard deviations (2 SD up and 2 SD down). Points on the graph show a gradual rise from below the mean to above the mean. All points are within 2 SD of the mean. The respiratory supervisor should A. monitor this machine closely B. replace the electrodes C. call the medical director for direction D. remove this machine from service

The correct answer is : A Explanation : Because all points on the blood gas graphs are within two standard deviations of the mean, the blood gas machine is technically in control. But the use of the word gradual rise means that there is a trend upward. Whenever there's a trend you need to monitor that machine because the trend may ultimately rise above the two standard deviations. You do not need to remove the machine from service or perform any kind of maintenance when a trend is observed - only monitor.

A patient has the following pulmonary function test results: SVC 4.2 L FVC 4.3 L Fev1 3.1 L Fev1/FVC% 72% DLCO 22 mL/CO/min/mm Hg The respiratory therapist should conclude

poor patient effort on the SVC.

A 36-week gestational age infant delivered 2 hours prior shows signs of ventilatory distress including grunting. In preparation for oxygen administration at 40% by oxygen hood, the therapist should utilize which of the following devices? A. aerosol heater B. isolette C. bubble humidifier D. heat-moisture exchanger

The correct answer is : A Explanation : Because maintaining adequate body temperature in an infant is paramount, inspired gases should be heated and humidified. This can best be accomplished through the use of an aerosol heater.

During morning ventilator rounds, the respiratory therapist observes the following results of an arterial blood gas done 1 hour prior with the patient receiving volume-controlled ventilation. pH 7.53 PaCO2 39 torr PaO2 85 torr HCO3- 19 mEq/L BE +8 mEq/L FIO2 0.45 SaO2 97% The respiratory therapist should suspect an error in recording which of the following values? A. HCO3- B. pH C. BE D. PaCO2

The correct answer is : A Explanation : Because the arterial blood gas results show a PaCO2 of 39 mmHg but the pH shows alkalosis, the problem must be metabolic rather than respiratory. This would mean that the bicarbonate must be very elevated. However, when we look at the bicarbonate it is nearly normal at 26 mEq/L. This would suggest an error in the recording of the HCO3-.

After performing minimum seal technique on an 80-kg (175-lb) patient who is orally intubated with an 8.0-mm ET tube, the cuff pressure is measured by manometer and found to be 36 cmH2O. Which of the following can best explain this? A. transesophageal fistula B. ruptured cuff C. tracheal stenosis D. tracheomalacia

The correct answer is : A Explanation : Because the minimum seal technique was used (also called minimum occluding volume), we know that the cuff is touching the tracheal tissues. However, the pressure required to do so is far above the normal pressure. This would indicate that the space inside the trachea is larger than normal. The cause of this is most likely a transesophageal fistula.

A respiratory therapist desires to modify the therapy of a patient who is receiving 2.5 mg of Albuterol via mouthpiece every 2 hours via small volume nebulizer because the patient has experienced cardiac palpitations and a headache. What is the appropriate modification in this scenario? A. Discontinue therapy. B. Change to 1.25 mg Proventil. C. Reduce the frequency. D. Reduce the dosage.

The correct answer is : A Explanation : Because the patient has not tolerated the therapy, the respiratory therapist should discontinue it and notify the ordering physician. A note about the patient's tolerance should also be made in the medical record.

A patient who was intubated for pneumonia and is receiving VC SIMV ventilation is ordered for bronchial hygiene assistance through high-frequency chest wall oscillation. Which of the following may also help the patient clear secretions? A. endotracheal suctioning B. teaching huff cough technique C. quad coughing D. serial coughing

The correct answer is : A Explanation : Because the patient is intubated, endotracheal suctioning must be employed to remove secretions once other methods have helped mobilize secretions from the lower distal areas up to the mainstem bronchus and trachea. Huff coughing, serial coughing, and Quad coughing are best done in non-intubated patients.

For which of the following responses to therapy should a respiratory therapist discontinue aerosolized albuterol? A. bleeding gums B. heart rate increases by 10 beats per minute C. patient reports no change in dyspnea D. poor cooperation by a 2-year-old patient

The correct answer is : A Explanation : Bleeding from the mouth, nose, or gums is a rare but serious symptom of an allergic response to albuterol.

Arterial blood gas results are obtained from a spontaneously breathing patient: pH 7.23 PaCO2 58 torr PaO2 74 torr HCO3- 24 mEq/L BE -1 mEq/L Which of the following is true regarding the interpretation of these results? A. acute respiratory acidosis B. compensated metabolic alkalosis C. metabolic acidosis D. compensated respiratory acidosis

The correct answer is : A Explanation : Blood gases reveal an elevated CO2 and an appropriate corresponding pH of 7.23. HCO3 is normal. Therefore, there is not metabolic compensation and the most appropriate interpretation is acute respiratory acidosis.

A patient with COPD is on non-invasive positive pressure ventilation to decrease work of breathing. Current settings and blood gas results are: IPAP 15 cm H2O EPAP 8 cm H2O FIO2 0.28 pH 7.31 PaCO2 65 torr PaO2 62 torr HCO3- 27 mEq/L BE +3 mEq/L The respiratory therapist should make which of the following changes? A. increase IPAP to 18 cmH2O B. decrease EPAP to 6 cmH2O C. increase both IPAP and EPAP by 4 cmH2O D. increase EPAP to 12 cmH2O

The correct answer is : A Explanation : Blood gases reveal the patient is hypoventilating and under oxygenating. When we have a problem with both ventilation and oxygenation, we should attempt to correct ventilation first. This is because the hypoxemia may be a result of the hypoventilation. To correct ventilation, in this case, we need only to raise IPAP.

Which of the following should be prevented with the proper application of cricoid pressure during manual bag/mask ventilation prior to intubation? A. gastric insufflation B. tachycardia C. pneumothorax D. right mainstem intubation

The correct answer is : A Explanation : By applying proper cricoid pressure, the rigid rings of the trachea are indirectly pressed down over the esophagus, which blocks air from entering the stomach during mask ventilation, and further prevents gastric contents from entering the oropharynx.

An emergency room physician orders an arterial blood gas with Carboxyhemoglobin level. This exam will help the physician determine which of the following? A. CO poisoning B. Presence of sickle cell anemia C. Presence of fetal hemoglobin D. Carbon dioxide serum levels

The correct answer is : A Explanation : Carboxyhemoglobin (COHb) levels are determined with an instrument called a co-oximeter, which directly measures arterial blood COHb. Both carbon monoxide poisoning and evidence of smoking, or exposure to cigarette smoke can be determined with the COHb level.

The most reliable test to determine if a pulmonary rehab patient has stopped smoking is a A. carboxyhemoglobin level B. treadmill trial C. methemoglobin level D. complete PFT

The correct answer is : A Explanation : Carboxyhemoglobin, as measured by a co-oximeter, is the most reliable test for determining levels of carbon monoxide created by smoking.

A physician suspects a patient has a pleural effusion. Which of the following radiography procedures is helpful in diagnosing the problem? A. PA chest radiograph B. AP chest radiograph C. apical lordotic radiograph D. lateral decubitus radiograph

The correct answer is : D Explanation : Pleural effusion is diagnosed through direct observation of a chest radiograph, specifically, the lateral decubitus x-ray.

Which of the following equations would determine cardiac index? A. C.O. / BSA B. stroke volume x heart rate C. C.O. / ejection fraction D. C.O. x BSA

The correct answer is : A Explanation : Cardiac index can be calculated by dividing the cardiac output by the body surface area. The body surface area is given in units of millimeters squared. Body surface area rarely deviates from about 2 m², regardless of the size of the patient. So essentially, we could take cardiac output and divide it by two to get cardiac index. This can be done even if the body surface area of the patient is unknown. For example, if the cardiac output is 4 L/min, the cardiac index is about 2. Conversely, if cardiac index is 1.8, cardiac output could be estimated to be 3.6 L/min.

Which of the following equations would determine cardiac index? A. (stroke volume x heart rate ) / BSA B. C.O. x ejection fraction C. stroke volume x heart rate D. C.O. / stroke volume

The correct answer is : A Explanation : Cardiac index can be determined by taking the cardiac output and dividing it by the body surface area. In these equations cardiac output divided by body surface area is not offered. However if you look closely stroke volume times heart rate is another way of calculating cardiac output therefore stroke volume multiplied by the heart rate divided by the body surface area will achieve cardiac index.

A bronchoscope has been used on a patient with a staphylococcus infection. To properly sterilize the equipment, perform a A. 15 minute soak in 13-day-old Cidex B. steam autoclave C. 1 hour soak in 30-day-old Sonacide D. 3 minute soak in one-day-old Sonacide

The correct answer is : A Explanation : Cidex, or alkaline glutaraldehyde, is most commonly used for bronchoscopy sterilization. A 10-hour soak is best, but all bacteria will be destroyed within 10 min. In this example, the primary concern is Staphylococcus, a bacteria. Alkaline glutaraldehyde must be changed no later than every 14 days.

After 15 seconds following birth, a neonate presents with vigorous crying and a strong cough. Obvious secretions and amniotic fluids are present from the airway and stomach. The baby has peripheral cyanosis. You would FIRST A. suction the mouth. B. check the heart rate. C. suction the nose. D. provide blow by oxygen.

The correct answer is : A Explanation : Clearing the oropahrynx or mouth of a newborn is a first step to prevent the baby from aspirating the secretions. Following the suctioning of the mouth, one should also gently suction the nose.

A patient with cystic fibrosis is receiving oxygen via a nasal cannula set at 5 L/min become somewhat unresponsive and confused. SpO2 is 99%. The therapist should recommend which of the following as a first action? A. Lower the oxygen flow rate B. Begin mechanical ventilation C. Increase the oxygen flow D. Switch to a non-rebreathing mask

The correct answer is : A Explanation : Cystic fibrosis is a disease associated with chronic obstruction and is therefore considered to be COPD. Patients with COPD should not receive more than 1-2 liters per minute by nasal cannula or more than 28% oxygen by any device. Excessive oxygen delivery can lead to suppression of the ventilatory drive and result in unresponsiveness and confusion. An oxygen saturation of 94% or less should be maintained.

Which of the following tests would be most helpful at diagnosing a patient with cystic fibrosis? A. sweat chloride test B. pulmonary function studies C. calorimetry D. metabolic studies

The correct answer is : A Explanation : Cystic fibrosis is primarily diagnosed with a simple test called a sweat chloride test.

The following arterial blood gas values are reported for a patient with emphysema receiving 3 L/min nasal cannula: Day 1 Day 2 Day 3 Day 4 pH 7.39 7.35 7.31 7.27 PaCO2 (torr) 55 57 59 61 PaO2 (torr) 55 60 65 70 What is the appropriate action? A. reduce O2 to 2 L/min B. calibrate the blood gas analyzer C. place on venture mask at 30% D. maintain current therapy

The correct answer is : A Explanation : Days 3 and 4 show that the patient's PO2 is increasing toward a normal range but that CO2 is climbing above the normal level for the patient. This is best manifested starting on day three with a low pH. This data suggests that the patient is receiving too much oxygen. Oxygen should be reduced to 2 L/min.

For a patient receiving volume-controlled ventilation, which of the following will not results in increased mean airway pressure? A. decrease in mandatory rate B. decrease in inspiratory flow rate C. use of an expiratory retard D. use of inspiratory pause

The correct answer is : A Explanation : Decreasing inspiratory flowrate, use of an expiratory retard, and use of inspiratory pause will all cause an increase in mean airway pressure. Decreasing rate will lower mean airway pressure.

The respiratory therapist records a patient's blood pressure to be 130/90 mm Hg as measured by a sphygmomanometer. Simultaneously, the therapist notes the blood pressure measured by an indwelling radial artery catheter and transducer is reading 105/50 mm Hg. Which of the following could explain the difference in these results? A. blood clots in the indwelling catheter B. the transducer is 18 inches below the level of the heart C. erratic movement of the arm with the arterial catheter D. improper reading of the sphygmomanometer

The correct answer is : A Explanation : Discrepancies between blood pressure taken by sphygmomanometer and by an arterial line (indwelling arterial catheter) are usually associated with clots or air bubbles in the ART line.

While performing an emergency oral intubation, the respiratory therapist discovers the light on a laryngoscope blade is not coming on. The first action of the therapist should be to A. attempt to tighten the bulb B. replace the handle C. replace the batteries D. use a small flashlight with another's assistance

The correct answer is : A Explanation : During an emergency intubation, a faulty laryngoscope light should be dealt with by first attempting to tighten the bulb. If that is ineffective, the next step is to replace the blade. Finally, the batteries in the handle of the laryngoscope should be replaced. The proper order of these corrections is related to the time that they take to perform. In an emergency setting the quickest option should always be attempted first.

A patient presents to the emergency room with fulminating pulmonary edema. Which of the following would be experienced with this emergency? A. elevated left ventricular filling pressure with low QT B. hypovolemia C. a good response to supplemental O2 D. CVP 10 torr and PAP 10 torr

The correct answer is : A Explanation : Fulminating pulmonary edema is a serious condition often resulting from left-sided heart failure. Left ventricular filling pressure, a.k.a. pulmonary capillary wedge pressure (PCWP), would likely be elevated while cardiac output (QT) would drop. The back pressure or congestion would become relieved in the lungs, creating severe pulmonary congestion.

Following a full cardiopulmonary arrest and successful resuscitation, the patient has received dopamine to raise blood pressure. The respiratory therapist his having difficulty obtaining arterial blood from the radial artery due to hypotension. From which of following sites should the therapist attempt to perform an arterial puncture? A. femoral artery B. umbilical artery C. pedal artery D. carotid artery

The correct answer is : A Explanation : During cardiopulmonary resuscitation blood pressure exists because of cardiac compressions and is usually low. Palpation of pulse, therefore, is difficult in the usual locations such as the radial or brachial arteries. When blood gases are needed during a code with a patient who has very low blood pressure, the femoral artery is the location of choice.

A post-operative female patient with atelectasis is ordered to receive SMI every hour while awake. A respiratory therapist instructs the patient to self-administer a single-patient-use, volume-oriented incentive spirometer with a flow rate sensor. The volume column reaches 1250 mL, 50% of her predicted inspiratory capacity. The flow sensor immediately reaches the top of the column regardless of the patient's varying flowrates. You would A. replace the spirometer. B. continue therapy with the spirometer. C. coach the patient to observe the volume column only. D. send the spirometer to central for cleaning.

The correct answer is : A Explanation : Even though a respiratory therapist may believe that malfunctioning equipment is working "well enough", it is never appropriate to continue using broken or malfunctioning equipment. In this case the incentive spirometer should be replaced because it is not working properly.

A 183-cm (6-ft), 87-kg (192-lb) male is receiving PC, A/C ventilation on the following settings: PEEP 8 cm H2O Set inspiratory pressure 18 cm H2O Mandatory rate 18 Total rate 18 FIO2 0.60 I:E 1:2 VT (exhaled) 350 mL Flow 50 L/min ABG are as follows: pH 7.28 PaCO2 52 torr PaO2 119 torr HCO3- 20 mEq/L BE +1 mEq/L A respiratory therapist should recommend an increase in which of the following? A. inspiratory pressure B. deadspace ventilation C. inspiratory flow rate D. FIO2

The correct answer is : A Explanation : Examination of this blood gas reveals acidosis secondary to hypoventilation. Because the patient is receiving pressure control ventilation, the primary method for controlling arterial carbon dioxide is through adjustments of the inspiratory pressure. In this case, inspiratory pressure should be increased to blow off CO2.

Which of the following is a significant indicator of negative effects from positive pressure ventilation? A. decrease urine output B. drying of pulmonary secretions C. increased alveolar distention D. increase in physiological deadspace

The correct answer is : A Explanation : Excessive pressures from positive pressure ventilation will first and foremost cause decreased venous return. Of the options offered the next most significant response is decreased urine output.

A 31-week-gestional age infant has a respiratory rate of 30/min and a heart rate of 70/min while breathing room air. Five minutes after birth, the infant is acrocyanotic and has a weak cry. Following suctioning of the oropharynx the respiratory therapist should A. provide manual ventilation with the bag-valve B. begin chest compressions C. place the infant in an oxyhood at FIO2 0.40 D. administer Atropine sulfate

The correct answer is : A Explanation : Five minutes after birth this infant is still showing signs of significant respiratory distress. Supporting ventilation with a bag-valve is appropriate.

BID is a common frequency order for which of the following respiratory medications? A. Flovent B. Ativan C. Albuterol D. Xopenex

The correct answer is : A Explanation : Flovent (fluticasone) is a corticosteroid used generally to prevent and relieve inflammation of airway walls. Inflammation is one of the components of asthma so corticosteroids are key in treatment. The frequency is twice per day or b.i.d. Flovent may cause Candidiasis, (oral yeast infection), which may be prevented by judiciously rinsing the mouth after inhaler use.

Which of the following is an appropriate goal for a patient in pulmonary rehabilitation who is in the advance stages of pulmonary emphysema? A. reduced hospitalizations B. return the patient to normal life C. normal pulmonary function values D. reduction of ADLs

The correct answer is : A Explanation : For a patient in pulmonary rehabilitation, appropriate goals include: increasing the quality of life from the patient's point of view, reducing incidence of infections and hospitalizations, and increasing the ability to perform activities of daily living. Because most lung disease is not reversible, returning the patient to "normal life" or "normal pulmonary function values or blood gas values" is not considered a goal of pulmonary rehabilitation.

A 62-year old male presents in the emergency department with the following arterial blood gas results while receiving supplemental oxygen by nasal cannula at 5 L/min. pH 7.53 PaCO2 28 torr PaO2 48 torr HCO3 - 22 mEq/L BE -2 mEq/L The respiratory therapist should conclude which of the following regarding the patient's condition? A. intrapulmonary shunting is occurring B. right-side heart failure is present C. increase in systemic vascular resistance D. metabolic alkalosis is present

The correct answer is : A Explanation : For a question like this, each option must be considered separately to determine if true or false. Only one of these options would be considered true. Examination of the pH and PCO2 shows a respiratory cause of the alkalosis. There is not enough data to indicate right heart failure or any changes in systemic vascular resistance. This leaves the option of intrapulmonary shunting. To determine if a patient is shunting, the FIO2 and PaO2 should be observed and compared. In this case, the patient is receiving 5 L/min by nasal cannula. On this setting PaO2 should be in excess of 100 mmHg. But, this patient is suffering from moderate hypoxemia. Even though the specific A-a gradient is unknown, it can still be assumed that pulmonary shunting is occurring.

Which of the following disease conditions is associated with purulent green sputum, separated into layers and has a foul odor? A. bronchiectasis B. cystic fibrosis C. mycoplasma pneumonia D. rocky mountain spotted fever

The correct answer is : A Explanation : Foul-smelling, green sputum that is described as "separated into layers" is associated with bronchiectasis.

A patient is suspected to have a kidney disease process that causes fluid shifting. Which of the following would help evaluate this process? A. creatinine B. hemoglobin C. Theophylline level D. COHb

The correct answer is : A Explanation : Kidney disease often results in fluid shifting. When fluid shifting is present, as manifested through pitting edema or other clinical signs, the situation may be further evaluated by obtaining a creatinine level. Blood urea nitrogen (BUN) is another test that would be useful in further evaluating this condition, although not as accurate as creatinine.

A patient with cystic fibrosis is receiving mechanical ventilation with an HME inline. Over the past 48 hours, the respiratory therapist notes that secretions are becoming increasingly thick and mucoid. Which of the following should the therapist change? A. Remove the HME, start heated humidification B. Document the secretions and advise frequent changes of the HME C. Report a need to clean the HME every two hours D. Administer N-acetylcysteine (Mucomyst)

The correct answer is : A Explanation : HME devices should only be used in a temporary situation and when the patient does not have significant or thick secretions. The presence of thick secretions is a clear indicator for heated humidification.

A physician suspects a patient is bleeding internally following surgery to repair multiple internal injuries sustained from a motor vehicle accident. Based upon the following data, which of the following represents an accurate estimation of the hemoglobin level? PaO2 94 torr COHb 2.5% HCT 24 SpO2 97% WBC 11,000 cu mm A. 8 gm/dL B. 15 gm/dL C. 45 gm/dL D. 19 gm/dL

The correct answer is : A Explanation : Hemoglobin level may be estimated if HCT or RBC values are known. These three have a relationship. Hemoglobin is roughly 3 x the RBC. HCT is roughly 3 x the hemoglobin. Therefore, if you know HCT you can simply divide by three to get an estimated hemoglobin level. In this case HCT, which is 24, divided by three is an estimated hemoglobin level of 8 gm/dL.

A patient with a history of hyper-reactive airway disease is having difficulty expectorating because the sputum is thick and tenacious. Which of the following medications should the respiratory therapist recommend? A. Acetylcysteine (mucomyst) B. Solu-Mederol C. Beclamethasone (Beclovent) D. Spiriva (tiotropium bromide)

The correct answer is : A Explanation : Hyperactive airway disease is associated with diseases such as asthma and consist of bronchoconstriction and inflammation. This patient appears to have difficulty with bronchoconstriction and thick secretions. Therefore, a mucolytic, such as acetylcysteine, and Solu-Medrol are appropriate.

A patient who is receiving volume-controlled ventilation has the following arterial blood gas and clinical data: Mode Assist/control Rate 14/min FIO2 0.50 (analyzed) VT 500 mL Arterial blood gas analysis was done at standard temperature and pressure (STP). Pressure (H2O) is 47 cm H2O pH 7.40 PaCO2 41 torr PaO2 358 torr HCO3- 25 mEq/L BE +1 mEq/L The respiratory therapist can conclude which of the following? A. The PaO2 result is erroroneous B. There is a fault in the Sanz electrode C. There is increased permeability in the alveolar capillary membrane D. The patient has a significant shunt

The correct answer is : A Explanation : If the patient is receiving 50% oxygen, the maximum oxygen tension in the alveoli is about 300 mmHg. This patient has a blood gas that shows the arterial oxygen level of 358 mmHg, which is impossible. It is impossible for the arterial oxygen level to be greater than the maximum alveolar oxygen tension level. Therefore, this blood gas represents an analyzer error.

The respiratory therapist should recommend which of the following to initially check the placement of a 7.5 mm oral endotracheal tube? A. Auscultation of the chest bilaterally B. Arterial blood gas analysis C. Palpation of the trachea D. Chest radiograph

The correct answer is : A Explanation : In order to check the position of an endotracheal tube there are several methods that may be used. The best method to initially assess the location is to visualize the symmetry of chest movement. The second best method is to auscultate breath sounds. The third best method for initial assessment is a chest x-ray. The idea here is to choose the examination that is the quickest for the initial assessment. In this case a visual assessment is not offered and therefore auscultation of the chest bilaterally is the quickest method for initially determining the location of the endotracheal tube. A chest x-ray is time-consuming but is very confirming. In this case we were not asked for confirming evidence.

In preparing to cap a fenestrated tracheostomy tube on a patient who is scheduled for speech therapy, the respiratory therapist should remember to A. deflate the cuff and remove the inner cannula B. switch to a liquid diet 8 hours prior to speech therapy C. evacuate the foam cuff D. inflate the cuff

The correct answer is : A Explanation : In order to configure a fenestrated tracheostomy tube for speech, the button or cap must be placed on the tube after the inner cannula is removed and the cuff is deflated.

The following clinical information is documented after you assess a home oxygen patient's SpO2 in the following situations. What is the appropriate oxygen treatment plan for home care for this patient? SpO2 #1: room air, resting: 91% SpO2 #2: moderate walk for 150 -ft on room air: 88% SpO2 #3: 1 L/min nasal cannula at rest: 96% SpO2 #4: moderate walk for 150 -ft on 1 L/min nasal cannula: 93% SpO2 #5: 2 L/min Nasal Cannula at rest: 97% SpO2 #6: fast walking for 250 -ft on 2 L/min nasal cannula: 94% A. 1 L/min while walking or exercising B. 1 L/min continuously C. discontinue supplemental oxygen D. 2 L/min continuously

The correct answer is : A Explanation : In order to provide sufficient oxygen for this patient to complete activities of daily living (ADLs), 1 L/min is required. A moderate walk is the highest reasonable activity. It is not necessary to oxygenate the patient at 2 L/min for a fast walk.

Two days prior an 85 kg (187-lb), 168-cm (5-ft 6-in) female patient was placed on mechanical ventilation for acute respiratory failure secondary to pulmonary hypertension. She has the following data after a one-hour spontaneous breathing trial at an FIO2 of 0.40. pH 7.35 PaCO2 45 torr PaO2 75 torr HCO3- 24 mEq/L Respiratory Rate 34 VT 375 ml What should the respiratory therapist do FIRST A. Place the patient on PCV B. Discontinue mechanical ventilation C. Increase FIO2 to 0.50 D. Extubate and provide continuous Albuterol therapy

The correct answer is : A Explanation : In this case we are asked to assess whether or not a patient is successfully weaning during a trial period. The blood gases are generally acceptable along with tidal volume, however, the respiratory rate is too high and the patient has failed the trial. Therefore, the only option that returns the patient to mechanical ventilation is to place the patient on PCV. The therapist may be uncomfortable with this mode of ventilation, but it is the best choice because it addresses ventilation, the first of the vital functions of life.

A 31-year-old female exhibits a stuffed-up nose, a periodic cough, and bouts of dyspnea. She indicates the symptoms return every spring. Which of the following drug classifications should the respiratory therapist recommend to help control the patient's symptoms? A. leukotriene inhibitor B. IgE immunoglobulin agonist C. beta I agonist D. Mucolytic

The correct answer is : A Explanation : Leukotriene medication is helpful in long-term control of inflammatory processes in the upper and lower airways.

The following information is available on the ventilator flow sheet of a 55-year old patient who is 3 days post appendectomy. Following the procedure, the patient sustained a pneumothorax and has been recovering in the intensive care unit. 8:00 AM 10:00 AM 12:00 PM Peak Pressure (cm H2O) 25 35 36 Plateau Pressure (cm H2O) 18 23 22 PEEP (cm H2O) 5 10 10 VT (mL) 500 500 500 The therapist can conclude which of the following is present? A. increasing airway resistance, no change in pulmonary compliance B. pulmonary stiffening, decreasing dynamic compliance C. increased pulmonary elastance D. decreasing pulmonary compliance, increasing airway resistance

The correct answer is : A Explanation : In this data, increased airway resistance is manifested by an increase in peak pressure. It is tempting to assume decreased pulmonary compliance because plateau pressure is also rising. However when one looks closely, it is noted that PEEP has undergone a change during the study, which accounts for the change in plateau pressure. This will also accordingly change the peak inspiratory pressure but this data shows an increase greater than the amount of increase in the PEEP and therefore airway resistance is increasing.

Which device is most appropriate to send to the central processing department for sterilization after use on an infectious patient? A. Battery-powered laryngoscope handle B. Infant ventilator C. Stylet D. Fiber optic laryngoscope blade

The correct answer is : A Explanation : In this example the laryngoscope handle is an electronic device and may be damaged with customary soaking in glutaraldehyde. A fiber-optic laryngoscope blade and a reusable stylet can both be soaked in a glutaraldehyde solution. An infant ventilator cannot be soaked or sent for sterilization. A ventilator is wiped down. The use of filters prevents contamination of the inner parts.

A patient with CHF has the following input/output history Yesterday Today IN 2800 mL 2300 mL OUT 1200 mL 1100 mL This data is most consistent with A. hypervolemia B. decreased CVP C. increased left end-diastolic pressure D. hypovolemia

The correct answer is : A Explanation : In this problem it is obvious over the course of two days that the patient is taking in more fluid than he is putting out. This must mean the patient is retaining fluid and is therefore hypervolemic.

A 75-kg (165-lb) male drug overdose patient is receiving mechanical ventilation by a volume-controlled ventilator in the assist/control mode on the following settings: Mandatory rate 16 VT 500 mL FIO2 0.6 PEEP 5 cm H2O ABG results: pH 7.47 PaCO2 33 mm Hg PaO2 85 mm Hg HCO3- 24 mEq/L BE 0 mEq/L Which of the following represents an appropriate action? A. add 100 mL of deadspace B. increase rate to 18 C. decrease VT to 400 mL D. decrease rate to 14

The correct answer is : A Explanation : In this problem, the CO2 is slightly low. CO2 can be raised by one of three different methods - decreasing rate, decreasing tidal volume, and adding dead space. In this case, adding 100 mL of dead space is most appropriate because the change we need to make in CO2 is small. Decreasing the tidal volume would also be appropriate except that the patient's current tidal volume is already at the low end of the range. Decreasing the tidal volume to 400 mL would place it outside the range which is inappropriate for the patient based on their ideal body weight. Decreasing rate from 16 to 14 is too drastic and would result in a CO2 change that is too large. Increasing the rate would further decrease the CO2, which is a move in the wrong direction.

A 70-kg (154-lb) male patient is being weaned from mechanical ventilation. The following spontaneous breathing trial data is recorded in the patient's record. 1 hr 2 hr 3 hr 4 hr VT (mL) 450 420 320 280 VC (L) 1.8 1.7 0.9 0.7 MIP (cm H2O) -34 -33 -24 -20 SpO2(%) 92 92 87 85 Which of the following is an appropriate plan for the next breathing trail? A. trial should not exceed 2 hours B. 5 hour trial C. trial should not exceed 3 hours D. 6 hour trial

The correct answer is : A Explanation : In this question the patient is undergoing T-piece trials, which is a form of weaning from mechanical ventilation. After one hour and two hours the patient appears to be stable. But, by the third and fourth hour the patient appears to be unable to sustain adequate ventilation and oxygenation. This indicates that future trials should not exceed two hours.

A post-op adult patient achieved an inspiratory capacity of 1800 cc with an incentive spirometer prior to surgery. After the surgery, the patient appears to be confused on how to use the incentive spirometer. The therapist notes the patient is having difficulty getting the balls to rise. The therapist should A. switch to a volume-type device B. assess the patient for the cause of the confusion C. administer oxygen at 5 L/min nasal cannula D. coach the patient more diligently

The correct answer is : A Explanation : Incentive spirometry comes in two types: a flow type, which requires keeping balls floating, and the volume type, which simply measures inspiratory capacity. The volume type of incentive spirometer is considered to be the easiest. It is appropriate to switch to a volume spirometer when the patient is having difficulty with a flow type spirometer.

A patient receiving volume-controlled ventilation has thick secretions that are difficult to suction. The patient has an 8.0-mm oral endotracheal tube and is being suctioned with a 10 Fr catheter at a pressure of 100 mmHg. Which of the following would be most helpful at increasing the efficiency of suctioning? A. switch to a 12 Fr suction catheter B. increase suction pressure to 110 mmHg C. increase suction time and frequency D. increase suction pressure to 120 mmHg

The correct answer is : A Explanation : Increasing suction efficiency can be most improved by increasing the size of the suction catheter. However, the diameter of the suction catheter cannot be greater than one half of the internal diameter of the endotracheal tube. The patient currently has an 8.0 mm ET tube in. One half of 8 mm is 4 mm. We then must multiply it times three to obtain French units. 4x3 is 12 French. This means the maximum catheter size that may be used is 12. The current size being used is 10 French. This means we may benefit from increasing the catheter size from 10 to 12. This change is far more effective than increasing suction pressure or suction time.

Immediately after oral intubation of an apneic patient, the respiratory therapist begins manual ventilation with a bag-valve assembly. The patient is simultaneously connected to an end-tidal carbon dioxide monitor. Which of the following should the respiratory therapist expect to observe when looking at the capnographic waveform? A. first a rise, then a fall in CO2 B. stair-step shifts in the CO2 tracing C. steady CO2 reading D. fall in CO2 followed by a subtle rise

The correct answer is : A Explanation : Initial ventilation attempts after a patient has been apneic will show reduced end-tidal CO2 followed by a steady rise as ventilation is provided. Ultimately, as ventilation continues the end-tidal CO2 will begin to decrease.

Which of the following weaning parameters would NOT be useful in determining whether the patient is ready for cessation of mechanical ventilation for a 75-kg (165-lb) male patient? A. MVV of 75 L in 15 seconds B. Vital capacity of 650 mL C. Spontaneous tidal volume of 375 mL D. PaCO2 increase of 8 torr during weaning trial

The correct answer is : A Explanation : MVV, or maximum voluntary ventilation is a test completed during a complete pulmonary function evaluation. It is not a part of weaning parameters assessed on mechanically ventilated patients. It is also a test that should be completed on patients who are otherwise healthy and in their best condition, not a critically ill patient receiving mechanical ventilation. The other tests are appropriate to assess when deciding to terminate mechanical ventilation.

A patient with suspected laryngotracheobronchitis is demonstrating moderate stridor with a barking cough. Which of the following should the respiratory therapist consider? A. administration of an aerosolized decongestant B. heated bland aerosol by mask C. aerosolized Lidocaine by mask D. nebulized epinephrine

The correct answer is : A Explanation : Moderate stridor may be treated with racemic epinephrine. This drug is classified as a decongestant. This alpha-1 medication can constrict vessels and tissues and reduce inflammation. If stridor is considered to be severe or marked, racemic epi not the best choice. An airway should be established immediately. This action is preferred over racemic epinephrine because of the time it takes for the medication to have effect. Therefore, racemic epinephrine should be used in cases involving mild and moderate stridor.

A 19-year old patient with mucoviscidosis will be travelling alone to various foreign countries for two months. The patient normally receives daily assistance with bronchial hygiene by chest percussion from family members. Which of the following can the respiratory therapist recommend to facilitate bronchial hygiene during travel? A. vibratory PEP therapy B. IPPV C. VEST therapy D. quad coughing

The correct answer is : A Explanation : Mucoviscidosis is another name for cystic fibrosis, which is a condition known for excessive sputum production. For this patient who would be traveling to foreign countries for an extended period of time, bronchial hygiene that can be performed easily by oneself and does not require complex equipment or another person's help, is imperative. VEST therapy can be complex, expensive, and may require specific electrical connection that may not be available in other countries. Quad coughing requires the help of another and is therefore inappropriate.

A patient experiencing exacerbation of asthma is receiving small volume nebulizer treatments with albuterol via aerosol mask. The patient suddenly complains of nausea, tingling of the digits and seems very anxious. The best course of action would be to A. discontinue therapy and report your findings to the doctor B. instruct the patient to breathe less deeply during the treatment C. continue current therapy D. change to a mouthpiece

The correct answer is : A Explanation : Nausea, tingling of the digits, and anxiousness indicate that the patient is not tolerating the medication being delivered. The therapy should be discontinued and the physician should be notified.

The respiratory therapist observes an ECG wave form on a patient that is consistent with atrial tachycardia. The patient is complaining of chest pain, dizziness, and nausea. The respiratory therapist should recommend A. sychronized defibrillation B. unsynchronized defibrillation C. Atropine sulfate D. epinephrine

The correct answer is : A Explanation : Non-deadly arrhythmias, such as this one, may be addressed through cardioversion, also called synchronized defibrillation. Cardioversion is a form of defibrillation with low wattage and with the synchronization set to "active". This allows the shock to be sychronized to the R wave.

The laboratory results of a sputum culture and sensitivity have returned for a patient who has bilateral bacterial pneumonia. The culture reveals streptococcus, a gram-positive bacteria. The medical records indicates the patient is allergic to penicillin. Which of the following should the respiratory therapist recommend? A. Cephalexine (Keflex) B. Methacillin C. Nafcillin D. Amoxicillin

The correct answer is : A Explanation : Normally gram-positive bacteria may be killed by penicillin-type antibiotics. But, because the patient is allergic to penicillin, a suitable drug is cephalexine. Nafcillin and methacillin are suitable antibiotics when a patient is penicillin-resistant but not when they are allergic.

While measuring the cuff pressure of a patient on a ventilator, the respiratory therapist notes a pressure of 43 mmHg. Upon auscultation of the neck, significant air can be heard passing around the cuff at peak airway pressure. The therapist should respond by A. recommending replacement of the endotracheal tube B. decreasing cuff pressure to less than 20 mmHg C. adjusting the patient's position in bed D. adding air to the cuff

The correct answer is : A Explanation : Normally, an endotracheal tube cuff can seal in the trachea at around 16 to 20 mmHg pressure. In this case it is taking 43 mmHg of pressure while still showing evidence of air leaking around the cuff. The cuff is not touching the tracheal wall. Together, these two data suggests an endotracheal tube cuff that is too small. Replacing the endotracheal tube with a larger size is most appropriate.

During the assessment of a 3-year-old child, the respiratory therapist auscultates wheezing over the right lung field with normal breath sounds heard over the left. This finding is most consistent with A. foreign body aspiration B. croup C. unilateral bronchoconstriction D. diaphragmatic hernia

The correct answer is : A Explanation : Normally, bronchial constriction results in bilateral wheezing. The presence of wheezing on only one side of the lungs is most often related to foreign body aspiration.

An infant on a time cycled, pressure limited ventilator is showing signs of distress. Inspiratory pressure is set to 30 cm H2O with a pop-off set at 35 cm H2O. Close observation reveals the pressure manometer on the ventilator is only rising to 20 cmH2O with each breath. Which of the following changes should the therapist recommend? A. increase the flow B. decrease the pressure pop-off C. switch to a volume-cycled ventilator D. decrease I:E ratio

The correct answer is : A Explanation : On a time-cycled ventilator, when the set inspiratory pressure is not being reached it is likely due to an inadequate flow.

The results of a spinal tap on patient complaining of flu-like symptoms and sluggish lower extremities reveals significant presence of protein. Which of the following is associated with these findings? A. Guillain-Barre Syndrome B. Salmonella poisoning C. Muscular distrophy D. Myasthenia gravis

The correct answer is : A Explanation : Significant proteins found in the spinal fluid indicate a diagnosis of Guillain-Barre' Syndrome.

While transporting a patient from a helicopter landing pad to the emergency department (ED), the respiratory therapist notices the ET tube has become unsecured. To best confirm to location of the ET tube, the therapist should recommend A. chest radiograph B. palpate the trachea C. observe chest rise D. auscultate breath sounds

The correct answer is : A Explanation : One may quickly determine the location of an endotracheal tube by observing chest rise and auscultating breath sounds. However, neither of these methods are conclusive. The only way to confirm proper placement is through a chest x-ray.

Which of the following will be most helpful at preventing complications for a 48-year-old male patient who has just undergone bariatric surgery for obesity? A. Incentive spirometry every hour B. Small volume nebulizer therapy with Albuterol every 4 hours C. Ambulation twice a day D. IPPB with 3.0 mL normal saline every 4 hours

The correct answer is : A Explanation : One of the best methods to prevent postoperative complications is the use of incentive spirometry, also called maximal sustained inspiration or SMI.

T mixed venous blood PO2 C. PCO2 from umbilical venous blood D. PvO2 in the blood taken from the same site

The correct answer is : A Explanation : Patent ductus arteriosus can be diagnosed by comparing the PO2 values between normal arterial PO2 (usually taken from the brachial or radial arteries) and umbilical artery PO2. If these two values are greater than 15 mmHg apart, patent ductus arteriosus (PDA) is considered present.

In preparation for a patient who will be receiving oxygen by nasal cannula at 2 L/min at home, the respiratory therapist should recommend which of the following devices for primary oxygen delivery? A. molecular sieve device B. bank and manifold of H tanks C. 3-5 E cylinders D. bulk liquid oxygen conversion system

The correct answer is : A Explanation : Patients who are in need of low-flow oxygen at home (between 1-6 L/min) are best served by using an oxygen concentrator. This is also called a molecular seive device.

A 40-year-old female is brought to the emergency department (ED) after being found down in a burning building. A quick assessment reveals singed nasal hairs and cherry-red arytenoids. She is on a non-rebreathing mask and oxygen SAT is 97%. Which of the following interventions should be given priority? A. intubation with a bronchoscope B. COHb determination from arterial blood C. multiple wave-length spectrophotometry D. blind oral intubation

The correct answer is : A Explanation : Patients who have had exposure to extremely heated air masses, such as seen with firefighters, are at risk for swelling of the tissues in the upper airways and ultimate loss of that airway. Priority, therefore, is to ensure an airway through intubation. However, blind intubation is extremely dangerous because probing those tissues with an endotracheal tube may incite additional inflammation and cause complete closure of the airway. Intubating with direct visualization is most appropriate. Usually, this means the use of a bronchoscope.

A 12-year-old child is receiving IPPB therapy by mouthpiece. The machine continually fails to cycle on both the inhalation and exhalation phases. The first step the therapist should take to resolve the problem is to A. utilize nose clips B. decrease sensitivity C. switch to a mouth seal D. decrease inspiratory pressure

The correct answer is : A Explanation : Patients who received IPPB therapy by a mouthpiece can sometimes have a tendency to leak at the corner of the mouth during a positive pressure breath. The solution for this is to switch to a mask. However, prior to trying a mask, it is possible the patient is leaking through their nose. Therefore, an easier solution, if offered, may be to try nose clips. This is especially true of pediatric patients who often have not learned how to close their nose internally.

Which of the following is most accurately descriptive of PS ventilation? A. the size of spontaneous breaths are augmented B. PSV is a companion to A/C ventilation C. provides more sensitivity for a ventilator actuated breath D. promotes better deflation of the lungs and decreases air-trapping

The correct answer is : A Explanation : Pressure support ventilation, also called PS ventilation, is somewhat of a misnomer in that it really does not provide ventilation at all. Pressure support is a way of overcoming both mechanical and anatomical airway resistance, making it easier for a person to inhale. Thus, it decreases the work of breathing and is suitable in patients who are attempting to liberate from mechanical ventilatory support and who are needing extra help in weaning from the ventilator. Essentially, the use of pressure support increases spontaneous tidal volumes. But, it does not trigger or deliver a breath if the patient does not initiate the breath themselves.

Predicted Observed TLC (liters) 4.90 3.30 FRC (liters) 2.55 1.80 SVC (liters) 3.30 1.90 FEV1 (liters) 2.10 1.90 FEF50 (liters/sec) 3.80 3.70 Which of the following represents the most appropriate interpretation of the preceding spirometry results? A. kyphoscoliosis B. normal C. chronic bronchitis D. emphysema

The correct answer is : A Explanation : Pulmonary function data indicates normal flows, which means the patient is not obstructive. Flows however, are diminished significantly as shown by a decreased SVC. This means the patient has a restrictive lung defect. To find the correct answer, we must look for that disease which is restrictive. Of those offered, only kyphoscoliosis is restrictive.

A patient is brought to the emergency room after ascending too quickly while scuba diving in the ocean. The patient is complaining of abdominal pain. The respiratory therapist should recommend A. hyperbaric chamber B. oxygen by non-rebreathing mask C. heliox therapy D. Bi-level therapy

The correct answer is : A Explanation : SCUBA divers who ascend too quickly are subject to a problem called "the bends". This is caused from dissolved nitrogen coming out of solution inside the blood when there is a significant reduction in pressure. For a patient with "the bends", the best treatment is to artificially increase the pressure of their environment. This is done with a hyperbaric chamber. Once the patient is at a deeper simulated pressure, the environmental pressure may be reduced slowly.

A patient presents in the emergency room with blood pressure of 95/60 mmHg. Skin turgor is poor and urine output is less than 10 mL/hr. Which of the following tests would be helpful in further evaluation of the patient? A. serum electrolytes B. arterial blood gases C. multiple-wavelength spectrophotometry D. chest radiograph

The correct answer is : A Explanation : Serum electrolytes, especially potassium, can reveal the fluid status of the patient. Poor skin turgor and reduced urine output, as well as a low blood pressure all indicate a fluid status change in the patient. Serum electrolytes will help us to assess this problem.

A radiographic image shows a right upper lobe cavitation. Which of the following conditions may be associated with this finding? A. Tuberculosis B. Cancer C. Foreign body aspiration D. Lung Mass

The correct answer is : A Explanation : Single cavitations in the upper lobes are associated with tuberculosis. A foreign body aspiration would be associated with an object, not a cavitation. Cancer or lung masses would also fill a space on X-ray instead of creating a cavitation.

Which of the following sputum characteristics is NOT assessed for a patient with a productive cough? A. specific gravity B. odor C. consistency D. color

The correct answer is : A Explanation : Specific gravity of sputum is not typically assessed.

The following data is obtained from the patient's ventilator flow sheet: 8 am 8 pm Peak pressure (cm H2O) 20 30 Plateau pressure (cm H2O) 14 14 VT (L) 0.5 0.5 PEEP (cm H2O) 8 8 Which of the following can be accurately stated? A. static compliance remains steady B. distribution of gases is improving C. dynamic compliance is increasing D. pulmonary vascular resistance is increasing

The correct answer is : A Explanation : Static compliance is determined by evaluating the plateau pressures over time. An increase in plateau pressures indicates a decrease in pulmonary compliance and vice versa. In this case, plateau pressures remain steady and therefore static compliance is also remaining steady.

A respiratory therapist notices the cuff pressure on an endotracheal tube is 10 cm H2O. After introducing 10.0 mL of air, the cuff pressure is 5 cm H2O. The patient is receiving positive pressure ventilation. The therapist should recommend A. replacing the ET tube B. monitoring the patient C. clamping the pilot tube D. initiating high frequency jet ventilation

The correct answer is : A Explanation : The ET tube cuff that fails to increase in pressure after introducing additional air is most likely damaged. When a part of an ET tube is damaged, the only acceptable option is to replace it.

A respiratory therapist notices the waveform of a pulmonary artery catheter is repeatedly rising and descending from 25 mmHg to 8 mmHg. However, a closer observation reveals there is no dicrotic notch in the waveform. Which of the following can the respiratory therapist do with the pulmonary artery catheter to correct the situation? A. aspirate the catheter B. advance the catheter C. jiggle the catheter D. withdraw the the catheter

The correct answer is : A Explanation : The absence of a dicrotic notch in a pulmonary artery pressure waveform is called "pressure dampening". This does not harm the patient. However, it is an indication of a poor signal and should be resolved. This may be remedied by: First, aspirating from the catheter; second, flushing the catheter; and third, rotating the catheter. Advancement or withdraw of the catheter is not an appropriate remedy for pressure dampening.

The package insert quality-control material for a blood gas analyzer defines expected values and upper and lower tolerance levels as follows: Value Tolerance pH 7.40 0.5% PCO2 40 mm Hg 5.0% PO2 80 mm Hg 3% Which of the following runs should be closely reviewed? A. pH 7.45 PCO2 37 mmHg PO2 81 mmHg B. pH 7.36 PCO2 42 mmHg PO2 79 mmHg C. pH 7.44 PCO2 38 mmHg PO2 82 mmHg D. pH 7.40 PCO2 40 mmHg PO2 78 mmHg

The correct answer is : A Explanation : The answer that shows a pH of 7.45 and a CO2 of 37 should be examined closely as these values appear to be outside the range of tolerance. If these results are accurate the arterial blood gas machine requires calibration and should not be used to report patient results until maintenance has occurred.

A 1000 g, 28-week gestational age infant has the following umbilical artery blood gas results: pH 7.35 PaCO2 44 torr PaO2 39 torr HCO3- 27 mEq/L BE +1 mEq/L Which of the following should the respiratory therapist recommend to monitor the infant's oxygenation? A. pulse oximetery B. capillary blood gas analysis C. arterial blood gas analysis D. transcutaneous PCO2 monitor

The correct answer is : A Explanation : The arterial blood gas of this infant indicates hypoxemia. Normal PaO2 for an infant is between 50 and 80 mmHg. Therefore, this infant is hypoxic and should be monitored for oxygenation. However, routine blood gases are not indicated because tests that consume blood should be kept to a minimum due to the low blood volumes in infants. Therefore, monitoring oxygenation with pulse oximetry is most appropriate.

A 28-week gestational age infant is receiving ventilatory support by a high frequency oscillation ventilator (HFOV) after being administered surfactant to treat severe IRDS. Heart rate is 150/min and blood pressure is within normal limits. Arterial blood gas results on high frequency ventilation are as follows: pH 7.25 PaCO2 65 torr PaO2 62 torr HCO3- 27 mEq/L BE + 1 mEq/L The respiratory therapist should recommend A. increasing oscillatory amplitude B. decreasing expiratory pressure C. increasing FIO2 D. decreasing driving pressure

The correct answer is : A Explanation : The arterial blood gas on this patient shows hypoventilation but adequate oxygenation. To correct this, the oscillatory amplitude should be increased on the high-frequency ventilator. Oscillatory amplitude is related to tidal volume.

A patient is demonstrating increased work of breathing and remains hypoxic in spite of supplemental oxygen by nonrebreathing mask. The patient is thrashing about in bed. Which of the following should the respiratory therapist recommend? A. arterial blood gas analysis B. sedate the patient with Valium C. institute full mechanical ventilatory support D. administer an IPPB treatment with 3.0 cc normal saline

The correct answer is : A Explanation : The cause or reason for the hypoxemia should be investigated. Arterial blood gas analysis will be most helpful in assessing the degree and cause the hypoxemia.

A family is found sleeping in their vehicle on the side of the road during a long trip. The officer who found them reports the family was difficult to arouse. Which of the following would be the most appropriate examination? A. COHb B. drug toxicology screen C. pulse oximetry D. arterial blood gas analysis

The correct answer is : A Explanation : The circumstances surrounding this family suggest possible exposure to carbon monoxide. The most reliable way to assess carbon monoxide levels is to determine COHb. Pulse oximetry is not capable of measuring carbon monoxide attached to hemoglobin.

After a gastric bypass surgery, which of the following positions should the patient be placed in to facilitate effective ventilation? A. semi-Fowler's position B. 1/4 turn to either side C. prone D. supine

The correct answer is : A Explanation : The lungs are most effectively ventilated when patients are in semi-Fowler's position. This, of course, only applies to patients who are in bed. Standing always provides the best gas distribution and ventilation of the lungs.

A 52-year-old patient is receiving VC SIMV ventilation due to myasthenia gravis. The patient has no history of pulmonary disease and is alert and oriented. The following data is observed: Mode SIMV Set Rate 6 Total rate 12 VT(spont) 425 mL PEEP 5 cm H2O FIO2 0.40 pH 7.42 PaCO2 38 torr PaO2 122 torr HCO3- 24 mEq/L BE 0 mEq/L MIP -42 cm H2O The respiratory therapist should recommend A. extubation B. daily SBT for several days, then reevaluate C. CPAP 5 cm H2O, PS 15 cm H2O D. decreasing the set rate to 2/min, ABGs in 20 minutes

The correct answer is : A Explanation : The clinical and laboratory data for this patient indicates that the patient is able to ventilate independently and should continue the ventilator liberation effort. Since the patient has no history of pulmonary disease, extubating the patient from a rate of six is perfectly appropriate.

An 8-year-old hypotensive male patient has been brought in by paramedics to the emergency room following a house fire. He was found unconscious in the garage where smoke was prevalent. Which of the following is a definitive measure to assess carbon monoxide poisoning? A. Carboxyhemoglobin level B. Rapid measurement with a multiple wavelength spectrophotometer C. Arterial blood gas D. Methemoglobin level

The correct answer is : A Explanation : The clinical background strongly suggests smoke inhalation, which would result in carbon monoxide poisoning. The most definitive way to assess for this condition is by measuring the carboxyhemoglobin level in the blood. One might be tempted to select rapid measurement with a spectrophotometer, but the keyword "definitive" in the question indicates that a diagnostic procedure should be utilized.

A COPD patient has the following pre and post information after being placed on 4 L/min nasal cannula: Before 1 hour After Respiratory rate 22 16 Heart rate 109 92 Blood pressure (mm Hg) 131/90 122/78 Breath sounds Diminished Diminished The patient is difficult to arouse. Which of the following actions is appropriate? A. decrease oxygen to 1 L/min B. discontinue oxygen C. increase oxygen flow rate to 5 L/min D. decrease oxygen to 3 L/min

The correct answer is : A Explanation : The data for this COPD patient shows that one hour after an increase in oxygen percentage, the patient's respiratory rate is depressed. COPD patients should never have more than 1 to 2 L per minute oxygen or 28% or less (except in an emergency). It is likely, therefore, that the patient's drive to breathe has been reduced by the excess oxygen. The best solution would be to return the patient to 1 or 2 L/min. oxygen by nasal cannula or 24% or 28% oxygen by Venturi mask.

After a bout of violent coughing, the respiratory therapist notices that patient who is intubated with 7.5 mm oral endotracheal tube is difficult to ventilate with a bag-valve. Additionally, the right chest appears to be rising more than the left chest. Which of the following is most likely the cause? A. Inadvertent advancement of the ET tube B. Pleural effusion C. Right sided pneumothorax D. Kinked endotracheal tube

The correct answer is : A Explanation : The decrease in dynamic compliance (difficulty squeezing the resuscitator bag) and the asymmetrical chest rise indicates the endotracheal tube has been inadvertently advanced into the main stem bronchus and is ventilating one lung side more than the other.

Immediately after delivering a small volume nebulizer treatment with albuterol to a patient with an acute asthmatic episode, the respiratory therapist observes breath sounds that were diminished before the treatment and now indicate wheezing. The patient's heart rate has changed from 110/min pre treatment to 102/min after the treatment. The therapist should recommend which of the following? A. administer another treatment with albuterol B. change the medication to Spiriva C. switch to aerosolized Xopenex D. administer intravenous methylprednisolone

The correct answer is : A Explanation : The evolution of breath sounds from diminished to wheezing suggests that the patient's airways are opening up. Wheezing can only occur when air passes through airways. As constriction resolves, more air passes through airways which can cause a period of increased wheezing. This is a good indication that the medication is effective and should be continued.

A respiratory therapist is assisting a physician with a bronchoscopy when an active bleed is found in the bronchus. After instilling epinephrine down the bronchoscope directly onto the bleeding site, the next step to stop the bleeding is to A. compress the site with the bronchoscope B. manually ventilate the patient with a bag/valve C. instill 10.0 cc of cold normal saline D. instill 3.0 cc of Heparin down the bronchoscope

The correct answer is : A Explanation : The first step in addressing a bleed that occurs during a bronchoscopy procedure is to instill epinephrine on the bleeding site through the bronchoscope. Following the installation, the site should be compressed with the bronchoscope. If the bleeding continues, a Fogarty catheter should be inserted to tamponade the site.

During a routine ventilator check, the respiratory therapist hears a ventilator alarm in another room. When the therapist arrives, it is observed that the patient is extremely restless and is thrashing about in bed. The low return volume is continuously sounding and oxygen saturation is 82%. The first action of the therapist should be to A. provide manual ventilation to the patient B. sedate the patient C. order a chest radiograph, STAT D. check endotracheal tube markings and position

The correct answer is : A Explanation : The first step in responding to an alarm from a ventilator is to ensure the ventilation and oxygenation of the patient by disconnecting the ventilator and assuming manual ventilatory support with a bag-valve.

A patient has an Hb level of 6.0 g/dL and the following blood gas results: pH 7.55 PaCO2 27 mmHg PaO2 110 mmHg HCO3- 24 mEq/L BE 0 mEq/L Which of the following is an accurate statement? A. hyperventilation secondary to hypoxemia is present B. hyperoxemia is present C. patient has respiratory acidosis D. metabolic alkalosis is present

The correct answer is : A Explanation : The first thing we see on this patient is that the hemoglobin is low. When a patient's hemoglobin is low they suffer from hypoxemia regardless of their PaO2. In this case the patient's PaO2 is high. They are also over ventilating as a result of the hypoxemia that is present due to the low hemoglobin. The best interpretation of this condition is hyperventilation secondary to (caused by) hypoxemia.

How should the respiratory therapist set the high-pressure alarm? A. 10-15 cmH20 above baseline peak pressures B. 5-10 cm H20 above the baseline peak pressures C. 10-15 cmH20 above the average peak pressures D. 5-10 cm H2O above the average peak pressures

The correct answer is : A Explanation : The high-pressure alarm on a volume-cycled mechanical ventilator should be set 10-15 cm H2O above the baseline peak pressures.

What is the maximum flow setting for a nasal cannula on an adult receiving oxygen therapy? A. 6 L/min B. 8 L/min C. 10 L/min D. 5 L/min

The correct answer is : A Explanation : The maximum flow setting for a nasal cannula on adult receiving oxygen therapy is 6 L/min. This is due to a limitation in the size of the reservoir that is found in the nasopharynx. Flows higher than 6 L/min will not benefit the patient and will likely cause additional harm by drying out mucosal tissues.

A patient is receiving volume-controlled ventilation. Which of the following blood gas results is a clear indication for an increase in minute ventilation? A. pH 7.25, PaCO2 55 torr, PaO2 51 torr B. pH 7.35, PaCO2 60 torr, PaO2 65 torr C. pH 7.55, PaCO2 26 torr, Pao2 88 torr D. pH 7.50, PaCO2 30 torr, PaO2 82 torr

The correct answer is : A Explanation : The need to increase minute ventilation can be determined by examining arterial CO2. In this question, two options have an elevated PaCO2, indicating a need to increase ventilation. However one of these options shows a corrected pH, suggesting that the elevated CO2 is normal for that patient. Therefore, the correct answer is the blood gas results that show elevated CO2 with an uncompensated pH. This is called uncompensated respiratory acidosis.

Immediately following arterial blood extraction and the removal of the needle from a patient's radial artery, the respiratory therapist should first A. Compress the puncture site B. Place gauze on the site with a pressure bandage C. Label the syringe D. Discard the needle in a single entry biohazard device

The correct answer is : A Explanation : The next action that should be taken, once the syringe is removed from the artery, is compression of the puncture site.

A patient with acute epiglottitis is awaiting transport to surgery for a tracheotomy under anesthesia. SpO2 is 88% on room air. While waiting the patient would benefit most from which of the following? A. heliox therapy with 60%/40% mixture B. high flow Venturi mask set at 50% C. simple mask at 6 L/min D. nasal cannula at 2 L/min

The correct answer is : A Explanation : The patient has an arterial oxygen saturation of 88% on room air. This is consistent with hypoxemia and indicates a need for supplemental oxygen. Because the patient has acute epiglottitis, Heliox therapy is appropriate to reduce airway resistance. Additionally, the patient would benefit from an adult therapeutic dose of oxygen, which starts at 40%. Therefore, 60-40% Heliox mixture is most appropriate.

A 33-year-old female patient presents to the emergency room with droopy eyelids and facial muscles. She reports a history of Myasthenia Gravis at a prior time, but was never ventilator-dependent. She now has the following clinical data: Vital Capacity Tidal volume MIP 0.9 L 350 mL -22 cm H2O What should the respiratory therapist recommend? A. Tensilon challenge with resuscitation equipment available B. NRB mask C. manual ventilation D. Atropine

The correct answer is : A Explanation : The patient is showing sufficient signs of a potential returning myasthenia gravis and therefore requires a Tensilon challenge, which should always be administered with resuscitation equipment close by.

A patient with a history of asthma reports to the emergency department after self-treating bronchoconstriction for the past 3 days. Periodic relief has been achieved but the patient is in current distress and is wheezing bilaterally. Which of the following treatment actions would be most appropriate? A. continuous aerosolized bronchodilator at 7 mg/hr B. full pulmonary function testing with DLCO measurement C. sildenafil, IV D. administer Sublimaze, PO

The correct answer is : A Explanation : The patient shown in this question obviously demonstrates lack of response to bronchodilator therapy. Of the options offer, IV methylprednisolone and continuous aerosolized bronchodilators are appropriate.

A patient who weighs 65-kg (143-lb) is receiving mechanical ventilator support on the following settings with the following corresponding blood gas values: Mode Assist/control Mandatory rate 16 VT 450 mL FIO2 0.40 PEEP 15 cm H2O pH 7.33 PaCO2 46 mm Hg PaO2 98 mm Hg HCO3- 23 mEq/L BE -1 mEq/L The respiratory therapist should recommend: A. remove deadspace B. increase tidal volume to 700 mL C. increase rate to 18 D. decrease PEEP

The correct answer is : A Explanation : The patient's carbon dioxide is slightly high. To correct this, an increase in rate, an increase in tidal volume, or a decrease in deadspace is indicated. Increasing rate is too significant of a change and would result in an increase in mean airway pressure. Increasing the tidal volume to 700 mL would place the tidal volume outside the patient's range for their height and weight. The best way to decrease CO2, in this case, is to remove deadspace. This will not increase mean airway pressure but will bring CO2 down.

A physician has ordered administration of 80% / 20% heliox therapy to a spontaneously breathing patient. Which of the following gas delivery devices should the respiratory therapist plan on using? A. nonrebreathing mask B. nasal cannula C. air-entrainment mask D. simple mask

The correct answer is : A Explanation : The proper modality to administer Heliox therapy is a non-rebreathing mask.

While performing a forced vital capacity maneuver, a 49-year-old male patient fails to inhale after a forced expiratory effort. The patient is sitting upright in the PFT lab, unconscious and cyanotic. No breathing is detected. What should the respiratory therapist do FIRST? A. press the code button-call for help B. perform a precordial thump C. check for a pulse D. position the patient supine on the floor

The correct answer is : A Explanation : The protocol for suspected cardiac and respiratory collapse is to call for help or to activate emergency response teams that may be in the area.

During the placement of a pulmonary artery catheter, the respiratory therapist observes the waveform on the monitor and notices it is repeatedly rising and falling from 0 mmHg to 25 mmHg, which is an indication that the tip of the catheter is in the right ventricle of the heart. Based on this information, the respiratory therapist will recommend A. inflate the catheter balloon to sail the tip to the proper location B. rotate the catheter to achieve a dicrotic notch in the waveform C. suture the catheter in place D. withdraw the catheter back to the upper vena cava

The correct answer is : A Explanation : The pulmonary catheter should be terminated in the pulmonary artery, which is beyond the right ventricle. Therefore, advancing the catheter is appropriate. The catheter may be advanced by sailing it into position, which is done by inflating the catheter balloon and allowing blood flow to carry the catheter into position and then deflating the balloon once it is in the proper position.

The results of a V/Q scan is inconclusive following what appears to be the development of a pulmonary embolus. What further testing should the therapist recommend?

pulmonary angiography

Which of the following would be a sufficient reason to discontinue a spontaneous breath trial? A. respiratory rate increases from 20 to 32 per minute B. blood pressure increases from 110/70 to 119/79 mmHg C. heart rate increases from 78 to 94 bpm D. SPO2 decreases from 98% to 94%

The correct answer is : A Explanation : The purpose of T-piece trials is to build pulmonary muscle strength and determine how long the patient may breathe spontaneously before permanently removing mechanical ventilatory support. The point at which a patient fails the trial is when blood pressure rises more than 10 mmHg above baseline, rate rises more than 10 above baseline, heart rate rises more than 20 above baseline, and oxygen saturation falls below 92%. Any single one of these criteria is an indication to stop the trial. In this case, the increase in respiratory rate by more than 10 is the only indication the trial has failed or that the patient has reached his or her limit.

A respiratory therapist must gather equipment for an arterial puncture on an adult female patient. The correct needle size is A. 22 gauge B. 25 gauge C. 18 gauge D. 12 gauge

The correct answer is : A Explanation : The respiratory therapist is often most concerned with the procedure involved with arterial puncture, or complicated blood gas analyses. However, it is necessary to know the basic recall facts such as the needle size for an adult which is most commonly a 22 gauge needle.

A respiratory therapist is discarding a ventilator circuit. Upon doing so, the therapist notes water remains in the tubing. The therapist should dispose of the tubing and its contents by A. discarding everything into a routine trash receptacle B. discarding everything in a red biohazard bag C. emptying the water in the sink and discarding the tubing in a trash receptacle D. double bagging the circuit and place in a regular trash receptacle

The correct answer is : A Explanation : The soiled ventilator circuit on a patient who does not have a highly infectious disease may be disposed of in a normal trash receptacle. A disposable ventilator circuit from an active tuberculosis patient that is obviously soiled would need to be red-bagged, and/or placed in a biohazard receptacle.

A recovering motor vehicle accident patient with multiple trauma suddenly becomes short of breath and develops cyanosis from the waist up. Which of the following conditions should the respiratory therapist suspect? A. pulmonary embolism B. myocardial infarction C. acute ventilatory failure D. pulmonary edema

The correct answer is : A Explanation : The sudden development of shortness of breath and the development of cyanosis from the waist up is associated with pulmonary embolism. This is considered an emergency and the patient should be given 100% oxygen. Only after 100% oxygen is in place should efforts be made to confirm the diangosis (VQ scan, etc).

A parent of a 7-year-old child with asthma reports having difficulty controlling the asthmatic episodes. The patient has an albuterol inhaler and is taking beclovent by MDI twice daily. The mother indicates that episodes have been increasing and she is confused regarding when to continue treatment and when to call the doctor. The respiratory therapist should A. create an asthma action plan with the parent and patient B. set an appointment for allergen testing C. recommend switching from albuterol to Xopenex D. replace Beclovent with Azmacort

The correct answer is : A Explanation : The use of an asthma action plan can help the patient and/or caregiver manage the asthma, especially during episodic attacks. The plan helps direct when and how to treat oneself, as well as when to seek medical attention. The plan consists of personal monitored data translated to a green yellow red light status.

A respiratory therapist is measuring the gas volume from a patient who exhales maximally after inhaling to inspiratory reserve volume. Which of the following volumes is the respiratory therapist attempting to observe? A. vital capacity B. expiratory reserve volume C. total lung capacity D. inspiratory reserve volume

The correct answer is : A Explanation : The volume exhaled maximally after a maximum inhalation is called vital capacity.

A respiratory therapist is having difficulty suctioning secretions from a 6-year old cystic fibrosis patient. The patient is being suctioned with an 8 Fr suction catheter through a 5.5 ET tube at a pressure of 85 mmHg for 10 seconds. To increase the effectiveness of suctioning, the therapist should increase which of the following? A. suction pressure to 100 mmHg B. suction time to 20 seconds C. catheter size to 10 Fr. D. catheter length

The correct answer is : A Explanation : There are several ways to increase suctioning effectiveness. The best way is to increase the suction catheter diameter size. The next best way is to increase suction pressure. Finally, increasing suction time up to 15 seconds is also appropriate. Since the suction catheter diameter size is already as big as is permitted the next step would be to increase suction pressure. A pediatric patient should have a suction pressure range of 80 to 100 mmHg. Thus, the best option would be to increase the suction pressure to 100 mmHg, which is at the top of the range.

A respiratory therapist is having difficulty effectively removing secretions through endotracheal suctioning of a 7-year-old child with cystic fibrosis. The ET tube size is 6.0-mm. The suction pressures is set at 75 mmHg and the suction catheter size is an 8 Fr. The therapist should do which of the following to increase the efficacy of suctioning? A. increase suction pressure to 80 mmHg B. instill 20 cc of normal saline prior to each suction attempt C. increase catheter size to 10 Fr D. increase suction during per pass

The correct answer is : A Explanation : There are three ways to increase suction efficiency. In order: 1) increase suction catheter size 2) increase suction pressure 3) increase suction time. However, prior to any of these three one must first ensure adequate minimal pressure for the patient's age is being used. This is a pediatric patient and suction pressure range should be between 80 and 100 mmHg. However, the suction pressure is only set at 75 mmHg. So, this should be corrected first before considering any other action to improve suction efficiency.

A physician orders a chest radiograph on an ambulatory patient in the emergency department (ED) in response to a suspicion of pneumonia. Which of the following should be ordered? A. PA radiograph B. series of oblique-angle radiographs C. AP radiograph D. apical lordotic

The correct answer is : A Explanation : There are two types of chest x-rays that can be used - a PA x-ray and an AP x-ray. The PA x-ray is more accurate but requires the patient to be mobile. An AP x-ray is used for patients who are bedridden. In this case the patient is clearly ambulatory so A PA radiograph is most appropriate.

Which of the following devices is needed to determine a patient's airway resistance? A. Body box B. turbine pneumotachometer C. Geissler tube nitrogen analyzer D. Wheatstone bridge helium analyzer

The correct answer is : A Explanation : There is a procedure involving panting where airway resistance can be determined. In order to do that procedure all conditions must be standardized, including temperature, pressure, and even humidity to some degree. Only a body box can provide this controlled environment.

Results of a quality control maneuver for a spirometer using a 3.0 L calibration syringe as follows: Volume 1 2.65 L Volume 2 2.68 L Volume 3 2.66 L According to ATS Standards, the spirometer is A. inaccurate B. proof that the syringe requires calibration C. operating correctly D. lacking in precision

The correct answer is : A Explanation : These calibration results are all very close together, indicating the machine is very precise. However, the results are too far from the 3.0 L of gas introduced by the calibration syringe. The maximum variance is 2.85 L - 3.15 L. Therefore, although the machine is precise, it is considered inaccurate and should not be used for patient testing and reporting.

A patient is being paralyzed and intubated in preparation to receive positive pressure ventilation with a volume-cycled ventilator in the control mode. Which of the following alarm settings is most important? A. low PEEP alarm B. high-pressure alarm C. low minute ventilation alarm D. low return-volume alarm

The correct answer is : A Explanation : This is a difficult question. When a patient is intubated after being paralyzed, they do not have the ability to signal or perform any kind of physical manifestation should they accidentally become disconnected from the ventilator. Thus, a ventilator alarm is paramount. The most sensitive alarm that would signify an accidental disconnection is the low PEEP alarm. On some ventilators this is called the disconnect alarm. Keep in mind, this alarm is more sensitive than the low pressure or low-volume alarms. In other words, the low PEEP alarm responds faster than any other alarm when a disconnection has occured.

An adult female patient has severe airway resistance for which she is being treated with an 80/20 mixture of heliox via a NRM. The problem is improving but not yet resolved. ABGs are: pH 7.36 PaCO2 45 torr PaO2 67 torr HCO3- 6.2 mEq/L What should the respiratory therapist recommend? A. Change to 70/30 heliox mixture. B. Replace heliox with nasal cannula at 5 L/min. C. Change to 60/40 heliox mixture. D. Replace heliox with CPAP, FIO2 0.40.

The correct answer is : A Explanation : This is an oxygenation problem as seen by the PaO2 of 67 torr. Therefore more oxygen is required. A 70/30 heliox mixture would provide approximately 10% higher FI02.

What is the appropriate treatment for a patient on air with the following arterial blood gas results? pH 7.56 PaCO2 44 mm Hg PaO2 62 mm Hg HCO3- 35 mEq/L BE +11 mEq/L SAT 86% A. supplemental oxygen B. bicarbonate C. hydrochloric acid D. rebreathing device

The correct answer is : A Explanation : This patient is demonstrating metabolic alkalosis and hypoxemia. Of these two problems, hypoxemia should be addressed first. Therefore, the most appropriate action is to administer supplemental oxygen.

After a motor vehicle accident causing blunt trauma to the chest, a male patient in the emergency room has air beneath the skin about the chest, neck and face. What would the respiratory therapist expect to auscultate in the affected areas? A. Dry crackles throughout B. Moist crackles upon expiration C. Moist crackles throughout D. Dry end inspiratory crackles

The correct answer is : A Explanation : This patient is experiencing subcutaneous emphysema secondary to chest trauma. The most common sign associated with subcutaneous emphysema is dry crackles upon auscultation. Moist crackles are associated with pulmonary edema or secretions in the airways. Dry end inspiratory crackles are associated with atelectasis.

A patient with significant pulmonary shunting is receiving volume-controlled ventilation on the following settings and has the following arterial blood gas results: Mode Assist/control Mandatory rate 18 VT 500 mL FIO2 0.70 PEEP 25 cm H2O C.O. 3.2 L/min Heart rate 118 pH 7.36 PaCO2 45 torr PaO2 54 torr HCO3- 26 mEq/L BE +2 mEq/L The therapist should decrease which of the following? A. PEEP B. rate C. flow D. FIO2

The correct answer is : A Explanation : This patient is hypoxic. To correct this either PEEP or FIO2 should be increased. However, closer examination of hemodynamic data, namely cardiac output, shows instability. Normal cardiac output is 4-8 L/min. This patient has low cardiac output most likely brought about by excessive levels of PEEP. The most appropriate action, in spite of hypoxemia, is to decrease PEEP.

A spontaneously breathing patient has the following arterial blood gases: pH 7.25 PaCO2 55 torr PaO2 54 torr HCO3- 25 mEq/L BE -1 mEq/L Which of the following should the respiratory therapist recommend? A. VC AC ventilation B. supplemental oxygen by partial rebreathing mask C. supplemental oxygen by nonrebreathing mask D. Bi-level therapy, IPAP of 15 cm H2O, EPAP of 5 cm H2O

The correct answer is : A Explanation : This patient is in acute ventilatory failure, as shown by an increased CO2 and a pH of 7.25 or less. When acute ventilatory failure is present, the patient would benefit most from positive pressure ventilation, such as mechanical ventilation.

A 60-kg (132-lb) female patient is being weaned from mechanical ventilation. The following data and ventilator settings are available: Mode SIMV Mandatory rate 8 Total rate 24 VT 450 mL VT (spont) 200 mL FIO2 0.4 PEEP 5 cm H2O PS 5 cm H2O Which of the following represents an appropriate recommendation? A. increase pressure support B. decrease mandatory rate C. increase mandatory rate D. discontinue mechanical ventilation

The correct answer is : A Explanation : This patient is in the beginning stages of ventilator weaning. When a patient is weaning it is most helpful to observe the total rate compared to the set rate, and the spontaneous tidal volume compared to the set tidal volume. In this case the patient has a set rate of 8/min but his breathing at a total rate of 24/min. This rate is excessive. Further examination reveals the patient's tidal volume is 200 mL, which is far less than the 5 mL per kilogram tidal volume needed to sustain life. Together, the high rate and low spontaneous tidal volume show an increased work of breathing. Ultimately, the patient will be unable to sustain independent ventilation. Pressure support is a way that we can assist the patient by increasing their spontaneous tidal volume which will cause a lower respiratory rate and preserve the patient's energy.

While administering a treatment with Albuterol to a patient with COPD, the therapist notes a sudden change in the patient's color to marked cyanosis. The patient appears apneic and the heart rate on the pulse oximeter indicates 30/min. The therapist should FIRST

begin mouth-to-mouth resuscitation

An 80 kg (176 lb) adult male is receiving volume-controlled ventilation. Current ventilator settings and corresponding arterial blood gas results are: Mode SIMV Mandatory rate 4 Total rate 12 Tidal volume 550 mL VT(spont) 450 mL PEEP 5 cm H2O FIO2 0.40 pH 7.46 PaCO2 38 torr PaO2 90 torr HCO3- 25 mEq/L BE +1 mEq/L The respiratory therapist should recommend A. discontinue mechanical ventilation B. decrease FIO2 to 0.35 C. decrease mandatory rate to 2/min D. add pressure support

The correct answer is : A Explanation : This patient is obviously weaning from mechanical ventilatory support as evidenced by the SIMV mode and a rate of 4/min. In this condition, the patient appears to be ventilating adequately with the tidal volume that is greater than 5 mL/kg. Blood gas results are good. This data indicates the patient is ready to be weaned from mechanical ventilatory support.

A patient who weighs 85 kg (187 lb) is weaning from mechanical ventilatory support on the following settings Mode SIMV Mandatory rate 8 Total rate 30 Set Tidal volume 600 mL Spontaneous VT 200 mL FIO2 0.60 PEEP 5 cm H2O Pressure support 8 cm H2O Which of the following should be increased in order to decrease the work of breathing? A. pressure support B. tidal volume C. PEEP D. respiratory quotient

The correct answer is : A Explanation : This patient is weaning from mechanical ventilatory support but is having difficulty. The total rate of 30 is excessive and a spontaneous tidal volume of 200 mL is insufficient to sustain life. Sufficient tidal volume is at least 5 mL per kilogram. To solve this problem, pressure support may be used to increase the size of spontaneous tidal volumes, which will ultimately result in a reduced rate and reduced labor of breathing.

A 52-year-old patient with polyneuropathy has received volume-controlled ventilation for 9 weeks. In an attempt to slowly wean the patient from dependence on mechanical ventilatory support, spontaneous breathing trials with a tracheostomy collar have been implemented over the last few days. The following data is available: 30 Minutes 1 Hour 2 Hours Pulse rate 89 98 119 SpO2 (%) 97 95 89 MIP (cm H2O) -30 -28 -18 Vital capacity (L) 1.7 1.6 0.8 Respiratory rate 20 24 32 The respiratory therapist should recommend A. limit the next SBT to 1 hour in duration B. switch to a T-tube C. limit the next SBT to 2 hours in duration D. discontinue SBT trials for 3 days

The correct answer is : A Explanation : This patient is weaning from mechanical ventiltory support by undergoing spontaneous breathing trials. After a 30-min and 1-hour trial, the patient appears to be stable. However, after two hours, a significant degradation in the patient's status is noted. This would indicate that future trials should not exceed one hour.

Which of the following is required to determine RAW in a spontaneously breathing ambulatory patient? A. plethysmograph B. helium analyzer C. galvanic oxygen analyzer D. nitrogen analyzer

The correct answer is : A Explanation : To determine airway resistance, otherwise known as RAW, a body box is needed. Another name for a body box is a plethysmogaph.

Which of the following is NOT used to determine the optimum level of PEEP for a patient receiving mechanical ventilation? A. VD/VT ratio B. steady QT of 4.4 L/min C. inflection point of a volume-pressure loop graphic D. sufficient oxygenation with no hemodynamic instability

The correct answer is : A Explanation : To determine optimal PEEP, the pressure which may be found at the upper inflection point of a flow pressure ventilator graphic, hemodynamic stability, and cardiac output are needed. Deadspace-tidal volume ratio is NOT helpful in determining optimal PEEP.

A patient has just undergone oral endotracheal intubation. Which of the following should be examined FIRST to determine proper positioning of the ET tube? A. breath sounds B. chest radiograph C. symmetry of diaphragmatic excursion D. tracheal deviation

The correct answer is : A Explanation : To determine proper positioning of the endotracheal tube the respiratory therapist should first assess that which is quickest. Of the options given, the quickest method would be to auscultate breath sounds. This method is not conclusive but is quick. If asked to provide conclusive evidence, a chest radiograph is most appropriate.

A patient is receiving mechanical ventilator support with a Hamilton adult ventilator. The following data is available: pH 7.42 PB 747 torr PaCO2 40 torr PaO2 85 torr HCO3- 25 mEq/L BE +1 mEq/L PAO2 295 torr CaO2 17.5 vol% CvO2 13.0 vol% Mode SIMV Rate 12 VE 6.0 L FIO2 0.50 PIP 29 cm H2O PEEP 10 cm H2O The respiratory therapist should report which of the following values as an accurate A-aDO2? A. 210 torr B. 300 torr C. 185 torr D. 260 torr

The correct answer is : A Explanation : To determine the alveolar-arterial oxygen difference or gradient, one must first determine the alveolar oxygen tension. In this case PAO2 is given. When you take the PAO2 of 295 mmHg and subtract the PaO2, which is 85 mmHg, you get 210 mmHg, which represents the A-aDO2

During a disaster, an H-cylinder with 1000 psi is being used to power a Bird Mark 7 pressure ventilator that is set to deliver 100% oxygen at a tidal volume of 500 mL, respiratory rate of 12/min. How many hours will the tank last? A. 8 B. 10 C. 4 D. 6

The correct answer is : A Explanation : To determine the answer to this question, the liter flow per minute must be determined. At a rate of 12 per minute and a tidal volume of 500 mL, the minute ventilation is 6.0 L. Tank duration = 3.14 x 1000 psi = 3,140 L. 3,140 L / 6 L/min = 523 minutes. 523 min / 60 = 8.7 hours. The closest answer is eight hours.

A patient is receiving volume-controlled ventilation in the emergency department (ED). The following data is available: Mode Assist/control VT 550 mL Mandatory rate 10 FIO2 0.5 PEEP 5 cm H2O PetCO2 10 torr pH 7.39 PaCO2 40 torr PaO2 100 torr HCO3- 25 mEq/L BE +1 mEq/L The respiratory therapist should report which of the following as an accurate VD/VT ratio? A. 75% B. 50% C. 15% D. 25%

The correct answer is : A Explanation : To determine the deadspace-tidal volume ratio (VD/VT), we take arterial CO2 and subtract end-tidal CO2 and divide that by the arterial CO2. - (PaCO2-PetCO2)/PaCO2 - In this case, end-tidal CO2 subtracted from arterial CO2 is 30. 30 divided by 40 is equal to 75%.

What is the total flow of gas in L/min for a patient who is receiving 60/40% heliox through an oxygen flow meter that is indicating a flow of 12 L/min? A. 16.8 B. 19.2 C. 12.0 D. 21.6

The correct answer is : A Explanation : Use of an oxygen flow meter to administer various mixtures of Heliox will produce an erroneous result in terms of the indicated flow on the oxygen flow meter. Therefore, each Heliox gas mixture has a specific factor to convert the oxygen flow meter reading to the total gas flow. These conversion factors are: 1.8 for 80/20% mixture; 1.6 for 70/30% mixture; and 1.4 for 60/40% mixture. In this case, the factor of 1.4 should be used. Simply multiply that factor by the indicated flow on the oxygen flow meter, which is 12 L/min. 1.4 x 12 = 16.8 L/min.

During surgery, a patient is placed on an infrared absorption device. Capnographic waveforms are normal. The difference between measured PaCO2 by arterial blood gas analysis and expired gas measurement by capnography is within expectations. PaCO2 is 40 mm Hg. What is the patient's estimated VD/VT? A. 25% B. 60% C. 40% D. 50%

The correct answer is : A Explanation : VD/VT = (PaCO2 - PetCO2)/PaCO2. End tidal CO2 varies from PaCO2 by about 10 mm Hg. (reading lower) Therefore, (40-30)/40 = 0.25 or 25%.

A patient is being mechanically ventilated by volume controlled ventilation. A chest tube drainage system is in place and 1200 mL of fluid has been collected from the left chest. A low volume alarm on the ventilator begins to sound. The respiratory therapist notices vigorous bubbling in the water seal chamber. The following data is available: pH 7.44 PaCO2 38 torr PaO2 88 torr HCO3- 24 mEq/L BE -1 mEq/L Mode Assist/control VT 600 mL Exhaled VT 396 mL Mandatory rate 12/min Total rate 12/min FIO2 0.60 The respiratory therapist's first action should be to A. look for a leak between the water seal chamber and the patient B. discontinue chest tubes C. decrease suction pressure to the system D. increase rate to 14

The correct answer is : A Explanation : Vigorous bubbling in the water seal chamber is abnormal and indicates the presence of a leak in the chest tube drainage system. The leak may exist somewhere between the water seal compartment and the patient. However, the leak may be caused from a perforated lung.

A respiratory therapist has been ordered to alter the I:E ratio on a COPD patient. The current ratio is 1:2. The ordered ratio is 1:5. In completing this order, the therapist will have A. decreased the I:E ratio B. Altered the mandatory rate C. decreased inspiratory flow D. increased inspiratory time

The correct answer is : A Explanation : When I:E ratio is changed from 1:2 to 1:5, the I:E ratio is being decreased. This can be confusing because the number in the ratio appears to be increasing. But, in fact, inspiratory time is being decreased, allowing more time for expiration.

While completing the insertion of a nasopharyngeal airway, the respiratory therapist notices the patient begins coughing violently. The therapist should A. Remove the airway, reinsert a shorter airway B. Replace the airway with a small diameter lumen C. Replace with an oropharyngeal airway D. Secure the airway to restrict its movement

The correct answer is : A Explanation : When a patient begins coughing uncontrollably at the insertion of a nasopharyngeal airway, also called a nasal trumpet, the most likely cause is that the trumpet is too long. The airway should be removed and a shorter one should be inserted.

The respiratory therapist notices the chest does not rise on a patient who receives one rescue breath at the beginning of CPR. After repositioning the head and reattempting a rescue breath, the chest fails to rise again. Which of the following is the next appropriate action? A. begin abdominal thrusts B. leave to get help C. reattempt another ventilation D. perform a finger sweep of the patient's mouth

The correct answer is : A Explanation : When a patient demonstrates the inability to breathe, abdominal thrusts may be needed to clear the airway. Abdominal thrusts should be done only after attempting to reposition the head and neck and attempting to ventilate the patient.

A patient is receiving a positive pressure treatment through a fenestrated tracheostomy tube. To prepare the patient to be able to speak, the respiratory therapist should FIRST A. deflate the cuff B. insert the inner cannula C. cap the tube D. inflate the cuff

The correct answer is : A Explanation : When a patient has a fenestrated tracheostomy tube and requires transitioning from a positive pressure ventilation configuration to a speaking configuration, the first step is to deflate the cuff.

Which of the following is most associated with a pitting peripheral edema of +3? A. congestive heart failure B. hypovolemia C. cor pulmonale D. Guillain-Barre' Syndrome

The correct answer is : A Explanation : When a patient has pitting peripheral edema of +1, +2, +3, the pathological association is congestive heart failure. This pitting edema occurs because of fluid shifting and has involvement of the kidney. Follow-up examinations would include creatinine or BUN.

A patient is breathing rapidly and deeply while receiving oxygen by Venturi mask at 60% with a flowrate to 6 L/min. SpO2 is 88%. The therapist should A. increase the flow B. increase to 80% by Venturi mask C. decrease FIO2 to 0.4, flow rate 8 L/min D. intubate and mechanically ventilate

The correct answer is : A Explanation : When a patient is receiving oxygen by Venturi mask or any Venturi device, the respiratory therapist must ensure the total flow of gas going to the patient is meeting or exceeding the patient's inspiratory demand. This is especially true on high oxygen percentages. At 60% the air-oxygen ratio is 1:1. This would give us a total flow of 12 L per minute. A patient who is breathing deeply and rapidly is very likely exceeding 12 L per minute.

A 4-year old child was playing with toys when he suddenly became tachypneic. Auscultation reveals wheezing on the right side. Which of the following should the respiratory therapist prepare? A. a bronchoscope B. an Albuterol treatment C. aerosolized Lidocaine D. racemic epinephrine

The correct answer is : A Explanation : When a patient is wheezing bilaterally the most likely cause is bronchoconstriction. However, when the patient is wheezing unilaterally, or on just one side, the most likely cause is foreign body aspiration (a toy or perhaps food in the main stem bronchus). Of the options given, the most appropriate procedure to further assess the problem is a bronchoscopy. The other options consist of the delivery of medications which will not help remove the foreign body.

A patient has just self-extubated and is now experiencing inspiratory stridor. Vital signs are stable and SaO2 is 97% on a large-volume, non-heated aerosol device set at 30% oxygen. In addition to close monitoring, the respiratory therapist should also consider A. administering aerosolized racemic epinephrine B. immediately reintubating the patient C. examining the oropharynx with a tongue depressor D. providing heated mist

The correct answer is : A Explanation : When a patient self-extubates, it is likely that the air in the endotracheal tube cuff was not evacuated. Pulling a tube with an inflated cuff can cause damage and inflammation of the vocal folds and upper airway tissues, resulting in inspiratory stridor. When this happens, the administration of racemic epinephrine is appropriate. If, however, inspiratory stridor is described as severe or marked, reintubation of the patient is indicated.

Which of the following would best decrease the work of breathing for a spontaneously breathing patient with a fixed upper airway obstruction?

heliox therapy

A patient presents to the emergency room with shortness of breath and severe nasal/sinus congestion. SPO2 on air is 86%. She is placed on a nasal cannula at 4 L/min but complains that she cannot breathe through her nose. The therapist should A. change to a 35% air-entrainment mask B. change to 100% cool aerosol mask C. increase to 6 L/min nasal cannula D. change to a 40% venture mask

The correct answer is : A Explanation : When a patient's nose is occluded with secretions or for any other reason a nasal cannula may not be appropriate. Switching to a mask will allow gases to be delivered by mouth. The FIO2 coming from a nasal cannula is approximately 3 to 4% per liter. Thus, a 35% Venturi mask most closely approximates the FIO2 of a nasal cannula running at 4 L/min.

A patient in the emergency room has a blood pressure of 90/60 mmHg and a palpated heart rate of 110 bpm. Pulse oximetry shows a saturation of 70% and a heart rate of 60 bpm. The respiratory therapist should A. move the pulse ox electrode to different location B. recalibrate the pulse oximeter C. change the pulse oximeter sensor D. record the pulse ox saturation in the medical record

The correct answer is : A Explanation : When a pulse oximeter is reading a low oxygen saturation but has a different heart rate compared to that which can be palpated, the oxygen saturation reading is in error and should not be reported. The cause for this is likely poor peripheral perfusion. An appropriate response is to first move the pulse ox probe to a different location on the body. A secondary option is to replace the probe.

A 32-year old patient is receiving an initial small-volume nebulizer treatment with Albuterol. Prior to administration, the patient's pulse was 74/min. During the treatment the pulse rises to 88/min and remains steady. The respiratory therapist should A. Complete the treatment, continue to monitor pulse rate B. Decrease future doses to half strength C. Contact the physician to obtain direction D. Discontinue the treatment, assess the patient in 15 minutes

The correct answer is : A Explanation : When administering a breathing treatment, monitoring the patient for adverse reactions is important. A negative cardiac response is present when the heart rate rises more than 20 beats above baseline. In this case, the heart rate has risen only 14 beats above baseline and therefore is not considered an adverse reaction. The treatment should be continued, but the patient's heart rate should be continually monitored.

A patient with a tracheostomy is receiving supplemental oxygen via tracheostomy collar connected to a large volume nebulizer set at 40%. The respiratory therapist analyzes the FIO2 at the tracheostomy collar with a galvanic fuel cell analyzer. The analysis shows the FIO2 to be 55%. Which of the following could be the cause of the increase in FIO2? A. too much water in the circuit B. the aerosol tubing is too short C. clogged jet orifice in the nebulizer D. calibration error in the galvanic fuel cell

The correct answer is : A Explanation : When administering oxygen by any device that has a venturi mechanism, back pressure on the venturi will slow the speed of gas, decrease room air entrainment, and result in an increase in FIO2. Of the options offered, only excess water in the tubing would cause this type of back pressure.

An IPPB machine cycles prematurely into inhalation before a patient attempts to inhale. Which of the following will mostly likely correct this problem? A. decrease sensitivity B. decrease inspiratory pressure C. decrease inspiratory flow rate D. decrease expiratory time

The correct answer is : A Explanation : When an IPPB machine begins to cycle prematurely into the inhalation phase before the patient has initiated a negative pressure or breath, the most likely cause is that the sensitivity is set too high. The answer is to decrease sensitivity.

When evaluating a patient's pulmonary condition, the therapist should include which of the following in the assessment? A. occupation and smoking history B. marital status and disease history C. insurance and disease history D. language skills and social status

The correct answer is : A Explanation : When evaluating a patient's pulmonary condition, occupation may reveal long-term exposure to pulmonary irritants. Smoking history would also be helpful. Marital status and language skills do not directly relate to a patient's pulmonary condition.

The respiratory therapist notes the following results of an arterial blood gas on a patient on 40% Venturi mask: pH 7.40 PaCO2 41 torr PaO2 260 torr HCO3- 24 mEq/L BE 0 mEq/L The respiratory therapist should A. run the blood sample on another blood gas analyzer B. discontinue supplemental oxygen C. decrease FIO2 to 28% D. report the blood gas values to the physician

The correct answer is : A Explanation : When looking at these blood gases, the extremely high PaO2 should be in question since the patient is only on 40% oxygen. Determining the maximum possible alveolar oxygen tension reveals a maximum possible PAO2 of 230 mmHg. It would be impossible for the arterial oxygen tension (PaO2), to be higher than this.

A 39-week gestational age infant is receiving supplemental oxygen by oxyhood. An air/oxygen blender is set at 30% and the heated large volume aerosol is set at 100%. A capillary blood sample reveals the following values: pH 7.30 PcCO2 49 torr PcO2 48 torr HCO3- 23 mEq/L BE -1 mEq/L SpO2 97% Which of the following is an accurate interpretation of this data? A. hypoventilation and respiratory acidosis B. metabolic alkalosis with mild hypoxemia C. acute-on-chronic metabolic acidosis D. hypoxemia and respiratory acidosis

The correct answer is : A Explanation : When looking at what appears to be arterial blood gas data, a closer examination reveals the blood gas levels are not determined from arterial blood but rather, capillary blood. It must be remembered that capillary pH and CO2 data can be trusted but capillary oxygen data cannot. Therefore, we cannot say the patient is hypoxic because of a capillary oxygen tension of 48 mmHg. We can, however, state that the patient is hyperventilating and has respiratory acidosis as manifested by high CO2 and a low pH.

During weaning from mechanical ventilation in the SIMV mode with a mandatory rate of 8, the respiratory therapist notices the patient's spontaneous tidal volume is gradually decreasing and the total respiratory rate is increasing. Which of the following may benefit the patient? A. pressure support B. expiratory retard C. mechanical sighs D. decrease mandatory rate

The correct answer is : A Explanation : When patients are weaning from mechanical ventilation it is important that they have adequate tidal volumes. They must be able to sustain 5 mL per kilogram of their ideal body weight. In other words, if they weigh 75 kg their spontaneous tidal volume must be at least 375 mL. During the course of weaning if tidal volumes fall below 5 L per kilogram one way we can assist the patient is to implement pressure support. Pressure support is specifically used to increase the size of spontaneous tidal volumes.

Which of the following benefits from continuous low-flow supplemental oxygen should be expressed to a patient with COPD who is participating in a pulmonary rehabilitation program? A. increase ability to perform ADLs B. reverse lung disease C. establish normal pulmonary function volumes D. return to normal life

The correct answer is : A Explanation : When performing any kind of pulmonary rehabilitation or home care with the patient, it is important for the practitioner to understand they cannot reverse lung disease or return the patient to normal life. Patients with lung disease will never return to normal life because their condition is not reversible. Therefore, the appropriate goals and intentions of rehabilitation and education is to increase their ability to perform activities of daily living, generally increase exercise tolerance, reduce the incidence of infection and hospitalization, and generally improve their quality of life from their point of view.

Which of the following should be used on a patient who is intubated with a tracheostomy tube and is receiving mechanical ventilation for 7-10 days? A. heated humidification B. HME C. non-heated passover humidification D. ultrasonic nebulizer

The correct answer is : A Explanation : When providing humidity to the patient on a ventilator, because the natural humidification processes of the body are bypassed, 100% humidity must be provided through the ventilator. Only heated humidification can accomplish this. Non-heated passover humidification is inadequate. An HME is meant for short-term use only (a few hours). A centrifugal and large volume nebulizers are not devices used with the mechanical ventilation.

A patient receiving oxygen by nasal cannula at 5 L/min complains of dryness of the nasal passages. The respiratory therapist notices the patient is using a bubble humidifier and that the reservoir is one quarter full. The respiratory therapist should help the patient by A. Adding sterile water to the reservoir B. Applying a petroleum based ointment to the patient's nares C. Switching to a Venturi mask D. Utilizing a heated cascade humidifier

The correct answer is : A Explanation : When receiving flows greater than 1-2 liters per minute by nasal cannula, patients have a tendency to experience a drying of the nasal mucosal tissues. To combat thi,s humidification of the delivered gas by a bubble humidifier is appropriate. In this case, the patient's humidifier reservoir is nearly empty. As the bubble humidifier loses water its effectiveness decreases significantly because there is less distance from the bubbles to travel from the bottom of the humidifier to the top water level. Ensuring an adequate level of sterile water in the reservoir is important.

A respiratory therapist is having difficulty advancing a suction catheter down a fenestrated tracheostomy tube while the inner cannula is removed. The cannula appears to meet resistance within a few inches from insertion. The therapist should A. twist the catheter while attempting to advance B. remove the tracheostomy tube C. inflate the cuff D. obtain a smaller suction catheter

The correct answer is : A Explanation : When suctioning somebody with a fenestrated tracheostomy tube, it must be remembered that the suction catheters can sometimes get stuck on the fenestration hole inside the tube. The best remedy for this is to twist the suction catheter until the end is clear of the hole.

What is the maximum negative pressure that should be used to suction a 4-year-old patient who is intubated with a 5.0 mm endotracheal tube? A. -100 mm Hg B. - 80 mm Hg C. -110 mm Hg D. -120 mm Hg

The correct answer is : A Explanation : When suctioning, use the following indicated "gauge" pressures: _x000D_ Adult: 100-120 mm Hg (negative), Pediatric: 80-100 mm Hg (negative), Infant: 60-80 mm Hg (negative)

A COPD patient is in the emergency room. A quick assessment reveals a respiratory rate of 28, spontaneous tidal volume of 200 mL, use of accessory muscles, and venous distension. The patient is receiving oxygen by nasal cannula at 2 L/min and oxygen saturation by pulse oximetry is 88%. Which of the following would provide most help to the patient at this time? A. NIPPV B. adult mechanical ventilation C. flutter valve therapy D. non-rebreathing mask

The correct answer is : A Explanation : When the COPD patient has ventilatory difficulty beyond and above their baseline, managing their work of breathing is paramount. If not managed, the COPD patient can end up on a mechanical ventilator quite easily. The key modality used to decrease work of breathing is bilevel therapy, or noninvasive positive pressure ventilation. This allows time for the patient to breathe easier while other processes in the body are corrected. For instance, a COPD patient with an infection and consequent ventilatory difficulty may benefit from bilevel therapy until antibiotics have taken affect.

A patient is receiving heliox therapy with a mixture of 80% / 20% by nonrebreathing mask to reduce airway resistance during an asthmatic episode. The respiratory therapist notes the patient seems to be experiencing moderate respiratory distress. The therapist also observes the reservoir bag completely collapses with each inspiration. The therapist should A. increase total flow of gas mixture to the mask B. discontinue heliox therapy C. switch to a 60%/40% heliox mixture D. switch to a Venturi mask set at the highest possible FIO2 setting

The correct answer is : A Explanation : When the reservoir bag of a non-rebreathing mask collapses completely, which each breath, and this problem persists beyond a few minutes, the flow to the reservoir bag should be increased.

A patient who has an indwelling arterial catheter is being monitored for wide fluctuations in blood pressure. The ART line is zeroed while the pressure transducer is level with the lower abdomen. Ten minutes later, a blood pressure alarm sounds and the therapist discovers the patient has raised his arm above his head, causing the transducer to now be level with the head. Which of following would the therapist expect to observe? A. low indicated blood pressure B. accurate reported blood pressure C. high indicated blood pressure D. no reported blood pressure due to an error

The correct answer is : A Explanation : When the transducer of an ART line is raised above the level at which it was zeroed and calibrated, the blood that pumps against the transducer will exert less pressure as it attempts to climb up to the transducer. This, therefore, will cause the indicated blood pressure to appear lower than actual. The opposite is also true. If the transducer is placed below the point at which it was zeroed, gravity will cause the blood to exert additional pressure against the transducer diaphragm, which will produce an indicated blood pressure that is higher than actual.

A 38-week gestational age infant delivered 4 hours prior shows signs of hypoxemia. In preparation for oxygen administration at 30% by oxygen hood, the therapist should utilize which of the following devices? A. blender B. heat-moisture exchanger C. high-flow hydrator D. bubble humidifier

The correct answer is : A Explanation : When using an oxygen hood, it is appropriate to premix gases by use of an oxygen-air blender. Use of a large-volume nebulizer at low FIO2 could cause excessive noise transmitted to an oxygen hood which could promote hearing damage and restlessness of the infant. For this reason use of blender is more appropriate.

Pulmonary function testing is done on a patient with muscular dystrophy. The following pulmonary function data is recorded: Percent of Pred Actual value Fev1 80% FEF200-1200 75% FEF25-75 82% SVC 69% FVC 61% DLCO 24 CO/min/mmHg Which of the following most likely represents the patient's condition? A. kyphoscoliosis B. chronic bronchitis C. emphysema D. asthma

The correct answer is : A Explanation : When we examine these pulmonary function test results we see that the flow rates are normal as manifested by 80% of the predicted in the FEV1. When flows are normal, the patient is not obstructed. Answers that are obstructive diseases, therefore, cannot be correct. If the emphysema, chronic bronchitis, and asthma are all obstructive diseases, Kyphoscoliosis must be the only restrictive disease listed. We know the patient is restrictive because the SVC is less than 80% of predicted.

A respiratory therapist is asked to provide patient education for a newly diagnosed COPD patient being discharged home with continuous supplemental oxygen along with Combivent and Flovent MDIs. The respiratory therapist should not attempt to teach the patient how to A. improve pulmonary function values. B. reduce the work of breathing. C. change out a portable tank regulator. D. use a spacer with metered-dose inhalers.

The correct answer is : A Explanation : While these medications may actually improve pulmonary function values, it is not part of a teaching plan to encourage a patient to think this way because the patient does not understand this language and will become confused.

A patient is receiving oxygen by air-entrainment heated aerosol set at FIO2 0.60. The flow meter shows a flow of 12 L/min. What is the approximate total flow of gas to the patient? A. 36 L/min B. 24 L/min C. 60 L/min D. 12 L/min

The correct answer is : B Explanation : Each air-entrainment device, depending on the FIO2 chosen, has a specific ratio of oxygen to air. For 60% the air-oxygen ratio is 1:1. To determine total flow one must add each element of the ratio (in this case one plus one), and multiply is by the indicated flow on the oxygen flow meter, which in this case is 12 L/min. Therefore, the total flow is 2x12 L/min which equals 24 L/min.

An adult patient has a respiratory rate of 28, a pulse of 138, and a blood pressure of 85/65 mm Hg. Breath sounds are absent on the left and diagnostic percussion reveals hyperresonance on that same side. The respiratory therapist should FIRST recommend A. insert a chest tube in the left chest B. insert a 14-guage needle into the left chest C. radiographic examination of the chest D. schedule a V/Q scan

The correct answer is : B Explanation : The data shown here, especially pulse and blood pressure, indicate an emergent condition. Hyperresonance and absent breath sounds is suggestive of a tension pneumothorax. Because the tension is likely building, the patient is experiencing hemodynamic degradation that is dangerous. This is manifested by the low blood pressure. This problem should be remedied in an emergent way. This can be done by inserting a 14-gauge needle into the left chest, in between ribs. This will allow immediate venting of air and minor amounts of fluid to be ejected from the lung space and will provide immediate hemodynamic improvement.

Prior to beginning patient pulmonary function testing, a respiratory therapist uses a 3-L calibrated and certified syringe to confirm proper function of the pulmonary function testing equipment. The measurements are: 2.85 L, 2.99 L, 3.14 L. Which of the following should the therapist conclude? A. The PFT equipment requires calibration or maintenance B. The equipment is acceptable for patient testing. C. The syringe requires recalibration or recertification D. Room temperature should be lowered before retesting

The correct answer is : B Explanation : The three-liter calibrated syringe should be trusted above the pulmonary function equipment. Therefore, after injecting air into the pulmonary function machine and finding these values, it can be concluded that the machine is inaccurate and requires maintenance or calibration. Although the syringe used to calibrate is 3 L, some deviation is allowed. That deviation is 2.85 to 3.15 or plus +/- 5%. In this case, all values obtained are within the margin of error and are acceptable enough to proceed with patient testing.

A patient has a fenestrated tracheostomy tube in place. To prepare the patient for speech therapy, the therapist should do which of the following prior to placing a cap on the tracheostomy tube? A. insert the obturator B. ensure the cuff is deflated C. ensure cuff seal with minimum leak technique D. insert the inner cannula

The correct answer is : B Explanation : A fenestrated tracheostomy tube is capable of two different configurations. One configuration allows the patient to speak through a hole in the outer cannula of the tube. However, when the inner cannula is in place and the cuff is inflated, the tube may be used for positive pressure breathing. When changing from one configuration to another, several steps must be taken. Those steps include removing the inner cannula, deflating the cuff, and installing a tracheal button. This will allow the redirection of airflow over the vocal folds which are required for speech.

A 30-week gestational age infant is experiencing respiratory distress within an hour after delivery. A chest radiograph shows bilateral radiolucency with a ground glass appearance. Which of the following would be most beneficial in the care of the patient? A. Atropine B. Survanta (beracant) C. Aminophylline D. Albuterol sulfate

The correct answer is : B Explanation : A ground glass appearance on an infant's x-ray is indicative of lung immaturity. This may be treated with surfactant therapy. Of the options listed, only Survanta is a surfactant agent.

A ventilator-dependent patient is scheduled for an MRI, which will require a transport of 90 minutes. The patient is receiving humidification with a heated- humidification system. A transport ventilator is available. How should a respiratory therapist assure that humidification will be provided to the patient during the MRI procedure? A. Provide ventilation with a manual resuscitator B. Provide an HME C. Use the same humidification system in use on the primary ventilator D. Provide a pass-over humidifier

The correct answer is : B Explanation : A heat moisture exchanger (HME) is appropriate for a transport of only 90 min. Furthermore, the scenario does not suggest that the patient has thick, retained secretions, therefore an HME may be applied.

What is the primary purpose for a nitrogen washout test? A. determine closing volume B. determine FRC C. directly measure TLC D. evaluate evenness of pulmonary gas distribution

The correct answer is : B Explanation : A nitrogen washout test is used to determine three different lung volumes: TLC, RV, and FRC. For the NBCR exam the most important of those volumes is the FRC. Thus, when asked what a nitrogen washout test is for (or helium dilution test), the correct answer is FRC.

A physician in the emergency department has asked that a patient be evaluated for pulmonary obstruction while in the emergency room. Which of the following tests should the therapist plan on performing? A. chest radiographic film at full expiration B. beside pulmonary function testing C. maximum inspiratory pressure D. pulmonary assessment by body box

The correct answer is : B Explanation : A patient in emergency room will be unable to perform complex pulmonary function testing. However, to determine if the patient is obstructive or restrictive, a bedside pulmonary function test is adequate. Pulmonary function testing such as DLCO, nitrogen washout, helium dilution, and airway resistance determined by body box would not be appropriate at bedside in the emergency room.

A patient with a history of asthma has been receiving Aminophylline by IV over the last 24 hours in an attempt to promote bronchodilation. Which of the following data would be helpful in assessing the effectiveness of therapy? A. Determine the PD50 B. Evaluate Theophylline level C. Recommend a V/Q scan D. Perform a bronchoscopy to assess the degree of bronchocontstriction

The correct answer is : B Explanation : A patient taking xanthine medication, such as Aminophylline, should be monitored through a blood test called a theophylline level. The appropriate theophylline level should be between 10-20 micrograms. A level less than that is considered sub-therapeutic and is an indication that the amount of medication should be increased. A level higher than that range is associated with seizures and other adverse reactions that can be deadly. If this is found, the medication should not be merely reduced, but should be stopped until the level falls back into the acceptable range.

A patient is being evaluated for idiopathic dyspnea that occurs unpredictably at rest and during exertion. The following pulmonary function test reveals: Pre-bronchodilator Post-bronchodilator FEV1 2.6 L 2.6 L FVC 3.2 L 3.1 L Chest radiograph and ECG are normal. Which of the following should the therapist recommend NEXT? A. V/Q scan B. cardiopulmonary stress testing C. pulmonary angiography D. analysis of CSF

The correct answer is : B Explanation : A patient who complains of periodic dyspnea may benefit from pulmonary function testing. However, these pulmonary function results do not provide helpful information or show a possible reason for the dyspnea. Therefore, a greater examination is indicated. Cardiopulmonary stress testing can determine if the heart is the source of the problem. Analysis of cerebral spinal fluid, a VQ scan, and pulmonary angiography are specific tests that confirm diagnoses in other areas and are premature in this case.

A patient with cystic fibrosis complains of shortness of breath during a postural drainage and percussion treatment after the head of bed is down for 10 minutes. A bronchogram has revealed consolidation of secretions in the lateral basal segment of the left lower lobe. The respiratory therapist should modify the therapy in which way? A. Switch to IPPB therapy B. Reduce the time in Trendelenburg position C. Utilize reverse Trendelenburg position D. Use a pneumatic precursor

The correct answer is : B Explanation : A patient who experiences difficulty tolerating the head of bed down during postural drainage and percussion may benefit from decreasing the duration of the treatment.

A patient is receiving oxygen by non-rebreathing mask at a flow of 10 L/min. Spontaneous tidal volume is 500 mL and spontaneous respiratory rate is 20 /min. What change should the therapist recommend? A. Ensure an adequate mask seal on the patient's face B. Increase flow to 14 L/min C. Order arterial blood gas analysis D. Increase flow until reservoir bag collapses completely with each breath

The correct answer is : B Explanation : A patient who is breathing a tidal volume of 600 mL at a rate of 20/min has a minute ventilation of 12 L. If the non-rebreathing mask is set at only 10 L/min, the total flow to the patient is insufficient. The flowrate should meet or exceed the inspiratory demand of the patient. Therefore, increasing to 14 L/min is most appropriate.

Which of the following clinical data can be obtained from a patient who is comatose? A. dyspnea level B. oxygenation status C. subjective information D. vital capacity

The correct answer is : B Explanation : A patient who is comatose is unable to comply with verbal commands. Of the options offered, the only data that requires compliance to a verbal command is "vital capacity".

A patient is receiving IPPB therapy by mouthpiece following abdominal surgery and is still somewhat lethargic from the anesthesia. During the treatment, the respiratory therapist notes the IPPB machine frequently fails to cycle into expiration. Which of the following should the therapist do to correct the problem? A. Increase the flow B. Replace the mouthpiece with an inflatable mask C. Lower the pressure D. Increase the sensitivity

The correct answer is : B Explanation : A patient who is unable to keep their lips tightly sealed around the mouthpiece, while taking and IPPB therapy treatment, may benefit from the use of a lip-seal device or an inflatable mask.

A 49-year-old unconscious male is brought to the emergency room after a suspected suicide attempt. He was found in his garage with the car running. Which of the following should be recommended first? A. arterial blood gas analysis B. 100% oxygen by cool aerosol C. 10 L/min partial-rebreathing mask D. COHb analysis

The correct answer is : B Explanation : A patient who was found in a garage with a vehicle running should be suspected for carbon monoxide poisoning. The treatment for carbon monoxide poisoning is maximum FIO2. This could be accomplished by use of a 100% cool aerosol.

The respiratory therapist measures the functional residual capacity with a body box and determines the related value is 4.3 L. The FRC value is 3.4 L when determining via helium dilution method. Which of the following can explain the difference? A. hyperventilation secondary to panic in the body box B. significant amount of non-ventilated alveoli C. depleted helium analyzer D. inconsistent patient effort

The correct answer is : B Explanation : A patient with COPD has significant non-ventilated lung space. The measurement of total lung capacity, FRC, and RV done by helium dilution and nitrogen washout will likely be less accurate and show a smaller FRC compared to pulmonary function testing done by a body box. A body box can indirectly measure non-ventilated alveolar space.

A mal-positioned tracheostomy tube is detected by chest radiograph for a patient who is still receiving partial ventilation through the tube. Subcutaneous emphysema is present in the upper chest. Palpation of the affected area would produce A. soft tissue hyperlucency. B. popping or crackling sensations. C. air accumulation below the skin. D. dry crackles.

The correct answer is : B Explanation : A patient with an inappropriately positioned tracheostomy tube may experience air in the dermal and subdermal spaces of the skin around the neck and upper chest. Upon palpation air bubbles below this may move around or may burst, causing crackling or popping sensations during palpation.

A patient in a current myasthenic crisis is being monitored every 2 hours. The following data is available: 1 pm 3 pm 5 pm VC (L) 2.8 2.4 1.6 VT (mL) 500 485 400 MIP (cm H2O) -48 -35 -30 The respiratory therapist should recommend A. administer Tensilon B. continue to monitor the patient closely C. manually ventilate with a bag-valve-mask D. intubate and initiate mechanical ventilatory support

The correct answer is : B Explanation : A patient with myasthenia gravis experiences progressive paralysis of the muscles associated with ventilation, especially the diaphragm. This data shows a gradual decrease in ventilatory ability. However, the patient has a vital capacity well over 1 L, indicating that mechanical ventilatory support is not yet needed. However, this patient should be monitored closely for further degradation in the ventilation status. Once VC falls below 1 L, mechanical ventilatory support is indicated.

A respiratory therapist notes a pressure-volume loop on a patient receiving mechanical ventilation has a pronounced beak. What strategy is useful in this case? A. increasing flow B. decreasing tidal volume C. decreasing flow D. increasing PEEP

The correct answer is : B Explanation : A pressure volume loop on a ventilator that shows a significant beak indicates inappropriate ventilation. It indicates that a massive increase in pressure is required to deliver the final part of a tidal volume. This is undesirable because an increase in mean airway pressure has many negative affects on the patient, including decreased venous return, among other things. The solution is to sacrifice some of the tidal volume and therefore reduce pressure. A proper pressure volume loop will show a very tiny beak. This beak indicates the point of over distention of the lung and is a good indicator that we are filling the lung properly. Achieving this small beak will help to increase alveolar recruitment, aerate distal parts of the lungs, and help mobilize and remove secretions.

A patient with adult respiratory distress syndrome is receiving mechanical ventilation with a pressure-cycled ventilator. Which of the following alarms is most important? A. low pressure alarm B. low-volume alarm C. high pressure alarm D. I:E ratio alarm

The correct answer is : B Explanation : A pressure-cycled ventilator delivers a breath at a specific set pressure in spite of the volume achieved. The machines focuses on pressure without regard to delivered volume. Therefore, the respiratory therapist should be focused on volume. The most important alarm on a pressure-cycled ventilator is a low return volume alarm.

Which of the following data, if observed, would indicate improper positioning of an oral endotracheal tube? A. chest radiograph shows tube 2 cm above the carina B. ET tube markings at 29 cm C. radiologist reports the ET tube is level with the aortic arch D. symmetrical chest rise

The correct answer is : B Explanation : A properly placed endotracheal tube will cause even chest rise and symmetry during ventilation. ET tube markings will be in the low 20s, if an oral intubation. A chest x-ray will show the end of the tube 2 to 5 cm above the carina or level with the aortic arch. Thus, all options indicate inappropriate positioning of the endotracheal tube.

A patient is in the intensive care unit receiving volume-controlled ventilation and is hemodynamically unstable. A new balloon-tipped, flow-directed, pulmonary artery catheter has been placed. The pulmonary artery waveform on the monitor is rising and falling with inflection points at 25 mmHg and 8 mmHg. Which of the following can the respiratory therapist conclude about the positioning of the pulmonary artery catheter? A. the catheter should be withdrawn B. the catheter is properly positioned C. the balloon should be deflated D. the catheter should be advanced

The correct answer is : B Explanation : A pulmonary artery catheter should be placed so that the distal end rests in the pulmonary artery. When monitoring the pressure in the pulmonary artery, we may look at mean pressures or the data may be in the form of a systolic and diastolic pressure. In this case, the systolic pressure of 25 mmHg is given and the diastolic pressure is 8 mmHg. This indicates that the pulmonary artery catheter is properly placed.

The following pulmonary function data is obtained from a 65-year-old adult male who is 5 ft 2 in (157-cm) tall and weighs 120-lbs (55-kg). Percent (%) of Predicted SVC 75 FVC 72 FEV1.0 39 FEF25-75 52 FEF200-1200 76 DLCO 89 Which of the following most accurately represents the patient's condition? A. moderate obstructive defect B. mild restriction with severe obstructive defect C. normal spirometry D. severe restrictive defect only

The correct answer is : B Explanation : A slow vital capacity of 75% indicates a mild restriction. An FEV1 of 39% of predicted indicates a severe obstruction.

A 32 year-old female patient reports to the emergency room complaining of a sudden onset of coughing. She indicates the coughing started when she was eating. Which of the following would most definitively rule out a cancerous mass in the lungs: A. V/Q scan B. spiral CT scan C. lateral neck radiograph D. BUN

The correct answer is : B Explanation : A sudden onset of coughing while the patient was eating is most likely associated with foreign body aspiration (food in the main stem bronchus). However, nonproductive coughing could also be caused by a cancerous mass in the upper airway. To rule this out a procedure that shows the upper airway with three dimensionality is most appropriate. A ventilation perfusion scan and a BUN blood test would not do this. It may be tempting to get a lateral neck x-ray, but the results are not three dimensional and therefore would not be as confirming in nature. The most confirming examination would be a spiral CT scan.

The best way to determine the accuracy of a vane respirometer is a A. Geizler tube ionizer B. 1.5-liter calibrated syringe C. Douglas bag D. galvanic analyzer

The correct answer is : B Explanation : A vane respirometer is a device that measures volume and must be calibrated from time to time. This can be done with a three-liter super syringe or a syringe that comes in 1.5 L. This syringe is certified and calibrated for accuracy and should be trusted above all other instruments in the pulmonary function laboratory.

A V/Q scan would be most helpful in evaluating which of the following? A. restrictive lung disease B. pulmonary embolism C. obstructive lung disease D. ARDS

The correct answer is : B Explanation : A ventilation-perfusion scan is helpful at examining the blood flow around the alveoli and in conjunction with the air flow into the alveoli. Together these two elements make up perfusion and ventilation respectively. A V/Q scan cannot be used to diagnose restrictive or obstructive lung disease and is not helpful in diagnosing adult respiratory distress syndrome.

A patient with asthma monitors their peak flow in the morning and documents that peak flow is 60% of his usual baseline. Based on the NAEP and the patient's asthma action plan, the patient should A. check peak flow again in 2 hours B. take a short-acting beta 2-agonist, continue to monitor peak flow C. contact their physician D. report to the hospital

The correct answer is : B Explanation : According to the national asthma guidelines, a patient who is self-monitoring peak flow and is only able to achieve 60% of baseline should first take a short-term bronchodilator (a short acting beta-II agonist), and continued to monitor peak flows.

Following a cardiac arrest and emergency intubation, breath sounds are bilaterally equal with manual ventilation on a male patient in the emergency room. Capnography shows an end-tidal CO2 of 5 mm Hg. What would the respiratory therapist expect to happen to the ETCO2 over the next few minutes with adequate ventilation provided? A. a sudden rise to normal B. gradually increase C. gradual decrease D. remain low

The correct answer is : B Explanation : After cardiac arrest, which includes ventilatory arrest, end-tidal CO2 will be very low because niether ventilation nor perfusion have been occurring, and there will be very little CO2 in the alveoli. As circulation and ventilation increase with manual ventilation, the CO2 should start to enter the lung, causing a gradual increase in exhaled CO2.

A respiratory therapist is evaluating a patient who is receiving volume-controlled ventilation through a tracheostomy tube. After inserting 15 mL of air into the tracheostomy tube cuff, the therapist measures the cuff pressure and notes it is at 5 cm H2O. The therapist should A. add 15 mL additional air to the cuff B. replace the tracheostomy tube C. cut the pilot tube D. record the pressure in the patient's medical record

The correct answer is : B Explanation : After inserting air into the tracheostomy tube cuff and noticing that the cuff pressure does not rise, the therapist can conclude that the tube is defective and should be replaced. Although there are other methods to temporarily keep the cuff inflated, the best answer is always to replace the tube because it is considered damaged. Damaged equipment should not be used.

A 56-year-old patient with a history of asthma, and who takes beta-blocking medication, reports no relief when taking Albuterol as a rescue medication. The respiratory therapist should recommend: A. doubling the frequency of Albuterol treatments B. switching to ipratropium bromide (Atrovent) C. doubling the dose of the Albuterol D. combining the Albuterol with a corticosteriod

The correct answer is : B Explanation : Albuterol is an example of a beta adrenergic medication used for bronchodilation. However, if the patient is on beta-blockers, it is possible it will render the albuterol ineffective. A suitable alternative would be ipratropium bromide (Atroven).

Which of the following should be monitored for a ventilator-dependent newborn with IRDS? A. pH B. fluid input and output C. urine specific gravity D. gas distribution (SBN2) in the lungs

The correct answer is : B Explanation : All choices are indicated while monitoring a critically ill neonate receiving mechanical ventilation. I & O fluid monitoring is essential. Managing acid-base balance and the artificial airway are also essential.

What pulmonary function maneuver will reveal information required to diagnose COPD? A. SVC B. FVC C. SBN2 D. DLCO

The correct answer is : B Explanation : Although FVC is a volume, which is not directly helpful in diagnosing COPD, the actual name of the maneuver that is used to establish flow rates such as FEV1, is forced vital capacity (FVC). This can be confusing because the FVC is an actual volume, but when compared to time it also produces flow data, which is used to diagnose obstructive lung disease.

A patient who is receiving mechanical ventilation is scheduled for fiberoptic bronchoscopy to investigate a lesion in the right mainstem bronchus. Which of the following represents the most significant threat to ventilation during the procedure? A. stimulation of the vagal nerve and bradycardia B. airway obstruction C. loss of PEEP through suctioning D. excessive airway resistance

The correct answer is : B Explanation : Although a bronchoscopy procedure is necessary to investigate deep pulmonary problems, such as lesions, the procedure itself can cause problems because of the width of the scope relative to the size of the airway. During the procedure the scope acts as a large airway obstruction. If the patient is receiving mechanical ventilatory support, it is likely that the high-pressure alarm will sound during this procedure. Therefore, it is appropriate to temporarily increase the high-pressure limit until the procedure is complete.

A spontaneously breathing patient has the following clinical and laboratory data: Respiratory rate 30/min Exhaled VT 340 mL FIO2 0.40 pH 7.34 PaCO2 45 torr PaO2 78 torr HCO3- 26 mEq/L BE +1 mEq/L The respiratory therapist should recommend? A. Hyperbaric therapy B. Bi-level therapy C. CPAP therapy D. Place the patient on a nonrebreathing mask

The correct answer is : B Explanation : Although the ventilation and oxygenation status of this patient are technically adequate, the increased respiratory rate and reduced spontaneous tidal volumes are an indication that the patient is experiencing increased work of breathing and may ultimately experience ventilatory failure. This may be best resolved by administering pressure support ventilation. Of the options shown, Bi-level therapy will be most helpful.

Which of the following should be monitored in a patient receiving aminophylline for long-term bronchodilation? A. serum electrolytes B. blood theophylline level C. ABGs D. CBC

The correct answer is : B Explanation : Aminophylline is a xanthine medication used to establish long-term bronchodilation in patients who have asthma and other types of airway constriction diseases. When a patient is receiving this type of medication, the proper blood level must be maintained. This blood test is called a theophylline level. It should be maintained between 10 and 20 µg. A level that is too high can cause seizures and death. A level that is too low would be sub therapeutic and not helpful.

One minute after birth, a 30-week gestational age infant has an APGAR score of 2. Which of the following should the therapist be doing to assist the infant? A. chest physiotherapy B. basic life support C. provide mechanical ventilation D. surfactant therapy

The correct answer is : B Explanation : An APGAR score between 0-3 necessitates basic life support, including cardiac compressions. Scores between 4-6 indicate a need for supplemental oxygen and general stimulation of the infant. Scores greater than 6 (7-9) are normal and require no specific intervention other than routine treatment of the infant.

Which APGAR score necessitates the delivery of basic and/or advanced cardiac life support for a newborn? A. 5 B. 2 C. 10 D. 7

The correct answer is : B Explanation : An APGAR score is a 10-point scoring system that helps to know what kind of intervention for a newborn is appropriate. An APGAR score between 0-3 indicates immediate cardiopulmonary resuscitation. A score between 4-6 suggests that supplemental oxygen and general stimulation of the infant is required. A score between 7-10 indicates a normal, healthy infant, which means routine care is appropriate.

In preparation for an arterial puncture in the right radial artery, the respiratory therapist performs a modified Allen's test. When the ulnar artery is released, color returns to the hand in 35 seconds. The therapist should A. perform the puncture in the right radial artery B. perform an Allen's test on the left ulnar and radial arteries C. perform a brachial artery puncture D. perform a femoral artery puncture

The correct answer is : B Explanation : An Allen's test is an assessment for collateral blood flow in to the hand in preparation for a radial arterial puncture. The presence of bilateral blood flow into the hand reduces risk of the arterial puncture because there is another source of blood if the radial artery is damaged. The procedure occurs by occluding both the radial and ulnar artery and allowing the hand to blanche or the blood to leave the hand. When the ulnar artery is released the amount of time required to not color in the hands is observed. Blood must return in under seconds. A duration of 35 seconds is too long in indicates poor blood flow through the ulnar artery. This would suggest an alternative site should be considered.

Which pH level is expected for a COPD patient having an acute on chronic exacerbation? A. 7.49 B. 7.44 C. 7.30 D. 7.35

The correct answer is : B Explanation : An acute on chronic episode occurs when a COPD patient with compensated respiratory acidosis experiences an exacerbation, which causes their respiratory rate or overall minute ventilation to significantly increase, usually due to hypoxemia. The "acute" increase in minute ventilation drives the pH upward and even the PaCO2 is reduced from its normally very high level to a new level that still may be higher than the normal range but low for the patient. PaO2 is often significantly low in these scenarios. HCO3- will be significantly increased. These are difficult blood gases to interpret, and require critical thinking by the respiratory therapist.

A patient with a history of cor pulmonale is receiving oxygen at 2 L/min by nasal cannula. The physician orders a target SpO2 of 90%. The following clinical and laboratory data is obtained: RR 18 HR 110 BP 145/92 mm Hg SpO2 88% pH 7.35 PaCO2 72 torr PaO2 55 torr HCO3- 33 mEq/L BE +2 mEq/L The respiratory therapist should recommend which of the following FIRST? A. place on NIV B. switch to an air-entrainment mask at 28% C. administer oxygen by nonrebreather mask D. increase oxygen flow to 4 L/min

The correct answer is : B Explanation : An air-entrainment device is superior at delivering a consistent FIO2, which could help this patient achieve an SpO2 of 90%.

A patient undergoing pulmonary function testing in a body box has an airway resistance (Raw) of 2.7 cm H2O/L/sec. The respiratory therapist should provide which of the following interpretations? A. Guillain-Barre Syndrome B. asthma C. restrictive pulmonary disease D. normal results

The correct answer is : B Explanation : An airway resistance of 2.7 cmH2O/L/sec is considered high. Elevated airway resistance is associated with asthma.

An artificial nose is A. sometimes called a nasal trumpet. B. ineffective with copious secretions. C. ideal for post operative patients with hypothermia. D. helpful for thinning thick secretions.

The correct answer is : B Explanation : An artificial nose, also known as a heat moisture exchanger (HME), may easily become clogged with secretions and rapidly loses its effectiveness in the presence of thick, copious sputum. It should be replaced with a conventional heated humidification system when secretions are a problem for the patient.

A patient is undergoing a full cardiopulmonary arrest. The patient is intubated and is being monitored with an infrared capnographic device. PetCO2 data is showing 3%. Which of the following best explains this value? A. increased PAO2 B. poor alveolar perfusion C. decreased C(a-v)O2 D. poor systemic perfusion

The correct answer is : B Explanation : An end-tidal CO2 of only 3% is low. This indicates poor ventilation. In looking at the answers the best one is poor alveolar perfusion. Poor systemic perfusion does not relate and increased alveolar oxygen tension does not relate to low alveolar CO2.

While coaching a patient on the use of a volume-type incentive spirometer, the respiratory therapist notices the patient is exhaling forcefully into the mouthpiece and that there is no rise of the indicator (no volume is achieved). The therapist will provide which of the following instructions? A. "perform an inspiratory capacity maneuver" B. "inhale as deeply as possible" C. "exhale more forcefully" D. "breathe in and out rapidly"

The correct answer is : B Explanation : An incentive spirometer is a device that measures the inspiratory capacity of a patient and therefore requires the patient to perform an inspiratory maneuver. In this case, the patient is found to be exhaling into the device rather than inhaling. Appropriate instruction should be provided to correct the problem.

A postoperative patient for cardiac surgery is to perform sustained maximum inspiration therapy with an incentive spirometer. Prior to surgery the patient achieved 1600 mL. The respiratory therapist should use which of the values as a postoperative, initial goal? A. 1200 mL B. 800 mL C. 1800 mL D. 1600 mL

The correct answer is : B Explanation : An initial goal for incentive spirometry postoperatively is one half of the patient's achievement prior to surgery. In this case, the patient was able to achieve 1600 mL prior to surgery. The initial goal should be 800 mL. If the patient is unable to obtain that amount or comes far from that goal, the goal should be lowered to a volume that is closer to the patient's achievement. This will incentivize the patient keep trying.

A patient is found on the side of the road, asleep in his car. CO poisoning is suspected. Which of the following is helpful to rule in or out CO poisoning?

hemoximetry

How many hours will an H cylinder with 1400 psi last for a patient receiving oxygen at 7 lpm? A. 6 hours B. 10 hours C. less than 1 hour D. 4 hours

The correct answer is : B Explanation : And H cylinder has a tank factor of 3.14. 3.14x1400 PSI = 4396 L. 4396 L / 7 L per minute = 628 min. 628 min./ 60 min.= 10.4 hours, or about 10 hours. When you get an answer that is not exact, pick the closest number possible. It is common on the exam not see the exact answer from your calculation. This is because the NBRC knows that you're estimating.

A respiratory therapist receives a phone call from a home care patient who is receiving oxygen by pulse-dose oxygen delivery system. The patient complains that she does not feel enough oxygen is coming out. The therapist should instruct the patient to A. report to the emergency room B. switch the flow meter to continuous mode C. discontinue oxygen use until a therapist arrives D. instruct the patient to call their physician

The correct answer is : B Explanation : Any time a home care patient complains of not receiving enough oxygen, before troubleshooting or attempting to repair the situation, the patient should be instructed to switch to a different modality to ensure oxygen delivery. The patient should not be asked to find the problem or to troubleshoot the problem before switching to a modality that is working. In this case, a pulse-dose oxygen delivery device can be switched to continuous delivery mode.

A patient is receiving supplemental oxygen therapy at FIO2 0.60 with heated humidity by large volume air-entrainment nebulizer. What can the respiratory therapist expect to occur with FIO2 as excessive water develops in the aerosol tubing? A. decrease B. increase C. will remain unchanged D. will rise to 1.0

The correct answer is : B Explanation : Anytime an entertainment device encounters back pressure that develops in the device as a result of a kinked tube or water in the tubing, the result will be an increase in FIO2. This is because back pressure on the air-entrainment device will cause the gas passing through the device to slow. The slowing of gas causes the relative pressure to increase. When the pressure increases, less room air is entrained. This results in an increase in FIO2.

A respiratory therapist notes during a ventilator/patient check that the RAW is lower than in previous assessments. One explanation for this change is a reduced A. waveform. B. flowrate. C. fever. D. I:E.

The correct answer is : B Explanation : As flow rate increases airway resistance also increases due to the turbulence of air movement. Therefore, reductions in flow rate will likewise reduce airway resistance.

A patient with asthma is receiving a second dose of albuterol, 1.25 mg. During the treatment, the patient's HR increases from 90 to 119. The respiratory therapist should A. administer budesonide B. switch to Xopenex 0.63 mg C. switch to tiotropium bromide (Spiriva) D. decrease the dosage of albuterol

The correct answer is : B Explanation : As this patient's heart rate changes with the albuterol treatment from 90 to 119, it must be assumed that the patient is experiencing an adverse reaction to the albuterol. Normally, reducing the dose would be appropriate. However, this patient is already on the lowest therapeutic dose as listed in the PDR. Therefore, of the options given, a change in bronchodilator medication is most appropriate. Xopenex offers the most appealing change as it is a similar class and type of bronchodilator.

An oxygen-dependent patient ordered for 2 L/min is planning a trip that will last 24 hours. The plane will allow up to 4 extra E-cylinders per passenger. What should the respiratory therapist recommend? A. reduce flow to 1.5 L/min B. pulse-dose unit C. cancel the trip D. go on the trip with 4 extra E-cylinders

The correct answer is : B Explanation : At 2 L per minute a 24-hour trip will require at least 5 E cylinders to complete. Of the options offered use of a pulse-dose unit is the only option that will conserve oxygen enough to allow the patient to take only four tanks. Reducing the flow will also accomplish this but is considered a deviation from physician orders and is not clinically appropriate for the patient

A known COPD patient presents to the emergency room with extreme shortness of breath and copious amounts of thick, yellow sputum produced over the past three days. Respiratory rate is 32/min. ABGs on 1 L/min nasal cannula are: pH 7.45 PaCO2 47 torr PaO2 49 torr HCO3- 33 mEq/L BE +4.1 mEq/L The proper interpretation for the arterial blood gas is A. compensated metabolic alkalosis B. compensated respiratory acidosis C. the blood gas analysis is in error D. compensated respiratory alkalosis

The correct answer is : B Explanation : At first, the respiratory therapist may believe that the patient has experienced alkalosis because the pH is on the high-end of the normal range. However, with a closer review you will see that the patient has a very high HCO3-, is known to have COPD, and is likely experiencing an acute on chronic episode. This is further confirmed with a very low PaO2 of 47 torr. In fact, the low oxygen status is causing a much higher than normal minute ventilation due to air hunger. This is the cause of the pH being in the high-end of the range. The patient's normal blood gas values probably show a pH on the low side of the range, a high PaCO2, with a high HCO3-.

A 3 day-old premature infant has a heart rate of 50/min. Which of the following medications would be most beneficial to the patient? A. administer racemic epinephrine B. administer 0.2 mg Atropine sulfate C. administer dopamine HCl D. administer subcutaneous epinephrine 1:10,000

The correct answer is : B Explanation : Atropine a be administered to an infant in order to increase heart rate. 0.2 mg of atropine is an appropriate dose.

A pulmonary rehab patient has the following arterial blood gas on air: pH 7.37 PaCO2 50 torr PaO2 62 torr HCO3- 30 mEq/L SaO2 87% COHb 6.5% What should the respiratory therapist suspect about this patient? A. there is no long-term lung damage B. recent exposure to cigarette smoke C. rehab should be discontinued D. rehab is effective

The correct answer is : B Explanation : Because COHb is increased (more than 1-2%), the patient should be suspected for recent cigarette smoking.

A patient is receiving ventilatory support with a time-cycled ventilator. Assuming the inspiratory time and mandatory rate remain unchanged, a decrease in flow rate would result in which of the following? A. increase in FIO2 B. decrease in tidal volume C. increase in inspiratory time D. decrease in expiratory time

The correct answer is : B Explanation : Because inspiratory time and mandatory rate are fixed, a decrease in flow would result in a decrease in delivered tidal volume.

While performing endotracheal suctioning on a cystic fibrosis patient who is intubated and receiving VC SIMV ventilatory support, the respiratory therapist discovers that the suction has immediately stopped. Which of the following is the most likely cause?

The suction canister is full.

Which of the following do NOT contribute to total airway resistance for a mechanically ventilated patient with an endotracheal tube? A. main bronchi B. upper half of the trachea C. bronchoconstriction D. endotracheal tube

The correct answer is : B Explanation : Because the upper half the trachea is not communicating with the airway when an endotracheal tube is in place, it does not contribute to total airway resistance. The endotracheal tube cuff is inflated in the lower half of the trachea.

A patient is receiving continuous oxygen by transtracheal catheter with a flow rate of 1 L/min. The patient complains that he is not getting enough air. Which of the following is most appropriate? A. place the patient on a nasal cannula at 1 L/min B. place the patient on a nasal cannula at 2 L/min C. remove the catheter D. flush the catheter with saline

The correct answer is : B Explanation : Before troubleshooting the transtracheal oxygen catheter is appropriate to switch the patient to a more reliable oxygen source. When switching from a transtracheal oxygen catheter to a nasal cannula, the flowrate should be doubled. This will approximate the FIO2 achieved by the transtracheal catheter.

A physician has inserted a CVP line in a 78-year-old patient who has cor Pulmonale. The respiratory therapist should expect to see a placement radiograph that shows the end of the catheter positioned in which of the following areas? A. pulmonary artery B. superior vena cava C. right ventricle D. level with the aortic knob or notch

The correct answer is : B Explanation : CVP is determined by measuring the pressure in the superior vena cava or right atrium.

A patient is being monitored hemodynamically with a balloon-tipped, flow directed pulmonary artery catheter. The following hemodynamic data is available: PAP 22 mm Hg CVP 8 mm Hg PCWP 16 mm Hg C.O. 3.2 L Which of the following mostly likely represents the patient's condition? A. cor pulmonale B. mitral valve stenosis C. dehydration D. pulmonary hypertension

The correct answer is : B Explanation : CVP is high. PAP is also high. This indicates no problem in the right heart. PAP as high and pulmonary capillary wedge pressure is also high. This suggests no problem in the lungs. PCWP is high but cardiac output is low. This suggests there is a problem in the left heart. The only option that is associated with a left heart condition is mitral valve stenosis.

What equipment should a respiratory therapist gather when standing by to assist the physician with a cardioversion? A. mechanical ventilator B. manual resuscitator C. IPPB D. bronchoscope

The correct answer is : B Explanation : Cardioversion should be done with resuscitation equipment available because the sedation may severely compromise a patient's airway, and the procedure itself may cause the patient to slip into more deadly rhythms like pulseless V-tach or V-fib.

A radiographic image shows an upper lobe cavitation. Which of the following conditions is most closely associated with this finding? A. Asbestosis B. Tuberculosis C. Bronchiectasis D. Pneumonia

The correct answer is : B Explanation : Cavitations in the upper lobes are generally associated with tuberculosis. Bronchiectasis is a condition affecting the bronchioles or airways and is seen throughout the lung. Pneumonia is not a cavitation and can also be seen throughout the lung on X-ray. Asbestosis is seen throughout the lung with reduced lucency.

A respiratory therapist is examining the chest radiograph of a patient who is receiving volume-controlled ventilation and who has a chest tube drainage system in place in the right side, fourth interspace, mid axillary line. Where should the respiratory therapist expect to see the end of the chest tube if it is properly placed? A. in the right lung B. in the pleural space C. in the chest cavity D. in the parietal area of the lung

The correct answer is : B Explanation : Chest tubes, for the purpose of draining air or pleural fluid, should be positioned in the pleural space.

A respiratory therapist is making a list of recommendations that could be helpful to a patient with obstructive sleep apnea. The patient is 5-ft (152-cm) tall and weighs 86-kg (190-lb). Which of the following are appropriate recommendations? A. Dopram, nocturnal full face mask CPAP B. weight loss and nocturnal nasal CPAP C. Aminophylline, bi-level therapy by nasal mask D. supplemental oxygen at night

The correct answer is : B Explanation : Correction of obstructive sleep apnea can be accomplished immediately though the administration of nocturnal nasal CPAP. Long-term corrections may include weight loss and surgical resection of soft tissues in the upper airway.

Which of the following pulmonary function test trails represents the best patient effort? Trial 1 Trial 2 Trial 3 FEV1 (L) 2.2 2.2 2.7 FVC (L) 2.7 2.7 2.3 IC (L) 2.4 1.9 2.1 FEF25-75% (L/sec) 1.8 1.3 1.3 A. Trial 2 B. Trial 3 C. Trial 1 D. set of trials do not meet ATS standards

The correct answer is : B Explanation : Determining the best patient effort in a pulmonary function test is done by adding the FEV1 and FVC. The trial with the highest sum is considered the "BEST TEST".

After performing a forced vital capacity maneuver as a part of a complete pulmonary function study, the respiratory therapist inspects the data to determine the consistency of the values. In doing so, the respiratory therapist is assessing for A. quality control B. precision C. calibration D. accuracy

The correct answer is : B Explanation : Determining the consistency of measured values for pulmonary function machine is related to assessment of precision. Precision relates to how close the values are to one another but does not relate to the accuracy. In other words, a pulmonary function machine may be precise but not accurate.

A respiratory therapist is assigned to monitor a mechanically ventilated male patient with the following settings and clinical data: Mode Pressure/control Mandatory Rate 20/min FIO2 0.75 PIP 30 cm H20 I:E ratio 2:1 PEEP 22 cm H20 pH 7.35 PaCO2 50 torr PaO2 60 torr Based on this data, which of the following conditions is most likely? A. Decreased QS/QT B. Acute Respiratory Distress Syndrome C. High A-aDO2 value D. Elevated oxygen index (OI)

The correct answer is : B Explanation : Due to the high FIO2 and PEEP associated with a low PaO2, ARDS is very likely. In such a case, A-aDO2 and oxygen index (OI) would likely be decreased. QS/QT would likely be increased.

Which of the following patient conditions would CONTRAINDICATE the use of Trendelenburg's position? A. Independent lung ventilation B. Increased intracranial pressures C. Massive blood loss D. Lobectomy

The correct answer is : B Explanation : Placing the patient's head-of-bed down would be harmful to the patient, who has increased intracranial pressures. This could cause further increase which could result in herniation of the brainstem and permanent neurological damage.

Polysomnography reveals a patient has chest movement with no nasal airflow and severe oxygen desaturation. Which of the following therapies should the respiratory therapist recommend for the patient? A. nocturnal mechanical ventilatory support B. nasal CPAP C. oxygen via nasal cannula when sleeping D. Doxapram (dopram)

The correct answer is : B Explanation : During a sleep study, the presence of chest movement indicates the patient is attempting to breathe. The absence of nasal airflow indicates obstruction, or the absence of air movement. Together, these two data support a diagnosis of obstructive sleep apnea. The gold standard for treatment, in this case, is nocturnal nasal CPAP or BIPAP.

A patient is undergoing a maximal exercise tolerance test. During the exam, as workload is increased, an increase in heart rate from 90 to 120 bpm. Blood pressure has increased from 110/80 to 128/90 mmHg. Which of the following can be correctly stated about the exam results? A. abnormal cardiac response, abnormal response in blood pressure B. normal cardiac response, normal response in blood pressure C. abnormal cardiac response, normal response in blood pressure D. normal cardiac response, abnormal response in blood pressure

The correct answer is : B Explanation : During cardiac or exercise stress testing, blood pressure and heart rate should increase with workload. In some cases, one may increase without the other. When this happens, the patient has exceeded his or her maximum exercise tolerance level. In this case the heart rate increased as did blood pressure. Therefore, the patient is said to have a normal cardiac response and a normal blood pressure response to an increase in workload. This is what we expect to see any normal patient.

A patient receiving volume-controlled ventilation in SIMV mode has a total respiratory rate of 26/min and is showing signs of increased labor of breathing. The mandatory rate is 14/min. Which of the following would most likely help the patient? A. Increase the machine flow rate B. Use of pressure support C. Increase PEEP D. Switch to pressure control ventilation

The correct answer is : B Explanation : During ventilator weaning, a patient must maintain a moderately low respiratory rate, an adequate sized tidal volume, and low work of breathing. In this case, the patient is experiencing increased labor of breathing and an increase in respiratory rate. This is likely due to a reduced spontaneous tidal volume. Although this data is not shown, this condition can be assumed. The solution for a low spontaneous tidal volume and increased work of breathing during weaning is to provide pressure support.

While performing routine oxygen rounds, the respiratory therapist notes a COPD patient who is receiving supplemental oxygen at 2 L/min is markedly cyanotic and has a heart rate of 30/min. The therapist should FIRST A. increase flow to 4 L/min by nasal cannula B. switch to FIO2 1.0 C. go get help D. obtain an arterial blood gas

The correct answer is : B Explanation : Even though a COPD patient should rarely receive more than 2 L/min oxygen, there are emergency circumstances that would dictate more supplemental oxygen. The use of the word "markedly" is an indication of an emergency. In this case the patient is markedly cyanotic and therefore has an oxygenation emergency. Switching to 100% oxygen is appropriate.

A 45-year-old female with leukemia is receiving 10 cm H2O CPAP with an FIO2 0.50. Arterial blood gas results are below: pH 7.38 PCO2 42 torr PO2 109 torr HCO3- 28 mEq/L BE 0 mEq/L WBC 2,000 cu mm Hb 8.0 g/dL RBC 2.0 g/dL The respiratory therapist should conclude that the patient has A. methemoglobinemia B. hypoxemia C. compensated respiratory acidosis D. a bacterial infection

The correct answer is : B Explanation : Examination of this blood gas PO2 reveals that the patient is over oxygenating. However, a closer examination reveals anemia as shown by a hemoglobin level well below normal limits. When a patient is anemic they are considered to be hypoxic, regardless of the PaO2.

A neonate born 48 hours ago is receiving time-cycled, pressure-limited mechanical ventilation in the NICU. How should the patient's fluid output be measured? A. Insert a urinary catheter B. Weigh the diaper C. Periodically weigh the neonate D. Press and assess duration for blanched skin to return to normal

The correct answer is : B Explanation : Fluid control is vital in managing the care of a neonate. It may not seem very scientific to weigh the diaper, but it is the most reliable and practical of the choices offered.

Which of the following is used to evaluate a patient for fluid imbalance? A. minute ventilation B. pitting edema C. P50 D. cerebral perfusion pressure

The correct answer is : B Explanation : Fluid imbalance may result in pitting edema, changes in sensorium, and altered capillary refill time. Minute ventilation, P50, and CPP is not helpful.

A patient with a history of COPD is receiving mechanical ventilatory support in SIMV mode with a flow-triggering baseline flow of 18 L/min. During spontaneous expiration, the respiratory therapist notices the pressure manometer needle remains higher than the set PEEP level. The therapist should A. monitor the patient closely for 20 minutes B. decrease the baseline flow C. increase the level of PEEP D. obtain an arterial blood gas in 20 minutes

The correct answer is : B Explanation : For a patient who is using flow-triggering during mechanical ventilatory support, if the pressure manometer needle remains higher than the PEEP, the appropriate response is to decrease the baseline flow.

A patient has just been diagnosed with obstructive sleep apnea and obesity hypoventilation syndrome. In addition to nasal nocturnal CPAP, on what else might the therapist educate the patient with regard to treatment options? A. neutrally adjusted ventilatory assist devices B. weight loss surgery C. respiratory stimulating medication such as Dopram (doxapram) D. smoking alternatives

The correct answer is : B Explanation : For a patient with obstructive sleep apnea, likely due to obesity, there are several options that can help treat the issue. One is weight loss, which can be done surgically through bariatric surgery. Another surgical procedure, though not offered here, consists of resection of various soft tissue structures in the oropharynx.

A volume-cycled mechanically ventilated male patient intubated with a 6.5 mm endotracheal tube is on the following settings: Mode Assist/control Set rate 10 Tidal volume 600 mL Peak pressures are consistently exceeding 35 cm H20, with static compliance in the normal range. In order to correct the high peak pressures, the respiratory therapist would FIRST consider which of the following? A. Change to PCV. B. Replace the endotracheal tube with a larger size. C. Increase the flowrate. D. Suction the patient every 30 minutes.

The correct answer is : B Explanation : In this example the endotracheal tube at 6.5 mm is likely too small, creating increased airway resistance. This is resulting in high peak pressures. A larger diameter endotracheal tube should result in reduced peak pressures, making the ventilator more manageable.

A patient with status asthmaticus is receiving 70%/30% helium-oxygen by nonrebreather mask after repeated attempts at traditional bronchodilation, including aerosolized albuterol at 10 mg/hr and intravenous solumedrol administration. The heliox mixture is being provided through an oxygen flow meter with an indicated flow of 10 L/min. The patient's lips and ears are demonstrating cyanosis. The therapist should A. increase the indicated flow to 15 L/min B. switch to a 60/40% heliox mixture C. discontinue heliox therapy, apply 100% oxygen D. administer 80/20% heliox

The correct answer is : B Explanation : Helium-oxygen mixtures come in three different concentrations -- 80/20%, 70/30%, 60/40%. The latter number represents the amount of oxygen. Therefore, 70/30% represents 70% helium and 30% oxygen. As the patient is persistently demonstrating evidence of hypoxemia, the percent of oxygen delivered should be increased. Thus, a 60/40% heliox mixture is most appropriate.

A high-frequency chest wall occilation device would be the most appropriate tool for use in which of the following patients? A. 20-year-old patient with cystic fibrosis tolerating CPT in trendelenburg B. 5-year-old child with cystic fibrosis not cooperating with CPT C. adult patient with ARDS resulting in decreased lung compliance D. adult patient with long-term requirements for home mechanical ventilation

The correct answer is : B Explanation : High-frequency chest wall oscillation is an alternative device to help mobilize secretions. It is especially helpful for a patient who cannot tolerate the head-of-bed down position that is associated with normal chest physiotherapy.

Which of the following would be helpful during a bronchoscopy procedure to ensure adequate continuous ventilatory support for a patient who is receiving mechanical ventilation and is dependent on high levels of PEEP? A. closed in-line suction catheter B. high frequency jet ventilation C. Halcinon D. resuscitation bag with a PEEP valve

The correct answer is : B Explanation : High-frequency jet ventilation may be helpful when a patient is undergoing a bronchoscopy. This sort of ventilation keeps mean airway pressures down while the bronchoscope is in place.

A patient is ordered to be placed on a high frequency jet ventilator in response to a bronchopleural fistula. Which of the following ventilator parameters can the respiratory therapist expect to manipulate in place of the tidal volume control, which can be found on a volume-controlled ventilator? A. Injector line B. Drive pressure C. % expiratory time D. Rate

The correct answer is : B Explanation : High-frequency jet ventilators do not have a control to set tidal volume directly. Delivered volume is set indirectly through a control called drive pressure, sometimes called oscillatory amplitude.

Hyperbaric therapy may be useful in A. decreasing PVR B. facilitating wound healing C. increasing blood nitrogen content D. improving cardiac function

The correct answer is : B Explanation : Hyperbaric therapy may be combined with high level oxygen to help wounds healing. PVR and cardiac function will not be effected. Hyperbaric therapy may be used to decrease nitrogen.

A 48-year-old female completes the several FVL maneuvers. Both the expiratory and inspiratory side of the loops demonstrate a similar inverse pattern that is round in appearance. Which of the following could the respiratory therapist potentially conclude about this pattern? A. an obstructive pulmonary defect is likely present B. a fixed upper airway obstruction is present C. a restrictive pulmonary defect is likely present D. the shape of the loop demonstrates normal pulmonary function

The correct answer is : B Explanation : In a normal flow volume loop, the inspiratory side of the loop (the topside), is vastly different than the bottom side of the loop. However, when upper loop resembles the bottom part of the loop in shape (this is often called a round loop) it indicates that the patient is having difficulty getting air in AND out. This is most commonly associated with a fixed upper airway obstruction or vocal cord paralysis or cancer.

A patient with status asthmaticus has been admitted to the intensive care unit from the emergency department (ED). Wheezing persists in spite of multiple treatments with beta sympathomimetic medications. To help alleviate bronchoconstriction, the respiratory therapist should suggest A. nasal CPAP B. prednisone C. aerosolized albuterol D. acetylcysteine

The correct answer is : B Explanation : In addition to beta sympathomimetic medication (short term bronchodilators) a patient with persistent wheezing may also benefit from steroids such as prednisone or methylprednisolone.

After completing three flow-volume loops on a bedside pulmonary function screening device, the therapist notices that there are FVC maneuvers that are higher than some of the SVC maneuvers. The therapist should have the patient A. accept the results as normal. B. repeat the SVC maneuvers C. repeat the FVC maneuvers D. repeat the entire flow-volume loop maneuver

The correct answer is : B Explanation : In pulmonary function testing, a patient should reliably be able to produce a higher slow vital capacity compared to a forced vital capacity. This is due to the difference in laminar and turbulent air flow that occurs in the patient's airways upon exhalation. When exhalation is forceful, turbulent airflow is present and less gas may be exhaled. Therefore, in this case, because the patient has produced FVC maneuvers that are higher in volume than their SVC maneuvers, it must indicate that their original slow vital capacity maneuvers were not at the patient's full effort or capability. Thus, the SVC maneuvers must be repeated.

A mechanically ventilated patient is experiencing tachycardia with decreasing oxygen saturations while the high-pressure alarm is sounding with each breath. Delivered tidal volumes are set at 600 mL. Returned tidal volumes are 300 mL. Potential causes include which of the following? A. leaky cuff B. partially blocked endotracheal tube C. partially dislodged endotracheal tube D. unplugged capnograph sensor

The correct answer is : B Explanation : In this example it is not appropriate to select an option which includes a leak because the high-pressure alarm is sounding. This tells us that peak pressures are too high. The only option that would result in higher peak pressures is a partially blocked endotracheal tube.

Which of the following patient instructions is appropriate for a 12-year-old male patient who is about to have an appendectomy? A. "Once your anesthesia wears off we will start you on incentive spirometry." B. "After surgery, you will need to take several deep breaths every hour." C. "When you cough you must splint your incision." D. "During surgery, we will be monitoring your end tidal CO2."

The correct answer is : B Explanation : Lung expansion following abdominal surgery is important in order to avoid pulmonary complications such as atelectasis. Also, in this example, several answers contain the use of terminology that would not be understood by a 12-year old patient.

A patient is receiving SIMV volume-cycled mechanical ventilation. A respiratory therapist is using slow, incremental reductions in the set respiratory rate for weaning. Which low alarm should the respiratory therapist place the most emphasis on? A. tidal volume B. minute ventilation C. pressure D. PEEP

The correct answer is : B Explanation : In this example we are tested about our knowledge to withdraw mechanical ventilation. While one may believe that low tidal volume could be the best answer, it is not as significant as the sum of several tidal volumes in a minute, or minute ventilation. While weaning, a patient may vary in VT, while maintaining an acceptable ventilatory drive. Pressures such as PEEP or low-pressure are not as critical for monitoring while weaning with slow incremental reductions of the mechanical rate.

An MVA victim with severe facial trauma has been trached in the emergency room. The respiratory therapist should now provide A. heated/humidified aerosol at FIO2 0.50 B. heated/humidified aerosol at FIO2 1.0 C. cool aerosol at FIO2 0.50 D. cool aerosol at FIO2 1.0

The correct answer is : B Explanation : In this example, the patient's natural upper airway has been by-passed and the respiratory therapist should be concerned about delivery of cold dry gas. Heat and humidification are the therapist's primary concern. 100% oxygen is also necessary due to the emergent nature of the patient's condition having suffered a motor vehicle accident.

The following graph is from a patient receiving mechanical ventilation in the assist/control mode. Which of the following would be of potential concern? A. excessive inspiratory flow rate B. development of AutoPEEP C. insufficient PEEP D. increased work of breathing

The correct answer is : B Explanation : In this graph it must be observed that expiratory flow is not naturally returning to zero before another breath is delivered. This means that the inspiratory phase is beginning before the patient has exhaled completely. This would develop autoPEEP. AutoPEEP may also be referred to as intrinsic PEEP.

A mechanically ventilated patient who with an 8.0 mm trachestomy tube in place has a strong cough, but is unable to expectorate thick, dry secretions. During an attempt to suction the patient, the tube becomes dislodged, but promptly placed back to its proper position. Following the procedure, the respiratory therapist notes dry crackles in the right upper lobe and neck upon auscultation. These breath sounds would most likely be associated with A. bleb emphysema B. subcutaneous emphysema C. pneumothorax D. retained secretions in the airway

The correct answer is : B Explanation : In this scenario the respiratory therapist should assume that while the tube was dislodged, breaths delivered by the ventilator caused air to be forced into soft tissue areas under the skin outside of the trachea. This condition is known as subcutaneous emphysema. Auscultation reveals dry crackles and crepitus upon palpation.

A respiratory therapist is assisting in a cardiopulmonary stress testing of a patient. After several increases in workload by increasing the incline on the treadmill, heart rate is 120/min, blood pressure is 130/95 mmHg and O2 Sat is 97%. Increasing the treadmill further shows no further increase in these values, but the patient appears pale in color. The therapist should recommend A. decrease workload B. discontinue the test, document findings C. monitor the patient closely D. after two minutes, increase workload

The correct answer is : B Explanation : Increasing the workload by increasing the inclination of the treadmill should result in an increase in heart rate and blood pressure on a normal patient who has normal cardiac and blood pressure response. When workload is increased and there is no increase in heart rate or blood pressure, the patient has reached his or her maximum workload. The tests should be discontinued at this time and the previous level of workload should be recorded as the patient's maximum exercise ability

Immediately after oral intubation of an apneic patient, the respiratory therapist begins manual ventilation with a bag-valve assembly. The patient is simultaneously connected to an end-tidal carbon dioxide monitor. Which of the following should the respiratory therapist expect to observe when looking at the capnographic waveform? A. fall in CO2 followed by a subtle rise B. first a rise, then a fall in CO2 C. steady CO2 reading D. stair-step shifts in the CO2 tracing

The correct answer is : B Explanation : Initial ventilation attempts after a patient has been apneic will show reduced end-tidal CO2 followed by a steady rise as ventilation is provided. Ultimately, as ventilation continues the end-tidal CO2 will begin to decrease.

Which of the following medication would be most helpful in anesthetizing a patient's airway just prior to a flexible bronchoscopy procedure? A. anectine (Succinylcholine Chloride) B. lidocaine HCL (Xylocaine) C. benzocaine D. cardizem

The correct answer is : B Explanation : Insertion of a bronchoscope into the patient's airway can cause a significant cough reflex. This can complicate the procedure and place the patient in great distress during the process. To prevent or decrease the cough reflex, aerosolized lidocaine may be administered to the patient's airway prior to the procedure. Lidocaine comes in many preparations such as Xylocaine and benzocaine and many others.

Periodic weighing of a newborn's diaper is associated with A. documenting weight loss B. measuring fluid output C. IRDS in a premature neonate D. assessing response to breast feeding

The correct answer is : B Explanation : It is not practical to insert a urinary catheter while caring for the neonate. Weighing the diaper is the standard of care for measuring fluid output.

A patient following abdominal surgery has severe bilateral atelectasis in both bases. What would one expect to hear upon auscultation? A. diffuse inspiratory crackles B. end-inspiratory crackles C. pleural rub D. bronchial breath sounds

The correct answer is : B Explanation : Late inspiratory crackles, or end-inspiratory crackles are commonly auscultated with a patient experiencing atelectasis. Crackles are also known as rales.

Within 2 hours of abdominal surgery, a patient has a blood pressure of 70/45 mmHg while receiving 10 mcg/kg/min Dopamine HCL, IV. The patient has not awakened from anesthetics and is ashen in color. The respiratory therapist should suspect the patient A. is in a state of shock B. is hemorrhaging C. has cor Pulmonale D. has left heart failure

The correct answer is : B Explanation : Low blood pressure, ashen color of the patient, and the fact that the patient has just been discharged for surgery, all indicate the likelihood of loss of blood. This is most likely occurring from internal hemorrhaging related to the surgical procedure. This patient should be stabilized with maximum supplemental oxygen and returned to surgery for exploration.

A patient has been gradually weaning from mechanical ventilation for the past two weeks. The physician requests data to determine the patient's current ability to breathe on his own. Which of the following tests would not be indicated? A. VT B. MVV C. VC D. MIP

The correct answer is : B Explanation : MVV, or maximum voluntary ventilation is a test completed during a complete pulmonary function evaluation. It is not a part of the weaning parameters assessed on mechanically ventilated patients. It is also a test that should be completed on patients who are otherwise very healthy and in their best condition, not a critically ill patient receiving mechanical ventilation.

After removing 5 inches of mechanical deadspace on a ventilator circuit, what change would the respiratory therapist expect to see? A. Small reduction in PaCO2 with slight increase in MAP B. Small reduction in PaCO2 with no increase in MAP C. Unchanged MAP with slight increase in PaCO2 and PaO2 D. Increase in pH with a reduction in MAP

The correct answer is : B Explanation : Mechanical deadspace is the tubing on the patient side of the wye on a ventilator circuit. This includes the endotracheal tube. The deadspace reduces the efficiency of ventilation, which affects the PaCO2. Therefore, removing deadspace will reduce PaCO2. Increasing deadspace will increase PaCO2. Changes to mechanical deadspace do not affect the mean airway pressure in the chest.

A patient complaining of shortness of breath presents with severe hyperglycemia, marked hyperpnea, and normal oxygen levels. What should the respiratory therapist expect when analyzing an ABG? A. anemic hypoxemia B. metabolic acidosis C. respiratory failure D. compensated respiratory alkalosis

The correct answer is : B Explanation : Metabolic acidosis results in a significantly low pH, which causes an increased respiratory drive in order to attempt to normalize the pH. High glucose levels are often seen with metabolic acidosis. Oxygenation remains normal.

Metabolic alkalosis is the correct interpretation for which of the following blood gas values? A. pH 7.50, PaCO2 34 mmHg, HCO3- 25 mEq/L, BE +2 mEq/L B. pH 7.50, PaCO2 44 mmHg, HCO3- 32 mEq/L, BE +9 mEq/L C. pH 7.50, PaCO2 24 mmHg, HCO3- 23 mEq/L, BE -1 mEq/L D. pH 7.50, PaCO2 35 mmHg, HCO3- 19 mEq/L, BE -6 mEq/L

The correct answer is : B Explanation : Metabolic alkalosis is indicated when the pH is high, the CO2 is normal or low, and HCO3- level is elevated.

Thin, watery secretions are aspirated through the suction catheter on a patient during nasotracheal suctioning. Prior to repeating the procedure, the respiratory therapist should A. decrease suction pressure B. ensure the patient is in sniffing position C. place an oropharyngeal airway D. place a rolled-up blanket under the patient's shoulder

The correct answer is : B Explanation : Nasotracheal suctioning is facilitated by placing the patient with their head in sniffing position. Any other position could cause the suction catheter to enter the esophagus or prevent advancement.

One indication to change from administering a front door bronchodilator to a back door bronchodilator is A. the patient's heart rate does not increase with the front door bronchodilator B. nausea and anxiousness induced by the front door bronchodilator C. ineffectiveness of a side door bronchodilator D. diminished lung sounds result in wheezing after the front door bronchodilator is administered

The correct answer is : B Explanation : Nausea, anxiousness, tingling of the digits, & excessive increase in heart rate are all examples of poor tolerance by front door (sympathomimetic) bronchodilators. It is appropriate to try either a side door or back door bronchodilator should poor tolerance occur. If diminished lung sounds result in wheezing after a bronchodilator is administered, it is an indication that increased air movement is occurring and the therapy should continue.

A patient receiving volume-controlled mechanical ventilation has the following values obtained from a pulmonary-artery catheter: PAP 24 mm Hg CVP 6 mm Hg PCWP 6 mm Hg C.I. 2.5 L/min/m2 Which of the following would be administered? A. 40% venturi mask B. nitric oxide C. balloon pump D. heliox

The correct answer is : B Explanation : Nitric oxide is used to treat pulmonary hypertension. By evaluating the hemodynamic values, particularly noting a high PAP and normal PCWP, it is evident that there is a problem in the lungs and likely hypertension. Nitric oxide therapy will help to lower pulmonary hypertension.

Which of the following conditions could produce a central venous pressure of 9 torr, and a pulmonary artery pressure of 11 torr? A. excessive airway pressure B. Cor pulmonale C. hypervolemia D. mitral valve stenosis

The correct answer is : B Explanation : Normal CVP is approximately 4 or 5 mm Hg. Normal PAP is 14 mm Hg. Since the CVP is high and PAP is low, it tells us that the right heart is not functioning properly and blood cannot flow correctly through it. CVP is the pressure entering the right heart and PAP is the pressure leaving the right heart. Cor pulmonale is a right heart condition.

Following abdominal surgery, a patient has a hematocrit level of 21%. CVP is 1 torr, PAP 10 torr, PCWP 5 torr. C.O. 3.9 L. The patient would most benefit from which of the following? A. Osmitrol B. administration of whole blood C. lactated ringers D. Lasix (furosemide)

The correct answer is : B Explanation : Normal hematocrit is 45%. This patient has a hematocrit of 21%. If we divide by three that would give us an estimated hemoglobin level of 7 g/dL. This low hemoglobin level, in conjunction with the fact the patient has just had surgery, is suggests the patient is hemorrhaging internally. This patient should immediately receive blood and be sent back to surgery to explore for a leaky vessel.

Which of the following may be used to determine a patient's ability to cough forcefully? A. minute ventilation B. MEP C. RSBI D. VT

The correct answer is : B Explanation : Of the options offered, a maximum expiratory pressure value can be used to assess a patient's ability to generate a forceful cough. Minute ventilation, tidal volume, and rapid shallow breathing index will not be helpful in assessing the effectiveness of a patient's cough.

Which of the following is a determinant of Optimal PEEP level? A. VD/VT B. consistent cardiac output with increasing plateau pressures C. decreasing dynamic complianc D. RSBI

The correct answer is : B Explanation : Of the options offered, cardiac output relative to plateau pressures is used to determine optimal PEEP.

A high frequency jet ventilator system includes which of the following items? A. Clark electrode B. flow interrupter C. pop-off spring valve D. nitrogen analyzer

The correct answer is : B Explanation : Of the options offered, only a flow interrupter is a control associated with high-frequency jet ventilation.

A patient has been receiving IPPB therapy after surgery to reduce pulmonary complications. The respiratory therapist should use which method to determine the effectiveness of the therapy? A. SBN2 B. Periodic chest radiograph C. Arterial blood gas analysis D. Airway resistance

The correct answer is : B Explanation : Of the options offered, the purpose of an IPPB treatment is to prevent post-operative complications, including consolidation of sputum in the alveoli. Monitoring pre and post breath sounds and obtaining a periodic chest x-ray is most helpful in assessing the effectiveness of the IPPB therapy.

A respiratory therapist is assisting a patient who has an SpO2 of 85% while receiving oxygen by nonrebreather mask at 12 L/min flow. Laboratory results show a WBC of 10,000 cu mm and an Hb of 8.1 g/dL. The patient has a DNI order and an advanced directive that indicates no blood administration. Which of the following can be done to maximize arterial oxygenation? A. one unit of packed RBCs B. NIPPV C. turn the flow to the nonrebreather to flush D. installation of a tracheostomy

The correct answer is : B Explanation : On this patient who has orders indicating 'DO NOT INTUBATE' and an advanced directive that includes no blood administration, the best and most effective way to increase arterial oxygenation is through the use of positive airway pressure. In this case NIPPV (noninvasive positive pressure ventilation). Installation of a tracheostomy is considered a form of intubation and would violate the patient's wishes. Administration of RBCs is a type of blood product and would go against the patient's advanced directives.

Results obtained from a multiple wave-length spectrophotometer are consistent with the results obtained from a Co-oximeter, but not with the blood gas analyzer. The respiratory therapist determines that the blood gas analyzer is inaccurate. How should this problem be corrected? A. Run QC only on the Co-oximeter B. Run a two-point calibration on the blood gas analyzer, followed by QC C. Accept the results that seem most consistent with the clinical scenario D. Run a two-point calibration on the Co-oximeter, followed by QC

The correct answer is : B Explanation : Once a respiratory therapist determines that an instrument is inaccurate, the machine should be either removed from service or calibrated. After calibration, quality control material should be utilized to confirm that the calibration efforts were successful and that the machine is now reading accurately.

An 18-year-old male patient is admitted to the emergency department (ED) after being rescued from an area lake in the middle of the winter. Core body temperature is 32 deg C. (90 deg F) He has the following blood gas values on room air. The blood gas values are not corrected for temperature: pH 7.33 PaCO2 46 mmHg PaO2 60 mmHg HCO3- 24 mEq/L BE 0 mEq/L The respiratory therapist should A. provide a warming blanket to the patient B. start a heated aerosol by mask set at 45% C. place the patient on mechanical ventilation D. provide manual resuscitation

The correct answer is : B Explanation : One effective way to raise core body temperature is to administer heated aerosol.

Regarding patient safety while performing a heel stick on a newborn, the most important thing to consider is A. warming the heel for one minute prior to puncture. B. location of the puncture. C. level of the capillary tube while filling. D. depth of skin puncture.

The correct answer is : B Explanation : One might be tempted to select the depth of the puncture, but actually more concerning is the location. Heel sticks should not be performed on a neonate in an area of their foot that they will walk on in the future as they grow. Damage to this area of the foot can result in difficulty with future ambulation. Therefore, heel sticks are performed on the lateral area of the heel.

Which of the following is an important instruction given to the family who will be caring for a 7-year-old patient with cystic fibrosis at home? A. advanced cardiac life support techniques B. recognizing signs of infection C. how to perform an emergency tracheotomy D. titration of oxygen therapy according to the needs of the patient

The correct answer is : B Explanation : One of the most important aspects to teach a family who is administering home care to a chronic child is that of recognizing signs of infection. If the onset of infection can be recognized early enough, it can be prevented or decreased before significant pulmonary compromise and/or hospitalization is required.

An infant remains cyanotic after the administration of high level supplemental oxygen. The physician is concerned about the presence of a condition, patent ductus arteriosis, and asks the respiratory therapist to gather data to confirm or rule out that condition. Which of the following data will the therapist require? A. blood glucose B. blood gases from the umbilical and radial arteries C. CT scan with contrast D. chest radiography

The correct answer is : B Explanation : Patent ductus arteriosus (PDA) is a congenital abnormality that may be found in some infants. If found, the correction is surgery. To diagnose PDA, the PO2 is analyzed from the right brachial artery and the umbilical artery. If PO2 differs by more than 15 mmHg then PDA is present.

Following abdominal surgery, a patient is attempting to perform sustained maximal inhalation (incentive spirometry) but repeatedly falls asleep during the procedure. The therapist should modify the therapy in which of the following ways? A. administer Narcan B. switch to IPPB with face mask C. hold therapy until the patient becomes more alert D. intubate and begin mechanical ventilation

The correct answer is : B Explanation : Patients often receive incentive spirometry after surgery. This therapy is considered to be most effective at preventing postoperative complications. However, some patients come out of surgery heavily sedated and sleepy and are unable to comply with the demands of incentive spirometry. In this case, a modification in therapy that accomplishes the same objective must be made. One appropriate modification is to use IPPB. A face mask is appropriate because the patient is not alert enough to use a mouthpiece.

A firefighter is brought to the emergency department with burns about the neck and face. Singed nasal hairs are also noted. SpO2 is 92% on a non-rebreathing oxygen mask. Which of the following interventions should be given priority? A. emergency tracheotomy B. intubation with direct visualization C. insertion of an LMA D. blind oral intubation

The correct answer is : B Explanation : Patients who have had exposure to extremely heated air masses, such as seen with firefighters, are at risk for swelling of the tissues in the upper airways and ultimate loss of that airway. Priority, therefore, is to ensure an airway through intubation. However, blind intubation is extremely dangerous because probing those tissues with an endotracheal tube may incite additional inflammation and cause complete closure of the airway. Intubating with direct visualization is most appropriate. Usually, this means the use of a bronchoscope.

An adult female weighing 168 lbs is receiving pressure-cycled noninvasive ventilation with a Bird Mark 7. The machine-delivered pressure breaths are not cycling off as expected. What should the respiratory therapist do? A. Observe the pressure/volume curve in the ventilator graphics B. Look for a leak in the circuit C. Reduce inspiratory time D. Suction the patient

The correct answer is : B Explanation : Pressure-cycled mechanical ventilation cycles into exhalation once the preset pressure is reached. If the breaths will not cycle off, a leak exists. The leak may be coming from the patient's mouth or the circuit itself. Therefore the best answer is to look for a leak in the circuit.

Which of the following provides the most significant evidence of improper placement of a nasal endotracheal tube? A. SpO2 of 90% B. ET tube markings are 21 cm at the right nare C. right hemi diaphragm is higher than the left D. breath sounds are bilateral

The correct answer is : B Explanation : Proper placement of an endotracheal tube is manifest by bilateral breath sounds. Of the options given the only one that indicates an improper placement of the nasal endotracheal tube is the ET tube markings at the right nare. When the patient is nasally intubated markings at the nose should be in the high 20s.

Based on the pulmonary function testing below, which of the following represents the best interpretation of the results? Pre-bronchodilator Post-bronchodilator FEF 25-75% 80% 90% FEF 200-1200 81% 82% FVC 71% 75% MVV 74% 78% A. mild obstructive defect, responsive to bronchodilator therapy B. restrictive defect C. mixed obstructive and restrictive defect D. normal spirometry

The correct answer is : B Explanation : Pulmonary function data shows normal flows. Volumes, however, are decreased. This indicates the presence of a restrictive defect.

The following data is available for a 62-year old patient with COPD. MAP 93 torr CVP 3 torr mPAP 16 torr PCWP 8 torr Q.T. 4.0 L/min PEEP 5 cm H2O C.I. 2.0 L/min/m2 The physician asks the therapist to determine pulmonary vascular resistance and record the findings in the patient's medical record. The therapist will record which of the following values? A. 320 dynes B. 160 dynes C. 80 dynes D. 1800 dynes

The correct answer is : B Explanation : Pulmonary vascular resistance is calculated by subtracting the PCWP from PAP and then dividing by cardiac output. In this case (16-8 = 8). In this question, cardiac output is provided and is called QT. To finish, (8/4 = 2). Finally, we must multiply the result by 80 to obtain the measurement in dynes. PVR = 2 x 80 = 160 dynes.

A respiratory therapist is preparing for defibrillation on a patient with pulseless ventricular tachycardia. Which of the following should NOT be part of that preparation? A. prepare to administer supplemental oxygen B. set the defibrillator's synchronization to ON C. set up for manual ventilation with a bag/valve D. set the defibrillator to deliver 360 joules

The correct answer is : B Explanation : Pulseless ventricular tachycardia should be treated with defibrillation. When defibrillating a deadly cardiac rhythm, the synchronization setting on the defibrillator should be set to "off".

Pursed-lip breathing with extended expiratory time will have what effect on the I:E ratio? A. inverse B. decrease C. increase D. no change

The correct answer is : B Explanation : Pursed-lips breathing, coupled with an increase in expiratory time will result in a reduced I:E ratio, where inspiratory time is decreased, and expiratory time is increased.

A respiratory therapist is called to the nursery to assess a 36-week gestational age neonate delivered five hours prior. Supraclavicular retractions are present. Respiratory rate is 70/min with nasal flaring and grunting. Radiological examination has a honeycomb pattern in the chest. The respiratory therapist should A. intubate and mechanically ventilate B. intubate and deliver surfactant C. place the patient on a nasal cannula at 2 L/min D. place the patient on a 40% oxygen hood

The correct answer is : B Explanation : Radiological descriptions such as "honeycomb" or "ground glass" patterns suggest RDS. These signs along with the respiratory distress of a premature neonate strongly suggest the need for surfactant therapy, which must be delivered through an endotracheal tube. Intubation is necessary for delivering surfactant.

The respiratory therapist is making a home visit to evaluate an oxygen-dependent patient. The patient has 3+ pitting peripheral edema and has not followed the physician's orders to limit fluid intake. You would expect which of the following hemodynamic values? A. low mean arterial pressure B. elevated right sided preload with a normal to low right ventricular after load C. reduced cardiac index D. elevated PCWP with a low cardiac output

The correct answer is : B Explanation : Right-sided preload is another name for CVP (central venous pressure). Peripheral edema is one of the signs associated with right heart failure, which results in an elevated CVP with a normal or low pulmonary artery pressure (PAP), a.k.a. right ventricular afterload.

The following graphic on an intubated ventilator patient is most likely caused by A. hyper-reactive airway disease B. secretions in the ET tube C. cancer of the vocal folds D. faulty PEEP valve

The correct answer is : B Explanation : Secretions in the ET tube will show up on a ventilator graphic as a fluttering expiratory flow.

After replacing the three-chamber chest tube drainage system, the respiratory therapist notices significant bubbling in the water-seal compartment. The therapist should A. decrease suction pressure B. examine the system for leaks C. monitor closely as this is a normal finding D. reposition the chest tube in the pleural space

The correct answer is : B Explanation : Significant or excessive bubbling in the water-seal chamber of a chest tube drainage system is usually associated with a leak somewhere between the patient's lung and the waterseal compartment. The system should be examined for leaks.

A 3-year-old patient is demonstrating ventilatory difficulty with use of intercostal accessory muscles. The patient is febrile and is drooling. A softened inspiratory stridor is audible. Which of the following mostly likely represents the patient's condition? A. Asthma B. Acute epiglottitis C. Bacterial pneumonia D. Laryngotracheal bronchitis (croup)

The correct answer is : B Explanation : Softened inspiratory stridor in conjunction with drooling is associated with acute epiglottitis. Acute epiglottitis is a bacterial infection, which can cause febrile conditions.

The following data is obtained from the patient's ventilator flow sheet: 3 am 9 am 3 pm Peak pressure(cm H2O) 20 21 25 Plateau pressure (cm H2O) 14 14 20 VT (L) 0.5 0.5 0.5 PEEP (cm H2O) 5 5 10 Which of the following can be accurately stated? A. dynamic compliance is steady B. static compliance remains steady C. decreasing ventilation/perfusion mismatch D. decreasing shunt

The correct answer is : B Explanation : Static compliance is monitored by examining the plateau pressures over time. Observation of these data appear to suggest an increase in plateau pressures, which is associated with a decrease in pulmonary compliance. However, closer observation reveals that PEEP was increased by 5 cm H2O. Therefore, the increase in plateau pressures at 3 PM are caused by an increase in the PEEP setting and are not a result of changing static compliance. This data indicates static compliance is remaining steady.

Which of the following is considered subjective information? A. breath sounds B. dyspnea C. chest movement symmetry D. vital capacity

The correct answer is : B Explanation : Subjective information are those data that must be reported by the patient. This kind of data is also known as symptoms. Data that can be observed independent of the patient's input is known as objective information, otherwise called signs. Of the options here, dyspnea must be reported by the patient and therefore is the only subjective information offered.

A respiratory therapist would like to review the physician's recent thoughts regarding the prognosis and anticipated care plan for the patient. In which section of the medical record should the therapist plan on reviewing? A. physician orders B. progress notes C. discharge plans D. nursing notes

The correct answer is : B Explanation : Technically, documentation in the progress notes does not constitute an official change in care. Notes in this section represent thoughts of healthcare workers regarding the potential problems, diagnoses, and treatment plans of the patient. To be made official, orders must be written in the chart as physician orders.

Which of the following clinical signs should result in discontinuation of weaning from mechanical ventilation? A. Respiratory rate above 25/min B. Frequent apneic periods of 20 seconds C. PaCO2 increases 8 torr above baseline D. VT < 6 mL/kg

The correct answer is : B Explanation : The 20-second periods of apnea strongly indicate the need to return to mechanical ventilation in this scenario. A respiratory rate above 30 per minute would also indicate the need to return to mechanical ventilation. The PaCO2 must increase by 10 torr or more to define weaning failure.

While coaching a postoperative patient on the use of an incentive spirometer, the respiratory therapist notices the patient is not exhaling completely before starting the inspiratory phase. The therapist should provide which of the following instructions? A. Pause at maximum inhalation for 2-3 seconds before exhalation B. Exhale completely, then pause before starting inhalation C. Perform the incentive spirometry therapy at tidal volume D. Decrease breath size

The correct answer is : B Explanation : The appropriate instruction for a patient performing sustained maximal inspiration with an incentive spirometer is to exhale completely, pausing before starting installation.

A respiratory therapist is instructing a patient with chronic obstructive pulmonary disease on the proper method of taking a bronchodilator treatment with a small-volume jet nebulizer. Which of the following should be included in the instructions? A. breathe in quickly, blow out passively and relax between breaths B. breathe in slowly, pause at the top of each breath, blow out passively and relax C. take a slow deep breath in, pause, blow out with moderate forcefulness D. take deep breaths, hold for 10 seconds, blow out quickly

The correct answer is : B Explanation : The appropriate instruction for normal breathing and for taking an aerosol treatment is to breathe in slowly, pause at the top of each breath, and exhale passively, relaxing between breaths.

A 6 year-old child is requiring frequent suctioning through a 5.0 mm endotracheal tube. Secretions are thick and copious and difficult to suction in spite of repeated instillation of normal saline prior to suction attempts. The suction catheter size is 6 Fr and suction pressure is set to -65 mm Hg. The respiratory therapist should do which of the following to improve the effectiveness of secretion removal? A. Instill atropine down the endotracheal tube B. Increase suction pressure C. Switch to a shorter catheter D. Use a 10 Fr suction catheter

The correct answer is : B Explanation : The appropriate suction pressure for an infant is between 60-80 mmHg. The patient is only receiving 65 mmHg of suction pressure and therefore has room for an increase in suction pressure. This will increase the effectiveness of secretion removal.

A home care patient with a tracheostomy tube uses an electrically-powered ventilator during sleeping hours. Recent nocturnal oximetry results reveal moderate hypoxemia while asleep on the ventilator. What should the respiratory therapist recommend? A. Provide a tracheostomy collar with FIO2 0.40 heated aerosol. B. Provide oxygen bleed-in by molecular sieve device. C. Increase the set respiratory rate by 2/min. D. Perform complete polysomnography.

The correct answer is : B Explanation : The best device for home oxygen therapy is a molecular sieve device, commonly known as an oxygen concentrator. This device is powered with electricity, which makes it appropriate for home use.

A patient has the following arterial blood gas results and ventilatory parameters: pH 7.08 PaCO2 27 mm Hg PaO2 101 mm Hg HCO3- 7 mEq/L BE -18 mEq/L RR 33 Which of the following blood tests would be indicated? A. INR B. glucose C. PTT D. Blood urea nitrogen (BUN)

The correct answer is : B Explanation : The blood glucose level should be evaluated in this case in order to determine the cause of the severe metabolic acidosis, which is likely to be diabetic ketoacidosis.

The following capnographic tracing is recorded for a patient receiving mechanical ventilation in the assist/control mode. Which of the following actions is most appropriate? A. increase inspiratory flow rate B. increase mandatory rate C. order a V/Q scan D. decrease minute ventilation

The correct answer is : B Explanation : The capnograph shows that the end-tidal CO2 is high. Remember the actual CO2 in the blood is usually about 10 mmHg higher than shown on the end-tidal CO2 monitor. From this data alone we can't deduce that the patient is hypoventilating. Of the options given, increasing mandatory rate is most appropriate.

A male has arrived in the emergency room after being found under a car in a closed garage with the engine running for several hours. A note was found in the patient's handwriting. Which of the following laboratory assessment should the therapist recommend? A. PaCO2 B. COHB C. SpO2 by pulse oximetery D. PaO2

The correct answer is : B Explanation : The conditions surrounding this patient suggest the patient has carbon monoxide poisoning and was attempting to commit suicide in that fashion. To assess the degree of carbon monoxide in the blood, a COHb is required.

A patient with cystic fibrosis develops rhonchi after 5 minutes of PEP therapy at 20 cm H2O. The respiratory therapist should A. begin cool bland aerosol therapy B. continue the therapy C. discontinue treatment, report to findings to the physician D. switching to flutter therapy

The correct answer is : B Explanation : The development of rhonchi in response to PEP therapy is considered to be a good outcome for the therapy. The purpose of the therapy is to mobilize and promote expectoration of secretions. The development of rhonchi is an indication that secretions are moving from small and middle-sized airways to the larger size airways where they may be naturally expectorated or suctioned. The therapy should be continued because it is effective.

Which of the following medications should the respiratory therapist recommend to provide a neurmuscular blockade and facilitate intubation? A. Xanax B. Pavulon C. Ativan D. Morphine

The correct answer is : B Explanation : The drugs most commonly used to provide a neurmuscular blockade (paralyze) to the patient and facilitate intubation are Anectine (succinylcholine), Pavulon (pancuronuim bromide), Curare (d-tubocurarine) and Vecuronium. The others listed here are merely anxiety controlling medication and will not provide paralysis.

Microorganisms are found on reusable mouthpieces that were soaked in a cleaning solution for 30 minutes. Which of the following could be a contributing factor for the persistent organisms? A. room temperature B. age of the solution C. aeration time D. barometric pressure

The correct answer is : B Explanation : The effectiveness of a cleaning solution, such as alkaline glutaraldehyde, is based on the amount of time the item is soaked in the solution, the type of solution, and the age of the solution. Although not offered, the resistance of the organism is also a factor. Room temperature and barometric pressures are not contributing factors.

An 81-kg (178-lb) adult male is orally intubated with a 6.5-mm endotracheal tube. The therapist reports difficulty removing secretions with the suction pressure set to 120 mmHg. The respiratory therapist will FIRST A. decrease suction pressure to 100 mmHg B. switch to an 8.0-mm ET tube C. increase suction pressure to 130 mmHg D. instill Albuterol down the ET tube

The correct answer is : B Explanation : The first problem that should be recognized is that the patient's endotracheal tube is too small for their ideal body weight. And 81 kg male should have an endotracheal tube size of 8 mm to 8.5 mm. Having the proper endotracheal tube size will permit a larger suction catheter to be used. This will drastically improve the efficiency of suctioning.

A post-operative patient is receiving positive-pressure ventilation with an IPPB while recovering from anesthesia. The mandatory rate is 10/min. Inspiratory pressure is set to 18 cm H2O. Gradually the rate increases to 16/min while the monometer needle is showing a significant negative deflection before inhalation begins. The pressure monometer no longer rises smoothly during inhalation. What should the respiratory therapist do? A. sedate the patient B. increase sensitivity C. decrease inspiratory flow rate D. wean the patient

The correct answer is : B Explanation : The gradual increase in respiratory rate indicates that the patient is waking up from anesthesia, as expected. The negative deflection before inhalation indicates that the machine sensitivity is too low-requiring the patient to work too hard to trigger inspiration. The manometer not rising smoothly indicates the flow rate is also too low. It is not appropriate to sedate the patient but rather allow him to wake up with more applicable settings.

A 72-kg (158-lb) patient is receiving volume-cycled mechanical ventilation on the following settings: Mode SIMV Rate 12 VT 550 ml FIO2 0.40 PEEP 10 cm H2O Which of the following alarm settings is most appropriate? A. low PEEP alarm of 2 cm H2O B. low VT alarm of 450 mL C. high VT alarm of 700 mL D. low VT alarm of 325 mL

The correct answer is : B Explanation : The low tidal volume alarm should be set at about 100 mL below the preset/returned tidal volume. Be prepared to lower the low VT alarm when the patient's lung compliance increases.

A 28-year old male reports to the emergency department with dizziness, nausea, and some degree of vertigo. The patient's record indicates the patient is on a skiing vacation but normally lives on the coast. The physician orders sildenafil. For which of the following is the patient mostly likely being treated? A. nitrogen narcosis B. altitude sickness C. decompression sickness D. avian flu

The correct answer is : B Explanation : The medication sildenafil, also known as Viagra, can be used to treat altitude sickness.

Which of the following chemicals and soak-times should be used to sterilize a bronchoscope after being used on a patient with known active tuberculosis? A. Steam autocalve B. Cidex (alkaline gluteraldehyde) for 10 hours C. Irradiation followed by ethylene oxide (ETO) D. Sonacide (acid gluteraldehyde) for 20 minutes

The correct answer is : B Explanation : The most appropriate sterilization method for a bronchoscope would be Cidex (alkaline glutaraldehyde). Cidex will kill the tubercle bacilli in 20 min. and will kill everything in 12 hours.

A patient with thick and tenacious secretions would benefit most from which of the following? A. aerosolized hypertonic saline B. oral consumption of water C. postural drainage and percussion D. Albuterol

The correct answer is : B Explanation : The most effective way at thinning a patient's thick secretions is the oral consumption of water. Other procedures listed are helpful at mobilizing secretions but not as effective at hydration achieved by drinking water.

Auscultation of the chest on an adult patient intubated with an 8.0 mm endotracheal tube reveals diminished breath sounds on the left side. This is mostly likely caused by A. pleural effusion B. intubation of the right mainstem bronchus C. left-sided pneumothorax D. left-sided fluid consolidation

The correct answer is : B Explanation : The most likely cause of breath sounds that are diminished only on one side, especially the left side, is most caused by inadvertent intubation of the right mainstem bronchus.

A patient with COPD has received mechanical ventilatory support for 2 days. Recent settings and clinical data include: Mode SIMV Mandatory rate 4 VT 500 mL FIO2 0.30 PEEP 5 cmH2O Total rate 12 VT (spont) 430 mL MIP -30 cm H2O The therapist should recommend: A. add pressure support of 10 cmH2O B. extubate the patient C. return to full ventilatory support D. increase mandatory rate to 6

The correct answer is : B Explanation : The patient in this question is obviously weaning. Because the rate is set to four. The patient appears to be adequately ventilating, the next step is to remove the patient from the ventilator. In this case, extubating the patient is the best option.

A COPD patient on 5 L/min nasal cannula in the emergency room is complaining of continuing shortness of breath and has the following arterial blood gas results: pH 7.37 PaCO2 48 mm Hg PaO2 39 mm Hg HCO3- 29 mEq/L BE +2 mEq/L What is the appropriate intervention? A. 8 L/min nasal cannula B. NRB mask C. CPAP 5 cmH20 D. intubation

The correct answer is : B Explanation : The patient is experiencing severe hypoxemia to a level that is not consistent with maintaining life function. While we are often uncomfortable increasing oxygen flows to COPD patients, this should not stop us from treating the problem. 8 L /min is not appropriate because it exceeds the limits of a nasal cannula.

An adult male has had a unilateral wheeze, when auscultating the chest, for several months. Which of the following is the most likely cause? A. bronchoconstriction B. a cancerous mass in the pulmonary tree C. ARDS D. idiopathic pulmonary fibrosis

The correct answer is : B Explanation : The presence of a unilateral wheeze that has persisted for several months is not likely related to bronchoconstriction. When bronchoconstriction is present, wheezing is usually noted bilaterally. The unilateral wheeze, in conjunction with its persistence for several months, is most likely caused by a mass in the pulmonary tree. This mass may be cancerous or benign.

A patient in the emergency room is being treated for an asthmatic episode. After a treatment with Albuterol and Ipratropium Bromide (Atrovent), auscultation of the chest reveals increased wheezing. Peak flow rates have increased. The therapist should A. administer cromolyn sodium (Intal) B. repeat the bronchodilator treatment C. administer Acetylcysteine (mucomyst) D. wait 4 hours and re-evaluate

The correct answer is : B Explanation : The presence of increased wheezing, after the delivery of a bronchodilator medication, indicates improvement by the patient or responsiveness to the bronchodilator medication. This improvement is manifested by an increase in peak flow rates. This evidence indicates the need to continue bronchodilation therapy.

Ten minutes after extubation of a 20-year old male diagnosed with barbiturate overdose, the respiratory therapist observes deceased SPO2, intercostals retractions, and marked stridor. The therapist should immediately A. provide cool aerosol at previous FIO2 B. orally intubate the patient C. continue therapy, monitor closely D. apply Lidocaine spray to the oropharyngeal tissues

The correct answer is : B Explanation : The presence of marked stridor is a clear indication to protect the airway. In this case oral intubation is most appropriate, even though the patient was just extubated moments ago.

A 70-kg (154-lb) male patient is receiving volume-controlled mechanical ventilation on the following settings: Mode SIMV Mandatory rate 16 Tidal volume 600 mL FIO2 0.4 PEEP 5 cm H2O Insp flow rate 55 L/min The following ventilator graphic is associated with the current state of ventilation. Which of the following should the respiratory therapist recommend? A. increase inspiratory flow rate B. decrease tidal volume C. switch to assist/control mode D. decrease pressure limit

The correct answer is : B Explanation : The pressure-volume ventilator graphic shown demonstrates a pronounced beak. A pronounced beak shows a significant increase in pressure to accomplish a very small amount of volume. This is because the volume is too large for the lung and the lung is becoming overdistended. A proper pressure-volume loop has a very small beak, which is an indication that the lung is full but not grossly overdistended. To correct this situation the tidal volume should be lowered. While this will sacrifice a small amount of volume, it will reduce mean airway pressure significantly.

The sudden sounding of a high-pressure ventilator alarm is noted on a patient receiving volume controlled mechanical ventilation. The respiratory therapist notes the endotracheal tube is at 27 cm at the teeth and that breath sounds are absent when auscultating the left hemithorax. The therapist should immediately A. advance the endotracheal tube until breath sounds can be auscultated over the left chest B. withdraw the endotracheal tube until breath sounds are bilateral C. check the air pressure of the tube cuff D. prepare for insertion of chest tubes in the left thorax

The correct answer is : B Explanation : The proper placement of an oral endotracheal tube is when the teeth or lip line matches with markings on the tube somewhere in the low 20s. This endotracheal tube is 27 cm at the teeth, indicating the tube is advanced too far. In this case, it is likely the tube has advanced into the right mainstem bronchus, which would cause a high-pressure alarm to occur. The appropriate action is to withdraw the endotracheal tube until breath sounds are bilateral.

A patient is receiving an initial pulmonary rehabilitation visit to the clinic. To orient the patient which of the following should be taught to the patient concerning the goals and benefits of pulmonary rehabilitation? A. achieve normal pulmonary function B. increase exercise tolerance and improve performance of ADLs C. return to normal life D. reduce the need for oxygen

The correct answer is : B Explanation : The purpose of a pulmonary rehabilitation program for a patient with COPD relates to improving their life from their point of view. Legitimate goals include recognizing signs of infection, reducing hospitalizations, increasing their ability to perform activities of daily living, and generally increasing exercise tolerance. Because lung disease cannot be reversed it is unreasonable to think that we can return the patient to normal life or reduce their need for oxygen.

Which of the following pre/post bronchodilator pulmonary function test results is the most meaningful in suggesting that a bronchodilator is indicated? A. FEV1 increases by 150 mL B. FEF 200-1200 increases by 17% C. FVC increases from 2.5 L to 2.6 L D. FEV1.0/FVC % increases by 10%.

The correct answer is : B Explanation : The two primary considerations in pre-and post-bronchodilator studies are (1) whether flows increased by at least 12% or more and (2) whether the FEV1 increases by at least 200 mL. The FEF200-1200, indicating the condition of the large airways, increases by well over 12%, and is therefore the correct choice.

Quality control data plotted on a daily chart shows points above the mean followed by a group of points below the mean. All points are within 2 standard deviations from the mean. This data can best be categorized as A. a variance B. a shift C. a trend D. out-of-control

The correct answer is : B Explanation : The quality control data plotted on a daily chart shows the ability for the blood gas machine to produce accurate results. If all points plotted are within the upper and lower control limits, the machine is said to be in control and may be used to report patient results. The closer the points are to the mean, or the middle, the more accurate and precise is the analyzer. The respiratory therapist must monitor these points plotted on a graph to ensure the machine's accuracy. If points on the graph are described as above the mean and suddenly below the mean, this is known as a shift. If points are described below the mean or above the mean and gradually move to the other side of the mean line, this is known as a trend. The machine must be monitored during trends closely to ensure it does not trend to an out-of-control state.

A chest radiograph on an adult indicates a concave superior boarder. This finding is most closely associated with which of the following? A. complete pneumothorax B. pleural effusion C. lobular pneumonia D. pulmonary edema

The correct answer is : B Explanation : The radiological description of a chest x-ray that indicates the presence of a "concave superior border", sometimes called a "superior border interface", is most closely associated with a pleural effusion.

A patient is experiencing reduced tidal volumes on a volume-cycled ventilator. There is a gurgling sound coming from his mouth with each breath and the endotracheal tube marking is 19 cm at the lips. The respiratory therapist should A. replace the endotracheal tube. B. add air to the pilot balloon while auscultating over the neck. C. extubate the patient and provide manual ventilation. D. advance the endotracheal tube 5 cm.

The correct answer is : B Explanation : The scenario presents an airway managment problem including an air leak with an endotracheal tube. The cause is likely due to a lack of air in the cuff, but could be several other things, such as a hole in the cuff, or a misplaced tube. By first adding air to the cuff the respiratory therapist will either solve the problem, or further isolate the problem. For example, if a hole exists in the cuff, the leak will shortly return, and the therapist will know to replace the tube.

While performing manual ventilation with a self-inflating resuscitator bag through a properly placed endotracheal tube, a respiratory therapist observes no chest rise. To troubleshoot the problem, the therapist should A. adjust the PEEP valve B. check for proper function of the air inlet valve. C. increase the flowrate. D. assure the large bore reservoir tubing is connected.

The correct answer is : B Explanation : The scenario suggests that the ET tube is properly placed, therefore the problem must be in the resuscitator. When the therapist squeezes the device, the air is not being delivered to the patient, which is why there is no chest rise.

The low exhaled volume alarm is sounding on a patient receiving volume controlled ventilation. Which of the following could correct the problem? A. check for a pneumothorax B. add air to the ET tube cuff C. drain excess water in the ventilator circuit D. suction the patient

The correct answer is : B Explanation : The solution to this problem is to pick those things that could cause a low volume alarm to sound. In most cases this would be a leak of some sort. When you examine the answers you do not directly see the causes of the alarms but you see the possible solutions. You must examine each solution and determine if the alarm would be a high pressure or a low-pressure alarm or something else. Suctioning the patient would presumably be because of excess secretions and would result in a high-pressure alarm. Adding air to the ET tube cuff, if the cuff was deflated, would result in a low-pressure alarm. So, that option has to be a correct one. Excess water and the ventilator circuit and the possibility of a pneumothorax would both result in a high-pressure alarm, and therefore cannot be the correct answer. Two only is the correct answer.

Quality control test results of a recently certified 3.0 Liter calibration syringe used in conjunction with a spirometer are as follows: Volume 1 3.10 L Volume 2 3.12 L Volume 3 3.15 L Which of the following is an accurate statement, according to ATS standards? A. The syringe is defective B. The spirometer is accurate C. The spirometer is inaccurate D. The spirometer lacks precision

The correct answer is : B Explanation : The spirometer is considered accurate if the results are no more than 3% above or 3% below than the 3 L calibration syringe. This means the accuracy range is 2.85 L -3.15 L. Because all values are within this range the spirometer is considered to be accurate.

A 65-kg (143-lb), 170-cm (5-ft, 7-in) patient who is nasally intubated and receiving mechanical ventilatory support has a sudden drop in SpO2 from 97% to 82%. Breath sounds are clear on the right, absent on the left. The endotracheal tube is at the 28 cm mark at the right nare. Which of the following action should the therapist take first? A. Insert chest tubes in the right chest B. Increase FIO2 to 1.0 C. Manually ventilate the patient with a bag-valve D. Increase the pressure limit of the ventilator

The correct answer is : B Explanation : The sudden drop in oxygen saturation and breath sounds that are absent on the left but present on the right, Is associated with a sudden pneumothorax. This is considered an emergency and requires the administration of 100% oxygen.

The respiratory therapist records a patient's blood pressure to be 120/80 mmHg as measured by a sphygmomanometer. Simultaneously, the therapist notes the blood pressure measured by an indwelling radial artery catheter and transducer is reading 145/90 mm Hg. Which of the following could explain the difference in these results? A. the sphygmomanometer is likely in error B. the transducer is lower than the level to which it was originally zeroed and calibrated C. erratic movement of the arm with the arterial catheter D. the transducer is 8 inches or more above the level of the heart

The correct answer is : B Explanation : The transducer associated with an indwelling arterial catheter should be level with the heart in order to produce an accurate blood pressure reading. If the transducer is lower than the heart, increased blood pressure will be demonstrated on the monitor and the reported pressure will be erroneously high. If the transducer is above the level of the heart, the reported blood pressure on the monitor will be erroneously low, or less than the actual blood pressure of the patient.

Which of the following would be an indicator for the use of pressure control mechanical ventilation? A. decreasing plateau pressure below 30 cm H2O B. peak inspiratory pressure in excess of 50 cm H2O C. increasing compliance D. pulmonary emphysema

The correct answer is : B Explanation : The use of pressure-control ventilation is primarily indicated when peak inspiratory pressures are in excess of 50 cmH2O. This, however, is only true if the increased peak pressures are a result of decreasing pulmonary compliance and are not caused by temporary conditions such as excess secretions in the airways or bronchoconstriction

A patient undergoes a V/Q scan after the sudden onset of shortness of breath. The scan shows adequate ventilation and poor perfusion over a specific area of the lung. The patient would benefit most from which of the following? A. increase in minute ventilation B. Streptokinase C. Lasix (furosemide) D. administration of volume expanding fluids

The correct answer is : B Explanation : The ventilation perfusion scan that shows adequate ventilation but poor perfusion is associated with a pulmonary embolism. This clot in the pulmonary vasculature needs to be removed. Of the options offered streptokinase is a medication that can dissolve the clot.

A respiratory therapist is monitoring the volume of gas a patient can inhale after a passive exhalation of tidal volume. The therapist is attempting to observe which of the following? A. inspiratory reserve volume B. inspiratory capacity C. total lung capacity D. vital capacity

The correct answer is : B Explanation : The volume achieved when the patient inhales maximally after a passive exhalation of tidal volume is called inspiratory capacity. Comparatively, the maximum volume inhaled after complete active exhalation is called vital capacity.

A respiratory therapist notes the word cachectic used in the medical documentation to describe a patient. This is most consistent with A. obesity B. a wasted appearance C. obtunded D. non-attentiveness

The correct answer is : B Explanation : The word "cachectic" refers to a wasted appearance, skinny, and poor skin turgor. A person with AIDS or pulmonary tuberculosis may demonstrate this.

The physician orders mechanical ventilator settings: Mode assist/control VT 600 mL Mandatory rate 10/min I:E 1:2 FIO2 0.40 PEEP 5 cm H2O Which of the following represents the minimum inspiratory flow setting the respiratory therapist should select? A. 24 L/min B. 18 L/min C. 40 L/min D. 60 L/min

The correct answer is : B Explanation : There are several methods to determine the minimum flow needed to accomplish specific minimum flow settings on a mechanical ventilator. One shortcut method is to add the I:E ratio numbers together and multiply it by the minute ventilation. In this case, 1+2 = 3. Minute ventilation = (.6L x rate of 10) = 6.0 L. 6.0 L x 3 = 18 L/min. This is the minimum flow. When answering this question if the exact number is not available in the options, the correct choice would be the next highest number. For instance, if 18 L per minute was not an available option, the next best answer in this question would be 24 L per minute.

Which of the following is an appropriate strategy when caring for a patient with ARDS who is receiving mechanical ventilation? A. keep the patient in reverse trendelenburg position B. keep FIO2 at 60% or below if possible C. strive for C.I. values less than 2.0 L/min/m2 D. keep PEEP as high as possible

The correct answer is : B Explanation : There are two primary strategies for patients who have adult respiratory distress syndrome and are receiving mechanical ventilatory support - keep mean and peak airway pressures as low as possible and keep FIO2 below 60% as much as possible.

A respiratory therapist is determining a VD/VT ratio. Which of the following is needed to complete this analysis? A. VT and VA B. PECO2 and PaCO2 C. FIO2 and PaO2 D. PECO2 and PaO2

The correct answer is : B Explanation : There are two things required to determine a VD/VT ratio - end tidal CO2 and PaCO2.

The respiratory therapist is making a plan of care for a patient with mycoplasma pneumonia who needs assistance with airway clearance of secretions. In which order should therapy be performed? A. percussion, postural drainage, bronchodilator, coach coughing B. bronchodilator, percussion, postural drainage, coach coughing C. coach coughing, bronchodilator, percussion, postural drainage D. postural drainage, percussion, bronchodilator, coach coughing

The correct answer is : B Explanation : There is an oder at which procedures should be done to mobilize and remove secretions. The first steps are to open the airway, dislodge the sputum, move dispute into the upper airway, and then remove the sputum through coughing or suctioning.

Which of the following would NOT be needed in preparation for a bronchoscopy? A. normal saline B. Magill forceps C. 10 cc syringe D. anesthetic

The correct answer is : B Explanation : There is no purpose for bringing Magill forcepts to assist with a bronchoscopy procedure. Magill forcepts are used primarily for nasal intubation.

After abdominal surgery, a patient becomes short of breath and has shallow respirations. An arterial blood gas on a Venturi mask at FIO2 0.50 is done and results show: pH 7.48 PaCO2 32 torr PaO2 47 torr HCO3- 23 mEq/L BE -1 mEq/L The respiratory therapist will do which of the following to correct the condition? A. Administer a small volume nebulizer treatment with Albuterol B. Implement non-invasive positive pressure ventilation C. Use pressure support ventilation D. Institute mechanical ventilatory support

The correct answer is : B Explanation : This arterial blood gas reveals hyperventilation and hypoxemia. The hypoxemia must be corrected first. Of the options offered, implementing noninvasive positive pressure ventilation, also called BiPAP, will provide positive expiratory pressure, expand alveoli, and increase oxygenation. Providing mechanical ventilatory support is excessive because the patient is not demonstrating respiratory failure. There is no evidence of bronchoconstriction and therefore no need to administer a bronchodilator. Use of pressure support ventilation is used to increase spontaneous tidal volumes and minute ventilation. Clinical data does not show that there is a problem with ventilation.

Immediately after insertion of a nasopharyngeal airway the patient begins to cough violently and does not stop. The respiratory therapist should A. switch to an oral pharyngeal airway B. obtain and insert a shorter airway C. remove and lubricate the airway, then reinsert through the other nare D. spray Lidocaine spray in the oropharynx

The correct answer is : B Explanation : When a patient begins to cough uncontrollably after the insertion of a nasopharyngeal airway, also called a nasal trumpet, the airway is likely too long. A shorter airway should be used.

The following arterial blood gas results are obtained on a patient who has been on mechanical ventilation for 3 days. pH 7.55 PaCO2 40 torr PaO2 310 torr HCO3- 28 mEq/L BE -8 mEq/L FIO2 0.55 SaO2 98% Ventilator settings are: Mode Assist/control Mandatory rate 12 VT 550 mL FIO2 0.40 The respiratory therapist should evaluate the function of which of the following in the blood gas analyzer? A. Severinghaus electrode B. Clark electrode C. multi-wave length oximetery light D. Sanz electrode

The correct answer is : B Explanation : This blood gas reveals a problem. The PaO2 of 310 torr would be impossible with an FIO2 of only 40%. We know this from estimating the alveolar oxygen tension or doing the alveolar air equation. At 40% the maximum oxygen tension in the alveoli is about 230 torr. It is impossible for the arterial oxygen tension to be greater than the alveolar oxygen tension. Therefore, there is likely an error in the measurement of the PaO2. Of the options given, the Clark electrode is another name for the PO2 electrode.

The following data were obtained while an adult patient was receiving VC A/C ventilation with an FIO2 0.50. 0800 0815 0830 0845 HR 88 98 103 110 BP (mm Hg) 132/85 125/83 115/80 102/72 SpO2 (%) 82 90 94 96 Static Compliance 22 38 52 43 PEEP (cm H2O) 5 12 17 20 Which of the following PEEP levels should the respiratory therapist recommend? A. 5 cm H2O B. 17 cm H2O C. 20 cm H2O D. 12 cm H2O

The correct answer is : B Explanation : This group of data shows that with increasing PEEP the patient exhibits improved oxygen saturation. However, as PEEP is increased, there is an ultimate degradation in hemodynamics that may result. According to this data, the patient's hemodynamic status deteriorates significantly at a PEEP level of 20 cmH2O. This is manifested by a sudden decrease in pulmonary compliance from 52 to 43 mL/cmH2O. A Sharp decrease in blood pressure is also an indicator. Thus, the previous PEEP level of 15 is most appropriate.

An appropriate way to respond to an asthmatic patient who experiences nausea and tingling in her digits with albuterol therapy is to A. explain in simple words that these are normal, harmless side effects. B. reduce the dosage of the medication within limits. C. reduce the flowrate to the nebulizer during therapy. D. tell the patient not to breathe so deeply during the treatment.

The correct answer is : B Explanation : This is an example of therapy modification. It is important to maintain all therapy within the therapeutic range as modifications are made. For example, if one option suggested that the medication be reduced to a non-therapeutic level, it would be incorrect. In this case we have an option that permits a reduction of the dosage within proper limits, which is the correct choice.

While performing nasal tracheal suctioning, the respiratory therapist notices a drop in heart rate from 80/min to 62/min. After discontinuing the suction procedure, the therapist should A. make a recommendation to never suction the patient B. monitor the patient's heart rate closely C. inject Lidocaine down the airway prior to suctioning D. administer atropine

The correct answer is : B Explanation : This patient experiences a significant decrease in heart rate during the suction procedure. Once the procedure is discontinued, the heart rate should be monitored closely to ensure it rebounds to the normal baseline before the procedure.

A patient in the emergency room has the following arterial blood gas results: pH 7.18 PaCO2 30 mmHg PaO2 80 mmHg HCO3- 18 mEq/L BE -7 mEq/L SAT 94% This data is most typical of which of the following patient diagnoses? A. Flail chest B. diabetic ketoacidosis C. ARDS D. COPD

The correct answer is : B Explanation : This patient has a profound state acidosis as manifested by a low pH of 7.18. CO2 is low, which is associated with alkalosis. In this case the alkalosis is being caused by a profound decrease in HCO3-. This combination is associated with diabetic ketoacidosis.

A 95 kg (210 lb) male patient who is 5 ft. 5 in. (165 cm) tall is admitted to the intensive care unit after undergoing a coronary artery bypass graft. The patient required 100% oxygen during the entire course of the surgery. There are no spontaneous respirations. SpO2 is 97%. Which of the following would be appropriate initial ventilator settings? A. VC, SIMV, rate 12, FIO2 1.0, VT 650 mL, PEEP 5 cmH2O B. VC, A/C, rate 16, FIO2 1.0, VT 450 mL, PEEP 5 cmH2O C. VC, SIMV, rate 12, FIO2 0.5, VT 350 mL, no PEEP D. VC, SIMV, rate 18, FIO2 1.0, VT 750 mL, PEEP 5 cm H2O

The correct answer is : B Explanation : This patient is obese and therefore the correct tidal volume range should be calculated after determining the ideal body weight of the patient. This patient should weigh about 62 kg. Appropriate tidal volume is 6-10 mL/kg and rate should be set between 10-20. Because the patient has been receiving 100% oxygen they should continue to do so when placed on a ventilator. PEEP between 4-6 cm H2O is appropriate for an initial setting.

A 38-year-old female patient is admitted to the ICU with acute ventilatory distress and pneumonia. Arterial blood gas results show: pH 7.22 PaCO2 59 torr PaO2 62 torr HCO3- 24 mEq/L BE -2 mEq/L FIO2 0.40 The physician orders the institution of mechanical ventilation. The patient is (5 ft 1 in) 155 cm and weighs 95 kg (210 lb) The respiratory therapist will recommend which of the following initial settings? A. Rate 12, VT 350 mL, FIO2 0.40, PEEP 10 cm H2O B. Rate 16, Vt 400 mL, FIO2 0.40, PEEP 5 cm H2O C. Rate 16, Vt 550 mL, FIO2 0.65, no PEEP D. Rate 14, Vt 650 mL, FIO2 0.40, PEEP 5 cm H2O

The correct answer is : B Explanation : This patient is obese. Therefore, we must calculate the patient's predicted body weight before calculating appropriate tidal volume range. The patient should weigh about 48 kg. Based on the patient's height and weight, appropriate tidal volume is between 288 and 480 mL. Initial rate should be 10-20. FIO2 should be 30-60% (better to be 40-55%). PEEP should be 4 to 6 cm H2O

A 38-year old male is receiving ventilatory support by a high frequency jet ventilator (HFJV). Heart rate is 120/min and blood pressure is within normal limits. Arterial blood gas results on high frequency ventilation are as follows: pH 7.26 PaCO2 64 torr PaO2 82 torr HCO3- 26 mEq/L BE 0 mEq/L The respiratory therapist should recommend A. switching to volume-controlled ventilation B. increasing frequency C. switching to pressure control ventilation D. increasing amplitude

The correct answer is : B Explanation : This patient, who is on a high-frequency ventilator, shows evidence of hypoventilation as manifested by high PaCO2. Oxygenation appears adequate. To correct the hyperventilation, the most appropriate action is to increase frequency. This is equivalent to increasing the mandatory rate.

Despite repeated efforts to verbally coach a patient to cough and expectorate upper airway secretions as manifested by the presence of rhonchi, the patient is unable or unwilling to following commands. The respiratory therapist decides to perform NT suctioning. Which of the following is the most important consideration when performing NT suctioning?

Utilize ample water-soluble lubricant and be gentle.

While following orders to reduce the length of an ET tube, the therapist accidently cuts the ballard suction catheter that was inadvertently inserted into the ET tube by a few centimeters. The therapist is unable to locate the catheter fragment. The therapist should immediately A. extubate the patient B. recommend a bronchoscopy C. perform an emergent tracheotomy D. send the patient to MRI

The correct answer is : B Explanation : This situation suggests that during the severing of the endotracheal tube, the suction catheter was also inadvertently cut. Although the respiratory therapist cannot locate the severed portion visually, it can be assumed that the fragment has entered in through the endotracheal tube and is possibly in the patient's lungs. Although, a chest radiograph could identify the radio-opaque line on the suction catheter and confirm its location, a bronchoscopy could provide both visual evidence and therapeutic response, which would allow immediate removal of the fragment.

A patient in the intensive care unit has the following clinical and laboratory data: PB 747 mm Hg PH2O 47 mm Hg pH 7.28 PaCO2 52 mm Hg PaO2 70 mm Hg HCO3- 27 mEq/L BE +2 mEq/L FIO2 0.6 SaO2 1.0 CvO2 12 vol% CaO2 17 vol% Hb 15 gm/dL Which of the following represents the C(a-v)O2? A. 200 vol% B. 5 vol% C. 2 vol% D. 25 vol%

The correct answer is : B Explanation : To answer this question, one must complete two calculations. First, CaO2 and CvO2 must be determined. Once CvO2 is subtracted from CaO2, 5 vol% is the closest answer.

In preparation for a helium dilution study, a respiratory therapist is calibrating the helium analyzer. While exposing the analyzer to ambient room air, what will the analyzer read for helium concentration? A. 79% B. 0% C. 2% D. 21%

The correct answer is : B Explanation : To calibrate a helium analyzer, sometimes called a Wheatstone Bridge, the device must be calibrated to room air for the low calibration and to a known level of helium for the high calibration. Because room air has no significant level of helium, helium analyzers should read 0% when exposed to ambient room air conditions.

Which of the following should be done to improve gas distribution for a patient receiving mechanical ventilation and who has high airway resistance during the inspiratory phase? A. SBN2 study B. inspiratory plateau C. increase inspiratory flow D. lower the rate

The correct answer is : B Explanation : To improve gas exchange in the lungs, inspiratory time must be increased. This is usually done by decreasing the inspiratory flow rate. However, that is not an option in this question. Of the options listed, instituting an inspiratory plateau, or pause, will result in increased inspiratory time and improved gas exchange. Lowering the rate will increase expiratory time but will not affect inspiratory time. A SBN2 study will assess the evenness of distribution in the lungs but will not actually promote or encourage better gas distribution.

The following data is available for a 62-year old patient with arterial sclerosis. MAP 93 torr CVP 3 torr mPAP 18 torr PCWP 8 torr Q.T. 5.0 L/min PEEP 5 cm H2O C.I. 2.5 L/min/m2 The physician asks the therapist to determine systemic vascular resistance and record the findings in the patient's medical record. The therapist should record which of the following values? A. 160 dynes B. 1440 dynes C. 1200 dynes D. 1600 dynes

The correct answer is : B Explanation : To obtain a systemic vascular resistance (SVR) value, CVP should be subtracted from MAP. (93-3) = 90. Then, the result should be divided by cardiac output. In this case, cardiac index should be multiplied by 2 to estimate cardiac output. This gives us a cardiac output of 4 L. (90/5 = 18). Finally, we must multiply that result by 80 to obtain the measurement unit of dynes. SVR = 18 x 80 = 1440 dynes.

A patient receiving 30% supplemental oxygen by Venturi mask is complaining of claustrophobia and will not comply with the ordered therapy. The therapist should suggest which of the following as an alternative? A. Simple mask at 6 L/min B. Nasal cannula at 3 L/min C. Nonrebreathing mask at 12 L/min D. Sedate the patient

The correct answer is : B Explanation : To prevent claustrophobia, switching to a nasal cannula for oxygen delivery is appropriate. To approximate the oxygen percentage of 30%, the oxygen flow rate to the nasal cannula should be set to 3 L/min. With a nasal cannula, 1 L approximates a 3-4% change in oxygen. Therefore, 3 L/min approximates a 9% increase in oxygen percentage above room air. 21% + 9% = 30%.

A respiratory therapist should open the airway by using the jaw-thrust maneuver for a patient experiencing A. a difficult intubation requiring application of cricoid pressure B. a serious motor vehicle accident C. vomiting D. a carotid massage

The correct answer is : B Explanation : Trauma caused by motor vehicle accidents may result in injury to the spinal cord, or vertebrae. Additional movement or manipulation of the neck or upper back may result in additional damage, which could result in permanent paralysis. The jaw-thrust maneuver will prevent further damage to the spine.

A patient is brought to the emergency room after a motor vehicle accident. Wounds are severe and massive bleeding is occurring internally. The respiratory therapist should place the patient on A. partial rebreathing mask B. nonrebreathing oxygen mask C. Venturi mask D. nasal cannula

The correct answer is : B Explanation : Trauma victims who have massive bleeding externally or internally are at risk of death from profound hypoxemia due to the loss of hemoglobin, or the oxygen-carrying capacity of the blood. The most appropriate action is to administer maximum FIO2. Of the options listed, this can best be done by using a non-rebreathing oxygen mask.

A 40-week gestational age neonate is receiving time-cycled, pressure-limited ventilation with an I-time of 0.4 seconds, PIP 28 cm H2O, RR of 25/min, FIO2 0.45 and PEEP 5 cm H2O. As the flowrate is increased, what will happen to the I:E ratio? A. Decrease I-time, Increase E-time B. No change C. Decrease D. Increase

The correct answer is : B Explanation : Unlike volume-cycled mechanical ventilation, changing the flow rate will not change inspiratory or expiratory time. However, changing the flow rate will change the delivered tidal volume. Tidal volume is affected by peak inspiratory pressure, flow rate, and inspiratory time.

A patient is receiving volume-controlled ventilation in the emergency department (ED). The following data is available: Mode Assist/control VT 500 mL Mandatory rate 14 FIO2 0.5 PEEP 5 cm H2O PetCO2 20 torr pH 7.39 PaCO2 40 torr PaO2 100 torr HCO3- 25 mEq/L BE +1 mEq/L The respiratory therapist should report which of the following as an accurate VD/VT ratio? A. 75% B. 50% C. 15% D. 25%

The correct answer is : B Explanation : VD/VT ratio calculation is (PaCO2-PetCO2)/PaCO2. In this case 40 -20 = 20. 20/40 = 50%

A mechanically ventilated asthma patient has recently responded to bronchodilator therapy resulting in increased dynamic compliance. The patient is coughing, but high peak-pressure alarms are not sounding. The respiratory therapist should A. sedate the patient. B. turn down the high-pressure limit. C. discontinue mechanical ventilation. D. hyperoxygenate.

The correct answer is : B Explanation : When a patient coughs on a ventilator, it should trigger the high-pressure alarm. Cough is the most common response to secretions in the large airways, which require suctioning. Also, when pulmonary dynamic compliance increases it is easier to ventilate the patient's lungs causing the peak pressures to go down, resulting in the need to reduce the high-pressure alarm limit.

Which of the following is associated with hypokalemia? A. respiratory acidosis B. flattened T waves C. significant Q waves D. inverted P waves

The correct answer is : B Explanation : When a patient has hypokalemia the ECG waveform will show flattened T waves. Conversely, if the patient has hyperkalemia, spiked T waves may be observed.

For which of the following developments should the therapist monitor carefully for several hours after a patient self-extubates? A. friction rub B. stridor C. hyperventilation D. wheezing

The correct answer is : B Explanation : When a patient inadvertently or purposefully removes an endotracheal tube, he or she has not removed the air from the ET tube cuff. While pulling the tube out, it is possible for the inflated cuff to injure or inflame the upper airway. This could result in stridor.

While attempting to administer abdominal thrusts on a patient who appears to have aspirated food and is unable to breathe, the respiratory therapist is unable to reach all the way around the patient's abdomen due to obesity. The therapist should next A. obtain an abdominal support belt B. perform chest thrusts C. perform a modified abdominal thrust with only one fist D. utilize the back of a chair

The correct answer is : B Explanation : When a patient is too obese to receive abdominal thrusts, chest thrusts are appropriate. This is also true of pregnant women in need of airway clearance maneuvers.

A patient admitted to the hospital for pneumonia is receiving oxygen therapy by transtracheal catheter. Twenty minutes after the completion of a meal, the patient complains of shortness of breath and expresses the device is not working properly. The respiratory therapist should first A. increase the oxygen flow rate to the catheter B. initiate oxygen by air-entrainment mask C. apply suction to the catheter D. remove the catheter

The correct answer is : B Explanation : When a problem is encountered with one oxygen delivery modality, the first best response is to initiate oxygen therapy with a different modality and then troubleshoot the problem.

A chest radiograph reveals the location of a pulmonary artery pressure catheter within moments after its insertion. The tip of the catheter appears over the lower right lung field. Based on this observation, the catheter A. should be withdrawn. B. is properly placed. C. should be advanced. D. has perforated the right ventricular septal wall.

The correct answer is : B Explanation : When a pulmonary artery catheter is positioned correctly, the tip will appear over the right lower lung field.

Which of the following questions would be best to determine the location and intensity of chest pain in an adult with no identified language or communication barriers? A. "Is the pain you are experiencing bearable?" B. "Can you please describe the location and intensity of the pain?" C. "Do you have a radiating pain emanating from your left chest?" D. "Would you describe the pain in your chest as a 10?"

The correct answer is : B Explanation : When asking an patient to describe the location and intensity of chest pain, it is important that the inquiry be done with open-ended questions. In other words, questions that may result in a simple "yes" or "no" should be avoided.

A patient's tissue oxygen consumption has increased and cardiac output has decreased. Which of the following is also likely true? A. A-aDO2 has decreased B. C(a-v)O2 has increased C. cardiac index has increased D. SaO2-SVO2 has decreased

The correct answer is : B Explanation : When cardiac output decreases, the speed of blood flow across the tissue bed is also decreased. As blood flows slower over tissues, the tissues have less ability and time to absorb oxygen molecules and therefore will demonstrate increased oxygen consumption. As oxygen consumption increases, the difference in arterial oxygenation and venous oxygenation also increases. Thus, an increase in C(a-v)O2 is observed.

A patient is receiving bi-level therapy at the following settings: IPAP 15 cm H2O EPAP 5 cm H2O FIO2 0.5 Recent arterial blood gas results reveal hypoxemia. To compensate, the respiratory therapist increases the EPAP setting to 8 cm H2O. What other change should be made to ensure a consistent level of ventilatory support is maintained during spontaneous breaths? A. Add a back-up rate B. Increase IPAP to 18 cm H2O C. Decrease IPAP to 10 cm H2O D. Switch to full ventilatory support with a mandatory rate and a PEEP of 8 cm H2O

The correct answer is : B Explanation : When correcting hypoxemia with noninvasive positive pressure ventilation, EPAP should be increased. However, so as not to decrease ventilation inadvertently, the IPAP setting should be increased by the same amount in order to keep the distance between EPAP an IPAP unchanged.

Which of the following should be looked at FIRST to assess for proper placement of an oral endotracheal tube? A. lateral neck radiograph B. left chest symmetry C. breath sounds D. chest radiograph

The correct answer is : B Explanation : When examining for proper placement of an endotracheal tube the first actions that should be taken should be those that are less time-consuming. Three of these options are all suitable for determining proper positioning of the endotracheal tube. However, the quickest way to determine positioning would be to examine the left chest symmetry. Breath sounds is appropriate but takes a little more time. Obtaining a chest radiograph will definitely help to confirm positioning but will take significant time. So, if asked "what is the first step one would take", the answer would be whatever is the quickest. But, if asked what is the best way to determine the position of the endotracheal tube, the best answer would be a chest x-ray.

A patient in the emergency room has been diagnosed with status asthmaticus. She has received IV steroids, MDI steroids, bronchodilators and antibiotics. Which medication should be discontinued first once the patient becomes stable? A. MDI steroids B. bronchodilators C. IV steroids D. antibiotics

The correct answer is : C Explanation : A patient with status asthmaticus would benefit from IV steroids such as methylprednisolone. Of the medications given IV steroids should be discontinued first.

A patient is on 80%/20% heliox therapy by nonrebreathing mask. If flowing through an oxygen flow meter, what is the actual flow of the mixture if the flow meter indicates 10 L/min? A. 8 L/min B. 18 L/min C. 16 L/min D. 14 L/min

The correct answer is : B Explanation : When helium-oxygen mixtures are run through flow meters that are intended for oxygen only, the indicated flow on the flow meter is inaccurate. This is because the helium-oxygen mixture is less viscous and does not push the indicator ball up as much as does oxygen. In this case, a correction factor must be used depending upon the mixture percentages. For an 80/20% mixture a correction factor of 1.8 must be used. A 70/30% mixture has a correction factor of 1.6 and a 60/40% mixture has a correction factor of 1.4. In this problem the 80/20% mixture factor of 1.8 must be multiplied by the indicated flow on the oxygen flow meter to calculate actual total gas flow. When we do this we get a total gas flow of 18 L/min.

Which of the following would result in an increase in CVP? A. hypokalemia B. hypervolemia C. decreased PVR D. increased SVR

The correct answer is : B Explanation : When the body retains fluid all hemodynamic values are increased especially CVP. By itself a high CVP most likely indicates fluid retention and hypervolemia.

A 30-weeks of gestation infant is receiving mechanical ventilatory support with a time-cycled, pressure limited ventilator at the following settings: PIP 35 cm H2O Rate 28 FIO2 0.4 PEEP 4 cm H2O I-time 0.75 sec The respiratory therapist notices the pressure manometer is not rising to the set pressure during each inspiration. Which of the following is the mostly likely cause? A. set pressure exceeds the pressure capability of the ventilator B. the pressure-limiting pop-off valve is set to low C. internal flow rate is too high D. rate is insufficient

The correct answer is : B Explanation : When the pressure manometer is not rising to the set pressure, the cause may be one of two sources. First, the problem may be that the operational flow in the ventilator is too low. Next, there is a possibility the pressure-limiting pop off valve is set too low. This is the most likely answer, of the options given.

A patient with a confirmed pulmonary embolism is receiving oxygen by non-rebreathing mask. When oxygen therapy was initiated, the reservoir collapsed completely with each breath. Thirty minutes later the reservoir bag collapses only partially with each breath. Respiratory rate has fallen from 28/min to 10/min. The therapist should respond by A. assisting ventilation with a manual resuscitator B. continuing current therapy C. increasing flow to the NRB mask D. beginning mechanical ventilation

The correct answer is : B Explanation : When the reservoir on a non-rereading mask collapses completely, one must suspect that the inspiratory flow the patient is producing is exceeding the total flow to the patient from the flow meter. The solution to this is to increase the flow. However, in this case after a few moments, the bag begins to collapse only partially which is normal. This indicates that the patient has become less distressed and that the oxygen therapy has been effective. Continuing current therapy is most appropriate.

A patient being treated for a left-sided hemothorax following multiple injuries sustained in a motor vehicle accident is receiving positive pressure ventilation by a PB 840 volume ventilator. A 3-chamber chest tube drainage system is in place and functioning properly. During a routine ventilator check, the respiratory therapist notes the low-return volume begins to alarm. Which of the following would be appropriate to check to determine the cause of the alarm? A. cuff pressure for possible excessive inflation B. degree of bubbling in the water-seal compartment C. possible excess condensate in the ventilator circuit D. possible ET tube occlusion

The correct answer is : B Explanation : When the ventilator emits a low return volume alarm and the patient is described as having chest tubes in place, one must be worried about volume being lost through a hole in the lungs. This hole is also called a perforation. One way to quickly determine whether this is the case is to examine the water seal compartment of the chest tube drainage system. Normally, only gentle bubbling should be observed in that compartment. If significant bubbling is observed, then ventilatory volume is likely being lost through the chest tube drainage system to a perforation in the lung tissue.

A patient being monitored by a balloon-tipped pulmonary artery catheter (Swan-Ganz) requires transfer to the radiology department for a V/Q scan. Which of the following should be monitored to ensure proper placement of the Swan-Ganz catheter during transport? A. SpO2 waveform B. pulmonary artery C. pulmonary capillary wedge pressure D. central venous pressure

The correct answer is : B Explanation : While transporting a patient with a pulmonary artery catheter, the most important consideration is the position of the catheter during movement. Since transports involve transfers from beds, it is sometimes easy to inadvertently change the catheter's position. Therefore, the most important monitoring technique includes watching the pulmonary artery pressure closely. If the pressure or the waveform changes significantly, an inadvertent change in pulmonary artery catheter position must be suspected.

Which of the following equipment is most helpful at measuring FRC in a patient who has significant non-ventilated lung spaces? A. peak flow meter B. plethysmograph C. helium analyzer for a helium dilution test D. nitrogen analyzer for a nitrogen washout test

The correct answer is : B Explanation : You can determine FRC in three different ways - body box, nitrogen washout, or helium dilution. If someone has non-ventilated lung space, nitrogen washout and helium dilution are not helpful and may be inaccurate. FRC determined by body box will be higher because it can access non-ventilated lung space and therefore is more accurate.

Massive diaphoresis would most likely be associated with which of the following conditions? A. tuberculosis B. CHF C. ARDS D. pulmonary hypotension

The correct answer is : B Explanation : Massive diaphoresis, of the options given, is most closely associated with congestive heart failure.

The respiratory therapist notes an order in a patient's record for 3 mg of a drug. The normal stock concentration of the drug is 0.15%. How many mL will be required to meet the ordered dosage? A. 20.0 B. 2.0 C. 4.5 D. 1.6

The correct answer is : B Explanation : To determine the number of mL needed, one must first determine how many mg per mL exists in that drug at a specific concentration. This can be done by taking the concentration (strength) and multiplying it by 10. ie, 0.15% x 10 = 1.5 mg/mL. In this case, 3 mg is needed. Thus 3 mg/1.5 mg/mL = 2.0 mL.

Following extubation, after shoulder surgery, a 15-year-old male patient is experiencing 30-second periods of apnea. The respiratory therapist should A. provide SVN with normal saline 0.3% B. intubate the patient C. ventilate with a manual resuscitator D. place on 100% NRB mask

The correct answer is : C Explanation : 30-second periods of apnea indicate the need for immediate assistance with ventilation. Of the choices offered, only ventilation with a manual resuscitator meets this requirement. One may be tempted to choose to intubate the patient, but intubation does not automatically imply that mechanical ventilation will be provided. Intubation is for airway protection.

A patient has a sleep study (PSG) which shows an AHI of 45 and an average oxygen desaturation of 8%. Which of the following should the respiratory therapist recommend? A. use apnea impedance monitor when sleeping B. supplemental oxygen at night C. nasal CPAP titration study D. nocturnal administration of Doxapram

The correct answer is : C Explanation : A PSG is another name for a sleep study. AHI stands for apnea-hypopnea index. An index greater than 30 indicates severe sleep apnea. The gold standard treatment for this is nasal CPAP. But, a titration study should first be done to determine the appropriate level of positive airway pressure that should be prescribed for the patient.

A 58-year-old female admitted for mycoplasma pneumonia with significant infiltrates throughout all lung fields has a PaO2 of 58 torr while on FIO2 0.60 and PEEP of 15 cm H2O and is receiving volume-controlled ventilation. Which of the following should the respiratory therapist recommend FIRST to improve the patient's oxygenation? A. Administer tetracycline B. Switch to pressure-controlled ventilation C. Increase PEEP to 20 cm H2O D. Increase FIO2 to 1.0

The correct answer is : C Explanation : A PaO2 of 58 mmHg with an FIO2 of 0.6 is an indication to utilize positive end-expiratory pressure, or PEEP. Since PEEP is already set to 15 cmH2O, increasing to 20 cmH2O is most appropriate.

A patient with a tracheostomy tube is receiving heated aerosol via T-piece with a large-volume nebulizer set at 0.35. The T-piece repeatedly disconnects when the patient changes position from side to side. The respiratory therapist should recommend A. Sedating the patient B. Instructing the patient not to turn so often C. Replacing the T-piece with a tracheostomy collar D. Securing the T-piece with cotton ties

The correct answer is : C Explanation : A T-piece connects directly to the tracheostomy tube and is sometimes inadvertently disconnected when the patient turns from side to side or otherwise changes position. This can be addressed by replacing the T-Piece with a tracheostomy collar. This device hangs loosely around the opening of the tracheostomy tube and can tolerate significant movement of the patient.

A patient is receiving mechanical ventilatory support on the following settings: Mode Assist/control Mandatory rate 14 Tidal volume 550 mL FIO2 0.5 PEEP 5 cm H2O Insp Flow 45 L/min The following capnographic data can be explained by which of the following conditions? A. increased minute ventilation B. excess water on the infrared sensor C. hypoventilation D. secretions on the capnographic sensor

The correct answer is : C Explanation : A capnograph that shows an end-tidal CO2 of 40 mmHg or greater is indicative of hypoventilation. Remember, an end-tidal CO2 of 40 mmHg is equivalent to an arterial CO2 level of 50 mmHg

Which of the following types of indicators is used to show that a piece of nondisposable equipment did not go through a proper sterilization process? A. torn package B. misshapen equipment C. chemical tape indicator D. biological spores indicator

The correct answer is : C Explanation : A chemical tape indicator found inside an enclosed package of nondisposable equipment can be used to determine if the equipment was processed through the sterilization procedure correctly. From this method, practitioners can peer into the package and determine the likelihood of a sterile condition. Keep in mind that the tape may only indicate if the process was followed but does not indicate the presence or absence of viable spores (living organisms).

A patient with cystic fibrosis is receiving routine postural drainage and percussion. Which of the following may be used to determine the effectiveness of therapy? A. DLCO measurements B. Sputum analysis C. Chest radiograph D. Arterial blood gas analysis

The correct answer is : C Explanation : A chest radiograph, or x-ray, would be most helpful at determining the effectiveness of the postural drainage and percussion therapy. Arterial blood gas assessment and sputum analysis are too indirect to assess effectiveness. DLCO measurement is not related and not helpful.

Which of the following should NOT be included in the initial assessment of a comatose patient in the intensive care unit? A. Breath sounds B. Vital signs C. Symptoms D. General appearance

The correct answer is : C Explanation : A comatose patient is not cable of self-reporting problems. Therefore, subjective information, or symptoms cannot be determined. Signs, however, which are objective, can be determined by the caregiver, independent of the patient's ability to communicate.

Which of the following chest radiograph observations is most closely associated with pleural effusion? A. tree-in-winter pattern B. reticulogranular pattern C. concave interface border D. complete opacification

The correct answer is : C Explanation : A concave interface border is associated with the pleural effusion. A reticulogranular pattern is associated with adult respiratory distress syndrome. Complete opacification of the lung is related to pneumonia.

The addition of a heat moisture exchanger (artificial nose) to a ventilator circuit between the wye and the patient would result in which of the following? A. decrease in mean airway pressure B. decrease VD/VT ratio C. increase in arterial carbon dioxide tension D. decrease in arterial carbon dioxide tension

The correct answer is : C Explanation : A heat moisture exchanger, or HME, is considered deadspace in the ventilator circuit because it is added between the patient and the wye of the ventilator circuit. This will raise arterial carbon dioxide tension by 1-2 mmHg. The addition of deadspace will raise carbon dioxide while removing deadspace will reduce it.

A patient with suspected vocal cord dysfunction and paralysis is undergoing pulmonary function testing to further investigate the problem. Which of the following PFT exams will yield most information about the issue? A. SBN2 B. nitrogen washout C. FVL D. DLCO

The correct answer is : C Explanation : A pulmonary function exam can reveal the presence of vocal cord dysfunction and/or paralysis by a flow volume loop (FVL). The condition will manifest itself by showing what is called a round loop, or a loop whose inspiratory phase looks similar to the expiratory phase. Normally, these two phases should look drastically different.

A patient in the emergency room, who is breathing rapidly and deeply, is receiving heated aerosol by mask and large-volume nebulizer with FIO2 set at 1.0 and the flow set at 15 L/min. The therapist notices the aerosol disappears completely with each breath. Pulse oximetry shows an oxygen saturation of 88%. The therapist should recommend A. switch to a non-rebreathing mask at the same flow B. increase flow to 20 L/min C. a tandem aerosol device D. decrease FIO2 to 0.6

The correct answer is : C Explanation : A large volume nebulizer set at 60% with a flow of 15 L/min is producing a total gas flow of 15 L/min. Because this patient is breathing rapidly and deeply and is hypoxic, it is likely that the total flow is not meeting the inspiratory demands of the patient. Because the flow of the large-volume nebulizer may not be increased significantly due to back pressure the solution for this case is to add another tandem large-volume nebulizer device, producing a total gas flow of 30 L/min.

Which of the following is most likely in a patient who is complaining of a non-productive cough that started 2 months ago? A. foreign body aspiration B. bronchitis C. bronchial cancer D. emphysema

The correct answer is : C Explanation : A long-term, non-productive cough that that persists for periods ranging from three weeks to several months is usually associated with a growth or mass in the bronchial tree. The mass may be benign or cancerous but will irritate the bronchial tree and cause a nonproductive cough as it grows.

A respiratory therapist is alerted by a ventilator alarm of a 62-year-old patient with COPD who was intubated for bacterial pneumonia. Upon arrival, the therapist notes the activation of the low PEEP alarm. For which of the following should the therapist investigate to determine the source of the alarm? A. pinched inspiratory limb of the circuit B. lung tissue perforation C. reduced ET tube cuff pressure D. occlusion of the ET tube

The correct answer is : C Explanation : A low PEEP alarm occurs when there is a continuous leak in the system, disallowing PEEP pressure to accumulate and hold steady. Of the options given, a reduction in ET tube cuff pressure could be the only likely cause. A pinched circuit or an occlusion of the ET tube would trigger a high-pressure alarm. A perforation in the lung tissue could indeed cause a low pressure, low volume, or low PEEP alarm but it is very unlikely.

A pulmonary rehabilitation patient is being monitored for compliance to a smoking cessation program. Which of the following values would be most helpful to evaluate? A. metHb B. SPO2 C. COHb D. PaO2

The correct answer is : C Explanation : A patient on a smoking cessation program may be monitored for compliance by periodically assessing the COHb. If COHb remains elevated or increases, the patient is likely not compliant with the program.

Which of the following clinical data can be obtained from a patient who is comatose? A. dyspnea level B. vital capacity C. oxygenation status D. subjective information

The correct answer is : C Explanation : A patient who is comatose is unable to comply with verbal commands. Of the options offered, the only data that requires compliance to a verbal command is "vital capacity".

A respiratory therapist is alerted by a low-volume ventilator alarm on a patient who has a chest tube drainage system in place. Set tidal volume is 600 mL while return tidal volume is 190 mL. The therapist should FIRST A. insert an additional chest tube B. increase tidal volume to compensate C. clamp the chest tube near the patient D. disconnect the chest tube from wall suction pressure

The correct answer is : C Explanation : A patient who is receiving mechanical ventilation, and has a chest tube drainage system in place, must be monitored carefully for lost volume through the lung through the chest tube drainage system. The clinical evidence of this is found in the return tidal volumes. If the volume administered is far greater than the return volume, volume must be being lost in the system somewhere. A loss of volume may be noticed by the excessive bubbling in the water seal compartment. To determine how it is being lost, the first step would be to clamp the chest tube near the patient. If bubbling in the water seal compartment stops, the volume being lost is most likely coming from a perforation in the lung. The patient requires surgery to repair this.

A patient who is orally intubated has been dependent on mechanical ventilation for several weeks. In spite of multiple attempts, the patient has failed to wean from ventilatory support. Which of the following would be most helpful in the further weaning of the patient? A. utilize high-frequency jet ventilation B. avoid weaning for several weeks, then reattempt C. surgical intubation with a tracheostomy tube D. cold cessation of ventilation

The correct answer is : C Explanation : A patient who requires mechanical ventilatory support for several weeks or months, the patient would benefit from a tracheostomy tube. A tracheostomy is more suitable for long-term mechanical ventilation and will assist in weaning as it lowers airway resistance compared to a standard oral endotracheal tube.

A mal-positioned tracheostomy tube is detected by chest radiograph for a patient who is still receiving partial ventilation through the tube. Subcutaneous emphysema is present in the upper chest. Palpation of the affected area would produce A. air accumulation below the skin. B. soft tissue hyperlucency. C. popping or crackling sensations. D. dry crackles.

The correct answer is : C Explanation : A patient with an inappropriately positioned tracheostomy tube may experience air in the dermal and subdermal spaces of the skin around the neck and upper chest. Upon palpation air bubbles below this may move around or may burst, causing crackling or popping sensations during palpation.

A 38-year-old patient is 2 hours post surgery for a hernia repair. The patient is resting comfortably on 3 L/min oxygen through a nasal cannula. Which of the following would be most appropriate for evaluating the patient's response to oxygen therapy? A. arterial blood gas analysis B. monitor respiratory rate C. measure SpO2 D. monitor color of the nail beds

The correct answer is : C Explanation : A patient with hernia repair, and who has no history of pulmonary problems, is not likely having difficulty with ventilation. But, because the patient is receiving oxygen therapy, monitoring its effectiveness is important. Of the options offered, an arterial blood gas provides more data than is needed and is painful and difficult for the patient. Therefore, the most appropriate option to specifically measure the patient's response to oxygen therapy is to measure the oxygen saturation or SaO2.

A patient is in the intensive care unit following a motor vehicle accident. The respiratory therapist notices cyanosis from the waist up. Additional data shows: ABG pH 7.38 PaCO2 42 torr PaO2 78 torr HCO3- 24 mEq/L BE 0 mEq/L PetCO2 18% Hb 12 gm/dL CBC 9,000 cu mm Which of the following should the therapist recommend?

V/Q scan

A respiratory therapist is asked to quickly assess an intubated, mechanically ventilated patient for a possible pulmonary embolism. Which of the following would be most helpful? A. EZ-cap CO2 detector B. multiple wave-length spectrophotometer C. capnometer D. pulse oximeter

The correct answer is : C Explanation : A quick way to assess for the possibility of a pulmonary embolism is to compare exhaled CO2 with arterial CO2. If these two values are very far apart, a pulmonary embolism is highly suspected. Though, it is not diagnostic, it is a quick method of assessment. An example of this would be an arterial CO2 that is 45 with an exhaled CO2 a 16. This suggests that carbon dioxide is having difficulty leaving the blood and entering the alveoli. A clot in the pulmonary vasculature, which would prevent blood flow around a large portion of alveoli, could cause this clinical outcome.

A radiology report of a chest radiograph indicates a round-shaped infiltrate measuring 2 cm in diameter in the left lung. Which of the following tests would be helpful in further identifying the nature of this infiltrate? A. transesophageal echo (TE) B. sputum culture C. series of oblique chest radiographs D. V/Q scan

The correct answer is : C Explanation : A round-shaped infiltrate found on an x-ray is concerning because it may be a cancerous mass. If it is a mass it will have a three-dimensional quality to it and therefore a series of x-rays (radiographs) taken from different angles will show the three dimensional shape of the object. The purpose of chest x-rays taken from nontraditional angles is to focus on an object and get a three-dimensional understanding of its shape.

One minute after delivery by c-section, the respiratory therapist does NOT do which of the following on a neonate? A. check heart rate B. view color C. auscultate the lungs D. assess general appearance

The correct answer is : C Explanation : APGAR (appearance, pulse, grimace, activity, and respiratory effort) is done on every newborn after 1-minute and after 5-minutes of life. Assessment of respiratory effort is done by assessing the presence and strength of the baby's cry, not lung sounds.

A one-minute APGAR assessment reveals a score of three. What is the appropriate response by the respiratory therapist? A. provide positive pressure ventilation with a flow-inflating resuscitator B. warm the infant and provide tactile stimulation C. start chest compressions and positive pressure ventilation D. provide blow-by oxygen

The correct answer is : C Explanation : APGAR scores are calculated 1 and 5 minutes after birth. A score of less than 4 indicates that CPR is required.

During the administration of a breathing treatment with Albuterol, the patient suddenly complains of nausea. Heart rate has risen from 80/min to 102/min. The respiratory therapist should A. Continue treatment but monitor carefully B. Plan to reduce dosage for future treatments C. Discontinue the therapy D. Instruct the patient to breath slower

The correct answer is : C Explanation : Adverse reactions during any therapy should be responded to initially by discontinuing therapy and notifying the physician.

A 183-cm (6-ft), 87-kg (192-lb) male with ketoacidosis is receiving 60% oxygen by air-entrainment mask with the flow meter set at 15. SpO2 is fluctuating widely from minute to minute in spite of a good waveform and heart rate correlation. The most likely cause of this is A. poor patient position B. a shift in the oxygen dissociation curve C. inadequate total gas flow D. unstable blood glucose levels

The correct answer is : C Explanation : Although this patient is receiving supplemental oxygen through an air-entrainment device, SpO2 is fluctuating widely from minute to minute. This is likely due to the inability to meet the patient's inspiratory demand based upon their minute ventilation. A patient who is in a state of ketoacidosis will also be breathing at an elevated rate and increased tidal volume. This suggests a remarkably large inspiratory demand. At 60%, with the flow meter at 15 L/min, an air-entrainment mask would provide a total flow of 30 L per minute. It is likely that this does not meet the inspiratory demand of the patient. Therefore, the source of the fluctuating SpO2 is likely due to inadequate total gas flow. Although the question does not ask for a resolution, an appropriate way to deal with this would be to provide a tandem aerosol device, which would also deliver 30 L per minute. Together, 60 L per minute would likely exceed the patient's inspiratory demand and prevent wide variation in oxygen saturation.

To calculate alveolar minute ventilation, the respiratory therapist should A. multiply the RR by the VT. B. measure with a metabolic cart study. C. subtract anatomical deadspace from the VT, then multiply by the RR. D. measure while the patient is receiving mechanical ventilation.

The correct answer is : C Explanation : Alveolar ventilation is calculated by subtracting the anatomical dead space from the tidal volume then multiplying by the respiratory rate. If the weight is known, the absolute value in pounds can be used to estimate anatomical deadspace. For example a patient weighing 130 pounds would have 130 mL of anatomical dead space. If weight is unknown, anatomical dead space should be estimated at 150 mL.

Prior to performing an arterial puncture, a modified Allen's test is performed on the patient's right radial artery. When the ulnar artery occlusion is released, a pink color returns in 3 seconds. Based on this result the respiratory therapist should A. perform a right brachial artery puncture B. perform a femoral artery puncture C. proceed with the puncture of the right radial artery D. perform an Allen's test on the left radial artery

The correct answer is : C Explanation : An Allen's test is performed to ensure there is collateral circulation prior to performing an arterial puncture. The presence of collateral circulation, blood flow through the radial and ulnar arteries, helps to lower the risk of the puncture. The procedure is done by occluding both the ulnar and radial arteries simultaneously. Once the hand becomes blanched (white and seemingly devoid of blood) the ulnar artery is released and the hand is observed to see if color returns in a timely manner. In this case, color returned in 3 seconds indicating good circulation through the ulnar artery. This is an indication that the puncture may occur safely in the radial artery.

Which of the following is consistent with acute-on-chronic hypercapnia? A. pH 7.35, PaCO2 52 torr, PaO2 56 torr B. pH 7.30, PaCO2 50 torr, PaO2 65 torr C. pH 7.27, PaCO2 70 torr, PaO2 60 torr D. pH 7.52, PaCO2 42 torr, PaO2 55 torr

The correct answer is : C Explanation : An acute-on-chronic hypercapnia condition is indicated when the pH of a patient with COPD is below compensated values (7.35) and CO2 is higher than the normal baseline for that patient. In this case, a pH of 7.27 is considered uncompensated respiratory acidosis and the CO2 is well above the patient's normal predicted baseline.

The PetCO2 waveform is obtained from a patient receiving mechanical ventilation. The PetCO2 reading is 40 mm Hg. Breath sounds reveal bilateral wheezing. Which of the following actions is most appropriate for this patient? A. decrease expiratory time B. increase respiratory rate C. administer a bronchodilator D. increase inspiratory time

The correct answer is : C Explanation : An end-tidal CO2 of 40 mmHg is consistent with an arterial CO2 of 50 mmHg, which indicates hypoventilation. In addition to this data, breath sounds reveal bilateral wheezing. Of the options listed administering a bronchodilator is indicated.

A respiratory therapist has been paged to the neonatal intensive care unit to assess an infant who is 38 weeks of gestation, delivered 3 hours prior. Which of the following would be appropriate to include in the assessment? A. oxygen index B. R/Q value C. Record of APGAR, at 1 and 5 minutes D. symptoms

The correct answer is : C Explanation : An infant is incapable of communicating problems such as shortness-of-breath and pain. This type of information is considered to be subjective and is also referred to as "symptoms". APGAR scores can be very helpful in directing the further care of the patient. Oxygen index is generally used in complex oxygenation cases and is not really appropriate at this stage in the case.

An infant born 24 hours prior is experiencing frequent periods of apnea lasting more than 60 seconds. Which of the following medications would be helpful in stimulating the infant's respiratory drive? A. hormone B. Exosurf C. Aminophylline D. Fluticasone

The correct answer is : C Explanation : An infant who experiences apnea periods greater than 60 seconds may need respiratory stimulation. This may be accomplished by administering the medication Aminophylline. Aminophlylline is only intended for ventilatory stimulation in infants. The same medication is used for long-term bronchodilation in adults.

A 34 week gestational age infant was born 20 minutes ago and had a five-minute APGAR score of 6. A chest radiograph shows bilateral haziness and a reticulogranular pattern. Which of the following would be most beneficial to the patient? A. Aerosolized bronchodilator therapy B. Transillumination C. Surfactant therapy D. Echocardiography

The correct answer is : C Explanation : An infant's chest x-ray that shows bilateral haziness is related to pulmonary immaturity. Fetal lung immaturity is best remedied by the administration of surfactant.

What is the humidity deficit for patient who is intubated if the relative humidity of the inspired gas is 25%? A. 44 mg/L B. 11 mg/L C. 33 mg/L D. 22 mg/L

The correct answer is : C Explanation : Any time you are asked "what is the humidity deficit", the answer has to be in milligrams. When the gas is humidified at 100% the number of milligrams in a liter of gas is 44. The word "deficit" means how much is missing to make the relative humidity 100%. So if the relative humidity is 25%, there is only 11 mg of water per liter of gas. That would mean if you take 44 and subtract 11, the humidity deficit is 33 mg/L of gas.

A therapist is adjusting the cuff pressure for an orally intubated patient who has a 7.5-mm ET tube in place and is receiving VC ventilation. The therapist auscultates the neck region while inserting air into the cuff until the sound of air passage cannot be heard. The therapist is most likely utilizing which of the following techniques? A. Sledinger approach B. minimal leak technique C. minimum seal technique D. Rashkind

The correct answer is : C Explanation : Auscultation of the neck while adjusting gas pressure in the endotracheal tube cuff is associated with either the minimum seal technique or the minimal leak technique. Because the question describes the insertion of air until air passage around the cuff cannot be heard, this practitioner is using a minimum seal technique. This technique simply involves adding air to the cuff until no air passage around the cuff can be heard at the top of a ventilator breath. In contrast, a minimal leak technique consists of removing air from a sealed cuff until a very slight amount of air passage can be auscultated at the top of a mechanical inspiratory breath.

The supply of flu vaccination is limited as a result of short supply. The respiratory therapist would suggest prioritizing administration of the vaccine to which of the following groups of people? A. children B. teachers C. healthcare workers D. hospital administrators

The correct answer is : C Explanation : Because healthcare workers are important in helping others who are sick, vaccinations and other healthcare should be provided to healthcare workers first.

A patient is receiving volume ventilation through a tracheostomy tube. In response to a high pressure alarm, the respiratory therapist attempts suctioning but notices the suction catheter will not pass beyond the end of the tracheostomy tube. Which of the following would be the first action the therapist should take? A. Bronchoscopy B. Resume volume ventilation C. Deflate the cuff D. Increase catheter size and suction pressure

The correct answer is : C Explanation : Because the catheter will not pass beyond the end of the tracheostomy tube, the most obvious suspicion is a herniated tracheostomy tube cuff. To quickly determine if this is the cause of the high-pressure and the inability to pass the catheter, the cuff should be deflated and the catheter should be inserted again to determine if the blockage remains. If the cather is able to pass, then a herniated cuff is present, and the tube should be replaced.

The respiratory therapist should discontinue beta sympathomimetic aerosol therapy in which of the following scenarios? A. An asthmatic patient has a peak flow rate that is 75% of predicted or better B. A COPD patient is smoking cigarettes between therapy C. The patient experiences nausea and tingling in their extremities D. A pediatric patient does not cooperate with the therapy

The correct answer is : C Explanation : Beta sympathomimetic medications such as albuterol, Alupent, Xopenex, etc., should be discontinued if the patient does not tolerate them well. Tingling in the extremities, nausea, & increased heart rate greater than 20 bpm, are all examples of poor tolerance.

Auscultation of a child's breath sounds reveals unilateral wheezing in the right side. Which of the following should the respiratory therapist recommend to be most helpful to the patient? A. Albuterol B. Atropine C. Bronchoscopy D. Salmeterol

The correct answer is : C Explanation : Bilateral wheezing is associated with bronchoconstriction. When the bronchials constrict, they are not usually capable of constricting on one side of the lung only. Therefore, unilateral wheezing indicates the problem must be something other than bronchoconstriction and is most likely associated with foreign body aspiration. To remove the object a bronchoscopy is most appropriate.

A patient in the cardiac intensive care unit has a Swan-Ganz pulmonary artery catheter in place. The following data is available: CVP 6 torr mPAP 19 torr PCWP 12 torr C.I. 1.7 L/min/m2 Which of the following is the most likely cause of these data? A. fluid overload B. pulmonary hypertension C. affects of high PEEP D. right side heart failure

The correct answer is : C Explanation : CVP is essentially normal and mPAP is elevated. This does not indicate a problem with the right heart but rather a problem downstream. mPAP is elevated and PCWP is also elevated, indicating no problem within the vasculature of the lungs. PCWP is elevated but cardiac output is low. Remember, cardiac output is determined by doubling cardiac index. That would give a cardiac output of 3.4 L which is low. The problem, therefore, is in the left heart. The answer that can explain this most closely is the affects of high PEEP.

The following laboratory data is observed in a patient who has been experiencing massive vomiting for 12 hours: BUN 18 mg/dL K+ 2.8 mEq/L Cl- 82 mEq/L Na+ 101 mEq/L RBC 6.0 g/dL Hb 12 g/dL Which of the following may also be observed? A. involuntary muscle contraction in the extremities B. loose bowel C. flattened T waves on ECG D. elevated pulmonary artery pressure

The correct answer is : C Explanation : Close examination of this data reveals that the patient has hypokalemia, as shown by a potassium level that is well below normal. Normal potassium is about 4.0 mEq/L. Low potassium will cause the ECG tracing to demonstrate what is called flattened or dull T waves.

Volume-monitoring accuracy should be within plus or minus what percent of the measured tidal volume for a patient receiving volume-cycled mechanical ventilation? A. 5 B. 3 C. 10 D. 1

The correct answer is : C Explanation : During volume-cycled mechanical ventilation, returned tidal volume varies from preset tidal volume. The allowable variance is 10%. A variation greater than 10% requires troubleshooting by the respiratory therapist.

An oxygen-dependent patient will be traveling for 24 hours. How many E- cylinder oxygen tanks will the patient require for the trip if the flow is 2 L/min? A. 2 B. 8 C. 5 D. 3

The correct answer is : C Explanation : Each tank duration = 2200 x 0.28 = 616 L. 616 L / 2 L/min = 308 minutes. 308 min / 60 = 5.1 hours. Each E cylinder will last about 5 hours. For a 24 hour trip, at least 5 tanks will be required.

For which of the following is the respiratory therapist observing if obtaining an MEP measurement? A. RSBI B. presence of fixed upper airway obstruction C. strength and ability to cough D. inspiratory muscle strength

The correct answer is : C Explanation : Examination of the maximum expiratory pressure reading is helpful in determining a patient's strength and ability to cough but also helps determine their readiness to wean from mechanical ventilation. Inspiratory muscle strength is determined through an MIP. A fixed upper airway obstruction would best be observed by doing a flow volume loop, which is a pulmonary function test.

Which of the following pulmonary function volume calculations may be used to determine functional residual capacity (FRC)? A. VC - ERV B. ERV-RV C. TLC - IC D. VT + ERV + RV

The correct answer is : C Explanation : FRC may be calculated in a variety of ways by adding and subtracting specific lung volumes. To answer this question, one must take each option and do the calculation to determine if the desired volume can be achieved. In this case, FRC may be calculated by subtracting inspiratory capacity (IC) from total lung capacity (TLC).

A physician is about to perform a bronchoscopy for the purpose of tissue extraction and analysis to rule out carcinoma. The patient is receiving heparin therapy and the physician is concerned about bleeding. Which of the following should the therapist have ready in case of bleeding inside the airway? A. Atropine sulfate B. Alcohol in a 10 cc syringe C. Epinephrine D. 5.0 mL low molecular weight heparin

The correct answer is : C Explanation : For a patient who is receiving anticoagulant therapy, a bronchoscopy procedure can become quite risky. The risk relates to the inability to stop bleeding in the bronchial tree should it occur. Preventative actions prior to the procedure include the preparation of epinephrine and access to a Fogarty catheter. Epinephrine, when applied to a bleeding site, can stop bleeding. The Fogarty catheter can be used as a tamponade the bleeding site.

A patient is ordered to be placed on a high frequency jet ventilator in response to a bronchopleural fistula. Which of the following ventilator parameters can the respiratory therapist expect to manipulate when setting up the patient on the ventilator? A. tidal volume B. plateau pressure C. I:E D. jet orifice

The correct answer is : C Explanation : For a patient with a broncopleural fistula, keeping ventilatory pressures at a minimum is most appropriate. One alternative is to avoid traditional positive airway pressure and use high-frequency jet ventilation. These ventilators have different controls than traditional volume-controlled ventilators. Specifically, tidal volume is not a contol found on the machine. Of the options listed, PEEP, I:E, and frequency are controls that may be found on a high-frequency jet ventilator.

A patient with ARDS weighs 75 kg (165 lbs) and is receiving volume-controlled ventilation with the following ventilatory parameters and arterial blood gas results: Mode Assist/control Mandatory rate 20 VT 400 mL FIO2 1.0 PEEP 22 cm H2O Peak pressure 52 cm H2O Plateau pressure 35 cm H2O pH 7.35 PaCO2 45 mm Hg PaO2 80 mm Hg HCO3- 26 mEq/L BE +1 mEq/L The respiratory therapist should recommend A. Increase mandatory rate to 22 B. Increase VT 500 mL C. Switch to pressure-control ventilation D. Increase PEEP to 25 cm H2O

The correct answer is : C Explanation : For this patient, peak pressures are greater than 50 cmH2O. When high peak pressures are the result of decreasing pulmonary compliance, such as seen with ARDS, the patient would benefit from a change in ventilatory mode to pressure-control ventilation. If, however, the increased peak pressures are a result of temporary conditions, such as bronchoconstriction, or secretions in the airways, pressure-control ventilation is not necessarily indicated. Those types of problems may be corrected quickly and peak pressures may be lowered.

An infant is delivered and has a one-minute APGAR score of 5. The infant is placed on the radiant warmer. What other equipment would be most helpful at this time? A. croup tent B. nasal cannula C. oxygen hood D. non-rebreathing mask

The correct answer is : C Explanation : From examining Apgar scores one can determine the best therapy for the patient. An Apgar score of 1 to 3 requires CPR. An Apgar score of 4 to 6 necessitates supplemental oxygen and general stimulation. An Apgar score of 7 to 10 requires only routine care of the infant. No supplemental oxygen is required in this case, one must know that an oxygen hood is the most desirable method to deliver supplemental oxygen to the patient who has an Apgar score of five.

A 30-week gestational age infant is difficult to ventilate with a resuscitator bag and mask. A recent chest radiogram shows haziness and a reticulogranular pattern. Which of the following would be most helpful to the patient? A. Chest physiotherapy B. Vigorous suctioning C. Surfactant therapy D. Fluid distraction

The correct answer is : C Explanation : Haziness and a reticulogranular pattern noted on the chest x-ray of a premature infant is consistent with pulmonary immaturity. This may be treated by administering surfactant.

How is exhaled volume determined with IPPB? A. Peak- inspiratory pressure divided by flowrate B. Calculation using Dalton's law of partial pressures C. Directly measured at the exhalation valve outlet D. Pre-set using the tidal volume control knob

The correct answer is : C Explanation : IPPB machines are pressure ventilators without volume measuring devices. Tidal volume must be measured externally at the exhalation port.

Immediately following oral endotracheal extubation the patient should first be A. receive hydration B. suctioning C. directed to cough D. trained on PEP therapy

The correct answer is : C Explanation : Immediately following extubation the patient should be first instructed to cough. If effective this is the quickest method to remove secretions immediately from the upper airway. Suctioning is appropriate if the patient is unable to cough. Hydration is time-consuming and while important will not help to immediately remove secretions in the upper airway. PEP therapy is intended to mobilize secretions from the lower airways which is not the most immediate concern following extubation.

A patient is in the intensive care unit following the repair of a hip fracture. Within 2 hours after surgery, the patient is hypotensive and is increasingly short of breath. Heart rate has risen from 90 to 110 bpm. Which of the following test results would be most helpful at this time? A. COHb B. Creatinine C. Hb and HCT D. Arterial blood gases

The correct answer is : C Explanation : Immediately following surgery a patient that is hypotensive is most likely hemorrhaging from a leaky vessel from the operation. To confirm the loss of blood an Hb and HCT may be done to examine hemoglobin level. If low, the patient should be sent back to surgery to explore for a leak but not before blood is adminisitered, if the is an option.

Which of the following parameters is NOT consistent with weaning from mechanical ventilation? A. VT to 500 mL B. RSBI of 86 C. A-aDO2 of 320 mmHg D. VD/VT of 0.50

The correct answer is : C Explanation : In order to wean from a ventilator, the patient's dead space-tidal volume ratio must be below 60% or .60. The rapid shallow breathing index must be below 106. Spontaneous tidal volume must be greater than 5 mL per kilogram. The alveoli-arterial oxygen difference (A-a gradient) must be less than 300 mmHg. In this case, the dead space-tidal volume ratio is too high and the alveoli-arterial oxygen difference is too wide. These are the two criteria that indicate the patient is not ready to wean from the ventilator.

Which of the following is NOT normally part of a smoking cessation program? A. source of emotional support B. nicotine replacement therapy C. information on the cost of cigarettes and tobacco products D. education on the hazards of smoking

The correct answer is : C Explanation : In preparation to help the patient stop smoking, the healthcare giver may assess the emotional support required, evaluate the appropriateness of nicotine replacement therapy, and educate the patient on the hazards of smoking. Discussing the cost of cigarettes and other tobacco products will not be helpful.

A febrile patient has the following blood gas results: (results are not corrected for temperature) pH 7.35 PaCO2 44 mmHg PaO2 28 mmHg HCO3- 24 mEq/L BE 0 mEq/L O2 Sat 90% Which of the following requires attention? A. Sanz electrode B. spectrophotometer C. Clark electrode D. Severinghaus electrode

The correct answer is : C Explanation : In this arterial blood gas a PaO2 of 28 mmHg is reported. That is too low for the patient to still be alive and therefore this data is probably faulty. The question is essentially asking which electrode would you suspect is not working properly. The Clark electrode is another name for the PO2 electrode. The Sanz electrode is another name for the pH electrode and the Severinghaus electrode is called the CO2 electrode.

A pediatric patient with cystic fibrosis is receiving postural drainage and percussion for right lower lobe pneumonia with his head down at 30-degrees, three times a day for 20 minutes. The therapy is very effective, but after 15 minutes, the patient complains of difficulty breathing. What should the respiratory therapist recommend? A. Decrease the degree of trendelenburg. B. Change to PEP therapy. C. Reduce the duration of the therapy. D. Increase the degree of trendelenburg.

The correct answer is : C Explanation : In this case, the best option is to decrease the therapy time. Changing the position of the patient would dysfunctionalize the therapy and is not appropriate.

A female patient who is morbidly obese has an 8.0 mm tracheostomy tube. She has been placed on an ulcer prevention mattress in the Intensive Care Unit. Following transport to the new bed, the nurse calls a respiratory therapist to assess her because she is complaining of air hunger. Diffuse dry crackles are auscultated with crepitus noted upon palpation. These findings are most closely associated with A. pneumothorax B. atelectasis C. subcutaneous emphysema D. secretions in the airway

The correct answer is : C Explanation : In this case, the respiratory therapist should assume that the tracheostomy tube has become dislodged due to the patient transfer. Positive pressure ventilation through the tube while it was dislodged has resulted in subcutaneous emphysema, or air under the skin outside of the lung. Auscultation usually reveals dry crackles and palpation reveals a crepitus sensation.

A respiratory therapist is called to the room of a mechanically ventilated patient with a size 8.0 mm endotracheal tube. The low volume and low PEEP alarms are sounding. The respiratory therapist is able to see only 1.5 inches of the end of the endotracheal tube, which is located at the lips at the 22 cm marking. What should the therapist recommend? A. Withdraw the tube 2 cm. B. Add air to the cuff. C. Check for a leak. D. Silence the alarm and troubleshoot.

The correct answer is : C Explanation : In this example, it appears that the endotracheal tube has been cut and is therefore only showing 1.5 inches. However more importantly is the centimeter marking at the lips, which is 22 cm. The only reasonable option is to check for a leak.

A patient with an ideal body weight of 80-kg (176-lb) is being evaluated for cessation of mechanical ventilation. The following weaning parameters and clinical data are available: Vital Capacity 2.2 L RSBI 134Spont VT 200 mLResp Rate 33/min Which of the parameters is the most reliable indicator that the patient is ready for ventilator liberation?

VC

A respiratory therapist discontinues a small volume nebulizer started with mucomyst after 5 minutes due to a patient's poor tolerance of the therapy. SpO2 dropped from 95% to 86%, heart rate increased from 88 to 128 bpm. The patient returns to a normal condition shortly after the treatment is stopped. The next action would be to A. administer after adding a unit dose of Proventil. B. administer with 100% oxygen. C. document in the medical record and notify the physician. D. administer with a lower dose of mucomyst.

The correct answer is : C Explanation : In this example, the patient did not tolerate the therapy. Therefore, the respiratory therapist should discontinue therapy & notify the ordering physician. A note about the patient's tolerance should also be made in the medical record.

A morbidly obese 47-year-old male is admitted to the emergency department complaining of shortness of breath. Brief clinical and laboratory exams reveal the following: HR 98/min BP 135/90 mm Hg Temp 38.3 deg C (101 deg F) 2 PM 3 PM FIO2 0.21 0.50 pH 7.29 7.23 PaCO2 68 torr 73 torr PaO2 48 torr 55 torr HCO3- 34 mEq/L 35 mEq/L BE +5 mEq/L +4 mEq/L The patient remains stable, awake, and oriented to time, person, and place. He has a productive cough. The respiratory therapist should do which of the following NEXT? A. Continue current therapy B. Begin flutter valve therapy C. Initiate non-invasive ventilation D. Switch to nasal cannula at 3 L/min

The correct answer is : C Explanation : In this situation, a morbidly obese patient has blood gases that demonstrate increasing ventilatory distress. In truth, the blood gas demonstrates complete ventilatory failure. However, when you look at the options there is no answer to assume ventilation completely. Therefore, the best answer here is to initiate noninvasive ventilation.

A patient receiving mechanical ventilation has the following arterial blood gases: pH 7.29 PaCO2 55 torr PaO2 60 torr HCO3- 26 mEq/L BE 1.2 mEq/L After increasing minute ventilation, what new ABG would most likely be seen? A. ph 7.30, PaCO2: 45 torr, PaO2 60 torr B. pH 7.34, PaCO2: 45 torr, PaO2 60 torr C. pH 7.32, PaCO2: 48 torr, PaO2 68 torr D. pH 7.25, PaCO2: 59 torr, PaO2 65 torr

The correct answer is : C Explanation : Increasing minute ventilation should reduce PaCO2, and increase pH. Because CO2 is an acid, the higher the level of the CO2 in the blood, the lower the pH. You will notice also that the PaO2 has increased. It is important to solve the problem of insufficient ventilation prior to insufficient oxygenation because as ventilation improves, oxygenation should also improve to some degree

An oral pharyngeal airway is measured from the patient's jaw angle to the tip of the chin. After placement, the semi-conscious patient repeatedly pushes the airway out with his tongue. The therapist should A. secure the airway with tape. B. replace with a nasal endotracheal tube C. reinsert the oral airway. D. insert a larger airway.

The correct answer is : C Explanation : It is normal for a patient to reject an oral airway with their tongue. Reinserting the airway is most appropriate.

End inspiratory crackles are heard upon auscultation of a post operative hip patient with no known pulmonary history. The patient is febrile. These signs are indicative of A. dehydration. B. subcutaneous emphysema. C. atelectasis. D. hypovolemia.

The correct answer is : C Explanation : Late inspiratory crackles, or end-inspiratory crackles are commonly auscultated with a patient experiencing atelectasis. Crackles are also known as rales. Atelectasis is the presence of fully or partially collapsed alveoli.

Which of the following test results would NOT be helpful in determining whether a patient should receive Bi-level therapy with supplemental oxygen? A. VE 9.0 L/min B. MIP -22 cm H2O C. MVV 50 L in 12 seconds D. pH 7.29

The correct answer is : C Explanation : MVV, or maximum voluntary ventilation is a test completed during a complete pulmonary function evaluation. It is not a part of weaning parameters assessed on mechanically ventilated patients. It is also a test that should be completed on patients who are otherwise very healthy and in their best condition, not a critically ill patient receiving mechanical ventilation. The other tests are appropriate to assess when deciding to terminate mechanical ventilation.

A 65-kg (143-lb) 49-year-old patient in the cardiac intensive care unit is receiving mechanical ventilation on the following settings with the following data: Mode Assist/control Tidal volume 500 mL Mandatory rate 16 FIO2 0.8 PEEP 15 cm H2O Inspiratory flow 50 L/min CVP 8 mm Hg PAP 36/8 mm Hg C.O. 3.9 L The therapist should report which of the following values mPAP? A. 8 mm Hg B. 36 mm Hg C. 17 mm Hg D. 22 mm Hg

The correct answer is : C Explanation : Mean PAP, as with MAP, is determined by adding the systolic pressure to twice the diastolic pressure and dividing the sum by three. In this case, 36 + 8 + 8 = 52. 52/3 = 17 mm Hg.

Which of the following classes of medication is most helpful in treating an adult who has moderate stridor following endotracheal extubation? A. aminoglycocide B. corticosteroid C. decongestant D. vasoactive

The correct answer is : C Explanation : Mild stridor can be treated with a cool mist or racemic epinephrine. Moderate stridor should be treated with racemic epinephrine. Severe or marked stridor should be managed as an emergency and treated through intubation or an emergency tracheotomy.

Immediately following the extubation of an adult who was orally intubated for the past 2 weeks, the respiratory therapist hears moderate stridor. The therapist should recommend A. Immediately intubate the patient B. Delivery aerosol therapy with cool mist C. Administer racemic epinephrine D. Send to surgery

The correct answer is : C Explanation : Moderate stridor, caused by inflammation in the upper airway, may be treated with racemic epinephrine. Mild stridor may be treated with the racemic epinephrine and/or cool aerosol. Marked or severe stridor is treated by reestablishment of an artificial airway.

Which of the following ventilator controls will be primarily used when employing an alveolar recruitment strategy? A. high pressure limit B. inspiratory flow C. PEEP D. I:E ratio

The correct answer is : C Explanation : Most alveolar recruitment strategies include use of elevated PEEP levels for small periods of time. PEEP provides a baseline pressure that may be used to prevent alveolar collapse and recruit already collapsed alveoli.

While suctioning a patient through the inner cannula of a 7.0-mm fenestrated tracheostomy tube, the therapist observes the collected secretions show layers. With which of the following is this most consistent? A. COPD B. use of the antibiotic tetracycline C. bronchiectasis D. mycoplasma pneumonia

The correct answer is : C Explanation : Secretions that demonstrate differing layers of sputum is associated with bronchiectasis.

A 32-gestational week newborn is receiving mechanical ventilation via a 3.0 mm endotracheal tube. Which of the following are associated with ongoing assessment of renal function? A. PCWP B. mPAP C. diaper weight and blood draw volumes are recorded D. humidification is maintained at a non-invasive temperature

The correct answer is : C Explanation : NPO (nothing by mouth) indicates that a newborn is not feeding in the usual manner. Intravenous fluids are vital and must be recorded for an intubated patient. Fluid output is measured by diaper weight and carefully recorded blood draws. Humidification of gas through the endotracheal tube must be maintained at an invasive temperature rather than a noninvasive because the nasal passages are bypassed.

During treatment of a patient who is demonstrating bilateral expiratory wheezing, the patient complains of nausea and tremors with beta-agonist medication. The respiratory therapist should A. direct the patient to breathe at a slower rate B. reduce future doses to half-strength C. immediately discontinue the treatment, document the finding, notify the physician D. monitor carefully for the remainder of the treatment, notify the nurse and physician

The correct answer is : C Explanation : Nausea and tremors associated with the delivery of the beta agonist medication (like albuterol) is not considered a normal reaction, but is classified as an adverse reaction to the medication. As with any medication, when an adverse reaction is encountered, the very first step is to stop the therapy. Additionally the finding should be documented and the physician should be notified.

A patient with a 10-year history of chronic lung disease and cor pulmonale is the intensive care unit receiving volume-controlled ventilation in the A/C mode. The following data shows: mPAP 21 torr PWP 8 torr right atrial preload 10 torr C.I. 2.2 L/min/m2 Which of the following would be most helpful in reducing pulmonary vasculature pressures? A. Inhaled Lidocaine B. Inhaled NO2 C. NO therapy D. Heliox

The correct answer is : C Explanation : Nitric oxide therapy is useful in treating pulmonary hypertension. In this case, the patient has an increased mean pulmonary artery pressure with a normal pulmonary capillary wedge pressure. These two hemodynamic values, when observed together, suggest a build-up of pulmonary artery pressure due to the blood's inability to transition the vasculature of the lungs.

Which of the following is NOT part of the suction procedure of an orally intubated patient receiving volume-controlled ventilation in the assist/control mode? A. hyper-oxygenate prior to suctioning B. hyperventilate the patient through the ventilator C. switch to SIMV mode D. utilize sterile technique

The correct answer is : C Explanation : Normal suctioning procedure includes hyper oxygenation and hyperventilation of the patient prior to suctioning. A sterile technique is also used. Switching to an SIMV mode is not helpful.

A patient has a blood pressure of 65/45 mmHg while 10 mcg/kg/min of Dopamine HCL (Intropin) is being administered. The respiratory therapist is unable to get an SpO2 reading by pulse oximeter. The mostly likely reason for this is A. ace inhibitors B. the SpO2 monitor is in need of calibration C. poor peripheral perfusion D. increased systemic vascular resistance

The correct answer is : C Explanation : Obtaining an adequate SaO2 reading requires sufficient profusion in the extremities in order for the pulse ox to read properly. This patient has low blood pressure, which would result in poor peripheral perfusion and an inaccurate SaO2 reading.

Which of the following is most likely associated with hypervolemia? A. PCWP of 7 mmHg B. PAP of 15 mmHg C. CVP of 10 mmHg D. cardiac index of 2.0

The correct answer is : C Explanation : Of all the hemodynamic values, loss of fluid (hypovolemia) or (hypokalemia), the first and most significant hemodynamic manifestation will be found in the elevation of CVP. Normal CVP is about 5 mmHg. So, a CVP of 10 mmHg is high. Hypovolemia, or under hydration, is manifested by a low CVP. CVP is known by many other names. Be sure to know all of the names that are associated with CVP.

A patient has thick secretions after being mechanically ventilated for 3 days with a heated moisture exchanger in the circuit and no other source of hydration. The patient is intubated with a 7.0 mm ET tube. When performing suctioning with a 10 Fr closed system suction catheter, the patient experiences PVCs. The respiratory therapist should recommend which of the following? A. Administration of Lasix (furosemide) B. Switch to a 12 Fr suction catheter C. Administration of aerosolized acetylcisteine (mucomyst) D. Oral consumption of water

The correct answer is : C Explanation : Of the options given, the use of acetylcysteine will be most effective at thinning secretions which will help in their mobilization and removal. Switching to a 12 French suction catheter would also be effective but is not appropriate because it is too large for the 7.0 endotracheal tube.

Use of overnight oximetry at home is intended to determine the need for which of the following? A. continuous cardiac rhythm monitoring B. echocardiography C. polysomnography D. electroencephalography

The correct answer is : C Explanation : Of the options listed, the use of overnight oximetry is used to detect periodic oxygen desaturation, which may be connected to obstructive or central sleep apnea. Although the exam is not conclusive, it is enough to indicate further testing with regard to sleep. This kind of testing is called polysomnography.

Which of the following changes would result in an unchanged mean airway pressure for a mechanically ventilated patient? A. removal of an inspiratory pause B. reduction of tidal volume by 100 ml C. removal of mechanical deadspace D. increase in mandatory rate

The correct answer is : C Explanation : Of the options offered only removal or addition of deadspace will change CO2 without resulting in a change in mean airway pressure. Reduction of tidal volume will reduce mean airway pressure.

While observing a chest radiograph the respiratory therapist notes the apex of the right lung is higher than the apex of the left lung. The mostly like explanation for this difference is A. the radiograph was overexposed B. the radiograph was shot from an oblique angle C. the patient was rotated when the radiograph was taken D. the radiograph was underexposed

The correct answer is : C Explanation : On a chest x-ray film, when one apex of the lung is higher than the other, the patient was likely rotated inappropriately when the x-ray was taken. Chest x-rays are normally shot straight-on.

An 18-year-old patient is brought to the emergency department (ED) after being found unconscious on a park bench. The patient remains unresponsive. An open bottle of opioid medication is found on his person. The therapist notices erratic, shallow breathing. Which of the following would be helpful in directly assessing the adequacy of the patient's ventilatory drive?

VT and RR

The physician has asked the respiratory therapist to help determine oxygen consumption by the tissues. The therapist should suggest evaluation of which of the following? A. CvO2 B. CaO2 C. C(a-v)O2 D. PaO2

The correct answer is : C Explanation : Oxygen available at the tissue level is best determined by examining the arterial oxygen content or CaO2. However, to determine how much oxygen the tissue is consuming we must look at both the CaO2 and the CVO2. This allows us to determine how much oxygen existed before the tissues and how much oxygen exists after the tissues. When we subtract one from the other we get the C(a-v)O2. This value is most closely related with oxygen consumption at the tissue level.

A respiratory therapist is administering PEP therapy to a 7-year-old child diagnosed with cystic fibrosis. Which of the following, would be an indication to discontinue PEP therapy? A. epidermal infection of the outer ear B. pneumonia C. sinusitis D. small penetration in the ear drum

The correct answer is : C Explanation : PEP therapy is contraindicated by sinusitis, epistaxis, and a middle ear infection.

A patient is in the emergency room (ER) with paradoxical chest movement on the right side. The trachea is deviated from mid-line. Which of the following should the respiratory therapist suspect? A. pulmonary embolism B. pulmonary edema C. fractured ribs and a pneumothorax D. pleural effusion

The correct answer is : C Explanation : Paradoxical chest movement on the right side is an indication of the presence of broken ribs. Deviation of the trachea indicates the presence of a pneumothorax.

A patient presents in the emergency department (ED) with shallow, irregular respirations. Which of the following diagnostic procedures would best help rule out a pleural effusion? A. ventilation/perfusion (V/Q scan) B. pulmonary angiogram C. series of oblique chest radiographs D. PA chest radiograph

The correct answer is : C Explanation : Pleural effusions may be diagnosed through obtaining a lateral decubitus x-ray or by obtaining a series of x-rays from oblique angles and different positions.

Which of the following conditions would be a contraindication for IPPB? A. pulmonary edema B. a patient requiring bronchodilation AND lung expansion C. untreated pneumothorax D. fractured ribs

The correct answer is : C Explanation : Pneumothorax is a contraindication for IPPB due to positive pressure which will exacerbate barotrauma of the lung.

Which of the following could result from the application of positive end-expiratory pressure? A. reduced intra-thoracic pressure B. increased A-aDO2 C. reduced cardiac index D. reduced alveolar ventilation

The correct answer is : C Explanation : Positive end expiratory pressure, like all positive pressure ventilation techniques, leads to decreased venous return, decreased cardiac output, and decreased cardiac index.

A patient is being mechanically ventilated by volume controlled ventilation. A chest tube drainage system is in place and 600 mL of fluid has been collected. After several hours of drainage, the respiratory therapist notices the bubbling in the water seal chamber has become profuse. The following data is available: pH 7.32 PaCO2 47 torr PaO2 80 torr HCO3- 24 mEq/L BE -1 mEq/L Mode Assist/control VT 500 mL Exhaled VT 578 mL Mandatory rate 16/min Total rate 16/min FIO2 0.50 PEEP 5 cm H2O The respiratory therapist's first action should be to A. monitor the patient as usual B. switch to pressure controlled ventilation C. clamp the chest tube at the patient D. decrease mandatory rate to 14

The correct answer is : C Explanation : Profuse bubbling in the waterseal chamber indicates a leak in the chest tube drainage system. The leak may be found in the tubing between the waterseal chamber and the patient, or it may be found inside the patient. If the leak is occurring inside the patient, the cause is likely a perforation in the lung tissue. To determine the source of the leak, a clamp should be placed on the tubing as it exits the patient. If bubbling stops, the leak is inside the patient and suggests perforated lung tissue. This is corrected by surgery.

A respiratory therapist is asked to assist in the development of a teaching plan for a 7-year-old asthmatic patient at home. The plan should avoid instructions regarding A. improving the patient and family's understanding of infection control B. the proper technique for the use of MDIs C. the water-float technique to determine MDI canister content D. daily monitoring of peak-expiratory flow rates

The correct answer is : C Explanation : Proper MDI technique, peak-flow monitoring, and improving the understanding of infection control are all appropriate aspects of a teaching plan for a pediatric patient. Determining MDI canister content by floating the canister in water is never appropriate. The proper way to assess content is by actuating the MDI and observing the output.

A respiratory therapy supervisor is responsible for quality improvement in the respiratory department. Which of the following represents an appropriate quality improvement focus? A. daily patient charges B. total number of procedures done per hour C. incidence of ventilator acquired pneumonia D. number of blood gases done per shift

The correct answer is : C Explanation : Quality improvement is about improving patient care. Tracking daily patient charges, the total number of procedures done per hour, or the number of blood gases per shift does not improve quality of care to patients. Tracking the incidence of ventilator acquired pneumonia, however, may be helpful in directing the department's efforts to manage infection control more effectively. This has a direct result on patient care.

Which of the following statements is correct regarding suctioning of an adult patient with tuberculosis? A. Alert patients with spontaneous coughs should not be suctioned B. Insertion of a nasal pharyngeal airway should be facilitated with a petroleum based lubricant C. All suctioning should be done as needed only D. Suction pressure should not exceed -110 mmHg

The correct answer is : C Explanation : Regardless of the disease, suctioning should be reserved for PRN, or as needed situations. Suction pressure should not exceed -120 mm Hg. A nasopharyngeal airway should not be inserted with lubricants that are petroleum or oil-based. While it may seem unusual to suction alert patients with a spontaneous cough, it is permissible and indicated for a patient with rhonchi.

Which of the following medications would be helpful at reversing the affect of a benzodiazepine medication? A. Narcan B. Versed C. Romazicon D. Norcuron

The correct answer is : C Explanation : Romazicon is the only medication on the list that can reverse the effects of benzodiazepines.

A patient has idiopathic pneumonia with consolidation in the right lower lobe. The physician suspects a bacterial infection. Which of the following will provide conclusive data to rule out the physician's suspicions?

WBC

Three hours after endotracheal extubation, a patient demonstrates moderate stridor. Which of the following should the therapist administer? A. topical Lidocaine B. Albuterol sulfate C. racemic epinephrine D. viscous Xylocaine

The correct answer is : C Explanation : Stridor is produced by inflammation in the upper airway and may be seen occasionally post extubation. The solution is dependent upon the degree of the stridor. Mild stridor may be treated by cool mist or racemic epinephrine. Moderate stridor requires racemic epinephrine at a minimum. Marked or severe stridor requires immediate intubation or other airway protection procedures.

A 6 ft 3 in (191 cm) male who weighs 220 lbs (100 kg) is receiving positive pressure ventilation. The respiratory therapist palpates subcutaneous emphysema about the neck and upper chest. Which of the following is the mostly likely cause of this condition? A. Chronic air-trapping B. This is a normal finding in patients with COPD C. Tracheostomy tube is too small D. Status asthmaticus

The correct answer is : C Explanation : Subcutaneous emphysema is normally caused from a mal-positioned tracheostomy tube. However, it may also be caused from a tracheostomy tube that is too short. When this happens the end of the tracheostomy tube is nestled inside or around the tissues. Air that passes through the tube may have a tendency to enter subcutaneous areas. Of the options listed, the most likely cause is a tracheostomy tube that is too small.

Which of the following may occur as a result of stimulation of the vagal reflex during a suction procedure? A. first degree heart block B. increase in heart rate C. decrease in blood pressure D. PVCs

The correct answer is : C Explanation : Suctioning can cause a decrease in heart rate and a decrease in blood pressure. This natural, physiological response is associated with stimulation of the vagal nerve during suctioning. This is also called the vagal reflex.

While reviewing the progress notes on a patient's medical record, the respiratory therapist notes a new list of care recommendations related to ventilator weaning documented by the physician. The therapist should A. implement the changes from the progress notes B. discuss the recommendations with the nurse C. check for new respiratory care orders D. initiate any pertinent weaning protocols

The correct answer is : C Explanation : Technically, documentation in the progress notes do not constitute an official change in care. They represent thoughts of healthcare workers regarding the potential problems, diagnoses, and treatment plans of the patient. To be made official, orders must be written in the chart. Therefore, physician orders should be monitored for changes when documentation in the progress notes occur.

A patient on the general floor becomes a primary mouth-breather while asleep, resulting in a drop in SpO2 from 95% to 87% while on 3 L/min nasal cannula. The best remedy while asleep is to A. apply a chin strap. B. apply CPAP with oxygen bleed-in. C. change to a 30% air-entrainment mask. D. place the cannula in the patient's mouth.

The correct answer is : C Explanation : The FIO2 of a nasal cannula is approximated to be 3 to 4% per liter. Therefore, a 30% Venturi mask most closely approximates the FIO2 of a nasal cannula running at 3 L/min.

Which of the following chest PA radiograph results would suggest the patient was rotated improperly? A. there is a mediastinal shift B. heart shadow exceeds 1/2 the chest cavity laterally C. the apecies are different sizes D. tops of the lungs are not level

The correct answer is : C Explanation : The apexes (apecies) of the lungs are the very top triangular, cone-shaped aspects of each lung. When one apex is larger than the other on a radiograph, this is likely due to inappropriate patient rotation. This happens because one lung ends up closer to the film than the other and therefore appears smaller. If this is observed, the radiograph should be redone.

A patient is being sent home on a mechanical ventilator requiring oxygen bleed-in with the device. Which of the following should be recommended? A. E-Cylinders B. pulse-dose oxygen delivery C. oxygen concentrator D. liquid oxygen

The correct answer is : C Explanation : The best device for home oxygen therapy is a molecular sieve device, commonly known as an oxygen concentrator. This device is powered with electricity, which makes it appropriate for home use.

A patient has been on mechanical ventilation for 30 minutes. The physician would like to confirm the adequacy of ventilation and oxygenation. The respiratory therapist should suggest which of the following? A. Pulse oximetery B. End-tidal CO2 monitoring C. Arterial blood gas analysis D. Transcutaneous CO2 and O2 monitoring

The correct answer is : C Explanation : The best option to assess ventilation and oxygenation is arterial blood gas analysis. Pulse oximetry would only assess oxygenation and is not conclusive. End-tidal CO2 monitoring would only assess ventilation and is also not conclusive. Transcutaneous CO2 and O2 monitoring is usually associated with infants and is also not conclusive.

A patient is in the intensive care unit receiving volume-controlled ventilation and is hemodynamically unstable. A new balloon-tipped, flow-directed, pulmonary artery catheter has been placed. Immediately following insertion, a chest radiograph shows the tip of the catheter is positioned over the right mid lung near the hilum. Which of the following can the respiratory therapist conclude concerning the catheter? A. it has punctured the pericardial wall B. it should be withdrawn several centimeters C. the balloon should be inflated and the catheter advanced D. it should be rotated away from the hilum

The correct answer is : C Explanation : The chest radiograph reveals improper placement of the pulmonary artery catheter. To advance the catheter, the balloon should be inflated and the catheter should be sailed into place.

A ventilator is sounding two alarms - low temperature and low volume. This could be caused by which of the following? A. depleted humidification reservoir B. disconnect in the patient-ventilator interface C. disconnected temperature probe in the ventilator circuit D. ruptured ET tube cuff

The correct answer is : C Explanation : The combination of low temperature and low volume alarms is commonly caused by a problem related to the temperature probe. As the temperature probe is inserted inside the inspiratory limb of the ventilator circuit, it can sometimes become dislodged creating a leak in the circuit. If the probe becomes dislodged, it is no longer exposed to the heated gases and can therefore, trigger a low-temperature alarm. A low volume alarm will result from the leak through the port where the probe is normally nestled.

A patient with lobular pneumonia is receiving supplemental oxygen at 40% by Venturi mask. The following clinical and laboratory data is available: VT (spont) 600 mL RR 18/min pH 7.48 PaCO2 33 mmHg PaO2 49 mmHg HCO3- 23 mEq/L BE -1 mEq/L The respiratory therapist should recommend A. sedate the patient B. switch to nasal cannula at 2 L/min C. switch to a non-rebreathing mask D. increase oxygen to 50%

The correct answer is : C Explanation : The degree of hypoxemia indicates a need for maximum oxygen percentage. In this case a non-rebreather mask is most appropriate. All other options would either be insufficient, increase hypoxemia, or are unrelated.

The respiratory therapist should look to which of the following clinical data to determine the effectiveness of incentive spirometry? A. Maximum voluntary ventilation done periodically B. Inspiratory capacity predicted volume C. Breath sounds before and after every treatment D. Arterial blood gas analysis pre and post treatment

The correct answer is : C Explanation : The effectiveness of incentive spirometry can best be determined by auscultating breath sounds before and after the treatment and noting changes in air movement. While achieving inspiratory capacity is the goal, the real goal is to increase lung volume, improve alveolar recruitment, and prevent consolidation of sputum in the lungs.

In response to a ventilator alarm, the respiratory therapist enters the patient's room and notes the patient is awake, extremely agitated, and is moving violently about in the bed. Oxygen saturation is 86%. The therapist should do which of the following? A. restrain the patient and determine the nature of the alarm B. silence the alarm and verbally calm the patient C. manually ventilate the patient and recommend sedation D. temporarily increase FIO2

The correct answer is : C Explanation : The first step in responding to a ventilator alarm is to manually ventilate the patient, ensuring adequate ventilation and oxygenation. This patient, appears to be in need of sedation.

The following flow volume loop is obtained from a 62-year-old factory worker. Which of the following could represent the patient's diagnosis? A. chronic bronchitis B. emphysema C. pulmonary fibrosis D. cystic fibrosis

The correct answer is : C Explanation : The flow volume loop is tall and skinny, indicating normal flows but abnormal volumes. This is consistent with a restrictive lung defect. The best answer is the disease that is restrictive in nature. In this case, only pulmonary fibrosis qualifies.

A patient is engaged in a smoking cessation program. During a routine visit to the counseling clinic, the patient describes shortness of breath (dyspnea) after every meal. Which of the following instruction should the therapist provide? A. consume only low-fat meats and pure grains B. increase complex carbohydrate consumption between meals C. eat small meals more frequently, avoid too many carbohydrates D. drink 10-16 8oz glasses of water daily, even when not thirsty

The correct answer is : C Explanation : The general nutritional advice for a patient with chronic obstructive pulmonary disease is to eat smaller meals more frequently and to avoid excessive consumption of carbohydrates, which produce carbon dioxide and place a greater ventilatory load on the patient.

A respiratory therapist is called to the recovery room to find a 16-year-old male patient who was recently extubated experiencing anxiety and difficulty breathing. Breath sounds are clear over the chest, with a high-pitched noise heard over the upper airway. The respiratory therapist should recommend A. cool mist aerosol B. Decadron injection C. aerosolized racemic epinephrine D. heated aerosol

The correct answer is : C Explanation : The high-pitched noise heard over the upper airway is also known as stridor and is best treated by delivering aerosolized racemic epinephrine. Cool mist aerosol is also an option to be considered, however, in this case because the patient is experiencing difficulty breathing and anxiety, a more aggressive therapy should be selected.

A patient dependent on mechanical ventilatory support requires transport from the radiology department to the intensive care unit. Which of the following is required for the transport? A. multiple wave-length spectrophotometer B. a volume-cycled ventilator C. bag-valve resuscitator D. cardiac monitor with defibrillation capability

The correct answer is : C Explanation : The minimum equipment needed to transport a patient who is receiving mechanical ventilatory support is a bag-valve resuscitator.

Which of the following precautions should be taken when performing nasal tracheal suctioning on a patient who has a tendency for mucosal bleeding? A. Keep suctioning to no longer than 30 second B. Apply intermittent suctioning during withdraw of the catheter C. Be gentle D. Stop all blood-thinning medication prior to suctioning

The correct answer is : C Explanation : The most common hazard of suctioning is mucosal bleeding. The best way to prevent mucosal damage and injury is to use water-soluble lubricant and a gentle technique when suctioning.

Which of the following would be most helpful in determining if a patient regularly smokes tobacco products? A. DLCO B. end-tidal carbon dioxide C. COHb D. pulse oximetery

The correct answer is : C Explanation : The most reliable data that indicates the degree to which the patient has recently been smoking comes from a COHb analysis.

A newborn is experiencing nasal flaring, declining oxygen saturation values, tachypnea, and diminished breath sounds in the left lung following a meconium delivery. One possible cause of these findings is A. BPD. B. TTN. C. pneumothorax. D. IRDS.

The correct answer is : C Explanation : The patient is showing obvious signs of respiratory insufficiency with nasal flaring, low oxygenation, and tachypnea. Because we also have localized diminished breathsounds upon auscultation, along with a recent history of meconium, we must suspect a pneumothorax. Meconium creates a ball-valve effect when in the lung, creating air-trapping during ventilation that may result in a pneumothorax.

During the interview phase of collecting a patient's pulmonary history, the patient indicates he only has difficulty breathing when lying flat in bed. The respiratory therapist would use which term to document this finding? A. transient dyspnea B. supinal dyspnea C. orthopnea D. transient tachypnea

The correct answer is : C Explanation : The patient who has difficulty breathing while lying flat, or requires additional pillows to sleep, has a condition called orthopnea. This condition is commonly related to congestive heart failure.

While performing endotracheal suctioning with a 10 French catheter on a patient who is orally intubated with an 8.0 ET tube, the respiratory therapist notices it is sometimes difficult to withdraw the suction catheter. To correct this issue without sacrificing efficacy, the therapist should

applying water-soluble lubricant to the catheter

A patient reports to the emergency department with shortness of breath, bilateral wheezing, and a low-grade fever. This condition has persisted for several days. The patient has had multiple breathing treatments with only temporary relief. Which of the following is appropriate to recommend? A. IV Sublimaze B. oral antibiotics C. continuous bronchodilator therapy D. IV Doxapram (Dopram)

The correct answer is : C Explanation : The patient who has persistent bilateral wheezing but who is responding somewhat to bronchodilator therapy should continue that therapy. Of the options offered continuous bronchodilator therapy is most appropriate.

A patient in the cardiac intensive care unit has pitting peripheral edema rated at +2. What other laboratory data would be helpful to assess to further understand the nature of the problem? A. serum electrolytes B. PD50 C. creatinine and BUN D. arterial blood gases

The correct answer is : C Explanation : The presence of pitting edema is a fluid shifting problem that is related to the kidney and sometimes the heart. Any kind of fluid shifting problem can be evaluated further by looking at the creatinine and BUN. Of these tests, creatinine is considered more accurate. But both will supply the same kind of information.

Which of the following is an important strategy when caring for a patient with adult respiratory distress syndrome? A. promote AutoPEEP B. ventilate in Control mode with paralytics C. keep FIO2 below 0.60 when possible D. keep FIO2 higher than 0.60

The correct answer is : C Explanation : The primary strategy for a patients with adult respiratory distress syndrome who are on a ventilator is to keep the FIO2 as low as possible. Generally below 0.6 is most appropriate. This, however is a tough battle because hypoxemia is the cardinal attribute of adult respiratory distress syndrome. In order to compensate and keep hypoxemia at a minimum, PEEP is also used.

A 15-year-old patient with muscular dystrophy and pneumonia continue having difficulty expectorating secretions. Which of the following would provide most assistance with bronchial hygiene? A. huff coughing B. vibratory PEP therapy C. IPPV D. postural drainage and percussion

The correct answer is : C Explanation : The problem with muscular dystrophy is that the patient may have significant difficulty inspiring enough tidal volume to then generate a forceful exhalation and an effective cough. This can be assisted with IPPV, which will help increase inspired volume and increase the effectiveness of a cough.

A patient in the emergency room expectorates thick, yellow sputum. A chest radiograph reveals complete opacification of the left lung. Which of the following would be appropriate for further evaluation and treatment of the patient? A. thoracentesis of the left pleural space B. administration of furosemide (Lasix) C. culture and sensitivity D. chest tube placement on the left side

The correct answer is : C Explanation : The production of thick yellow sputum is associated with an infection. A chest x-ray that shows complete opacification of one or both lungs indicates the presence of mucus to the point it can be seen as a consolidation on the x-ray. This is the definition of pneumonia. Because the pneumonia is likely caused from an infection, the most appropriate action for the patient is a culture and sensitivity to determine the microorganism (bacteria) that is causing the infection, and the antibiotic that is most suitable to kill that particular microorganism. Placement of chest tubes or thoracentesis is for draining air or fluid from the pleural space. Lasix is used to diurese the patient. Neither of these actions are indicated.

Which of the following transdermal nicotine preparations might the respiratory therapist recommend to help a patient stop smoking? A. spray B. gum C. patch D. MDI

The correct answer is : C Explanation : The question is asking for a transdermal nicotine preparation. "Transdermal" means "through-the-skin". A patch is the appropriate preparation were looking for.

A respiratory therapist examines the patient's record to see the results of an acid-fast sputum stain. For which of the following conditions is the therapist investigating? A. Penicillin resistance B. Penicillin allergies C. pulmonary tuberculosis D. antibiotic sensitivity

The correct answer is : C Explanation : The results of an acid-fast sputum stain will rule in or rule out pulmonary tuberculosis.

A female patient weighing 150 lbs is receiving mechanical ventilation in the SIMV, volume-cycled mode. Set tidal volume is 500 mL. Returned volumes for mechanical breaths are 490 mL. The respiratory therapist should A. add air to the cuff. B. increase the set VT to 510 mL C. continue current therapy. D. check the circuit for leaks.

The correct answer is : C Explanation : The returned tidal volume will not always match the set tidal volume. In this example the loss of 10 mL per breath is not significant.

Which of the following is the correct postural position for drainage of the right lateral segment? A. flat on stomach, HOB down 18 inches B. supine and slightly rotated to the left, HOB down 12 inches C. left side, HOB down 18 inches D. right side, HOB down 12 inches

The correct answer is : C Explanation : The right lateral segment is drained with the head of bed down 18 inches with the patient lying on the left side.

While monitoring a newborn, the respiratory therapist notices the TcO2 tracing on the monitor suddenly rises sharply. Which of the following is the mostly likely explanation for this rise? A. the patient's arterial oxygenation has improved B. the transcutaneous electrode requires calibration C. transcutaneous electrode has become detached from the skin D. the transcutaneous electrode is below required temperature

The correct answer is : C Explanation : The transcutaneous oxygen probe is a device that is placed on an infant's skin that can analyze the PaO2 through the skin or transcutaneously. When the probe becomes dislodged, the electrode measures the oxygen tension in the room, which is always higher than the oxygen tension in the artery. Thus, a sharp rise will be noted.

What is the relative humidity of the inspired gas of a patient who is intubated if the humidity deficit is 33 mg/L? A. 50% B. 100% C. 25% D. 75%

The correct answer is : C Explanation : There are 44 mg of water per liter of gas if the gas is at 100% relative humidity. If you have only 11 mg/L of gas (deficit of 33 mg) then you have only 25% relative humidity because 11 mg is 25% of 44.

In preparation for ventilator weaning the physician requests a VD/VT ratio assessment. Which of the following is needed to determine the deadspace-tidal volume ratio?

arterial blood gases and capnographic data

Pulmonary function testing is done on a patient with a 65-pack-year history of smoking. The following pulmonary function data is recorded: Percent of Pred Actual value Fev1.0 58% FEF200-1200 75% FEF25-75 52% SVC 88% FVC 81% DLCO 18 CO/min/mmHg Which of the following most likely represents the patient's condition? A. kyphosis B. chronic bronchitis C. emphysema D. cystic fibrosis

The correct answer is : C Explanation : These pulmonary function results show that the patient obstructed. This is indicated because the FEV1 is less than 80% of predicted. The SVC and the FVC are normal, or about 80% of predicted. Therefore, the patient can not be restrictive. When we look at the answers we see there are three obstructive diseases to choose from. When this happens we need more information. When we return to the pulmonary function test results we further see that the DLCO is reduced and is below normal. Normal DLCO is 25 CO/min/mmHg. That means that less than 80% of predicted would be anything less than 20 CO/min/mmHg. In this case, the DLCO is 18 CO/min/mmHg. There is only one obstructive disease that is associated with the poor DLCO - pulmonary emphysema.

A patient is receiving VC AC ventilation with the following data: 3 PM 5 PM Plateau pressure (cm H2O) 33 33 Peak pressure (cm H2O) 39 46 VT (mL) 500 500 This data indicates A. pulmonary overdistension B. development of autoPEEP C. increase airway resistance D. decreased static compliance

The correct answer is : C Explanation : This data shows increasing peak pressures while plateau pressures are remaining constant. This combination suggests that the patient does not experiencing a change in pulmonary compliance but rather decreasing dynamic compliance. This can be caused by many things, including secretions in the airway, an occluded ET tube, or airway resistance through bronchoconstriction, among others.

A respiratory therapist is asked to interpret the following arterial blood gas for an 11-year old patient on a mechanical ventilator: pH 7.21 PaCO2 56 mm Hg PaO2 54 mm Hg HCO3- 26 mEq/L BE -2 mEq/L The patient is being ventilated with an adult circuit and an HME is in-line. What should the therapist recommend? A. change to a pediatric circuit B. replace HME with a heated-humidification system C. increase tidal volume D. decrease deadspace

The correct answer is : C Explanation : This patient is both hypoventilating and hypo oxygenating. Of these two problems hypoventilation should be addressed first. This is because fixing ventilation may also relieve hypoxemia. To increase ventilation either tidal volume or mandatory rate may be increased. Removing deadspace will also decrease CO2 but is usually insufficient when CO2 is more than 4 mm Hg off target. The best option would be to increase mandatory rate but this option is not offered. Therefore, the second best option is to increase tidal volume.

A patient with asthma receiving volume-controlled mechanical ventilation has the following arterial blood results on the settings below: Mode Assist/control ABGs Mandatory rate 18 pH 7.33 VT 400 mL PaCO2 47 torr FIO2 0.40 PaO2 78 torr Mech PEEP 5 cm H2O HCO3- 26 mEq/L Total PEEP 8 cm H2O BE +2 mEq/L I:E 1:2 Which of the following should be increased? A. tidal volume B. PEEP C. flowrate D. FIO2

The correct answer is : C Explanation : This patient is developing autoPEEP as manifested by a total PEEP that is higher than the set mechanical PEEP. This is caused from insufficient expiratory time and is remedied by shortening inspiratory time, which may be done by increasing flowrate.

A patient who weighs 68 kg (150 lb) has a minute ventilation requirement of 14 L/min to maintain a PaCO2 of 38 torr. Which of the following can explain the ventilatory requirements? A. excessive caloric intake B. obstructive apnea C. increased dead space ventilation D. CNS depression

The correct answer is : C Explanation : This patient is requiring a very high minute ventilation to achieve a normal PaCO2. The most likely cause of this is increased dead space ventilation. Dead space ventilation is likely due to collapsed alveoli which results in lower lung space. This would cause the patient to breathe quickly and deeply in order to move significant amounts of air to achieve adequate ventilation.

A patient has been weaning from mechanical ventilation and is currently on CPAP 5 cmH2O with pressure support 5 cm H2O. The following additional data is available: Respiratory rate 28 /min Pulse 92 /min VT(spontaneous) 200 mL MIP - 29 cmH2O The respiratory therapist should A. decrease pressure support, continue weaning for 30 minutes B. obtain arterial blood gases in 30 minutes C. switch to full ventilatory support D. monitor the patient, continue weaning

The correct answer is : C Explanation : This patient is weaning from mechanical ventilatory support but is showing signs of distress. Respiratory rate is a bit excessive and spontaneous tidal volume of 200 mL is insufficient to sustain life. Spontaneous tidal volume should be at least 5 mL/kg. this would be an indication to cease weaning and return the patient to full ventilatory support.

A 64-year-old patient with meningitis has received volume-controlled ventilation for 11 weeks. In an attempt to slowly wean the patient from dependence on mechanical ventilatory support, spontaneous breathing trials with a T-tube have been done over the last few days. The following data is available: 30 Minutes 1 Hour 2 Hours Pulse rate 89 112 122 SpO2 (%) 97 87 84 MIP (cm H2O) -30 -19 -17 Vital capacity (L) 1.7 0.9 0.7 Respiratory rate 20 30 36 The respiratory therapist should recommend A. discontinue weaning trials for 2 days B. the next SBT should not exceed 2 hours in duration C. the next SBT should not exceed 30 minutes in duration D. the next SBT should not exceed 1 hour in duration

The correct answer is : C Explanation : This patient is weaning from, mechanical ventilatory support by spontaneous breathing trials. After a 30-min. trial the patient appears to be stable. However, at one hour a significant degradation in ventilatory status is noted. This data indicates that future trials should not exceed 30 min.

The following arterial blood gas results are recorded for a patient during cardiopulmonary resuscitation: pH 7.10 PaCO2 46 torr PaO2 208 torr HCO3- 12 mEq/L FIO2 1.0 On the basis of these values and the following ECG waveform, the respiratory therapist should recommend A. decrease FIO2 B. administer sodium nitroprusside C. sodium bicarbonate administration D. increase manual ventilation rate

The correct answer is : C Explanation : This patient shows acidosis, as manifested by the pH, but has a nearly normal CO2. A further examination of the HCO3 shows that the source of the acidosis is metabolic. Administration of sodium bicarb (HCO3-) is appropriate.

A patient is suspected of being a chronic CO2 retainer. A blood gas analysis reveals a PaO2 of 64 torr and a PCO2 of 65 torr. What additional data may be used to determine if the patient is a chronic CO2 retainer?

arterial blood pH

A patient is receiving supplemental oxygen at FIO2 0.6 via cool aerosol. Arterial blood gas results are as follows: pH 7.40 PaCO2 41 mm Hg PaO2 69 mm Hg HCO3- 22 mEq/L BE -2 mEq/L Which of the following should the respiratory therapist recommend? A. Place the patient on a partial rebreathing mask B. Increase FIO2 to 0.8 via cool aerosol mask C. Place the patient on CPAP D. Place the patient on a non-rebreathing mask

The correct answer is : C Explanation : This patient shows continued hypoxemia in spite of an FIO2 of 0.6. This indicates significant shunting and suggests the patient would benefit from continuous positive airway pressure.

A postoperative patient receiving mechanical ventilatory support is waking from sedation. Ventilatory settings and arterial blood gases are as follows: Mode Assist/control Mandatory rate 14 Total rate 16 Tidal volume 450 mL FIO2 0.30 PEEP 5 cm H2O pH 7.41 PaCO2 39 torr PaO2 65 torr HCO3- 23 mEq/L BE -1 mEq/L In order to begin weaning, the respiratory therapist should recommend which of the following methods? A. decrease mandatory rate B. switch to bi-level ventilation C. SIMV with pressure support D. CPAP

The correct answer is : C Explanation : This postoperative patient is awaking from sedation while on a mechanical ventilator. Normally, when a patient awakens from surgery, they may be removed from mechanical ventilatory support very quickly. However, in this case, the patient's PaO2 still shows hypoxemia. Weaning with pressure support is appropriate.

Which of the following would suggest that ventilation liberation should not be attempted? A. spontaneous Vt of 6 mL/kg B. VC of 10 mL/kg C. VD/VT ratio > 60% D. MIP -31 cmH2O

The correct answer is : C Explanation : To be a candidate for ventilator weaning, deadspace-tidal volume ratio must be less than 60% while on the ventilator.

Which of the following data is needed to calculate minute alveolar ventilation? A. IBW, VD/VT B. PECO2, VT C. RR, VT, weight in lbs D. RSBI, VD/VT

The correct answer is : C Explanation : To calculate minute alveolar ventilation, three values must be known: the patient's weight in pounds, tidal volume, and respiratory rate. Respiratory rate multiplied by tidal volume is equal to minute ventilation. However, this question asks about minute ALVEOLAR ventilation. To determine alveolar ventilation one must first take tidal volume and subtract the dead space portion of each breath. That portion is estimated to be 1 mL per pound of ideal body weight. Once the dead space is removed from the tidal volume the remaining value may be multiplied by the rate to get the minute alveolar ventilation.

An 18-year-old female with a history of asthma since she was 9-years-old has the following pulmonary function results: Pre-bronchodilator Post-bronchodilator FVC (% predicted) 84 85 FEV1 (% predicted) 85 87 FEV1/FVC (%) 83 85 Peak flow (% predicted) 97 95 The respiratory therapist should recommend which of the following? A. SNB2 testing B. DLCO determination C. inhalation challenge D. maximum voluntary ventilation test

The correct answer is : C Explanation : To determine if a patient with asthma is responsive to bronchodilators, a variety of pulmonary function tests may be done. In this data, the pre-bronchodilator data suggests that there is no current constriction, making it difficult to check their responsiveness to any particular bronchodilator medication. Thus, it may be effective to use a practice that elicits bronchoconstriction first. This purposeful promotion of bronchoconstriction followed by the delivery of a bronchodilator is called an inhalation challenge. Methacholine and cold air are 2 methods of causing bronchoconstriction.

What is the static compliance in mL/cm H2O, given the following data on a patient receiving VC AC ventilation? Plateau pressure 15 cm H2O Peak pressure 25 cm H2O VT 500 mL PEEP 5 cm H2O A. 20 B. 25 C. 50 D. 33

The correct answer is : C Explanation : To determine static compliance, tidal volume is divided by plateau pressure after subtracting any PEEP. In this case, (500/(15-5) = 50 cm H2O.

Which of the following actions would most likely resolve a high-pressure alarm on a volume-cycled adult ventilator? A. add air to the ET tube cuff B. decrease heater temperature setting C. suction the patient D. tighten the patient-ventilator interface connection

The correct answer is : C Explanation : To determine the answer to this question one must first look at the cause of the high pressure alarm on the ventilator. The need to add air to the ET tube cuff would result in a low-pressure alarm. The need to tighten the patient ventilator interface connection would presumably be caused by a leak and would result in a low-pressure alarm. The heater temperature should have no effect. Therefore, suctioning the patient is the only solution that is an appropriate response to the high-pressure ventilator alarm

A patient is receiving 28% aerosol with the oxygen flow meter running at 6 L/min. What is the total flow of gas to the patient? A. 44 L/min B. 18 L/min C. 66 L/min D. 24 L/min

The correct answer is : C Explanation : To determine the answer to this question, one must have memorized the air-oxygen ratios. For 28%, the ratio is 10:1. This means that the Venturi mechanism will incorporate 10 parts air for every 1 part oxygen. One way to solve this problem is to add the ratio numbers together. (10+1= 11). Then, multiply that total by the indicated flow on the oxygen flow meter. In this case, we multiply 6 L per minute by 11. We get a total flow of 66 L/min.

Immediately following oral endotracheal intubation, the respiratory therapist should confirm proper placement by doing which of the following? A. Assess end-tidal CO2 with a colorimetric capnometer B. Auscultate the neck C. Obtain a anterior-posterior chest radiograph D. Ensure tube markings are between 20-24 at the teeth

The correct answer is : C Explanation : To determine the location and placement of an endotracheal tube a chest x-ray is appropriate. Because the patient is intubated it is not likely that the patient is ambulatory and therefore must undergo a chest x-ray in bed. When shooting a chest x-ray bed, the proper technique is called an AP chest radiograph, or anterior-posterior x-ray.

A respiratory therapist notices an increasing high-pitched noise coming from the upper airway of a child receiving 30% oxygen by heated aerosol. Which of the following would be most appropriate for further evaluation? A. CT scan of the upper airway B. bedside pulmonary function test with flow volume loop C. lateral neck radiograph D. MRI of the upper airway

The correct answer is : C Explanation : To evaluate the source of upper airway inflammation, a neck x-ray is most appropriate. This allows direct visualization of the tissue which can be helpful in determining the source and location of the inflammation.

A physician notes a round mass in the right lower lung field when observing the results of an AP chest radiograph. Which of the following would be most helpful to further evaluate the shape of the mass? A. lateral decubitus radiography B. lateral neck X-ray C. spiral CT scan D. PA chest radiography

The correct answer is : C Explanation : To further evaluate what appears to be a round mass in the lung field, a radiological procedure is appropriate. The most appropriate radiological procedure would be that which shows three dimensionality. In this case a spiral CT scan will provide that perspective. All other options offered would only show a two dimensional version of the concern.

A respiratory supervisor is calculating appropriate staffing levels for a shift. Which of the following should be included in calculation? A. frequency of refused therapy B. patient medical history C. patient areas and locations D. staff member preferences

The correct answer is : C Explanation : To plan for patient care, staffing must be appropriate. Conditions that affect the amount of staff needed include location of patients (how far staff must travel to get to patients), frequency of ordered therapy (how frequently staff must visit the patient), and the type of therapy (how long the therapy will take). The patient's medical history will not likely affect staffing.

A patient is receiving Anectine (succinylcholine chloride) to facilitate oral intubation. Two minutes after administering the medication, the respiratory therapist notices involuntary minor muscle twitching about the neck and upper thorax region. The therapist should A. administer rocuronium (Zemuron) B. allow time for the muscle twitching to cease C. proceed with the intubation D. administer antagonist medication immediately

The correct answer is : C Explanation : Upon administration of the drug Anectine, muscle twitching about the face, neck, and upper thorax is an indication that the drug has had sufficient time to impose paralysis. Therefore, it is a sign to proceed with the intubation procedure.

What radiographic findings are associated with active, untreated tuberculosis? A. Butterfly or batwing pattern in the lower lobes B. A single cavitation in one of the lower lobes C. Upper lobe cavitations D. Varying sized masses found in both upper lobes

The correct answer is : C Explanation : Upper lobe cavitations are associated with tuberculosis. The cavitations are larger and singular. These single cavitations are rarely if ever seen in the lower lobes. Butterfly or batwing patterns are associated with pulmonary edema. Varying sized masses are associated with cancer.

Which of the following is most likely to increase the incidence of ventilator-associated pneumonia? A. frequent suctioning with a closed system suction catheter B. use of minimum seal technique for ET tube cuff inflation C. frequent disconnection of the ventilator circuit D. HME device used for 72 hours

The correct answer is : C Explanation : VAP, or ventilator associated pneumonia, is caused by allowing bacteria and other microorganisms to inadvertently enter the ventilator circuit. Of the options given, only frequent interruption or disconnection of the ventilator circuit places the patient at risk. Long-term use of an HME device, while inappropriate, is not likely to increase the incidence of VAP. Frequent suctioning with a closed system suction catheter will protect the patient more than it will put the patient at risk. Ensuring adequate seal of the ET tube cuff will also help the patient avoid VAP.

A patient is receiving volume-controlled ventilation in the emergency department (ED). The following data is available: Mode Assist/control VT 500 mL Mandatory rate 14 FIO2 0.5 PEEP 5 cm H2O PetCO2 30 torr pH 7.39 PaCO2 40 torr PaO2 100 torr HCO3- 25 mEq/L BE +1 mEq/L The respiratory therapist should report which of the following as an accurate VD/VT ratio? A. 50% B. 75% C. 25% D. 15%

The correct answer is : C Explanation : VD/VT ratio calculation is (PaCO2-PetCO2)/PaCO2. In this case 40 -30 = 10. 10/40 = 25%

The emergency response team is monitoring a patient when the rhythm on the monitor transitions to ventricular tachycardia. The patient has a pulse. Prior to cardioversion, the respiratory therapist should do which of the following before handing the physician the paddles? A. set defibrillation to 360 joules B. administer sodium bicarbonate C. ensure synchronization is active D. call "Clear"

The correct answer is : C Explanation : Ventricular tachycardia with a pulse must be treated by cardioversion. Cardioversion differs from defibrillation by the number of watts or Jules administered and by how it synchronizes with the heart. Pure defibrillation requires no synchronization with a heart. However, cardioversion requires that the synchronization be set to active prior to delivery of the electrical shock.

A patient with a fenestrated tracheostomy tube is in full cardiopulmonary arrest with a blood pressure of 40/10 mmHg, a respiratory rate of 2, and a pulse of 20/min. To provide manual ventilation the respiratory therapist should FIRST A. suction the patient B. inflate the cuff C. remove the tracheostomy cap D. remove the fenestrated tube and reintubate with a regular tracheostomy tube

The correct answer is : C Explanation : When a fenestrated tracheostomy is in a speaking configuration but requires positive pressure ventilation with a resuscitation bag or ventilator, the first step is to remove the tracheostomy. When transitioning from one configuration to the other, one must remember to always do the steps in a order that allows the patient to breathe freely during the change.

A therapist is unable to obtain a reading with a galvanic-type oxygen analyzer when attempting to measure oxygen percentage inside an infant oxygen hood while heated humidity is also applied. The therapist should? A. change the batteries B. discontinue heated humidity C. change the electrode D. change the electrolyte solution

The correct answer is : C Explanation : When a galvanic-type oxygen analyzer fails to produce a reading, it is likely a problem with the power source. These type of analyzers do not have batteries. The electrode is in essence a battery. So, the solution is to change the electrode. The electrode is also called the fuel cell.

A homecare patient complains that her oxygen concentrator is not working correctly because she does not feel like she is getting enough oxygen. The therapist will immediately instruct the patient to A. push the reset button on the machine B. change the filter C. switch to an E cylinder oxygen tank D. check the circuit breaker

The correct answer is : C Explanation : When a patient complains they are not receiving enough oxygen, the first step is to ensure oxygen delivery by changing to a different modality. Then, the therapist can troubleshoot the problem and resolve it. In this case changing to an E cylinder is most appropriate.

After making the universal sign of choking, a person collapses. The observer should FIRST

perform abdominal thrusts

A 45-year old male is receiving mechanical ventilation on the following settings: Mode Assist/control Mandatory rate 16 Tidal volume 550 mL FIO2 0.60 Pressure limit 60 cm H2O PEEP 8 cm H2O I:E 1:1.5 The patient is developing autoPEEP. The respiratory therapist should do which of the following? A. Decrease PEEP to 5 cm H2O B. Discontinue PEEP C. Increase inspiratory flow rate D. Set PEEP to match the Auto PEEP value

The correct answer is : C Explanation : When a patient is developing auto PEEP, the solution is to either decrease mandatory rate or increase expiratory time. Because decreasing mandatory rate will likely negatively affect ventilation, the most appropriate action is to manipulate the I:E ratio. To lengthen expiratory time, inspiratory flow rate should be increased.

A patient with a pulmonary infection coughs up a mucus plug and is wheezing bilaterally. The respiratory therapist should administer A. mucomyst followed by heated aerosol by mask B. aerosolized Ipratroprium bromide (Atrovent) C. a bronchodilator followed by aerosol via ultrasonic nebulizer D. IPPB with beta-II agonist

The correct answer is : C Explanation : When a patient is wheezing and producing mucus, both bronchodilation and hydration are indicated. If asked which order these therapies should be administered, bronchodilation should be administered first, followed by hydration therapy. Mucomyst would help thin secretions but would likely increase bronchoconstriction. IPPB with the beta-II agonist (front door bronchodilator) would be helpful but does not address the need to thin secretions.

A 5-year old child in the emergency department is demonstrating wheezing on the right side while breath sounds are normal on the left. The respiratory therapist should FIRST do which of the following? A. perform bedside pulmonary function testing B. perform and ABG C. obtain a chest radiograph D. order a spiral CT scan

The correct answer is : C Explanation : When a patient is wheezing bilaterally the most likely cause is bronchoconstriction. However, when the patient is wheezing unilaterally, or on just one side, the most likely cause is foreign body aspiration (a toy or perhaps food in the main stem bronchus). To determine this a chest x-ray is most appropriate. A spiral CT scan may also see the object but is more expensive and time-consuming. The other examinations will not be helpful.

While eating, a patient makes the universal sign of choking to the therapist during a routine oxygen check. The observer should A. perform chest compressions B. administer rescue breathing C. perform abdominal thrusts D. check for a pulse

The correct answer is : C Explanation : When a person indicates the universal sign of choking, they are unable to verbalize because there is likely something caught in their airway. The person responding must first focus on removing the obstruction, which is done by performing abdominal thrusts. Administering rescue breaths would not be appropriate because the airway is obstructed. Checking for a pulse or performing chest compressions must be addressed after ensuring the patient's airway is unobstructed. This is because ventilation must be addressed before oxygenation, circulation, and perfusion.

The respiratory therapist notices an infant in an oxyhood at 28% is restless. SpO2 is 98%. To help calm the infant the therapist should A. increase FIO2 0.35 B. decrease flow to the nebulizer to 4 L/min C. replace the nebulizer with an air-oxygen blender D. switch to an incubator

The correct answer is : C Explanation : When an infant is receiving oxygen by oxygen hood with an air-entrainment device, a primary concern is the amount of sound transmitted into the oxygen hood. Too much sound can cause the patient to be restless and/or cause hearing damage. Air-entrainment devices that are set at low oxygen percentages entrain large amounts of air and emit an significant amount of sound. To resolve this problem, an air-oxygen blender may be used. It should be set at the appropriate FIO2 and the air entrainment device should be set at 100% so it does not entrain any room air. This will cause the whole system to operate more quietly and allow the infant to rest.

A respiratory therapist changes from a normal adult ventilator circuit to a heated-wire circuit. Arterial blood gases are as follows: pH 7.31 PaCO2 48 torr PaO2 81 torr HCO3- 24 mEq/L BE 0 mEq/L Which of the following changes is most indicated? A. revert to the normal non-heated wire circuit B. increase inspiratory flow rate C. remove 50-100 mL of deadspace between the wye and patient D. add 100 mL of deadspace at the patient-ventilator interface

The correct answer is : C Explanation : When arterial carbon dioxide is high, there are three options. Respiratory rate may be increased, tidal volume may be increased, or deadspace may be removed. Of the options offered removing deadspace is the best option and is the only option that will reduce carbon dioxide. What also makes this appropriate is the fact that CO2 is off target by a very small amount, making a change in deadspace appropriate.

A respiratory therapist is determining a patient's orientation to time, place, and person. Which of the following methods of communication should the therapist employ A. examine the medical record B. speak to the patient's family and/or friends and visitors C. open-ended questioning D. pictograms

The correct answer is : C Explanation : When interviewing a patient to perform a clinical or historical assessment or to assess the patient's orientation and mental condition, open-ended questions should be asked. These are questions that may not be answered with a simple "yes" or "no". People with language barriers will often answer in the simple affirmative or negative even when they do not understand the question.

While running quality control material through a blood gas analyzer, the therapist notes that 4 consecutive data points are beyond 2 SD from the mean. The therapist will A. monitor the machine closely for several days B. report the problem to the medical director C. remove the machine from service D. continue running quality control material until corrected

The correct answer is : C Explanation : When monitoring the quality control data for an arterial blood gas machine the points on the graph, which show the actual values of the quality control material, should be between the upper control limit and the lower control limit (two standard deviations above and below the mean for a total of a 4 standard deviation range). In this case there are several consecutive points that are beyond the two standard deviations and therefore the machine is considered out of control and should not be placed in service but removed. Blood gases should not be run on that machine until the problem has been resolved. More importantly, blood gas data should not be reported as part of the patient's medical record because they are likely inaccurate.

A hospital has an extremely low incidence of ventilator-associated pneumonia. To which of the following reasons may this be attributed?

periodic discontinuation of sedation

Over the last 2 hours, the respiratory therapist notices the peak pressure of a patient receiving volume controlled mechanical ventilatory support has risen from 22 to 31 cm H2O. Plateau pressures have remained steady at 14 cmH2O. Which of the following is the most likely explanation for these changes? A. reticulogranular changes on the chest radiograph B. pulmonary fibrosis C. secretions in the airway D. ARDS

The correct answer is : C Explanation : When peak pressures increase while plateau pressures remain virtually unchanged, the most likely cause is a temporary condition such as secretions in the airway, bronchoconstriction, occlusion of the endotracheal tube, etc. Pulmonary fibrosis and ARDS would cause a decrease in pulmonary compliance manifested by an increase in plateau pressures.

While performing minimal leak technique on a 70-kg (154-lb) male with a 7.0-mm tube, the therapist auscultates a high-pitched sound over the neck at the top of each mechanical breath. The therapist should A. reintubate with a larger ET tube B. add air to the ET tube cuff C. record results and monitor exhaled volume D. remove air from the ET tube cuff

The correct answer is : C Explanation : When performing a minimum leak technique to establish proper inflation of the ET tube cuff, if properly performed, the practitioner should be able to auscultate a high-pitched sound over the trachea at the top of each mechanical breath. This sound is coming from a tiny amount of air passing around the cuff and is purposeful. Ensuring a leak is the best way to ensure minimal pressures are being placed on the trachea wall.

Low-volume alarms are most important in which of the following situations? A. adult volume-cycled ventilation B. adult IPPB with bronchodilators C. infant time cycled, pressure-limited ventilation D. pediatric volume-cycled ventilation

The correct answer is : C Explanation : When providing mechanical ventilation to an infant with a time-cycled, pressure-limited ventilator, the low-volume alarms are critically important because a leak, or a change in lung compliance could easily result in a significant lack of sufficient ventilation.

An orally intubated patient is breathing spontaneously through an 8.0 mm ET tube. In the ambient air, humidity indicates only 22 mg per liter of air. Based on this, what humidity deficit will the respiratory therapist have to make up to completely meet the humidity needs of the patient? A. 44 mg/L of gas B. 100 mg/L of gas C. 22 mg/L of gas D. 12 mg/L of gas

The correct answer is : C Explanation : When relative humidity is 100%, the ambient air is holding 44 mg/H2O/L of gas. If the ambient air is only holding 22 mg/H2O/L, there is another 22 mg/H2O/L that is missing. This is known as the humidity deficit. Humidity deficit is always expressed in milligrams whereas relative humidity is expressed as a percentage.

A patient with pulmonary emphysema is unable to tolerate postural drainage and percussion to the basal pulmonary segments. Which of the following represents an appropriate modification in therapy? A. administer aerosolized Albuterol treatments B. administer aerosolized Mucomyst (Acetylcysteine) C. switch to high frequency chest wall oscillation D. perform postural drainage with bed in semi-fowlers position only

The correct answer is : C Explanation : When the patient is receiving postural drainage and percussion to the basal segments it means they have thier head of bed down significantly. Often patients cannot tolerate this. The respiratory therapist must, therefore, modify the therapy to something else that accomplishes the same objective. The administration of the bronchodilator or a mucolytic medicaion does not actually help secretions move out of the lungs. Performing postural drainage and percussion in semi-Fowler's position will not drain the basal segments and therefore is inappropriate. So, the only acceptable modification is the use of high-frequency chest wall oscillation, also called Vest therapy.

A patient who is receiving volume-controlled ventilation is showing signs of ventilatory distress. The low return volume alarm is sounding and there is excessive bubbling found in the water-seal compartment of a three-chamber chest tube drainage system. The respiratory therapist should first A. decrease suction pressure at the wall B. clamp the tubing between the collection chamber and the water-seal chamber C. clamp the chest tube at the patient's chest D. decrease tidal volume

The correct answer is : C Explanation : When these two conditions are observed, low return volume and excessive bubbling in the water seal compartment, the primary concern is that there is a perforation in the lung tissue. However, the excessive bubbling may be coming from a leak in the chest tubes between the patient and the collection chambers. To determine if this is the case, one starts by clamping the chest tubes at the patient's chest and observing for a decrease in bubbling in the water seal compartment. If bubbling does not decrease, the clamp is moved towards the chambers to determine if there is a leak in other parts of the tubing. However, if bubbling stops or decreases when the clamp is placed on the tubes where they exit the patient's body, the leak must be inside the patient and would suggest a perforated lung.

While turning a patient for a V/Q scan, the therapist suspects the endotracheal tube changed position. Currently the ET tube markings are 19 at the lip line. The therapist should FIRST do which of the following to assess tube position? A. advance the ET tube by 2 cm B. obtain a chest radiograph C. observe chest rise D. withdraw the ET tube by 5 cm

The correct answer is : C Explanation : When trying to quickly determine the location of the endotracheal tube the action that must be taken first is that which is the quickest. Of the options offered, observing chest rise is the quickest.

While turning a patient for a V/Q scan, the therapist suspects the endotracheal tube changed position. Currently the ET tube markings are 19 at the lip line. The therapist should FIRST do which of the following to assess tube position? A. obtain a chest radiograph B. advance the ET tube by 2 cm C. observe chest rise D. withdraw the ET tube by 5 cm

The correct answer is : C Explanation : When trying to quickly determine the location of the endotracheal tube the action that must be taken first is that which is the quickest. Of the options offered, observing chest rise is the quickest.

Which of the following defibrillator settings is appropriate for treating a patient with ventricular tachycardia? A. synchronize to the R wave B. synchronize to the T wave C. synchronization set to OFF D. synchronize to the P wave

The correct answer is : C Explanation : When using a defibrillator, one setting that must be adjusted is the synchronization. When defibrillating a patient who has ventricular tachycardia, the synchronization must be set to OFF. Only when cardioversion is being done should the synchronization be set to ON or ACTIVE.

A patient who requires 10 cmH2O of PEEP when mechanically ventilated is being transported. A spring PEEP valve will be used in conjunction with the resuscitation bag. Where should the PEEP valve be placed? A. proximal to the inlet valve B. distal end of the reservoir C. distal to the outlet valve D. between the bag and patient interface

The correct answer is : C Explanation : When using a resuscitation bag for a patient who requires contant PEEP, a PEEP valve may be placed at the distal end of the outlet valve. Placing the valve anywhere else will not provide positive end-expiratory pressure.

The disk fails to rise on a volume-oriented incentive spirometer in spite of a forceful exhalation into the device. The therapist should A. switch to a flow-oriented device B. examine the device for a leak C. instruct the patient to inhale through the device D. switch to IPPB by mask

The correct answer is : C Explanation : With incentive spirometers, patients commonly confuse the maneuver and exhale into the device rather than inhale. When this happens the respiratory therapist must instruct the patient on the proper use of the device, which is to perform inhale deeply through the device.

Which of the following is the best reason for reducing the high-pressure alarm limit for a patient who is receiving volume-cycled mechanical ventilation? A. decreased respiratory rate B. increased cough frequency C. increased lung compliance D. decreased patient alarm tolerance

The correct answer is : C Explanation : With increased lung compliance, peak pressures will diminish with constant delivered tidal volumes. Therefore, the therapist should adjust the peak pressures downward in order to remain 10-15 cm H20 above the baseline peak pressures.

The following chest radiograph is observed. Which of the following is evident in the film? A. tracheostomy tube, pneumothorax B. chest tube, pneumothorax C. pneumothorax, pleural effusion, chest tube D. tracheostomy tube, pleural effusion

The correct answer is : D Explanation : Examination of this chest radiograph reveals the presence of a tracheostomy tube and a pleural effusion. A pneumothorax would show a larger black area over a lung field and a chest tube would show up on the periphery in the pleural space. Neither of these are present in this film.

Which of the following will provide a continuous elevation of baseline pressure during inspiration and expiration on a patient who is breathing spontaneously? A. IPPB B. APRV C. NAVA D. CPAP

The correct answer is : D Explanation : The description provided in this question is most consistent with CPAP, also called continuous positive airway pressure.

A family of 4 is suspected to have CO poisoning from exhausted fumes that leaked into the cabin of their vehicle during a trip. Which of the following would be most helpful at quickly assessing the degree of carbon monoxide exposure? A. COHb by arterial blood analysis B. PaO2 by arterial blood gas C. O2 saturation by pulse oximetry D. multiple wavelength spectrophotometry

The correct answer is : D Explanation : A COHb is the gold standard for determining if a patient has carbon monoxide poisoning. When this is suspected normal pulse oximetry should not be trusted. However, multiple wavelength spectrophotometry is a method that works like pulse oximetry but is capable of reading more than just oxygen. It also has the ability and to determine carbon monoxide in the blood. A normal pulse oximeter is also called a single wavelength spectrophotometer whereas an oximeter capable of reading other gases, including CO, is called a multiple wavelength spectrophotometer.

A COPD patient who has bilateral expiratory wheezing would benefit most immediately from which of the following medications? A. Acetylcysteine (Mucomyst) B. Tilade (Nedocromil) C. Budesonide (Pulmicort) D. Albuterol

The correct answer is : D Explanation : A COPD patient would benefit most from short-term bronchodilation because the patient is currently wheezing. Of the options listed, only albuterol is considered to be a bronchodilator.

The respiratory therapist obtains the following blood gas data on a patient breathing spontaneously on room air: pH 7.35 PaCO2 45 torr PaO2 50 torr HCO3- 27 mEq/L BE +2 mEq/L The therapist could accurately estimate the patient's SaO2 to be which of the following? A. 85% B. 90% C. 75% D. 80%

The correct answer is : D Explanation : A PaO2 of 50 mmHg most closely correlates with an oxygen saturation of about 80% on the oxygen dissociation curve.

A respiratory therapist is educating a patient who will be receiving supplemental oxygen at home with an oxygen concentrator. Which of the following should the respiratory therapist recommend as a backup in the event electricity supply to the home is interrupted? A. bulk liquid oxygen supply B. a second oxygen concentrator C. manifold and bank of H tanks D. multiple E cylinders

The correct answer is : D Explanation : A an appropriate backup for an oxygen concentrator is an E cylinder.

A patient with ARDS has been receiving PC ventilation with an inspiratory pressure of 35 cm H2O and a PEEP of 20 cm H2O. A recent V/Q scan has confirmed a bronchopleural fistula in the right lung. The patient is schedule for surgery. Until surgery can occur, the respiratory therapist should recommend which of the following protective strategies? A. increasing inspiratory pressure to 40 cm H2O B. decreasing inspiratory pressure to 25 cm H2O C. removal all PEEP D. insertion of a double lumen endotracheal tube

The correct answer is : D Explanation : A bronchopleural fistula indicates that there is a hole in the lung, allowing some of the inspired gases to leak into the pleural space. This condition should be resolved surgically. In the meantime, however, all efforts should be centered on keeping mean airway pressure low. There are several ways to accomplish this but one common way is to provide independent lung ventilation. This means that two ventilators will be used, each to ventilate one lung. To facilitate this, insertion of a double lumen endotracheal tube is required.

Which of the following conditions would benefit most from bronchoscopy? A. emphysema B. ARDS C. chronic bronchitis D. foreign body aspiration

The correct answer is : D Explanation : A bronchoscopy will not help to resolve emphysema, chronic bronchitis, or adult respiratory distress syndrome. It is, however, helpful in removing a foreign body from the trachea or lungs.

A patient is receiving mechanical ventilation in the pressure-control mode. End-tidal CO2 is being monitored. The following tracing is available on the monitor in real-time. Which of the following interpretations is most consistent with this tracing? A. hyperventilation B. hypoventilation C. poor alveolar recruitment D. normal ventilation

The correct answer is : D Explanation : An end-tidal CO2 of 40 mmHg correlates with an arterial CO2 level of about 40 mmHg, which is normal.

A patient with a bronchopleural fistula is receiving volume-controlled ventilation. Arterial blood gas results are as follows: pH 7.35 PaCO2 45 torr PaO2 52 torr HCO3- 24 mEq/L BE -1 mEq/L The respiratory therapist should recommend which of the following? A. bilevel ventilation B. manual ventilation with a bag-valve C. inverse I:E ventilation D. high frequency jet ventilation

The correct answer is : D Explanation : A broncopleural fistula is a perforation in lung tissue that allows gases to move from the lungs into the pleural space. When this happens, volume is lost and gas exchange is decreased. To prevent expansion of the perforation, ventilation techniques which involve low peak pressures are important. High-frequency jet ventilation is one therapy that may accomplish this.

A 45-year-old 70-kg (154-lb) male patient with pneumonia is receiving mechanical ventilatory support in the assist/control mode. The patient awakens from sedation and is immediately agitated and seems air-hungry. Ventilatory parameters are: VT 500 mL Mandatory rate 16 FIO2 0.40 PEEP 5 cm H2O Insp flow 22 L/min I:E ratio 1:2 The therapist should A. decrease tidal volume B. decrease sensitivity C. lengthen inspiratory time D. increase flow

The correct answer is : D Explanation : A close examination of these ventilator settings shows a low flow rate. This flow rate may have been appropriate when the patient was not awake but once the patient has awaked and sedation has worn off, spontaneous ventilation is likely to increase. This increase in inspiratory demand may be accommodated by increasing flow and ensuring sensitivity is set appropriately.

A therapist is making several changes to the mechanical ventilator on a patient receiving volume-controlled ventilation in the SIMV mode. If the therapist decreases peak inspiratory flow but makes no changes in tidal volume or rate, which of the following could the therapist expect to occur? A. Increased peak inspiratory pressures B. Increased expiratory time C. Decreased inspiratory time D. Increased I:E ratio

The correct answer is : D Explanation : A decrease in inspiratory flow rates low will cause an increase in inspiratory time. This changes the I:E ratio.

A patient in the intensive care unit has just self-extubated. Stridor is audible. Which of the following is true about stridor? A. indicates severe asthma. B. is often treated with heated aerosol therapy. C. is created when a patient coughs through an open stoma. D. is a high-pitched sound coming from the upper airway.

The correct answer is : D Explanation : A high-pitched noise in the upper airway is also known as stridor. Stridor is related to inflammation in the upper airways resulting in a narrowing passageway for air movement. In some cases this is life-threatening.

A home care patient is ventilator dependent for more than 50% of the time. Which of the following is necessary to have? A. H-tank manifold with a bank of oxygen cylinders B. non-rebreathing mask with an E-cylinder C. bulk liquid oxygen supply D. back up ventilator

The correct answer is : D Explanation : A homecare patient who is ventilator-dependent more than 50% of the time should have a backup ventilator. A bag/valve is also important but does not qualify as a backup ventilator.

A home care patient is ventilator dependent for more than 50% of the time. Which of the following is appropriate to have on hand in the event of a power failure? A. compressed oxygen B. non-rebreathing mask with an E-cylinder C. back-up ventilator D. bag-valve resuscitator

The correct answer is : D Explanation : A homecare patient who is ventilator-dependent more than 50% of the time should have a backup ventilator. However, they should also have a bag/valve resuscitator in case of power failure.

A 24-year-old firefighter is brought to the emergency department after being trapped for 10 minutes in a collapsed, burning building. The respiratory therapist notes marked inflammation around the face and oropharynx. Nasal hairs appear to be singed. The following arterial blood gas data is noted while the patient is breathing air: pH 7.52 PaCO2 25 torr PaO2 102 torr HCO3- 23 mEq/L BE 0 mEq/L The therapist should FIRST recommend which of the following? A. check COHb B. initiate aerosolized racemic epinephrine C. hyperbaric oxygen therapy D. oral intubation

The correct answer is : D Explanation : A key concern for a patient who has had exposure to heated gases, such as might be seen in firefighters, is inflammation of the upper airway and potential complete obstruction. In this case, the presence of singed nasal hairs and inflammation visualize about the face and oropharynx is enough to suspect significant mucosal exposure to heated gases. Therefore, protection of the airway is appropriate through oral intubation.

Prior to performing an arterial puncture, a modified Allen's test is performed on the patient's right radial artery. When the ulnar artery occlusion is released, a pink color returns in 35 seconds. Based on this result the respiratory therapist should A. perform a femoral artery puncture B. perform a right brachial artery puncture C. proceed with the puncture of the right radial artery D. perform an Allen's test on the left radial artery

The correct answer is : D Explanation : A modified Allen's test is performed to ensure there is collateral circulation prior to performing an arterial puncture. The presence of collateral circulation, blood flow through the radial and ulnar arteries, helps to lower the risk of the puncture. This is done by including both the ulnar and radial arteries simultaneously. Once the hand becomes blanched (white and seemingly devoid of blood) the ulnar artery is released and the hand is observed to see if color returns in a timely manner. This should occur in a few seconds. In this case, color fails to return in a timely manner. This indicates that the right radial artery should NOT be used because the ulnar artery does not have good back-up circulation. The left ulnar artery should be checked for collateral circulation to determine if a puncture of the left radial artery is safe.

A patient with COPD is receiving volume-controlled ventilation with an I:E ratio of 1:2. A chest radiograph shows increased air-trapping. Which of the following will decrease air-trapping? A. increase minute ventilation B. decrease inspiratory flow C. decrease expiratory time D. decrease rate

The correct answer is : D Explanation : A patient receiving mechanical ventilation who does not have sufficient expiratory time may begin to trap gases in the lungs causing development of autoPEEP. The cause of this is either an inspiratory flow rate that is too slow or a rate that is too high. The primary solution is to increase flowrate. That option is not offered here. So, the next best solution would be to decrease mandatory rate.

While performing a routine check of a patient receiving mechanical ventilatory support, the respiratory therapist makes a small adjustment to the ET tube cuff pressure by injecting 2.0 cc of air into the cuff. Immediately after, the high-pressure alarm on the ventilator is activated. The therapist should

attempt to pass a suction catheter through the ET tube

During routine tracheostomy care for a patient with a two-week old trach, the patient coughs, resulting in the complete dislodgement of the tracheostomy tube, which falls on the floor. The respiratory therapist should NEXT A. cover the stoma with gauze. B. suction the patient through the stoma. C. clean the tube and re-insert. D. assess breathing.

The correct answer is : D Explanation : A patient who is breathing spontaneously through a tracheostomy tube that is suddenly ejected may be at risk for airway occlusion from soft tissues where a tracheostomy tube was placed. The first step following the extubation should be to monitor breathing and ensure adequate ventilation.

A 45-year-old patient has been receiving mechanical ventilation for 60 days. The respiratory therapist has been performing daily weaning attempts for the past week, but all attempts have failed. Which of the following should the therapist consider to facilitate weaning? A. pressure control B. small diameter ET tube C. 8 hr T-piece trials daily D. pressure support

The correct answer is : D Explanation : A patient who is having difficulty weaning may benefit from pressure support. Pressure support counteracts airway resistance and can decrease the labor of breathing during the weaning process.

Which of the following most effectively makes up the humidity deficit for a patient who is orally intubated and receiving mechanical ventilation? A. bubble humidifier B. HME C. centrifugal nebulizer D. heated humidity

The correct answer is : D Explanation : A patient who is intubated cannot provide 100% humidity to their lungs because the ET tube bypasses the natural humidification processes of the body. In such case a humidification device is needed to make up the entire humidity deficit. Of the devices listed in the options, only a heated humidifier can accomplish this. A bubble humidifier is used with a nasal cannula and an HME device does not provide sufficient humidity. A centrifugal nebulizer is not used in conjunction with a mechanical ventilator.

Which of the following may assist an ARDS patient receiving volume control ventilation and who has markedly decreased lung compliance? A. expiratory retard B. increased expiratory time C. increased inspiratory flow rate D. high frequency jet ventilation

The correct answer is : D Explanation : A patient with ARDS suffers from markedly decreased pulmonary compliance. Providing the patient positive pressure breathing at high peak airway pressures is risky as it may cause barotrauma and other problems. One of the primary strategies in the treatment of ARDS is to keep airway pressures as low as possible. One strategy for this is to use a high-frequency jet ventilator.

A 57-year-old cachectic male patient with known COPD is receiving oxygen by nasal cannula at 5 L/min. The patient is very drowsy with a respiratory rate of 8/min. Oxygen saturation is 100%. The therapist should A. place on a NRB mask. B. draw an arterial blood gas. C. obtain a CT scan. D. reduce supplemental oxygen delivery.

The correct answer is : D Explanation : A patient with COPD should not receive more than 1 to 2 L/min by nasal cannula or more than 28% oxygen. If excessive oxygen is administered the patient may experience a reduced ventilatory drive. Optimal oxygen saturation for a patient with COPD is between 92 and 94%.

A patient receiving volume-controlled ventilation in the assist/control mode has markedly decreased lung compliance. Which of the following would be most helpful at improving gas distribution in the lungs during the inspiratory phase of ventilation? A. decreased I:E ratio B. increased PEEP C. increased inspiratory flow rate D. inverse I:E ratio

The correct answer is : D Explanation : A patient with markedly decreased lung compliance, such as ARDS, may benefit from increasing inspiratory time and decreasing expiratory time. The increased inspiratory time will allow gases to penetrate the distal aspects of the lungs more fully and will increase gas exchange in general. When we increased inspiratory time such that it exceeds expiratory time, we have what is called an inverse I:E ratio.

A respiratory therapist notes tears in the rubber seals of a plethysmograph resulting in a failure to create an airtight environment. Which of the following test results would not be reliable? A. FVC B. FEV1 C. SVC D. TLC

The correct answer is : D Explanation : A plethysmograph (body box) is used to measure lung volumes such as residual volume, functional residual capacity, and total lung capacity. These volumes are measured indirectly.

A patient is suspected for carbon monoxide poisoning after being found lethargic in an enclosed space with a running motor vehicle. Which of the following would be helpful in further assessing the patient for this suspicion? A. Polargraphic oxygen analysis B. oxygen analysis with a Clark electrode C. PetCO2 monitoring D. multiple wave-length spectrophotometry

The correct answer is : D Explanation : A pulse oximetry cannot be used to measure true oxygen saturation as it can be erroneous in the presence of carbon monoxide poisoning. This is true because pulse oximetry provides single wavelength spectrophotometry. However, there are relatively new devices using multiple wavelength spectrophotometry that can detect carbon monoxide in the blood much the same way a pulse oximeter can detect oxygen. Because this method is quick and inexpensive it would be preferred over hemoximetry.

To determine the evenness of distribution of inhaled gases in an obstructive patient, the therapist should observe which of the following results? A. closing volume B. thoracic gas volume by body box C. phase I and II of an SBN2 test D. phase III of a nitrogen elimination test (SBN2)

The correct answer is : D Explanation : A single breath nitrogen elimination test (SBN2) I is useful in determining the evenness of gas distribution in the lungs. The results come in four phases as the patient exhales a single breath. Phase I is the exhalation of pure deadspace gas. Phase II consists of some deadspace and some alveolar gas. Phase III consists of pure alveolar gas and is the phase that indicates the evenness of distribution. Phase IV is called "closing volume".

The therapist is ordered to evaluate evenness of distribution in a patient with chronic obstructive pulmonary disease. Which of the following tests would be helpful for the requested evaluation? A. multiple breath DCO B. single-breath DLCO C. pulmonary angiogram D. nitrogen elimination (SBN2)

The correct answer is : D Explanation : A single-breath nitrogen elimination test (SBN2) is useful in determining the evenness of gas distribution in the lungs. The result comes in four phases as the patient exhales a single breath. Phase I is the exhalation of pure deadspace gas. Phase II consists of some deadspace and some alveolar gas. Phase III consists of pure alveolar gas and is the phase that indicates the evenness of distribution. Phase IV is called "closing volume".

You are asked to instruct a patient being discharged home to use inhalers instead of small volume nebulizers. The patient is 8-years-old and was generally non-compliant with aerosol therapy during the hospitalization. Which of the following should NOT be included in the teaching plan? A. instructions for the parents/legal guardians B. MDI self-administration with a spacer C. order and timing of ordered medications D. MDI self-administration without a spacer

The correct answer is : D Explanation : A spacer is indicated for a patient this age in order to reduce the need to rely upon coordination with an MDI, along with improving medication particle delivery. Including the family and taking multiple medications in their proper order are necessary points for teaching.

A capnographic tracing shows tidal waveforms within normal, expected ranges for the first hour of surgery. Suddenly, the waveform becomes flat and the value drops to nearly zero. This would be caused by A. low cardiac output. B. hypovolemia. C. apnea. D. a disconnection of the capnograph probe from the circuit.

The correct answer is : D Explanation : A sudden drop in CO2 would be caused by a mechanical malfunction, or disconnection from the patient. Too often we become focused on monitors and graphics, and lose our focus on the patient and basic equipment connections.

Which of the following conditions would benefit most from a thoracentesis? A. complete opacification of the right lung B. pericardial contusion C. atelectasis D. small pneumothorax

The correct answer is : D Explanation : A thoracentesis is a procedure that removes air or fluid from the pleural space. This would be appropriate with a small pneumothorax. A large pneumothorax, however, would require chest tubes.

The wick humidifier is A. unable to accomplish 100% of the body's humidification requirements. B. not indicated for thick secretions. C. designed for cool aerosol only. D. optimal for use on a pediatric patient at home with a tracheostomy.

The correct answer is : D Explanation : A wick humidifier is electronically powered and can provide sufficient heat and humidification for any patient including pediatric patients with a tracheostomy at home.

The respiratory therapist working in a clinic receives a call from a patient who has asthma and is only able to achieve 60% of their normal baseline peak flow measurement. The therapist should advise the patient to A. take a rescue medication every 15 minutes for 2 hours, recheck flows B. check flows again in one hour, call if less than 50%of expected C. take a rescue medication, check flows in one hour D. call the doctor immediately

The correct answer is : D Explanation : According to national asthma guidelines a peak flow measurement of 60% of baseline is a direct indication to seek additional medical help.

A patient with asthma monitors their peak flow in the morning and documents that peak flow is 40% of his usual baseline. Based on the NAEP and the patient's asthma action plan, the patient should A. report to the emergency room B. take a short-acting bronchodilator, check peak flow in 1 hour C. take a corticosteroid MDI D. contact the patient's physician

The correct answer is : D Explanation : According to the national asthma guidelines, a self monitored peak flow of 40% of baseline is an indication for the patient to contact their physician

A 49-year-old 70-kg (155-lb) adult female is receiving mechanical ventilatory support. Settings are as follows: Mode Assist/control Mandatory rate 12 Total rate 18 Tidal volume 350 mL FIO2 0.5 PEEP 5 cm H2O The patient is appears to be anxious. SpO2 is 84%. The respiratory therapist will recommend which of the following? A. decrease mandatory rate B. increase inspiratory flow rate C. decrease deadspace D. increase tidal volume

The correct answer is : D Explanation : All the answers in this question are suggestive of a need to change the ventilator settings. However, there is no arterial blood gas data available on which a change is normally based. When this happens, look closer at the current ventilator settings. In this case the patient weighs 70 kg and therefore has a minimum tidal volume of 420 mL (6 mL/kg). The set tidal volume is 350 mL. This must be corrected immediately to prevent respiratory acidosis.

A respiratory therapist observes the following data on a post bariatric surgery patient who has been receiving VC SIMV ventilation for 8 hours with the following settings and clinical and laboratory parameters: Spontaneous VT 400 mL MIP -35 cm H2O RR (spont) 16/min SpO2 98% FIO2 0.40 Mandatory rate 8/min VT (set) 500 mL PEEP 5 cm H2O WBC 18,000 cu mm K+ 4.1 mEq/L Cl- 101 mEq/L Na+ 135 mEq/L RSBI 94 The patient is alert and appropriately responsive to questions. Which of the following should the therapist recommend? A. spontaneous breathing trials B. PS ventilation at 5 cm H2O with CPAP of 5 cm H2O C. SIMV, rate 6, reduce rate by 2 every 4 hours until at 4/min D. avoid ventilator liberation at this time

The correct answer is : D Explanation : Although most all of the data listed here is consistent with readiness to wean from the mechanical ventilator, the presence of an elevated white blood cell count suggests the possibility of a bacterial infection. In such a case, ventilator liberation should be avoided until the underlying infection can be resolved.

With the addition of a spacer, the patient self-administering MDIs A. does not have to hold their breath after each maneuver. B. may take a smaller dose than ordered. C. should take a slower, deeper breath. D. does not have to closely correlate actuation and inhalation.

The correct answer is : D Explanation : An aerosol holding chamber, commonly known as a spacer, not only improves medication delivery when added to a metered-dose inhaler, but also reduces the significant amount of coordination required to properly self-administer an MDI.

The respiratory therapist notes the following results of an arterial blood gas: pH 7.40 PaCO2 41 torr PaO2 46 torr HCO3 -24 mEq/L BE 0 mEq/L The patient reports no symptoms. Pulse rate, blood pressure and color are within normal limits. Which of the following can the therapist appropriately conclude? A. hemoglobin level is low B. the patient has sickle cell anemia C. the analysis did not correct for temperature D. results are consistent with a venous blood sample

The correct answer is : D Explanation : An asymptomatic patient with a PaO2 of 46 mmHg is not normal. The arterial blood gas data should be questioned for its validity. Of the answers offered, the most likely cause of these results is an accidental analysis of venous blood rather than arterial blood. These values are consistent with a venous blood gas sample.

A patient is receiving volume-controlled ventilation. The ventilator graphic screen shows flow graphs that do not return to baseline before inspiration starts. This will cause the development of

autoPEEP

A 7-year-old patient is receiving mechanical ventilation with a PB 840 volume ventilator with an adult circuit. The end-tidal CO2 monitor is indicating a PetCO2 of 56 mmHg. Which of the following is most appropriate? A. remove 50 mL of deadspace B. switch to a pediatric circuit C. add 50 mL of deadspace D. increase mandatory rate

The correct answer is : D Explanation : An end-tidal CO2 of 56 mmHg approximates an arterial CO2 of about 66 mm Hg. This is a definite indication of hypoventilation and would best be remedied by increasing minute ventilation. This may be done by increasing tidal volume or increasing rate. Adding dead space would increase end-tidal and arterial CO2 even further. Removing dead space, while a step in the right direction, isn't a sufficient response. Changing to a pediatric circuit is not helpful.

Which of the following patients would benefit most from an inverse I:E ratio ventilation? A. COPD B. chronic bronchitis C. kyphoscoliosis D. ARDS

The correct answer is : D Explanation : An inverse I:E ratio is a term that indicates a longer inspiratory time than expiratory time. The patient with adult respiratory distress syndrom is an example of a patient who can benefit from an inverse I:E ratio. Their lungs are noncompliant and therefore require more inspiratory time to allow for better gas distribution.

A 62-kg (136 lb) male is receiving volume-controlled ventilation on the following settings: Mode SIMV Mandatory rate 10 Total rate 16 Tidal volume 400 mL Pressure support 6 cm H2O VT (spontaneous) 400 mL PEEP 10 cm H2O Arterial blood gas analysis reveals pH 7.29 PaCO2 53 torr PaO2 83 torr HCO3- 25 mEq/L BE -1 mEq/L Based on this data, the first recommendation of the respiratory therapist should be to A. increase tidal volume to 550 mL B. remove deadspace C. increase pressure support to10 cm H2O D. increase mandatory rate to 14

The correct answer is : D Explanation : Arterial blood gas analysis reveals hypoventilation but normal oxygenation. This may be resolved by increasing minute ventilation, which can be done by either increasing tidal volume or mandatory rate. Because CO2 is out of range by more than 4 mmHg the best option would be to increase mandatory rate to 14.

A male patient who weighs 72-kg (170-lb) and is 5-ft, 4-in tall is receiving mechanical ventilator support on the following settings with the following corresponding blood gas values: Mode SIMV Mandatory rate 14 VT 400 mL FIO2 0.50 PEEP 5 cmH2O pH 7.32 PaCO2 47 mm Hg PaO2 70 mm Hg HCO3- 25 mEq/L BE +1 mEq/L The respiratory therapist should recommend: A. increase PEEP to 8 cmH2O B. increase rate to 16 C. increase FIO2 to 0.6 D. increase tidal volume to 500 mL

The correct answer is : D Explanation : Arterial blood gases show mild hypoventilation and hypoxemia. Although increasing rate would correct ventilation, increasing tidal volume is most appropriate. Additionally, the CO2 is only off by a very small amount and a change in tidal volume is the better choice when making small changes in CO2 because it results in a smaller increase in mean airway pressure.

A 75-kg (165-lb) male is receiving mechanical ventilation by a volume-controlled ventilator in the assist/control mode on the following settings: Mandatory rate 12/min VT 500 mL FIO2 0.6 PEEP 8 cm H2O Arterial blood gas results: pH 7.30 PaCO2 52 mm Hg PO2 65 mm Hg HCO3- 27 mEq/L BE +3 mEq/L Which of the following represents the most appropriate action? A. increase tidal volume to 600 mL B. increase PEEP to 10 cmH2O C. increase FIO2 to 1.0 D. increase rate to 14

The correct answer is : D Explanation : Arterial blood gases show the patient is not ventilating and is under oxygenating. Because ventilation should be corrected first it is appropriate to either increase the tidal volume or increase the rate. In this case the PaCO2 is off by more than 4 mmHg and therefore should be addressed only with an increase in rate.

A 75-kg (165-lb) male is receiving mechanical ventilation by a volume-controlled ventilator in the assist/control mode on the following settings: Mandatory rate 14 VT 550 mL FIO2 0.4 PEEP 5 cm H2O Arterial blood gas results: pH 7.32 PaCO2 47 mm Hg PaO2 101 mm Hg HCO3- 25 mEq/L BE +1 mEq/L Which of the following represents the most appropriate action? A. increase VT to 700 mL B. decrease FIO2 to 0.3 C. increase rate to 18 D. remove deadspace

The correct answer is : D Explanation : Arterial blood gases show the patient is not ventilating and is under oxygenating. Because ventilation should be corrected first it is appropriate to either increase the tidal volume or increase the rate. In this case the PaCO2 is off by only 2 mmHg. A change in rate is too drastic. A change in tidal volume or a decrease in deadspace should be examined. The answers offered show an increase in tidal volume to 700 mL. This would exceed the maximum tidal volume range as determined by the patient's ideal body weight. Removing deadspace is the best option.

A 63-year-old female patient with myasthenia gravis and ventilator acquired pneumonia is currently receiving volume-controlled ventilation in the assist/control mode, FIO2 0.70, mandatory rate of 16/min, PEEP of 25 cm H2O. Recent arterial blood gas analysis shows the following: pH 7.35 PaCO2 45 torr PaO2 58 torr HCO3- 27 mEq/L BE +2 mEq/L Additional data shows CVP 5 torr PAP 14 torr PCWP 8 torr C.O. 5.5 L/min The respiratory therapist should address the patient's condition by A. Increasing FIO2 to 1.0 B. Perform an optimal PEEP study C. Increasing FIO2 to 0.80 D. Increasing PEEP to 28 cmH2O

The correct answer is : D Explanation : Arterial blood gases show the patient is ventilating adequately but is hypoxic. This may be remedied by either increasing FIO2 or increasing PEEP. Because the patient's FIO2 is is already at or above 60%, the next logical step is to increase PEEP. However, this may only be done if the patient demonstrates hemodynamic stability as manifested by normal cardiac output. Normal cardiac output is 4 to 8 L/min.

During a routine check of a patient on mechanical ventilation, the respiratory therapist palpates less chest rise on the left compared to the right during inspiration. The therapist should first do which of the following? A. Advance the endotracheal tube and recheck chest rise B. Prepare for needle decompression in the left chest C. Check the integrity of the ET tube cuff D. Auscultate breath sounds bilaterally for equal air movement

The correct answer is : D Explanation : Asymmetrical chest movement, or chest rise during mechanical ventilation, is an indication that the endotracheal tube may be improperly placed. The first, more immediate action that should be taken is to further assess the problem by auscultating the chest. If breath sounds are absent on one side, it is appropriate to withdraw the endotracheal tube until breath sounds can be heard bilaterally. After doing so, a chest x-ray should be ordered to confirm proper placement of the endotracheal tube.

The build-up of positive end-expiratory pressures in the lungs as a result of insufficient time to exhale is known as

autoPEEP

A patient is receiving volume-controlled ventilation in the assist/control mode. The respiratory therapist discovers that autoPEEP is increasing and that the patient is not completing exhalation before the inspiratory phase starts. Which of the following adjustments to the ventilator is most appropriate? A. Add expiratory retard B. Decrease mechanical deadspace C. Decrease the mandatory rate D. Increase flow

The correct answer is : D Explanation : AutoPEEP occurs when the patient fails to exhale all gases before the next mechanical breath begins. This promotes air-trapping and will expand the alveoli and result in many other negative effects that are associated with positive pressure ventilation. AutoPEEP can be corrected by allowing more time for exhalation. This can be accomplished by increasing inspiratory flow and decreasing inspiratory time. Additionally, AutoPEEP may be decreased by decreasing mandatory rate, but this is not a appropriate as increasing the flow because it will result in a shift in minute ventilation and may adjust PaCO2 negatively.

The parents of a 10-day old infant are reporting they feel their baby stops breathing sometimes when sleeping. The couple have lost a previous child to sudden infant death syndrome. Which of the following should the respiratory therapist recommend? A. non-invasive ventilatory support at night B. telling the parents that some apnea is normal for a 10-day-old infant C. caffeine injections daily for 20 days, then reassess the situation D. a home apnea monitor

The correct answer is : D Explanation : Based on the complaints of the parents, indicating the patient may be experiencing apnea, in conjunction with the history of a previous child lost to SIDS, placing a home apnea monitor in infants room may be helpful. An apnea monitor will both provide information to healthcare workers for analysis and create alarms for parents who may need to intervene to save their child's life in the case of significant apnea.

A patient receiving volume-controlled mechanical ventilation has the following values obtained from a swan-ganz catheter: PAP 21 mm Hg CVP 8 mm Hg PCWP 12 mm Hg CI 1.8 L/min/m2 Which of the following mostly likely represents the patient's condition? A. cor pulmonale B. pulmonary embolism C. increase pulmonary vascular resistance D. mitral valve stenosis

The correct answer is : D Explanation : Because both CVP and PAP are elevated, a right heart condition is ruled out and Cor pulmonale is not the answer. To determine if the problem is in the lungs we must look at PAP and PCWP. In this case, both are elevated, indicating no problem in the lungs, or pulmonary vasculature. This rules out a pulmonary embolism and increased PVR as sources of the problem. The only option left is mitral valve stenosis which is a left heart problem. A problem in the left heart is confirmed by an elevated PCWP with a reduced cardiac output. In this case, cardiac output is not given but cardiac index may be doubled and used as an estimated cardiac output. That would indicate a cardiac output of 3.6 L, which is low.

A respiratory therapist is suctioning a patient who is receiving positive pressure ventilation through a non-fenestrated tracheostomy tube when it is discovered that the suction catheter will not pass beyond the end of the tracheostomy tube. The patient is cyanotic and in obvious respiratory distress. After the therapist evacuates the cuff, the catheter passes easily. The therapist should first do which of the following? A. inflate the cuff and resume positive pressure ventilation B. remove the inner cannula and cap the tube C. increase the pressure limit on the ventilator, monitor exhaled volumes D. extubate the patient

The correct answer is : D Explanation : Because the catheter passes easily after the tracheostomy tube cuff is deflated, the most likely problem is that the tracheostomy tube cuff has become damaged and has herniated over the end of the tracheostomy tube. This means the tracheostomy tube should be changed to one that is not damaged. This starts with extubating the patient and providing ventilation and oxygenation, and generally ensuring the stability of the patient.

A patient has been ordered to receive Beclamethasone (Beclovent) by metered dose inhaler (MDI) once a day. Which of the following should be included in the instruction of the patient? A. follow treatment with Nystatin via MDI B. perform a breath hold for 10 seconds with each treatment C. avoid bronchodilator medication for 8 hours D. rinse mouth after each treatment

The correct answer is : D Explanation : Beclomethasone is considered to be a corticosteroid. Inhaled corticosteroids can cause an infection in the mouth known as candidiasis or thrush mouth. To prevent this, the patient should rinse his or her mouth out after taking inhaled corticosteroids of any type. If an infection does develope, the problem may be treated with Nystatin or Amphotericin

A 70-year-old male presents in the emergency department with shortness of breath and a non-productive cough. Auscultation of the chest reveals bilateral expiratory wheezes. Which of the following intervention would be most helpful to the patient? A. comolyn sodium (Intal) B. osmitrol (Mannitol) C. dexamethasone (Decadron) D. ipratropium bromide (Atrovent)

The correct answer is : D Explanation : Bilateral expiratory wheezing, or bronchial wheezing of any type, is best remedied by the administration of bronchodilator medication. Of the options listed, only ipratropium bromide, or Atrovent, is considered to be a bronchodilator. This type of bronchodilator, together with such drugs as albuterol, is considered to be a rescue medication. Cromolyn sodium is considered a long-term solution to prevent the release of histamine which will help prevent bronchoconstriction. The other drugs listed are not noted for achieving short-term bronchodilation.

A blood gas analyzer indicates an arterial blood sample taken from a normal healthy adult has a PO2 that is not in the reportable range. The therapist should A. replace the Sanz electrode B. perform a one-point calibration C. report the problem to the medical director D. perform a two-point calibration

The correct answer is : D Explanation : Blood gas machines have what is called a reportable range. This means there is a limit to how high and how low a PO2, pH, and PCO2 value the machine can detect. If the blood gas result is out of the reportable range for that machine, usually the machine will indicate an alarm and will not report because the results would not be accurate. To correct this problem the best solution would be to do a two-point calibration. A two-point calibration is most appropriate because a range has both an upper and lower limit, or two points.

Which of the following may occur as a result of stimulation of the vagal reflex during a suction procedure? A. first degree heart block B. PVCs C. increase in heart rate D. decrease in blood pressure

The correct answer is : D Explanation : Suctioning can cause a decrease in heart rate and a decrease in blood pressure. This natural, physiological response is associated with stimulation of the vagal nerve during suctioning. This is also called the vagal reflex.

A patient in the intensive care unit is receiving mechanical ventilation on the following settings: Mode Assist/control VT 450 mL Mandatory rate 18 FIO2 0.6 PEEP 8 cm H2O Other clinical data includes: CVP 12 mm Hg PAP 15 mm Hg PCWP 8 mm Hg C.O. 4.8 L/min Which of the following most likely represents the patient's condition? A. congestive heart failure B. left-sided heart failure C. pulmonary embolus D. cor pulmonale

The correct answer is : D Explanation : CVP is high. PAP is normal. This indicates a problem in the right heart. PCWP is normal and cardiac output is normal. This data suggests that the left heart and the lungs are okay. In the options offered only cor pulmonale is a right heart condition. Congestive heart failure is associated with a left heart problem and pulmonary embolism with a lung problem.

Which of the following units is associated with cardiac index? A. dynes/sec/cm2 B. L/min C. L/min/m3 D. L/min/m2

The correct answer is : D Explanation : Cardiac index is cardiac output divided by body surface area. Or if BSA is unknown, the cardiac index is estimated by dividing cardiac output by two. The unit of measure is L/min/m2.

Using the formula for calculation of cardiac index, how would a morbidly obese patient's C.I. compare to that of a person at ideal body weight? A. cardiac index would be unpredictable B. cardiac index would remained unchanged C. cardiac index would be higher D. cardiac index would be lower

The correct answer is : D Explanation : Cardiac index is determined by dividing cardiac output by body surface area. As body surface area is increased (from obesity) the cardiac index calculation will be less than a patient at ideal body weight.

What radiographic finding would be most likely associated with a patient who is experiencing active tuberculosis? A. Atelectasis in the upper lobes B. Cavitation in a lower lobe C. Mass in the left middle lobe D. Pleural cavitation in the upper lobes

The correct answer is : D Explanation : Cavitations in the upper lobes are generally associated with tuberculosis, and an X-ray with this finding should assure the respiratory therapist of the diagnosis. Plate-like or patchy infiltrates on X-ray are seen with atelectasis. A mass, which is solid, (opposite of a cavitation) is often associated with cancer.

A patient with known COPD has the following pre and post bronchodilator studies. Based on this information, what bronchodilator should the physician prescribe? Pre-Bronchodilator Post albuterol Post Combivent SVC (L) 2.7 2.7 2.8 FVC (L) 2.5 2.6 2.8 FEV1 (L) 1.5 2.0 2.0 FEF25-75 (% pred) 65% 80% 85% A. None B. Atrovent 0.5 mg C. Albuterol 2.5 mg D. Combivent

The correct answer is : D Explanation : Combivent is the best choice because improvement is seen across the board. Vital capacity improves, FEV1 improves, and FEF 25-75 improves significantly. Albuterol is helpful, but a greater impact is seen with Combivent.

A 19-year old cystic fibrosis patient presents to the emergency room with elevated leukocytes, fever, thick and sticky sputum, and shortness of breath. Which of the following is appropriate for the patient's care plan? A. IPPB B. Priscoline, IV C. NIPPV D. Pulmozyme (Dornase Alfa)

The correct answer is : D Explanation : Cystic fibrosis is considered a chronic COPD disease and therefore the patient could benefit from continuous oxygen and dornase alpha, which will help to thin secretions. Postural drainage is also a primary method of treatment. Because the patient has an elevated leukocyte count, which is associated with an infection, antibiotic therapy is also appropriate.

Which of the following laboratory results should a respiratory therapist look at to evaluate a patient for a suspected diagnosis of cystic fibrosis? A. Elisa test B. acid-fast sputum C. Tensilon challenge D. sweat chloride level

The correct answer is : D Explanation : Cystic fibrosis may be diagnosed through a laboratory test called a "sweat chloride level".

Which of the following will result in a decrease in mean airway pressure for a patient on a mechanical ventilator in the assist/control mode? A. use of expiratory retard B. institution of a 1.0 sec inspiratory plateau C. decreasing inspiratory flow D. decrease in inspiratory time

The correct answer is : D Explanation : Decreasing inspiratory time will lower the amount of time a patient is exposed to positive pressure and will therefore result in a decrease in mean airway pressure. Use of expiratory retard, increasing inspiratory flow, and institution of an inspiratory plateau will all lead to increased mean airway pressure.

A morbidly obese female patient with a large neck circumference is trached with a 7.5 mm tracheostomy tube. The tube frequently becomes dislodged with her cough. What should be done to resolve the problem? A. Increase to a size 8.0 mm tracheostomy tube. B. Tie the tracheostomy tube securely. C. Increase to a size 10.0 mm tracheostomy tube. D. Replace the tube with a longer tube.

The correct answer is : D Explanation : Due to the increased neck circumference of a morbidly obese patient, special tracheostomy tubes that have long proximal extensions allow for the increased distance from the interior tracheal wall to the opening of the stoma at the skin.

A patient in the cardiac intensive care unit has a Swan-Ganz pulmonary artery catheter in place. The following data is available: CVP 1 torr mPAP 10 torr PCWP 4 torr C.I. 1.7 L/min/m2 The therapist can conclude the presence of which of the following? A. left sided heart failure B. increased right atrial filling pressure C. hypervolemia D. hypovolemia

The correct answer is : D Explanation : Examination of the hemodynamic values in this case show that all values are low. When all hemodynamic values are low, dehydration is present. Thus, hypovolemia is the best interpretation of this data.

A postoperative patient is not progressing with sustained maximal inspiration therapy and refuses to cough because of pain along the incision site when doing so. The respiratory therapist should instruct the patient to A. avoid coughing until pain subsides B. pause at the top of each inspiration C. attempt coughing 1 hour after pain medication has been administered D. press a pillow against the incision site when coughing

The correct answer is : D Explanation : Explosive coughing by a postoperative patient can inadvertently open incision sites. To prevent this, splinting the site with a pillow is an appropriate option.

Which of the following can be done to increase the effectiveness of endotracheal suctioning? A. apply intermittent suction during removal of the catheter B. decrease suction catheter length C. use a coude-tipped catheter D. increase suction catheter size

The correct answer is : D Explanation : Suctioning effectiveness may be increased by increasing suction catheter size or diameter, by increasing suction pressure, or by increasing suction duration. Of the options given, increasing catheter size and increasing suction pressure are appropriate.

Immediately after placing a sterile tracheostomy tube in a patient who has maintained a stoma for 4 years, the patient coughs vigorously and expectorates the tube out onto the sterile napkin resting on the chest of the patient. The therapist should A. obtain a new, sterile trach tube B. immediately insert an obdurator into the stoma C. re-insert the expectorated trach tube D. obtain a larger tracheostomy tube

The correct answer is : D Explanation : For a patient who has maintained an open stoma for several years and easily expectorates a new tracheostomy tube, the patient is most likely in need of a larger airway.

If the respiratory therapist notices the reservoir bag on a nonrebreather mask is failing to partially collapse with each breath, the therapist should A. increase oxygen flow to the reservoir B. obtain a new nonrebreather mask C. remove the one-way valve disk D. tighten the elastic straps on the mask

The correct answer is : D Explanation : For a patient who is breathing oxygen through a non-rebreather mask, failing to cause a partial collapse of the reservoir with each breath is an indication that either the mask is not tight enough or that the flow to the mask is excessive. It is most likely that the mask requires tightening, which can be done by pulling on the elastic straps.

A patient admitted for acute asthma is being discharged to home. The respiratory therapist notices an order for oral theophylline. Which of the following additional orders might the therapist expect? A. point-of-care blood glucose monitoring B. Dilantin for treatment of seizure activity C. monthly measurement of DLco D. periodic theophylline blood level check

The correct answer is : D Explanation : For a patient who is taking oral theophylline, which is a xanthine medication, blood levels must be monitored to ensure adequate bronchodilation. Theophylline levels should be maintained between 10 and 20 µg. A theophylline level higher than that range can produce such negative clinical outcomes as seizures and death. A theophylline level lower than the range is simply said to be sub-therapeutic and does not provide long-term bronchodilation control.

Heel sticks are performed on the lateral area of a newborn's foot in order to A. reduce the pain for the patient. B. acquire better capillary blood flow. C. facilitate the procedure for the practitioner. D. prevent future ambulation problems.

The correct answer is : D Explanation : Heel sticks should not be performed on a neonate in an area of the foot that they will walk on in the future as they grow. Damage to this area of the foot can result in difficulty with future ambulation. Therefore, heel sticks are performed on the lateral area of the bottom of a newborn's foot.

What is the optimal location for capillary samples drawn from a neonate? A. capillary tubes should be held at a 50-degree angle B. samples are primarily venous blood C. temperature is not relevant to the accuracy of the results D. lateral section of the heel

The correct answer is : D Explanation : Heel sticks should not be performed on a neonate in an area of their foot that they will walk on in the future as they grow. Damage to this area of the foot can result in difficulty with future ambulation. Therefore, heel sticks are performed on the lateral area.

Which of the following sounds assessed by a respiratory therapist is potentially the most serious if heard on a pediatric patient in the emergency room? A. barking cough, otherwise clear B. rhonchi clearing with cough C. high-pitched continuous on exhalation over the larynx D. high-pitched continuous during inhalation over the larynx

The correct answer is : D Explanation : High-pitched continuous sounds in the upper airway, also known as stridor, may be life-threatening.

What would most likely be indicated by an elevated CVP? A. pulmonary embolism B. increased pulmonary vascular resistance C. left heart failure D. fluid overload

The correct answer is : D Explanation : Hypervolemia (fluid overload) is shown hemodynamically by an increase in all hemodynamic values including CVP, PAP, PCWP, and cardiac output. Among these values CVP is the first and most significant indicator when fluid levels in the body are high or low. Remember, CVP may be known by other names such as, right atrial pressure, right side preload, right ventricular filling pressure, and right ventricular end-diastolic pressure.

A decreasing CvO2 while CaO2 remains constant is most closely associated with A. increasing PVR B. decreasing SVR C. decreasing oxygen consumption D. increasing tissue oxygen consumption

The correct answer is : D Explanation : If CvO2 is decreasing while CaO2 remains the same, there is a widening between the arterial and venous oxygen content difference C(a-v)O2. A bigger gap between these numbers, or an increase in the C(a-v)O2, is associated with decreasing tissue oxygen consumption which is also associated with a decrease in cardiac output.

A respiratory therapist is unable to pass a 14 Fr suction catheter through a 7.0-mm ID nasotracheal tube. Lubrication of the catheter and minor repositioning does not seem to help. The therapist should do which of the following? A. deflate and reinflate the tube cuff with the proper pressure B. perform suctioning with a bronchoscope C. switch to an endotracheal tube D. utilize and 12 Fr catheter

The correct answer is : D Explanation : If a suction catheter has difficulty passing through an endotracheal tube, it could be that the suction catheter is experiencing too much friction as it is inserted into the airway. This can often be remedied by decreasing size or diameter of the suction catheter.

An intubated patient has a size 7.5 mm endotracheal tube in place. Diffuse rhonchi are auscultated. A respiratory therapist is using a 12 Fr suction catheter set to a pressure of -100 cm H2O, attempting to suction for 15 seconds. Only scant secretions are suctioned, and breath sounds do not improve after the procedure. The respiratory therapist should increase the A. catheter size to an 18 Fr. B. catheter size to a 14 Fr. C. suction duration. D. suction pressure.

The correct answer is : D Explanation : If suctioning is not adequate, correct the problem in this order: 1. check connections, change collection bottle if full 2. ensure suction pressure is in the right range 3. increase to the maximum size catheter within range 4. increase pressure within range 5. increase suction time

Which of the following patients does NOT require additional help to make up the humidity deficit during ventilation? A. A spontaneously breathing patient who is intubated nasally B. A patient with a tracheostomy C. A patient who is orally intubated with an endotracheal tube D. A patient using an oral pharyngeal airway

The correct answer is : D Explanation : Patients who are orally or nasally intubated or who have a tracheostomy tube in place, need additional assistance for humidification because the artificial airways are bypassing the natural humidification processes of the body. An oral pharyngeal airway does not accomplish this.

Which of the following cannot be measured or evaluated in a comatose patient? A. tidal volume B. objective information C. pupillary response D. symptoms

The correct answer is : D Explanation : If the patient is comatose, they cannot cooperate with procedures or follow commands. To answer this question one must pick the option that does not require compliance with verbal commands. Symptoms, coming from the root word sympathy, require the patient to tell you something. From the list only a tidal volume can be measured without help from the patient. But, the question is asking which of the following CANNOT be measured in a comatose patient. Therefore symptoms, vital capacity, maximum inspiratory pressure are all appropriate answers.

A respiratory therapist begins manual resuscitation on a patient who is found apneic for several minutes. Capnography with an infrared device is instituted. The therapist will expect to see which of the following initial capnographic trends? A. gradual decrease in end-tidal CO2 B. steady hypercapnia C. steady hypocapnia D. rise in end-tidal CO2

The correct answer is : D Explanation : If the patient is found not to be breathing for several minutes but then receives manual resuscitation, the initial end-tidal CO2 reading will be low but will be followed by a steady rise in the end-tidal CO2. This is because once ventilation begins CO2 begins crossing the alveolar capillary membrane and filling up the alveoli, and ulimately shows up on the capnograph.

An 85 kg (187 lb) postoperative hernia repair patient is receiving mechanical ventilatory support on the following settings: Mode SIMV Mandatory rate 12 Total rate 16 FIO2 0.40 Tidal volume (VT) 475 mL Inspiratory flow 50 L/min Corresponding blood gases show: pH 7.25 PaCO2 55 torr PaO2 69 torr HCO3- 26 mEq/L BE +1 mEq/L Which of the following should the respiratory therapist increase? A. FIO2 B. inspiratory flow C. mandatory rate D. tidal volume

The correct answer is : D Explanation : In addition to this patient being acidodic from hypoventilation, the set tidal volume is too low for the patient's height and weight. When the patient's height is not given we must assume that the reported weight is the patient's ideal body weight. This patient weighs 85-kg which means the minimum tidal volume is 510 mL. When CO2 must be corrected by this amount, an increase in rate is indicated. However, because set tidal volume is erroneously low, this should be corrected first.

Which of the following medication would be most helpful to a 35 week gestational-age infant who is experiencing periods of apnea exceeding 60 seconds? A. Progesterone B. Lidocaine C. Dopram (doxapram) D. Aminophylline

The correct answer is : D Explanation : In adults, Aminophylline is a long-term bronchodilator. It may be used on infants, however, as a ventilatory stimulant. Apnea among infants greater than 60 seconds is an indication for this medication.

Which of following parameters indicate a patient is ready to wean? A. PEEP of 10 cm H2O B. MIP of -18 cm H2O C. A-aDO2 of 410 mm Hg D. pulmonary shunt of 16%

The correct answer is : D Explanation : In order for a patient to be considered for ventilator weaning pulmonary shunting must be less than 20%, spontaneous respiratory rate must be less than 20 per minute, tidal volume must be at least 5 mL per kilogram or above 350 mL, and MIP must be greater than -28 cmH2O. The question is asking which of those parameters indicate the patient is ready to wean. All of them meet weaning criteria.

If a pressure-cycled mechanical ventilator had a leak in the circuit, what would the respiratory therapist expect to see? A. The pressure monometer would have a negative deflection during each breath B. Peak pressures would decrease C. The flowrate would automatically increase to compensate D. The mechanical breaths would not cycle off as expected

The correct answer is : D Explanation : In pressure-cycled mechanical ventilation the positive pressure breaths stop once the preset pressure is reached. If the circuit has a leak the preset pressure cannot be reached, and therefore the machine will fail to cycle into exhalation.

A routine screening blood gas performed on an asymptomatic patient prior to surgery shows the following: pH 7.38 PaCO2 42 mm Hg PaO2 80 mm Hg HCO3- 26 mEq/L SaO2 96% COHb 24% What should the respiratory therapist do? A. Place the patient on a NRB mask B. Report the findings and emphasize that the patient is likely a smoker C. Evaluate saturation using a single wave-length spectrophotometer D. Run quality control material and evaluate the accuracy of the Co-oximeter

The correct answer is : D Explanation : In this case the patient is presented as asymptomatic. A patient with a carboxyhemoglobin level of 24% would feel dyspnea and present with tachypnea. Therefore, the respiratory therapist should evaluate the accuracy of the laboratory results, specifically the carboxyhemoglobin (COHb). When checking accuracy of lab results, one would use quality control measures.

A respiratory therapist is unable to suction thick secretions from an adult patient who requires frequent NT suctioning through a nasal trumpet. The regulator is set to -120 mm Hg. The catheter is 14-Fr. What should the respiratory therapist recommend? A. Remove the nasal trumpet B. Increase the suction pressure C. Instill normal saline down the suction catheter prior to applying suction D. Increase the suction duration

The correct answer is : D Explanation : In this example the suction pressure cannot be increased because the maximum parameter has already been reached. The catheter size should not be increased because it will then exceed half of the diameter of the nasal trumpet. The only viable option is to suction for a longer period of time.

A patient with myasthenia gravis presents to the clinic with a fever, suspected pneumonia secondary to an infiltrate seen on a chest radiograph, and the following ABG results: pH 7.33 PaCO2 47 torr PaO2 85 torr HCO3- 26 mEq/L FIO2 0.21 Based upon this information, the patient needs A. pulmonary function testing. B. oxygen at 2 L/min nasal cannula. C. intubation. D. antibiotic therapy.

The correct answer is : D Explanation : In this example, we see a problem with ventilation as the PaCO2 is 47 torr. However, there is no option that will resolve ventilation, therefore we must look to the scenario for other problems. There is an underlying problem of infection for which antibiotic therapy is indicated.

A patient is experiencing atrial arrhythmias and is complaining of light-headedness and nausea. Which of the following should the respiratory therapist recommend to help remedy the problem? A. unsynchronized defibrillation B. administration of Lidocaine C. atropine sulfate D. cardioversion

The correct answer is : D Explanation : Lightheadedness and nausea are associated with atrial cardiac arrhythmias. These rhythms are usually not deadly but should be addressed as quickly as possible. Cardioversion is a form of defibrillation suitable to treat atrial arrhythmias. Cardioversion is done with at low wattage an the defibrillator set to ACTIVE synchronization.

A patient has a pulmonary artery catheter in place. The following data is obtained: PAP 10 torr CVP 1 torr PCWP 4 torr C.I. 1.8 L/min/m2 Which of the following is an accurate interpretation of this data? A. increased pulmonary vascular resistance B. increased left-sided preload C. hypervolemia D. decreased right atrial filling pressure

The correct answer is : D Explanation : In this problem all the hemodynamic values are low. This is indicative of hypovolemia. However, that is not the question. Of the options given, the only one that is true is that there is decreased right atrial filling pressure. Those specific words are nothing more than a way to say the patient has low CVP. The terminology is interchangeable.

A patient with a history of left heart failure has the following input/output history Yesterday Today IN 1600 mL 1900 mL OUT 2000 mL 2300 mL This data is most consistent with A. increased PAP B. increase in diastole of the PAP pressure C. increased right atrial filling pressure D. decreased CVP

The correct answer is : D Explanation : In this problem it is obvious over the course of two days that the patient is putting out more fluid than he is taking in. This must mean the patient is losing fluid and is therefore hypovolemic. Hypovolemia, or under hydration, is manifested through a decrease in all hemodynamic values, especially CVP. Remember that CVP is also known by other names such as right atrial filling pressure and right side preload as well as by a few other terms. Be sure to be familiar with all terms.

The following data is available for a patient receiving volume-controlled mechanical ventilation: 6 am 9 am Ppeak (cm H2O) 25 32 Pplat (cm H2O) 15 17 Set VT (L) 0.6 0.6 PEEP (cm H2O) 5 5 Which of the following is most consistent with these data? A. the patient has atelectasis B. the patient has ARDS C. the patient's pulmonary compliance is significantly decreasing D. the patient is in need of endotracheal suctioning

The correct answer is : D Explanation : In this problem, peak pressures are rising but plateau pressures are generally static. This indicates a decrease in dynamic compliance and is caused by a temporary condition such as a kinked ET tube, excess water in the circuit, bronchoconstriction, or secretions in the airway. Three of the options would cause a decrease in static compliance. Only endotracheal suctioning relates to dynamic compliance.

Physicians at the hospital have been increasingly ordering improper mixtures and combinations of similar class bronchodilators, causing significant delays in patient treatment. Which of the following would be MOST helpful at improving patient care with regard to bronchodilator therapy? A. communicate with each physician personally on proper ordering B. administer medications as ordered, but notify the medical director C. place educational posters in various areas in the hospital D. establish a bronchodilator protocol

The correct answer is : D Explanation : In this question, there is an obvious indication that there is a broad problem among physicians to incorrectly order bronchodilator therapy. Although physicians could be spoken to individually, a more efficient method is to create a bronchodilator protocol. This is a documented set of instructions that allows a physician to order bronchodilator therapy, without having to provide the specifics of how the therapy is given or what drug or dosage is used. This approach helps to standardize care and avoid problems with physician orders.

An adult patient is intubated with an 8.0 mm endotracheal tube. The high-pressure alarm is sounding. The tube is noted to be positioned at 28-cm at the lips. Chest rise is asymmetrical. What procedure should the respiratory therapist recommend? A. extubation B. suction with a directional-tip catheter C. arterial blood gas D. stat chest radiograph

The correct answer is : D Explanation : In this scenario, one may become anxious to choose an option that corrects the problem, with is likely to be an endotracheal tube that is advanced too far into the trachea. However, because there is no response like this, the respiratory therapist should confirm the concern with a chest X-ray, STAT.

A 62-year old male is admitted to the emergency department with increased somnolence and fever. The patient is stable, alert, and oriented, but quickly falls asleep with loud snoring when left alone. The following data is observed: HR 95/min BP 155/95 mm Hg Temp 38.3 deg C (101 deg F) FIO2 0.21 pH 7.28 PaCO2 67 torr PaO2 56 torr HCO3- 29 mEq/L BE -5 mEq/L The respiratory therapist should recommend A. Administer oxygen by nasal cannula at 2 L/min B. Orally intubate and being volume ventilation C. Begin oxygen by air-entrainment mask at 50% D. Initiate non-invasive ventilation

The correct answer is : D Explanation : In this situation, the patient has decreased ventilatory ability as demonstrated by a pH of 7.28. While this pH does not demonstrate complete ventilatory failure, it does show an impending problem. Therefore, it may be suitable to provide some level of ventilatory support through noninvasive ventilation. As a side note, it is concerning that the patient is demonstrating increased somnolence. In such a situation, the therapist should investigate the patient's ability to protect their own airway. Unfortunately, the question does not provide information that indicates the lack of ability to protect their airway and the best answer in this situation would be noninvasive ventilation. However, if there were evidence that the patient could not cough, could not follow instruction, or was not able to demonstrate orientation to time, person, and place, it would be appropriate to protect the patient's airway and provide invasive (intubated) mechanical ventilatory support.

A male postoperative patient has physician orders to receive SMI therapy. His preoperative inspiratory capacity was 2200 mL. The respiratory will set which of the following as an initial goal? A. 2200 mL B. 500 mL C. 3100 mL D. 1100 mL

The correct answer is : D Explanation : Initial goal for incentive spirometry (SMI therapy) after surgery should be about one half of the inspiratory capacity achieved prior to surgery. If the patient is unable to achieve this goal, the goal should be set just above the patient's ability to achieve.

The physician requests patient education for a newly diagnosed COPD patient being discharged home with continuous supplemental oxygen and inhalers. Which of the following are NOT indicated for the teaching plan? A. recognizing signs of infection B. increasing quality of life C. cleaning the MDI spacer D. achieving normal lung functions

The correct answer is : D Explanation : It is not realistic for a respiratory therapist to expect to achieve normal lung functions within COPD patients, therefore patient instructions should not include such false hopes.

A radiological report is placed in the medical record that describes the results of a chest radiograph. The report indicates plate-like infiltrates bilaterally. This finding is consistent with A. pleural effusion B. pulmonary edema C. pulmonary embolus D. atelectasis

The correct answer is : D Explanation : The term platelike infiltrates is associated with atelectasis.

Immediately after extubation of an adult male, the respiratory therapist auscultates mild stridor in the upper airway. SpO2 is 98% with FIO2 0.4 by air-entrainment mask. Spontaneous tidal volume is 450 mL and minute ventilation is 6.3 L. The respiratory therapist should recommend which of the following? A. provide heated mist B. immediately reintubate the patient C. administration of aerosolized epinephrine D. administration of cool aerosol by large-volume nebulizer

The correct answer is : D Explanation : Mild stridor may be addressed by administering a cool aerosol or racemic epinephrine. Do not be tempted to administer epinephrine. "Racemic epinephrine" is half-strength epinephrine.

For a patient with severe mitral-valve stenosis, a balloon-tipped flow-directed pulmonary artery catheter would be associated with which of the following clinical data? A. PAP 19 mm Hg and PCWP 7 mmHg B. CVP 9 mm Hg and PAP 9 mmHg C. PAP 25/8 mmHg D. PCWP 14 mm Hg and C.O. 4 L/M

The correct answer is : D Explanation : Mitral valve stenosis is a problem resulting from left heart failure. Problems in the left heart are manifested hemodynamically with high pulmonary capillary wedge pressure (PCWP) and low or normal cardiac output.

A chest radiograph of an orally intubated patient shows the end of the radio-opaque line on the 8.0-mm ET tube to be 1 inch above he carina. Breaths sounds are clear and diminished bilaterally. The respiratory therapists should A. advance the tube by 1 cm B. withdraw the tube by 2 inches C. replace the tube with a 7.0-mm size ET tube D. maintain tube position

The correct answer is : D Explanation : Normally, the end of an endotracheal tube should be positioned approximately 2 cm above the Carina. However, in some cases, that distance may be expressed as approximate inches. 1 inch is approximately 2.54 cm and therefore one-inch is considered acceptable, and tube position should be maintained.

An end-expiratory chest radiograph is best used to assess for which of the following? A. pulmonary vascular disease B. overall pulmonary aeration C. diaphragmatic hernia D. small pneumothorax

The correct answer is : D Explanation : Normally, we prefer to observe a chest x-ray at full inhalation. However, an end-expiratory chest radiograph is helpful at identifying a small pneumothorax.

Immediately following oral intubation, which of the following devices would be most helpful in determining if the ET tube is correctly positioned in the trachea? A. pulse oximetery B. SvO2 monitor C. transcutaneous CO2 monitor D. infrared CO2 detection device

The correct answer is : D Explanation : Of the devices listed, an infrared CO2 detection device may be used to determine if adequate CO2 is being exhaled from the ET tube during ventilation. If the endotracheal tube is inadvertently placed inside the esophagus end-tidal CO2 will read nearly 0, indicating a need to remove and reposition the endotracheal tube.

During endotracheal suctioning, the respiratory therapist observes that the patient's heart rate jumps from 95 to 110/min. Saturation falls from 93% to 88%. PVCs are noted on the ECG monitor. The respiratory therapist should change the A. suction pressure at the wall B. suction catheter diameter C. endotracheal tube D. suction duration

The correct answer is : D Explanation : Of the options given, the best way to prevent PVCs during the suction procedure is to reduce suction duration.

Which of the following would result from excessive PEEP? A. reduced pulmonary vascular resistance (PVR) B. increased auto PEEP C. reduced alveolar oxygen tension D. reduced cardiac index

The correct answer is : D Explanation : Of the options offered, PEEP will most likely cause a reduction in venous return which will cause a reduction in cardiac output and therefore, cardiac index.

What would occur on a time-cycled ventilator with a fixed rate if the inspiratory flow rate were reduced? A. decrease in inspiratory time B. increase in inspiratory time C. increase in tidal volume D. decrease in tidal volume

The correct answer is : D Explanation : On a time cycle ventilator, with a fixed rate and a fixed inspiratory time, if inspiratory flow rate is decreased, the result would be a decrease in tidal volume.

Which of the following clinical outcomes is a possible side effect of aerosolized racemic epinephrine? A. tachycardia B. bronchoconstriction C. blood anticoagulation D. rebound effect

The correct answer is : D Explanation : One potential problem with the use of aerosolized racemic epinephrine is something called the rebound effect. This means that after inhaling the substance and receiving relief, some patients, especially children, may experience a profound resurgence of the initial inflammation. That inflammation may exceed the initial presentation and therefore should be monitored closely. Commonly, children who are given racemic epinephrine in the emergency room, are held over for 24-hour monitoring.

Which of the following results should the respiratory therapist evaluate to determine the adequacy of oxygen transport on a patient who is diagnosed with carbon monoxide poisoning and is being treated with FIO2 of 1.0? A. oxygen consumption at the tissues B. cardiac output C. arterial-venous oxygen content difference D. arterial oxygen content

The correct answer is : D Explanation : Oxygen transport refers to the ability for blood to carry oxygen from the alveoli to the tissues. In a case involving carbon monoxide poisoning, it is transport that is most affected. This is true because hemoglobin becomes occupied with carbon monoxide rather than oxygen. Hemoglobin is 19 times more attracted to carbon monoxide compared to oxygen. Of the options listed, the best method to monitor the adequacy of oxygen transport would be to evaluate the arterial oxygen content. This value takes into account the amount of oxygen tied to the hemoglobin as well as the oxygen dissolved in the plasma of the blood.

A patient diagnosed with a pulmonary infection is febrile with a temperature of 39 deg C (102 deg F). A recent arterial blood gas shows the following: pH 7.50 PaCO2 30 torr PaO2 65 torr HCO3- 22 mEq/L BE -2 mEq/L Which of these reported values would be lower than the actual when not corrected for temperature? A. HCO3- B. pH C. SaO2 D. PaCO2

The correct answer is : D Explanation : PaCO2 and PaO2 would be lower than actual if the blood gas is not corrected for temperature.

Close monitoring of VC is indicated in which of the following situations? A. post abdominal surgery B. within 24 hours after cardiopulmonary resuscitation C. after the administration of a neuromuscular blockade D. neuromuscular disorder

The correct answer is : D Explanation : Patients with suspected neuromuscular disorders, like Guillain-Barré and myasthenia gravis, should be monitored for basic ventilation parameters, including vital capacity.

In which of the following situations would a point-of-care arterial blood gas analysis be most appropriate? A. at college for a cystic fibrosis patient B. at home for a COPD patient C. at a health fair for the public D. in surgery

The correct answer is : D Explanation : Point-of-care analytical devices are most appropriate in temporary situations or in places where quick information is required. Surgery is an example of this type of requirement. If a physician in surgery requires arterial blood gas data, it would perhaps take too long for the sample to leave the sterile area to be analyzed. Consequently an analyzer within the sterile area would provide data more quickly and allow for better care of the patient.

A patient receiving continuous positive pressure ventilation is most likely to have which of the following? A. decreased sodium B. increased bowel movement C. depressed CNS function D. decreased urine output

The correct answer is : D Explanation : Positive pressure ventilation promotes decreased venous return and decreased perfusion. This will first show up in urine output. When perfusion decreases, urine output also decreases.

A patient scheduled for abdominal surgery can be assessed for potential post-operative risk for complications by assessing which of the following? A. C(a-vDO2) B. alveolar oxygen tension on room air (FIO2 0.21) C. incentive spirometry with flow-type spirometer D. basic spirometry

The correct answer is : D Explanation : Potential postoperative risk may be assessed preoperatively through basic spirometry.

The following ECG waveform is noted on the monitor while performing endotracheal suctioning using a 12 Fr catheter at a wall suction pressure of 110 mmHg. Which of the following is the mostly likely cause? A. vagal nerve stimulation B. oxygen toxicity C. hypokalemia D. arterial hypoxemia

The correct answer is : D Explanation : Premature ventricular contractions (PVCs) can be caused by suctioning. Suctioning causes alveolar oxygen tensions to fall thus causing arterial hypoxemia. Vagal nerve stimulation also occurs but this is not associated with PVCs.

A chest radiograph that has been overexposed could result in which of the following? A. blunting of the costophrenic angles B. absence of bone density C. appearance of a pneumothorax D. pronounced vascular markings

The correct answer is : D Explanation : Pronounced vascular markings are associated with COPD and overexposure of a chest x-ray.

Which of the following medications is most suitable to provide sedation and analgesia for a patient who is post-operative for CABG and is receiving VC ventilation? A. Romazicon (flumazenil) B. Valium (diazepam) C. Lorazepam (Ativan) D. propofol (Diprivan)

The correct answer is : D Explanation : Propofol, also called Diprivan, is a great medication used to provide immediate sedation and analgesia. This drug is especially helpful because of its immediate mode of action as well as its short half-life when discontinued.

A patient with end-stage pulmonary emphysema is receiving oxygen at 5 L/min by nasal cannula. After 20 minutes, the patient becomes despondent and has shallow respirations at a rate of 22/min. Pulse oximeter is showing a SpO2 of 95%. The respiratory therapist should do which of the following? A. Change to a nonrebreathing mask B. Notify the physician C. Increase oxygen flow rate to 6 L/min D. Reduce oxygen flow rate to 1 L/min

The correct answer is : D Explanation : Pulmonary emphysema is one of the diseases associated with COPD. Therefore, this patient should receive no more than 28% oxygen by Venturi mask or 1 to 2 L/min by nasal cannula. When a COPD patient receives excessive amounts of oxygen, their hypoxic drive may be suppressed and respiratory rate and depth may decrease. This patient is receiving 5 L/min by nasal cannula which is too much. It is for this reason the patient is demonstrating despondency and other clinical manifestations of oxygen overdose. The best option is to reduce the oxygen flow rate to 1 L/min.

Pulmozyme is best used for A. COPD patients with airway inflammation B. tuberculosis patients with blood-tinged secretions C. asthma patients refractory to sympathomimetic bronchodilators D. cystic fibrosis patients with a FVC of at least 40% predicted

The correct answer is : D Explanation : Pulmozyme is helpful at helping patients with cystic fibrosis mobilize and expectorate secretions. However, to be effective the patient must be able to at least produce an FVC of 40% of predicted.

While instructing a home-care patient with COPD on strategies to minimize ventilatory distress, the respiratory therapist should include which of the following? A. avoid all activities of daily living that increases ventilatory demand B. hold breath for 1-2 seconds before exhalation when in distress C. breath at a higher rate but smaller breaths D. use pursed-lip breathing

The correct answer is : D Explanation : Pursed-lip breathing is a helpful method at preventing ventilatory distress by slowing expiration and decreasing FRC in patients with COPD. The patient should not be instructed to avoid activities of daily living or to hold their breath.

The therapist is observing the quality control results in order to determine the accuracy and precision of the blood gas analyzer. According to the documentation that came with the quality control material, pH is supposed to analyze at 7.40 with 0.5% upper and lower standard deviation. The PCO2 analyzer is supposed to measure 30 mmHg with 5% upper and lower standard deviation tolerance. Which of the following runs are consistent with an analyzer that is out-of-control? pH PCO2 (mm Hg) Run 1 7.37 25 Run 2 7.41 31 Run 3 7.34 29 A. 1 only B. 3 only C. 2 only D. 1 and 3

The correct answer is : D Explanation : Runs one and three, when calculated, exceed the pH or PCO2 limit and therefore are consistent with a blood gas analyzer that is out of control.

Which of the following units of measure are associated with systemic vascular resistance? A. L/min/m3 B. dynes-min/cm2 C. dm2 /min D. dynes/sec/cm5

The correct answer is : D Explanation : SVR = (MAP-CVP)/C.O. X 80 The unit of measure is dynes, or more specifically, dynes/sec/cm5.

Placement of a pulmonary artery catheter is associated with which of the following most common complications? A. hypotension B. pulmonic valve damage C. internal bleeding D. cardiac arrhythmias

The correct answer is : D Explanation : Several complications may arise from the placement of a pulmonary artery catheter, otherwise called a Swan-Ganz catheter. The development of cardiac arrhythmias is the most common complication of the options offered. Another serious complication is perforation of a vessel or cardiac muscle during the insertion.

Which of the following infection control methods is the most effective for eradicating harmful microorganisms on patient care equipment?

autoclave

A respiratory therapist notices the dicrotic notch is missing from the pulmonary artery catheter waveform. After unsuccessfully attempting to resolve the problem by aspiration, the therapist should A. withdraw the catheter B. advance the catheter C. replace the catheter D. rotate the catheter

The correct answer is : D Explanation : The absence of a dicrotic notch on a pulmonary artery waveform is an indication that the catheter is being occluded in some way. To correct this problem, the therapist should first aspirate the catheter. If this does not remedy pressure dampening, flushing the catheter is the next option. Finally, the catheter should be rotated. The only option given in this question that is appropriate is rotation of the catheter.

A patient with asthma is receiving bi-level therapy to decrease work of breathing. Current settings and blood gas results are: IPAP 20 cm H2O EPAP 10 cm H2O FIO2 0.35 pH 7.35 PaCO2 45 torr PaO2 70 torr HCO3- 24 mEq/L BE 0 mEq/L The respiratory therapist should make which of the following changes? A. decrease both IPAP and EPAP B. increase EPAP C. decrease EPAP D. increase IPAP and EPAP

The correct answer is : D Explanation : The arterial blood gas shows the patient is hypoxic but ventilation appears to be adequate. When we want to correct oxygenation we must look to the expiratory positive airway pressure. We must be careful here, however, because raising EPAP by itself will also inadvertently lower ventilation by decreasing the distance between the expiratory pressure and the inspiratory pressure. Thus, when we raised EPAP by a specific amount we must raise IPAP by that same amount if we do not wish to decrease ventilation.

Following a complete sleep study, it is determined that a 45-year-old male requires bi-level therapy at home, with supplemental nocturnal oxygen. Which of the following is indicated? A. blood pressure monitoring B. nocturnal oximetry study C. arterial blood gas D. oxygen bleed-in device

The correct answer is : D Explanation : The best device for home oxygen therapy is a molecular sieve device, commonly known as an oxygen concentrator. This device is powered with electricity, which makes it appropriate for home use.

A 75-kg (165-lbs) male is in acute ventilatory failure. In preparation for mechanical ventilatory support, the patient is to be orally intubated. Which of the following artificial airways would be most appropriate for this patient? A. Size 8.0 mm ET tube with a small volume, high pressure cuff B. Jackson tracheostomy tube C. Size 7.5 mm fenestrated tracheostomy tube D. Size 7.0 mm ET tube with a large volume, low pressure cuff

The correct answer is : D Explanation : The best way to determine the appropriate size of an endotracheal tube is to take the first digit of the patient's ideal body weight in kilograms. In this case, the patient weighs 75 kg. An endotracheal tube size of 7 or 7.5 is appropriate. The fenestrated tracheostomy tube is intended for long-term mechanical ventilation and is not appropriate in this case. The preferred ET tube is high-volume and low-pressure.

A 32-year-old male who weighs 60 kg (143 lb) is receiving VC SIMV ventilation following the surgical repair of a hernia with slow emergence from sedation. The following data is observed: pH 7.48 PaCO2 32 torr PaO2 110 torr HCO3- 23 mEq/L BE -1 mEq/L FIO2 0.45 Mandatory rate 12 Total rate 12 Tidal volume 650 mL The respiratory therapist should A. increase mandatory rate to 14 B. check autoPEEP C. decrease FIO2 to 0.21 D. decrease tidal volume to 450 mL

The correct answer is : D Explanation : The blood gas demonstrates a small amount of hyperventilation. Additionally, based upon the patient's predicted body weight, a tidal volume of 650 mL is too much. Thus, lowering the tidal volume to something more within range will help raise arterial CO2 to the proper level.

The respiratory therapist should respond to the following graph by checking for which of the following? A. hypercapnia B. poor alveolar perfusion C. hypercarbia D. disconnection in the patient-ventilator interface

The correct answer is : D Explanation : The capnograph wave form shows a sudden fall to Zero end-tidal CO2. This is most likely caused from a disconnection of the ventilator circuit.

A tracheostomy tube is changed on a patient who has been receiving continuous mechanical ventilation for the last several weeks. Within moments after intubation, the respiratory therapist observes the patient is suddenly agitated and SPO2 is 88% and dropping. This clinical change is most likely caused by A. a tracheostomy tube that is too small B. inadvertent closure of the tracheostomy opening C. a need to suction the patient D. malpositioning of the tracheostomy tube

The correct answer is : D Explanation : The crackling sensation, also called a popping sensation or sound, is most likely caused from subcutaneous emphysema. This is commonly caused from mal-positioning of the tracheostomy tube. Removal and repositioning is appropriate.

A patient is receiving volume-controlled ventilation through tracheostomy tube that was placed only twenty minutes prior. The therapist notices swelling about the neck and left breast. This is most likely caused by A. insufficient tracheostomy tube size B. pneumothorax C. hemothorax D. an improperly positioned tracheostomy tube

The correct answer is : D Explanation : The crackling sensation, also called a popping sensation, and swelling of the neck and breast, is most likely caused from subcutaneous emphysema. This is commonly caused from mal-positioning of the tracheostomy tube. Removal and repositioning is appropriate.

A patient receives postural drainage and percussion. During the treatment, the patient develops rhonchi. The respiratory therapist should NEXT A. administer an IPPB treatment B. discontinue treatment C. switch to PEP therapy D. perform nasal tracheal suctioning

The correct answer is : D Explanation : The development of rhonchi during postural drainage and percussion is an indication that the therapy is working - is mobilizing secretions effectively and moving them upward where they can be expectorated or suctioned.

A radiology report of a chest x-ray indicates the end of the radio-opaque line on an oral endotracheal tube is resting 1-cm above the carina. The respiratory therapist should suggest A. advance the ET tube 1 cm B. withdraw the ET tube 5 cm C. advance the ET tube 2 cm D. withdraw the ET tube 2 cm

The correct answer is : D Explanation : The endotracheal tube is properly positioned when the end is 2 to 5 cm above the carina. In this case, the tube is only 1 cm above the carina. Withdrawing the by 2 to 4 cm is appropriate.

Which of the following is an important considerations when preparing a patient to receive continuous mechanical ventilatory support at home, 24 hours a day, seven days a week?

back-up ventilator in the home

During the evaluation of a post-operative patient who completed surgery 24 hours prior, the respiratory therapist coaches the patient on incentive spirometry and notes the patient is able to achieve an inspiratory capacity of 750 mL. The goal for the patient should be A. 1100 mL B. 750 mL C. 1500 mL D. 900 mL

The correct answer is : D Explanation : The goal for incentive spirometry should be placed slightly higher than the patient's current inspiratory capacity achievement.

A respiratory therapist is performing a routine patient/ventilator check on a mechanically ventilated patient. Returned tidal volume is 150 mL less than delivered. A high-pitched audible leak is auscultated through the neck. What is the appropriate response? A. increase delivered tidal volume B. advance the endotracheal tube 1 cm C. reposition the patient D. add air to the cuff

The correct answer is : D Explanation : The high-pitched audible leak auscultating over the neck in conjunction with a low returned tidal volume is most likely an indication of insufficient pressure in the endotracheal tube cuff. Adding air to the cuff is most appropriate.

A patient is receiving volume-controlled mechanical ventilation on the following settings: Mode A/C Mandatory rate 16 FIO2 0.5 Tidal volume 600 mL PEEP 5 cm H2O Insp flow rate 45 L/min The following ventilator graphic is associated with the current state of ventilation. Which of the following should the respiratory therapist recommend? A. increase inspiratory flow rate B. switch to SIMV mode C. decrease tidal volume D. increase PEEP

The correct answer is : D Explanation : The key to this question is not to focus on oxygenation but rather the shape of the pressure-volume ventilator graphic. In a normal graphic, pressure and volume should increase at the same time. In this case, volume is increasing but pressure is not rising. This is shown by the flatness of the "football". This may be solved by increasing PEEP until pressure rises immediately with volume.

A patient is found to have uncompensated respiratory acidosis. This would correlate with what kind of change in the oxygen dissociation curve? A. shift to the left B. compressed waveform C. expanded waveform D. shift to the right

The correct answer is : D Explanation : The oxygen dissociation curve is a graphical depiction of the relationship between arterial PO2 and SpO2. It is based upon a fixed pH. When the pH changes, or the patient becomes alkaline or acidotic, the oxygen dissociation curve can shift to the right or left. In this case, acidosis will cause the curve to shift to the right, which will change the numerical relationship between PaO2 and SpO2. A shift to the right also indicates a decreased affinity between hemoglobin and oxygen molecules.

A 35-year-old female patient presents to the emergency room with droopy eyelids and facial muscles. She complains of frequent fatigue that improves with rest, but then returns. Which of the following diagnostic procedures are not indicated? A. Tensilon challenge B. arterial blood gases C. basic spirometry D. nitrogen washout

The correct answer is : D Explanation : The patient is showing signs of myasthenia gravis for which basic spirometry, a Tensilon challenge, and an arterial blood gas are indicated. A nitrogen washout would detect lung volume changes associated with chronic obstructive pulmonary disease, but has little to no value for a patient with myasthenia gravis.

Immediately after the insertion of a nasopharyngeal airway (nasal trumpet), the patient begins to cough uncontrollably. The therapist should consider A. spaying the oropharynx with benzocaine B. withdrawing the airway by 1 cm C. reinserting the airway with a viscous lidocaine lubricant D. replacing the airway with one that is shorter

The correct answer is : D Explanation : The patient's response to cough vigorously immediately after insertion of the nasal trumpet indicates the airway is likely too long. It should be replaced with a shorter one.

Which of the following would be most helpful in continuously monitoring the arterial oxygen and carbon dioxide levels in a newborn? A. single-wavelength spectrophotometry B. pulse oximetry C. arterial blood gas analysis D. transcutaneous monitoring

The correct answer is : D Explanation : The preferred method to provide continuous monitoring of arterial oxygen and carbon dioxide levels in a newborn is transcutaneous monitoring. This is a device that consists of a heated electrode that is placed on the skin of the infant. The electrode can detect minute perfused gases across skin and estimate PO2 and CO2. While it is not as accurate as arterial blood gas analysis, it does provide fairly precise data without removing precious blood volume from the infant.

A respiratory therapist is checking for the presence of right to left cardiac shunt. From what location should the therapist draw blood for gas analysis? A. femoral artery B. proximal lumen of a PA catheter C. right brachial artery D. distal lumen of a PA catheter

The correct answer is : D Explanation : The presence of a right-to-left cardiac shunt, which would produce mixed venous blood, can be determined by analyzing the blood from the distal lumen of the pulmonary artery catheter. Because the distal lumen is in the pulmonary artery this is the best place to obtain a mixed venous blood sample.

A patient complaining of chest pain has cold extremities, is diaphoretic, and is pale in color. The respiratory therapist should first do which of the following? A. administer blood-expanding medication B. administer blood C. perform a 12-lead ECG D. start supplemental oxygen

The correct answer is : D Explanation : The presence of chest pain, cold extremities, and diaphoresis indicate the patient may be having a myocardial infarction. The best first action is to provide the patient with supplemental oxygen which will help decrease the work of the heart.

A 62-year-old man admitted to the emergency room with chest pain complains of shortness of breath after a bronchodilator treatment. Further examination reveals the patient is diaphoretic. The ECG wave form shows occasional PVCs. Which of the following would be most helpful in further determining the cause of the patient's condition? A. cardiac catheterization with fluoroscopy B. cardiac stress test with Adenosine C. arterial blood gas analysis D. 12-lead ECG

The correct answer is : D Explanation : The presence of chest pain, diaphoresis, and PVCs are all indicators of cardiac distress. A 12-lead ECG will help to further evaluate the level of distress and potentially the cause of the problem. The concern would be myocardial infarction.

The following ECG tracing suddenly presents on a patient in the emergency department (ED) who was admitted for chest pain. The tracing is confirmed in two leads. The therapist should immediately asystole rhythm

begin chest compressions

Within 5 minutes of oral extubation, the patient demonstrates mild inspiratory and expiratory stridor, some accessory muscle use, and an SpO2 of 92% on 40% air-entrainment mask. The therapist should recommend A. reintubation B. benzocaine spray to the oropharynx C. heated bland aerosol D. aerosolized racemic epinephrine

The correct answer is : D Explanation : The presence of inspiratory and expiratory stridor can be addressed by the therapist through the administration of racemic epinephrine. This medication is an alpha-1 type drug that constricts tissue. However, this can only be used when the level of stridor is considered mild or moderate. In the presence of severe or marked stridor, immediate establishment of an airway is preferred.

An adult male patient receiving mechanical ventilatory support has the following pressure-volume graphics pre and post lung recruitment procedure. The respiratory therapist should conclude A. tidal volume has inadvertently been increased B. autoPEEP level has decreased C. further lung recruitment procedures should be placed on hold D. lung recruitment has been successful

The correct answer is : D Explanation : The pressure-volume loops shown in this set of ventilator graphics shows that the patient was initially over distended. However, without intervention, the over distention spontaneously dissipates. This would suggest that the lungs have essentially increased in their capacity, or in other words, alveoli have been recruited.

Which of the following can be done to most effectively increase the efficiency of suctioning through an endotracheal tube? A. increase suction time B. apply intermittent suction pressure during withdrawal of the catheter C. increase suction pressure D. increase suction catheter diameter (size)

The correct answer is : D Explanation : The proper way to increase suctioning effectiveness to an endotracheal tube is to first increase suction catheter size or diameter followed by increasing suction pressure and then increasing suction time. Therefore, the best answer is to increase the suction catheter diameter or size.

Pulmonary function testing is done on a patient with venous distension and a flattened diaphragm. The following pulmonary function data is recorded: Percent of Pred Actual value Fev1/FVC % 58% FEF200-1200 75% FEF25-75 52% SVC 88% FVC 81% DLCO 15 CO/min/mm Hg Which of the following most likely represents the patient's condition? A. asthma B. chronic bronchitis C. pulmonary fibrosis D. emphysema

The correct answer is : D Explanation : The pulmonary function data shows decreased flows but normal volumes. This is consistent with an obstructive defect. When looking at the answers three of the diseases are obstructive in nature. To discern which disease is most likely, one must look to the DLCO. In this case the DLCO is decreased indicating the patient has pulmonary emphysema. Of the five obstructive diseases only emphysema has a poor DLCO.

While performing pulmonary function testing, the respiratory therapist asks the patient to inhale and exhale maximally at a maximum rate for 12 seconds. Which of the following is being measured? A. FVC B. FEV1 C. minute alveolar ventilation D. MVV

The correct answer is : D Explanation : The pulmonary function maneuver expressed, when the patient is asked to exhale maximally at a maximum rate for 12 seconds, is called maximum voluntary ventilation or MVV. Since the patient is most likely unable to breathe in and out maximally for a whole minute, the patient may be asked to do so for 12 seconds and then the result is multiplied by 5. 15 seconds is also another interval that may be used and it would be multiplied by 4 to calculate the MVV.

While instructing a post-bariatric surgery patient on the technique regarding the use of an incentive spirometer, the therapist will include which of the following instructions? A. "this practice will increase your FRC and decrease your chance of pneumonia" B. "this exercise will help the doctor to know your inspiratory capacity as you recover" C. "this therapy will help improve gas-exchanging surface area in the alveoli" D. "the purpose of the therapy is to keep all parts of the lungs expanded"

The correct answer is : D Explanation : The purpose of a volume-oriented incentive spirometer is to keep all aspects of the lungs inflated and to prevent the collapsing of alveoli.

A 39-year-old female patient presents to the emergency room with wheezing and fever. She has a history of reactive airways disease secondary to pulmonary infections. What should the respiratory therapist recommend for initial treatment? A. oxygen by NRB B. complete PFT C. supplemental fluids D. antibiotics

The correct answer is : D Explanation : The respiratory therapist would likely be searching for a response that resolves wheezing resulting from RAD. There is no response available like this, therefore, following the order of priority, one will find an answer to resolve the underlying problem, which is a pulmonay infection. Antibiotics are indicated. A PFT would not be appropriate for a compromised patient. There is nothing in the scenario that suggests the patient needs 100% oxygen, nor is there anything that suggests supplemental fluids are needed.

A long-term care patient who experienced a stroke has left-sided paralysis about the face and mouth. The patient is unable to produce a seal with her lips while performing IPPB treatments. How would a respiratory therapist expect the IPPB machine to respond in this situation? A. Deliver a higher FIO2 B. Fail to cycle into inhalation C. Deliver a higher FIO2 D. Fail to cycle into exhalation

The correct answer is : D Explanation : The scenario suggests that a leak will occur when administering IPPB. Circuit leaks result in a failure to reach a preset pressure, which then results in failure to cycle into exhalation.

Prior to a pulmonary function test, a respiratory therapist conducts spirometry calibration with a 3.0 Liter super-syringe obtaining the following results: Trail 1 (L) Trial 2 (L) Trial 3 (L) 2.74 2.68 2.72 The spirometer is A. accurate. B. precise. C. imprecise. D. inaccurate.

The correct answer is : D Explanation : The spirometer is considered accurate if the results are no more than 3% above or 3% below the 3 L calibration syringe. This means the accuracy range is 2.85 L -3.15 L. In this case all values are outside this range and therefore the spirometer is considered to be inaccurate.

A patient in the emergency room is in respiratory distress. The report indicates the patient was stable one moment, but suddenly became short of breath with a respiratory rate of 30/min. Which of the following could be the cause of the change in the patient's condition: A. carbon monoxide inhalation B. chronic bronchitis C. congestive heart failure D. pulmonary embolus

The correct answer is : D Explanation : The sudden development of shortness of breath is brought on by a pneumothorax or pulmonary embolism. Most other disease processes or conditions cause changes to occur more slowly.

One important consideration when drawing an arterial blood sample is A. the type of antibiotics currently used by the patient B. heart rate C. the patient's potassium level D. needle size

The correct answer is : D Explanation : There are many important considerations when drawing an arterial blood gas. However, of the options listed in this question, the best answer is needle size. Needle size is something the average respiratory therapist may not routinely consider, however, it is a basic recall matter that one should know.

While moving a patient from the radiology procedure table to a transport gurney, the respiratory therapist suspects the oral endotracheal tube position has been inadvertently changed. To quickly assess airway position, the therapist should FIRST A. perform bronchoscopy B. auscultate breath sounds C. order a chest radiograph (x-ray) D. inspect symmetry of chest rise

The correct answer is : D Explanation : There are many ways to assess the position of an ET tube. This question is asking what should be done first to quickly assess the tube position. Of the options offered the most immediate data would come from a visual inspection of the symmetry of chest rise. The next best option would be auscultation of breath sounds, although not as quick as visual data. Finally, the last best option would be to get a chest x-ray. This option is time-consuming and not considered quick but is considered more conclusive than the other two. But keep in mind, the question is asking for the first evidence, not conclusive evidence.

During movement of an orally intubated patient from an emergency transport vehicle to the emergency department, the respiratory therapist suspects an inadvertent position change of the endotracheal tube. To confirm the position of the ET tube, the respiratory therapist should A. auscultate breath sounds bilaterally B. perform diagnostic chest percussion C. observe chest movement and symmetry of chest rise D. obtain a chest radiograph

The correct answer is : D Explanation : There are many ways to determine the position of the ET tube but only one way to provide conclusive evidence of its location. That one way is a chest x-ray which allows direct observation of the location of the end of the tube. The end of the tube should be located 2 to 5 cm above the carina.

The respiratory therapist should recommend which of the following to confirm the placement of an 8.0 mm oral endotracheal tube? A. Notation of the ET tube markings at the teeth B. Observation of chest rise and symmetry C. Auscultation of the chest D. Chest radiograph

The correct answer is : D Explanation : There are several methods in which one may determine the proper position of an endotracheal tube. Those methods include such observations as symmetry of chest rise, breath sounds, and radiographic assessment. However, of these options only a radiographic assessment is confirming in nature and will show the absolute position of the endotracheal tube.

Which of the following is needed to calculate minute alveolar ventilation? A. C(a-v)O2 and VT B. PAO2 and tidal volume C. VD/VT and PAO2 D. tidal volume, weight, respiratory rate

The correct answer is : D Explanation : There are three items you need to calculate a patient's minute alveolar ventilation- tidal volume, respiratory rate, and the patient's weight in pounds. If the tidal volume is not given you may need to calculate the tidal volume by looking at minute ventilation and dividing it by the rate first.

The respiratory therapist can BEST increase the effectiveness of endotracheal suctioning by doing which of the following? A. Increase the suction pressure B. Apply intermittent suction during removal of the catheter C. Increase the suction time with each pass D. Increase the diameter of the suction catheter

The correct answer is : D Explanation : There are three typical ways to increase the efficiency of suctioning. The most effective way is to increase the diameter of the suction catheter. This, however, is limited to the size of the endotracheal tube. The second best way is to increase suction pressure but this is limited to a maximum suction pressure of 120 mmHg for an adult. The third most effective way to improve suction efficiency is to increase suction duration. But, this is limited to a maximum of 15 seconds.

Which of the following can cause inverted T waves on a 12-lead ECG? A. hypokalemia B. second degree heart block C. hyperkalemia D. digitalis toxicity

The correct answer is : D Explanation : There are two causes of inverted T waves: cardiac ischemia and digitalis toxicity.

An infant is receiving CPAP to help resolve hypoxemia. Radiological examination reveals a perforation in the right lung. Cyanosis persists with increasing respiratory acidosis. The respiratory therapist should suggest A. increase CPAP B. decrease CPAP C. increase FIO2 D. high frequency jet ventilation

The correct answer is : D Explanation : There are two ways to deal with a perforation in one of the lungs. When the patient is receiving mechanical ventilation, the respiratory therapist must be careful not to expose that lung to too much pressure. This can be accomplished by either employing independent lung ventilation with a double lumen endotracheal tube, or by employing high-frequency jet ventilation, which will keep mean airway pressure low and avoid further damage to the lung.

While providing manual ventilation with a bag-valve assembly to a patient immediately following oral intubation, the respiratory therapist notices there is very little chest movement with each manual inspiration, the bag-valve is easier than normal to squeeze, and a "hissing" sound can be heard during manual inspiratory attempts. The respiratory therapist should A. increase the oxygen flow to the bag-valve device B. extubate and begin bag-valve ventilation with an inflatable mask C. remove the inlet valve on the resuscitation bag device D. obtain another bag-valve device

The correct answer is : D Explanation : This bag-valve is obviously not functioning correctly. When this happens, the first, quickest, and best action is to replace the device altogether. If this is not possible then some minor troubleshooting may be done if it can be done quickly.

While providing manual ventilation with a bag-valve and mask assembly to a patient who is in full cardiopulmonary arrest, the respiratory therapist is suddenly unable to squeeze the bag. After removing the bag-valve from the patient, the therapist tests the bag and finds the same results. The therapist should A. provide mouth-to-mouth ventilation B. change out the inlet valve assembly C. remove the valve assembly, reattempt ventilation D. obtain another bag-valve

The correct answer is : D Explanation : This bag-valve is obviously not functioning correctly. When this happens, the first, quickest, and best action is to replace the device altogether. If this is not possible then some minor troubleshooting may be done if it can be done quickly..

A patient in the cardiac care unit is found to have the following cardiac rhythm. Which of the following is the best initial action? A. administer Atropine B. unsynchronized defibrillation at 360 joules C. administer epinephrine D. confirm asystole in another lead

The correct answer is : D Explanation : This cardiac rhythm is asystole, which is associated with the absence of cardiac contraction. However, before beginning chest compressions, the absence of a cardiac contraction must be confirmed by attempting to palpate a pulse or by confirming the asystole rhythm in another ECG lead.

A patient is receiving volume-controlled ventilation in the assist/control mode. The following data are available: Peak pressure Plateau Pressure Exhaled VT 8 AM 35 cm H2O 22 cm H2O 600 mL 10 AM 53 cm H2O 24 cm H2O 600 mL The respiratory therapist should next perform which of the following? A. obtain a chest radiograph B. observe pressure/volume airway graphic C. decrease set tidal volume D. endotracheal suctioning

The correct answer is : D Explanation : This data shows a significant increase in peak airway pressures while plateau pressures remains nearly constant. This is consistent with a decrease in dynamic compliance but shows steady static compliance. A decrease in dynamic compliance is caused from temporary conditions such as secretions in the airway, bronchoconstriction, kinked endotracheal tubes, and water in the ventilator circuit, to name a few. Of the options offered, only endotracheal suctioning addresses the decrease in dynamic compliance.

The following data is available for a patient receiving mechanical ventilation: 6 PM 9 PM PEEP 5 cm H2O 5 cm H2O Static airway pressure 35 cm H2O 36 cm H2O Peak airway pressure 42 cm H2O 52 cm H2O Delivered tidal volume 550 mL 550 mL Inspiratory flow 50 L/min 50 L/min What is the next action the respiratory therapist should take? A. Decrease tidal volume B. Obtain a chest radiograph C. Increase inspiratory flow D. Perform bronchial hygiene

The correct answer is : D Explanation : This data shows that pulmonary compliance is remaining steady with static airway pressures that are not significantly changing. However, peak airway pressures are increasing. Increasing peak airway pressures while static pressures remain constant is an indication of either secretions in the airway or another artificial blockage of the airway. Of the options offered, performing bronchial hygiene is the only one that would address the decrease in dynamic compliance.

A patient with bilateral pneumonia is receiving pressure-controlled ventilation with the following arterial blood results on the settings below: Mode PCV Mandatory rate 22 PIP 25 cm H20 FIO2 0.65 PEEP 15 cm H2O pH 7.29 PaCO2 51 torr PaO2 65 torr HCO3- 26 mEq/L BE +2 mEq/L Which of the following should be increased? A. FIO2 B. Rate C. PEEP D. PIP

The correct answer is : D Explanation : This event is demonstrating hypoventilation as manifested by a high PaCO2. On a time-cycled ventilator the most appropriate control to adjust in order to decrease CO2 is the peak inspiratory pressure (PIP).

A patient is receiving bronchodilator therapy with the following results: Pre-treatment Mid-treatment Resp rate 14 15 Heart rate 90 120 The therapist should do which of the following? A. continue treatment and monitor closely B. instruct the patient to take smaller breaths C. continue treatment, notify the physician D. terminate the treatment and notify the physician

The correct answer is : D Explanation : This patient demonstrates an excessive change in heart rate during the bronchodilator therapy. Any change greater than 20 above baseline is considered a significant reaction to the bronchodilator medication. This is a direct indication to stop the therapy and to modify it by either decreasing dosage, switching to a different medication, or changing the modality altogether.

During the delivery of a small volume nebulizer treatment with Albuterol, the patient complains of tremors and nausea. The respiratory therapist notes a change in heart rate from 74 to 102/min. After discontinuing the treatment, the therapist should recommend which of the following? A. Decrease the frequency of Albuterol treatments B. Switch to IPPB C. Use half-strength Albuterol D. Switch from Albuterol to Xopenex

The correct answer is : D Explanation : This patient experiences an increase in heart rate of more than 20 bpm above baseline. This indicates an adverse reaction to the albuterol. Of the options offered, the best way to modify this therapy is to switch from albuterol to a different drug. In this case, Xopenex is most appropriate.

A patient who weighs 65-kg (143-lb) is receiving mechanical ventilator support on the following settings with the following corresponding blood gas values: Mode SIMV Mandatory rate 16 VT 450 mL FIO2 0.60 PEEP 5 cm H2O Inspiratory flow 50 L/min Corresponding blood gases show: pH 7.29 PaCO2 52 mm Hg PaO2 78 mm Hg HCO3- 26 mEq/L BE +2 mEq/L The respiratory therapist should recommend: A. increase tidal volume to 500 B. add pressure support of 5 cmH2O C. increase PEEP to 10 cmH2O D. increase rate to 18

The correct answer is : D Explanation : This patient has arterial blood gases that show hypoventilation. Mild hypoxemia is also present. When there is a problem with both ventilation and oxygenation at the same time, ventilation should be corrected first. This is because the hypoxemia may be caused by hypoventilation. The proper way to increase ventilation is to either increase rate, or increase tidal volume. Any time the CO2 is off by more than 4 mmHg a change in tidal volume is insufficient. An increase in rate is required.

A patient receiving volume-controlled ventilation weighs 75 kg (165 lb) and is 5 ft, 7 in (170 cm) tall and within ideal body weight. The following clinical data is available: Mode SIMV Mandatory rate 12 Total rate 32 FIO2 0.40 VT 500 mL Exhaled minute volume 9.6 L pH 7.44 PaCO2 32 torr PaO2 97 torr HCO3- 22 mEq/L BE +1 mEq/L The respiratory therapist should recommend which of the following changes? A. Reduce mandatory rate to 6 B. Add inspiratory pause C. Administer sedative medication D. Institute pressure support of 5 cm H2O

The correct answer is : D Explanation : This patient has reduced spontaneous tidal volumes. This may be determined by taking minute ventilation and dividing it by rate. This calculation reveals a tidal volume of 300 mL, which is far below the minimum 5 mL/kg tidal volume needed to sustain life. The elevated respiratory rate, in conjunction with a low tidal volume, is an indication that the patient is in need of pressure support. Pressure support will help by increasing the size of spontaneous tidal volumes, which will help naturally lower rate and decrease work of breathing.

A 90 kg (198 lb) adult male is receiving volume-controlled ventilation following abdominal surgery. Current ventilator settings and corresponding arterial blood gas results are: Mode SIMV Mandatory rate 16 Total rate 17 Tidal volume 650 mL PEEP 5 cm H2O FIO2 0.40 pH 7.48 PaCO2 33 torr PaO2 128 torr HCO3- 25 mEq/L BE +1 mEq/L The respiratory therapist should recommend A. switching to assist/control mode B. decreasing FIO2 to 0.35 C. decreasing mandatory rate D. discontinuing PEEP

The correct answer is : D Explanation : This patient is both hyperventilating and hyper oxygenating. Of these two problems, the over-oxygenation should be addressed first. To correct over-oxygenation either FIO2 or PEEP should be decreased. Normally, FIO2 is decreased first until below 0.6. and then PEEP is lowered. In this case, however, oxygen percentage is already below 60% and therefore discontinuing PEEP is the best option.

A patient with asthma receiving volume-controlled mechanical ventilation has the following arterial blood results on the settings below: Mode SIMV Mandatory rate 18 VT 600 mL FIO2 0.40 PEEP (set) 5 cm H2O Total PEEP 8 cm H2O I:E 1:2 pH 7.32 PaCO2 48 torr PaO2 70 torr HCO3- 26 mEq/L BE +2 mEq/L Which of the following should be decreased? A. flow B. PEEP C. FIO2 D. I:E

The correct answer is : D Explanation : This patient is developing autoPEEP as manifested by a total PEEP that is higher than the set mechanical PEEP. This is caused from insufficient expiratory time and is remedied by shortening inspiratory time, which is also termed as a decrease in I:E ratio. A decrease in flow would result in less expiratory time and an increase in autoPEEP.

While administering a breathing treatment with 1.25 mg Xopenex, the respiratory therapist observes an increase in HR from 90 to 122/min. The patient complains of nervousness and nausea. The therapist should A. switch to Azmacort, 2 puffs, q4 hrs B. Switch to 2.0 mg albuterol C. discontinue all Xopenex treatments for now, notify the medical director D. decrease the dosage of Xopenex

The correct answer is : D Explanation : This patient is exhibiting an adverse reaction from the drug Xopenex. Because the adult therapeutic dose range is from 0.63 mg to 1.25 mg, there is room for the dosage to be decreased. This is the best option.

A patient in the emergency room has the following arterial blood gas results: pH 7.55 PaCO2 35 mm Hg PaO2 84 mm Hg HCO3- 35 mEq/L BE +11 mEq/L SAT 95% Which of the following represents an accurate interpretation of the data? A. metabolic acidosis B. respiratory alkalosis C. mixed-respiratory and metabolic acidosis D. metabolic alkalosis

The correct answer is : D Explanation : This patient is in a state of alkalosis as manifested by a high pH of 7.55. Changes in pH are either caused by changes in CO2 or changes in HCO3-. In this case the patient's CO2 is in the normal range and therefore cannot be a cause of the alkalosis. This would mean that respiratory alkalosis is not present but metabolic alkalosis is evident.

A patient who weighs 60-kg (132-lb) is receiving mechanical ventilator support on the following settings with the following corresponding blood gas values: Mode SIMV Mandatory rate 6 Total rate 26 VT (set) 400 mL VT (spont) 180 mL FIO2 0.40 PEEP 5 cm H2O PS 5 cm H2O pH 7.48 PaCO2 33 mmHg PaO2 80 mmHg HCO3- 23 mEq/L BE -1 mEq/L The respiratory therapist should recommend: A. decrease mandatory rate to 4 B. increase mandatory rate to 8 C. increase set VT to 500 mL D. increase pressure support to 10 cmH2O

The correct answer is : D Explanation : This patient is obviously weaning, as noted by the SIMV mode and a rate of 6. Their total rate is 26, which is high, and their spontaneous tidal volume is 180 mL, which is far below the minimum 5 mL/kg needed to sustain life. If the patient continues in this fashion the increased work of breathing will ultimately cause weaning failure. To assist the patient, pressure support should be increased. This will increase the size of spontaneous tidal volumes and will ultimately lower the rate and decrease the work of breathing.

A 68-kg (150-lb) male is receiving volume-controlled ventilation secondary to cardiogenic shock. Ventilator settings and arterial blood gas data are below: Mode Assist/control FIO2 0.80 VT 500 mL Mandatory rate 14 Total rate 16 PEEP 22 cm H2O pH 7.30 PaCO2 50 torr PaO2 65 torr HCO3- 25 mEq/L BE +1 mEq/L C.I. 1 L/min/m2 A. add pressure support B. bronchodilator therapy C. increase PEEP D. decrease PEEP

The correct answer is : D Explanation : This patient is under ventilating and is hypoxic. However, examination of hemodynamic data shows cardiac instability as manifested by a low cardiac index. This condition is very dangerous for the patient and suggests that PEEP is too high. Of the options offered a reduction in PEEP is most appropriate.

A 9-year-old girl is receiving mechanical ventilation with an adult circuit with the following arterial blood gases values: pH 7.30 PaCO2 53 mm Hg PaO2 78 mm Hg HCO3- 25 mEq/L BE +1 mEq/L Based on these values, the respiratory therapist should A. change the circuit. B. decrease the rate. C. reduce 50 ml mechanical deadspace. D. increase the rate.

The correct answer is : D Explanation : This patient is under ventilating and under oxygenating. Of these two problems ventilation should be addressed first. This is best done by increasing the rate.

A patient is receiving non-invasive positive pressure ventilation with an IPAP of 16 and an EPAP of 8 cmH2O. The following blood gas data on these settings is available: pH 7.30 PaCO2 52 mm Hg PaO2 80 mm Hg HCO3- 24 mEq/L BE 0 mEq/L Which of the following change is most appropriate? A. decrease EPAP B. increase IPAP and EPAP C. decrease IPAP D. increase IPAP

The correct answer is : D Explanation : This patient is under ventilating but has adequate oxygenation. CO2 may be decreased by increasing the distance between the IPAP and EPAP pressures. This can be done by simply increasing IPAP.

A patient who weighs 75 kg (165 lb) is weaning from mechanical ventilatory support on the following settings: Mode SIMV Mandatory rate 4 Total rate 28 Set tidal volume 450 mL Spontaneous VT 170 mL FIO2 0.40 PEEP 5 cm H2O Which of the following can be done to increase spontaneous tidal volume while decreasing the work of breathing? A. increase PEEP B. decrease machine-delivered minute ventilation C. increase set tidal volume D. start pressure support

The correct answer is : D Explanation : This patient is weaning from the ventilator unsuccessfully. The total rate of 28 is excessive and a spontaneous tidal volume of 170 mL is insufficient to sustain life. The increased respiratory rate and insufficient tidal volume is an indication that the patient is experiencing increased work of breathing and will not likely be able to continue ventilation independently. To assist this patient, pressure support may be used to help increase the size of spontaneous tidal volumes, which will help to lower the rate naturally and ultimately decrease work of breathing, allowing the patient to continue weaning.

A patient is receiving non-invasive positive pressure ventilation with an IPAP of 28 and an EPAP of 12 cmH2O. The following blood gas data on these settings is available: pH 7.37 PaCO2 42 mm Hg PaO2 72 mm Hg HCO3- 24 mEq/L BE 0 mEq/L Which of the following change is most appropriate? A. decrease EPAP B. increase IPAP C. decrease IPAP and EPAP D. increase IPAP and EPAP

The correct answer is : D Explanation : This patient shows adequate ventilation but mild hypoxemia. To correct hypoxemia, expiratory pressure should be increased. However, increasing expiratory pressure alone will decrease the distance between the inspiratory and expiratory pressures and will inadvertently decrease ventilation. This is not desirable because CO2 is already appropriate. To prevent a change in ventilation IPAP must be increased by the same degree that EPAP is increased.

The following airway graphic is observed for a patient receiving mechanical ventilation in the SIMV mode. Which is true concerning the ventilation of the patient? A. the loop is indicative of bronchial cancer B. the patient is receiving PEEP therapy C. the flow-trigger sensitivity is set too low D. the patient has triggered a mandatory breath

The correct answer is : D Explanation : This pressure volume loop shows a "fishtail" pattern. This indicates a significant negative pressure deflection before the machine was triggered to administer a mandatory breath. Although it is normal to have some negative deflection of -1-3 cmH2O, this amount of deflection is too significant for the patient and will ultimately increase work of breathing. The presence of a "fishtail" is an indication that the patient has triggered the breath rather than the machine. If no fishtail exists, the suggestion is that the machine triggered the breath rather than the patient.

Based on the pulmonary function testing below, which of the following represents the best interpretation of the results? Post-bronchodilator Pre-bronchodilator (% of pred) (% of pred) FEF 25-75% 78% 90% FEF 200-1200 80% 92% FVC 82% 95% Peak Flow 74% 90% A. restrictive defect, not responsive to bronchodilator therapy B. mild obstructive defect, non-responsive to bronchodilator therapy C. mixed obstructive and restrictive defect D. normal spirometry, responsive to bronchodilator medication

The correct answer is : D Explanation : This pulmonary function data indicates normal spirometry. The 16% improvement in peak flows suggests the patient is responsive to bronchodilator therapy.

A comatose patient is brought into the emergency department (ER). Which of the following should the respiratory therapist immediately assess? A. pain level B. level of dyspnea C. symptoms D. respiratory rate and pattern

The correct answer is : D Explanation : This question is asking to identify assessments that may be done very quickly. The word "immediately" indicated this. Of the options offered, only an evaluation of color and respiratory rate and pattern will provide quick information. Level of dyspnea and presence of symptoms are subjective data, which requires an alert patient. Since this patient is comatose, this is not possible.

Quality control material has published values for pH of 7.30, PO2 of 85 mm Hg. Documentation indicates the range of pH is 0.5% and is 3% for PO2. Machine results are as follows: pH PO2 (mmHg) Run 1 7.33 83 Run 2 7.25 86 Run 3 7.29 80 Which quality control runs indicate the machine is out of control? A. 1, 2, and 3 B. 1 and 2 C. 1 and 3 D. 2 and 3

The correct answer is : D Explanation : This question is just a matter of math. It takes a few minutes to figure this one out. You must take each result and add and subtract 0.5% and 3% to the pH and PO2, respectively. You're looking for the Runs that are out of control. Runs two and three are the only ones out-of-control.

A patient is found to have the following rhythm on the ECG monitor. Which of the following is the best initial action of the respiratory therapist? A. Defibrillate at 50 joules B. Administer cardiac chronotropic medication C. Administer cardiac inatropic medication D. Provide cardiac pacing

The correct answer is : D Explanation : This rhythm is consistent with a third-degree heart block. Primary treatment for this rhythm is cardiac pacing.

A 64 kg (141 lb) female patient is admitted to the intensive care unit after undergoing a pneumonectomy of the left lung. The patient is receiving oxygen at 100% via bag-valve mask. There are no spontaneous respirations. SpO2 is 97%. Which of the following would be appropriate initial ventilator settings? A. VC, SIMV, rate 10, FIO2 1.0, VT 500 mL, PEEP 5 cmH2O B. VC, SIMV, rate 12, FIO2 1.0, VT 650 mL, PEEP 5 cm H2O C. PC, SIMV, rate 8, IP 34 cm H2O, I-time 1.0 sec, FIO2 0.5, no PEEP D. VC, A/C, rate 16, FIO2 1.0, VT 275 mL, PEEP 5 cmH2O

The correct answer is : D Explanation : Tidal volume should be set to 6-10 mL/kg. However, because a pneumonectomy has been performed, VT should be set at 1/2 the normal volume, or somewhat higher. FIO2 should be set to 100% because the patient is currently receiving an FIO2 of 1.0. Any PEEP of 4-6 cm H2O is adequate and appropriate.

How many portable oxygen E cylinders will be required for a 12-hour car trip with a flow rate of 2 L/min? A. 4 B. 5 C. 2 D. 3

The correct answer is : D Explanation : To answer this question you must know that the tank factor for an E cylinder is 0.28. A full E cylinder is 2200 PSI. Therefore 2200x0.28 is equal to 616 L. 616 divided by two is equal to 308 min. A 12 hour car trip is 720 min. Therefore the patient will need three E cylinders of oxygen to go on a 12 hour trip.

Which of the following values are required to determine a patient's inspiratory capacity (IC)? A. VC and RV B. ERV and IRV C. RV and ERV D. VT and IRV

The correct answer is : D Explanation : To answer this question, one must look at every option and mentally determine if the desired result can be reached. In this case, tidal volume can be added to inspiratory reserve volume (VT + IRV) to achieve inspiratory capacity (IC).

The respiratory therapist has an order to cap a patient with a fenestrated tracheostomy tube in preparation for speech therapy. Prior to applying the cap on the end of the tube, the therapist should also ensure A. the inner cannula is in place and the cuff deflated B. the inner cannula is in place and the cuff inflated. C. the inner cannula is removed and the cuff inflated D. the inner cannula is removed and the cuff deflated

The correct answer is : D Explanation : To configure a fenestrated tracheostomy to for speaking, the button must be removed, the inner cannula must also be removed, and the cuff should be deflated. This allows air to be deviated through a hole in the fenestrated tube and pass the vocal folds, allowing speech.

A physician suspects a patient has a restrictive pulmonary defect. Which of the follow tests would be most helpful at determining this? A. Forced vital capacity B. Fev1.0/FVC% C. Fev1.0 D. Slow vital capacity

The correct answer is : D Explanation : To determine if the patient has a restrictive pulmonary defect, pulmonary function volumes must be obtained. Specifically, slow vital capacity (SVC) or forced vital capacity (FVC) are used for this evaluation. An SVC is more accurate than a FVC. SVC should be chosen first if given the option.

A patient is receiving mechanical ventilator support with a Hamilton adult ventilator. The following data is available: pH 7.40 PB 747 torr PaCO2 40 torr PaO2 100 torr HCO3- 24 mEq/L BE +0 mEq/L CaO2 17.5 vol% CvO2 13.0 vol% FIO2 0.50 Mode SIMV Rate 12 VE 6.6 L PIP 24 cm H2O PEEP 5 cm H2O The respiratory therapist should report which of the following values as an accurate A-aDO2? A. 165 torr B. 250 torr C. 380 torr D. 200 torr

The correct answer is : D Explanation : To determine the alveolar-arterial oxygen difference or gradient one must first calculate the alveolar oxygen tension by using the alveolar air equation or by a shortcut. To determine PAO2, first take the oxygen percentage and multiply it by 7. 50% x 7 = 350. Next, subtract the CO2 + 10. 350 - 50 = 300 mmHg. Finally, if you subtract the PaO2 you get 200 mmHg, which represents the A-aDO2

The physician has asked the respiratory therapist to help determine oxygenation at the tissue level. The therapist should suggest obervation of which of the following? A. PAO2 B. QS/QT C. CvO2 D. CaO2

The correct answer is : D Explanation : To determine the amount of oxygen available to the tissues at the tissue level, we must look at the oxygen that is bound with hemoglobin, in addition to the oxygen that is dissolved the plasma. Because the oxygen bound with hemoglobin represents the most significant portion of oxygen in the blood, determing the arterial oxygen content is most helpful in determining the amount of oxygen available at the tissue level.

A bedside PFT is completed on a patient who is being evaluated for effects and progress of Myasthenia Gravis. Reproducible results are: FVC FEV1 SVC Trial 1 (L) 2.0 1.1 2.1 Trial 2 (L) 2.1 1.0 2.2 Trial 3 (L) 2.1 1.2 2.2 Trial 4 (L) 2.1 1.0 2.2 Which is the best trial? A. 4 B. 1 C. 2 D. 3

The correct answer is : D Explanation : To determine the best test, multiple FVC maneuvers should be done. Attempts should not differ by more than 5%. The highest sum of FEV1 + FVC is considered the "best test". In this case FVC 2.1 + Fev1 1.2 = 3.3

The respiratory therapist is determining the proper length of a nasopharyngeal airway. Which of the following methods will be helpful in properly estimating the appropriate size of the airway? A. determine the age and sex of the patient B. examine the distance between the angle of the jaw and tip of the chin C. determine the patient's ideal body weight in kilograms D. examine the distance between the earlobe and nasal septum

The correct answer is : D Explanation : To determine the proper length of a nasopharyngeal airway, also called a nasal trumpet, the distance between the earlobe and the nasal septum should be measured and matched roughly to the length of the nasal trumpet.

The following data is available for a patient receiving volume-controlled mechanical ventilation: Mode Assist/control FIO2 0.5 VT 400 mL Mandatory rate 16 PEEP 5 cm H2O PIP 30 cm H2O PetCO2 30 torr pH 7.40 PaCO2 40 torr PaO2 80 torr PvO2 60 torr HCO3- 22 mEq/L BE -2 mEq/L SaO2 99% SvO2 75% Hb 15 g/dL The respiratory therapist should record which of the following values as an accurate CvO2? A. 12.1 vol% B. 17.0 vol% C. 2.9 vol% D. 15.3 vol%

The correct answer is : D Explanation : To determine the venous oxygen content, the hemoglobin must be multiplied by the saturation which is then also multiplied by a factor of 1.34. In this case hemoglobin x 1.34 is approximately 20. 20 multiplied by 0.75 is approximately 15 vol%. Because we're using shortcuts the exact value in the answers may not be seen. However, it is suitable to pick the closest answer, which is 15.3 vol%_x000D_

A patient who is receiving mechanical ventilation in the assist/control mode with a PEEP of 10 cmH2O is being manually ventilated during a transport to CT scan. During the transport, the respiratory therapist notices desaturation and feels the PEEP should be increased. To do this, the therapist should A. increase oxygen flow rate to the bag B. cover the exhalation port C. squeeze the bag at a higher rate D. increase the tension on the PEEP spring

The correct answer is : D Explanation : To increase the PEEP of the patient receiving positive pressure ventilation by manual resuscitation bag, one must tighten or increase the tension of the PEEP spring. To do this, the PEEP valve should be turned clockwise.

A respiratory therapy supervisor is responsible for making suggestions in the initial development of a smoking cessation program. Which of the following would be part of an effective smoking cessation program? A. daily COHb analysis B. daily completion of an asthma action plan C. removing pulmonary irritants from the house D. Nicotine replacement therapy

The correct answer is : D Explanation : To stop smoking, both physiological and emotional barriers must be addressed. The patient may be helped best through nicotine replacement therapy and emotional support. Routine monitoring of the patient's progress is also helpful. Removing irritants from the house will not help the patient stop smoking.

A respiratory therapist auscultates a 3-year-old child in respiratory distress in the emergency department (ED). Auscultation reveals unilateral wheezing on the right and vesicular breath sounds on the left. The therapist should prepare which of the following to help the patient? A. a small volume treatment with Albuterol B. aerosolized racemic epinephrine C. aerosolized Ipratropium Bromide (Atrovent) D. a bronchoscope

The correct answer is : D Explanation : Unilateral wheezing in a small child is usually associated with foreign body aspiration. When the patient wheezes, normally, bilateral bronchoconstriction is present. Therefore, unilateral wheezing is more likely caused from an object or food in the airway on the affected side. To resolve this, a bronchoscope will both diagnose the problem and facilitate extraction of the object.

Which of the following may be helpful at relieving upper airway inflammation following endotracheal extubation? A. methylprednisolone B. Albuterol C. Azmacort (Triamcinolone acetonide) D. racemic epinephrine

The correct answer is : D Explanation : Upper airway inflammation may be treated with cool aerosol therapy or racemic epinephrine. If severe, intubation may be required. Other medications listed are not appropriate to reduce upper airway swelling.

A 42-year-old patient has undergone a laryngectomy for the removal of laryngeal cancer. A laryngectomy tube is in place. For the first 12 hours after surgery, the therapist should expect which of the following?

periodic expectoration of blood

A large-volume aerosol used as a direct gas source for an infant oxygen hood is associated with which of the following complications? A. low moisture B. unstable temperature C. unstable FIO2 D. hearing damage

The correct answer is : D Explanation : Use of a large-volume nebulizer with an oxygen hood can sometimes be of concern because of the sound the nebulizer may transmit to the oxygen hood. When this happens hearing damage may be caused and/or the infant may become restless. The most appropriate adjustment is to utilize an oxygen-air blender and set the large-volume nebulizer at 100% in order to avoid entraining air, which will help reduce sound.

A 90-year-old patient is having difficulty squeezing the metered-dose inhaler (MDI) in timing with inspiration. Which of the following devices should the respiratory therapist recommend: A. automatic MDI actuator B. a larger MDI canister C. stop watch D. spacer/chamber

The correct answer is : D Explanation : Use of an MDI requires a coordinated discharge and sufficient manual dexterity. For patients who may have difficulty with this, and to generally improve particle deposition, use of a spacer or chamber is helpful. The chamber reduces the need to time the discharge perfectly.

A patient with the following blood gas results has an end-tidal CO2 of 30 mm Hg. pH 7.38 PaCO2 40 mm Hg PaO2 80 mm Hg HCO3- 25 mEq/L What is the VD/VT? A. 62% B. 50% C. 30% D. 25%

The correct answer is : D Explanation : VD/VT = (PaCO2 - PetCO2) / PaCO2. In this case, VD/VT = (40-30)/40 = 10/40 or 25%.

A patient is receiving volume-controlled ventilation in the emergency department (ED). The following data is available: FIO2 0.5 VT 600 mL Mode Assist/control Mandatory rate 10 PEEP 5 cm H2O PetCO2 30 torr PaCO2 40 torr pH 7.39 PaO2 100 torr HCO3- 25 mEq/L BE +1 mEq/L The respiratory therapist should report which of the following as an accurate VD/VT ratio? A. 50% B. 75% C. 15% D. 25%

The correct answer is : D Explanation : VD/VT ratio calculation is (PaCO2-PetCO2)/PaCO2. In this case 40 -30= 10. 10/40 = 25%

The respiratory therapist should recommend which of the following to help evaluate a patient with suspected vocal cord paralysis? A. complete inhalation followed by completed forced exhalation B. slow vital capacity maneuver C. maximum voluntary ventilation for 12 seconds D. flow volume loop maneuver

The correct answer is : D Explanation : Vocal cord paralysis and other fixed upper airway obstructions are best diagnosed by doing a flow-volume loop pulmonary function test. If present, the flow-volume loop will be described as "round" in shape. This is because a fixed obstruction causes an equal amount of resistance on both inhalation and exhalation.

Which of the following would be most helpful in diagnosing vocal cord paralysis in an adult? A. FVC maneuver B. Thoracic gas volume measurement C. Neck radiograph D. Flow volume loop

The correct answer is : D Explanation : Vocal cord paralysis will cause both obstruction and restriction as airway resistance is increased in both the inspiratory and expiratory phases of breathing. This can be best observed through a pulmonary function maneuver called the flow volume loop. The loop will appear round in shape.

The galvanic-type oxygen analyzer is reading 5% when the respiratory therapist is attempting to calibrate it with 100% oxygen. To remedy the problem, the therapist should do which of the following? A. Change the electrolyte solution B. Change the batteries C. Discard the analyzer and get a new one D. Change the fuel cell

The correct answer is : D Explanation : When a galvanic-type oxygen analyzer is reading extremely low or zero, the problem is most likely related to a depleted power source. Because the analyzer does not have batteries but receives power from the probe (the fuel cell), the most appropriate action is to change the fuel cell. If this analyzer, however, is only slightly off the actual FIO2, , it is more likely the analyzer requires a high and low calibration.

A patient is receiving heliox therapy at 60%/40% mixture by nonrebreathing mask. The oxygen flow meter is set to 10 L/min. The patient has minute ventilation of 12 L/min. What is the total flow of the heliox gas mixture to the patient? A. 16 L/min B. 22 L/min C. 2 L/min D. 14 L/min

The correct answer is : D Explanation : When a helium-oxygen gas mixture is run through an oxygen flow meter, the indicated flow on the flow meter is less than the actual total flow of gas. This is because helium is less dense than oxygen. Therefore, each helium-oxygen mixture percentage has a correction factor. For a 60%/40% mixture, the correction factor is 1.4. In other words, the indicated oxygen flow of 10 L/min is multiplied by the correction factor of 1.4 to get a total gas flow of 14 L/min.

While performing initial suctioning of an infant who has aspirated meconium, the respiratory therapist notices the endotracheal tube is clogged. The therapist should A. apply continuous suction to the airway until the obstruction clears B. increase suction pressure, attempt to clear the obstruction C. clean the airway with normal saline D. remove the airway and insert a new one

The correct answer is : D Explanation : When a infant's endotracheal tube becomes clogged with secretions, it is not appropriate to dedicate significant effort in clearing the endotracheal tube, but rather a change of the tube is indicated. This is true even if the endotracheal tube must be replaced several times.

A physician suspects a patient has a cancerous mass in the right lobe as identified by a PA chest radiography. Which of the following should the therapist recommend to examine the nature and size of the mass? A. pulmonary angiography B. V/Q scan C. A-P chest radiograph D. series of oblique chest radiographs

The correct answer is : D Explanation : When a large round-shaped infiltrate is observed on a PA or AP x-ray, more information should be gathered. The suspicion would be that the patient has a cancerous mass in the lungs. This can, in part, be confirmed by doing another x-ray from a different perspective. A series of x-rays taken from different angles will help reveal the three-dimensional shape of the infiltrate. If the infiltrate is round shaped from other perspectives as well, the infiltrate is likely a mass of some sort. A biopsy would be indicated to determine if the mass is cancerous or benign.

A point-of-care blood gas analyzer produces results well outside the machines physiological reportable range. The respiratory therapist should A. examine the patient for consistent clinical signs B. replace the sample cartridge C. suspect a venous blood sample D. perform a 2-point calibration

The correct answer is : D Explanation : When any blood gas analyzer produces erroneous results that are outside the reportable physiological range, the device requires calibration. These types of devices offer one-point, and two-point calibrations. In this case, a two-point calibration would be more appropriate so that the machine is calibrated to both the high and low aspect of the range.

A patient on long-term supplemental oxygen via a transtracheal oxygen catheter is hospitalized on a general observation floor. The patient complains of shortness of breath and indicates the device is not functioning properly. The respiratory therapist should A. suction the catheter, recheck oxygen saturation B. remove the transtracheal catheter, begin mechanical ventilation C. administer aerosolized Ipratropium Bromide (Atrovent) by mask D. begin supplemental oxygen delivery by nasal cannula

The correct answer is : D Explanation : When a patient complains of shortness of breath or complains that the oxygen delivery equipment is not working properly, the best first option is to ensure oxygenation by changing to another device and then troubleshooting the problem.

Arterial blood gas results on an adult patient breathing room air is recorded in the patient's medical record. The sample was taken when the axillary temperature was 103 deg F (39.4 deg C). The results were not corrected for temperature when analyzed. The results are as follows: pH 7.35 PaCO2 45 torr PaO2 80 torr HCO3- 24 mEq/L BE -1 mEq/L FIO2 0.21 The respiratory therapist should recommend which of the following? A. Rerun the sample B. Monitor the patient C. Redraw the patient when no longer febrile D. Place the patient on oxygen at 5 L/min nasal cannula

The correct answer is : D Explanation : When a patient has a fever and blood gas results are not corrected for temperature, the reported results are erroneously high. This means that the reported PaO2 of 80 mmHg is higher than the actual PaO2 of the patient. This suggests that the real PaO2, which must be less than 80 mmHg, must be at a hypoxic level and therefore supplemental oxygen is indicated. The appropriate amount of oxygen is between 40 and 55%, which can be accomplished by a nasal cannula running at 5 L/min.

A patient is found apneic in his room. The period of apnea is at least 3 minutes. Following intubation, the patient is manually ventilated with a bag and an end-tidal CO2 detector in place. Which of the following PetCO2 patterns will the therapist expect to observe? A. high at first and persisting for several minutes of ventilation B. fluctuation of high to low for several cycles C. high at first, followed by a steady decrease D. low at first, followed by a steady rise

The correct answer is : D Explanation : When a patient has been apneic for several moments, the initial end-tidal CO2 reading will be low followed by a steady rise as ventilation occurs. The initial end-tidal reading is low because CO2 does not move freely from the blood across the alveolar capillary membrane into the alveoli when the patient is not breathing.

Which of the following laboratory examinations is most appropriate to further evaluate a patient with kidney failure and CHF? A. Theophylline level B. BUN C. urinalysis D. creatinine

The correct answer is : D Explanation : When a patient has congestive heart failure, one of the concerns is fluid shifting in the body. This fluid shifting shows up in the form of pitting edema, among additional physical signs. Not only does this relate to cardiac condition, it has implications regarding the kidney. There are two examinations that are helpful in evaluating this condition. One is creatinine and the other is BUN, or blood urea nitrogen. Both are good tests but creatinine is considered more accurate than BUN.

A patient with a history of status asthmaticus is admitted to the emergency room (ER) for the treatment of acute bronchoconstriction. After two treatments with aerosolized Albuterol, the patient demonstrates no relief. Peak flow measurements are unchanged. The respiratory therapist should recommend which of the following? A. switch to corticosteroid administration B. administer cromolyn sodium (Intal) C. repeat the Albuterol treatment, then reassess D. administer ipratropium bromide (Atrovent)

The correct answer is : D Explanation : When a patient is not responsive to a particular bronchodilator medication, it is appropriate to attempt bronchodilation with a different medication. In this case, Atrovent is a good alternative medication because it is also a short term rescue medication, like albuterol, but with a different mode of action.

A respiratory therapist notices the reservoir bag on a nonrebreathing mask collapses completely every time the patient inspires. The respiratory therapist should A. obtain a larger mask B. switch to a high-flow Venturi mask C. switch to a partial rebreather mask D. increase flow to the mask

The correct answer is : D Explanation : When a patient is using a non-rebreather mask, the reservoir should collapse slightly with each breath. The collapse indicates that the seal between the mask and the patient's face is adequate. If the reservoir collapses completely, this indicates there is not enough flow of oxygen to the reservoir bag to keep up with the patient's inspiratory demand. Increasing flow to the reservoir, or mask, is most appropriate.

A home-care patient complains to the respiratory therapist that the pulsed-dose oxygen delivery system does not appear to be working properly. The patient complains of shortness of breath. The therapist should first A. ensure the sensitivity is set appropriately for the patient's needs B. examine the flow from the regulator with a calibration device C. replace the double lumen nasal catheter D. place the patient on a nasal cannula with a continuous flow regulator and an E cylinder

The correct answer is : D Explanation : When a problem is encountered with one oxygen delivery modality, the first best response is to initiate oxygen therapy with a different modality and then troubleshoot the problem.

Which of the following should the respiratory therapist observe to confirm a chest radiograph was taken with proper technique? A. heart shadow should not be more than 1/2 width of the chest cage B. cardiac vessels can be seen clearly C. the spinal process should be slightly off-center D. spaces between vertebrae should be distinct

The correct answer is : D Explanation : When a respiratory therapist examines a chest x-ray he or she should observe specific landmarks to ensure the x-ray was done correctly and that the quality of the data is appropriate. The tops of the lung apecies should be level, spaces between the vertebrae should be distinct, and the clavicle bones should be level.

After running a blood gas, the respiratory therapist has concerns about the accuracy of the reportable physiological range of the PO2 electrode in the blood gas machine. The therapist should A. perform a one-point calibration B. replace the Sanz electrode C. replace the PO2 electrode D. perform a two-point calibration

The correct answer is : D Explanation : When a respiratory therapist has concerns about the accuracy of the result in the reportable range of a blood gas machine, performing a two-point calibration can correct the problem. A two-point calibration adjusts the machine at a high and low level which is appropriate when we are concerned about the accuracy of a range.

Blood pressure obtained by a sphygmomanometer reads higher than the indicated blood pressure from an arterial line on the same patient. To correct problem, the respiratory therapist should FIRST A. discontinue use of the sphygmomanometer B. flush the art line with sodium heparin C. advance the arterial line catheter D. check for air bubbles in the transducer dome

The correct answer is : D Explanation : When blood pressure by sphygmomanometer (cuff) differs from the blood pressure obtained from an indwelling arterial line, the blood pressure taken by cuff is considered more accurate and reliable. Problems that may occur in the ART line include clots in the line and bubbles in the transducer dome.

The respiratory therapist notices the blood pressure of a patient is higher when measured by a blood pressure cuff on the right arm compared to the blood pressure value obtained from an arterial line and transducer. The therapist suspects pressure dampening in the art line. Which of the following may be the cause of the pressure dampening? A. poorly calibrated art line B. kinked art line catheter C. movement of the patient's arm D. air bubble in the arterial line transducer

The correct answer is : D Explanation : When blood pressure taken by a blood pressure cuff differs from that which is observed by an arterial line, the blood pressure by cuff is considered to be more accurate. The arterial line may have a variety of problems that affect its accuracy. Air bubbles in the transducer dome or a clot in the line are examples that may affect the accuracy of blood pressure that comes from an arterial line.

Immediately following oral intubation, the respiratory therapist auscultates the chest and finds breath sounds are more pronounced on the right. The therapist should do which of the following? A. advance the ET tube by 1 cm B. order a chest x-ray C. perform minimal leak technique D. withdraw the ET tube by 1 cm and auscultate breaths sounds

The correct answer is : D Explanation : When breath sounds are more pronounced on the right for a patient who is receiving mechanical ventilatory support and is orally intubated, it is likely the endotracheal tube has inadvertently entered the right mainstem bronchus. The endotracheal tube may be removed by 1-3 cm and then breath sounds may be rechecked. This approach is helpful to quickly determine the ET tube's position but a chest x-ray will be required to confirm its position.

Excessive bubbling is noted in the water seal chamber of a chest tube drainage system. All connections are secure. The respiratory therapist should first A. disconnect and submerge the tube in water. B. reduce the suction pressure. C. replace the tubing. D. clamp where the tube exits the patient.

The correct answer is : D Explanation : When excessive bubbling is noted in the water-seal compartment of a chest tube drainage system, it is most likely caused from a leak somewhere between the water-seal compartment and the patient's lung tissue. To begin troubleshooting and find a leak a clamp should be placed on the tubing where it exits the patient. If excessive bubbling ceases then the leak must be inside the patient and is probably caused from a perforation in the lung tissue. If this is the case the patient should be sent to surgery. The clamp should be moved closer to the water seal compartment to determine if a leak is found in the tubing of the chest tube drainage system.

Which of the following is an important element of instruction to a home care patient who will be receiving continuous oxygen therapy at 2 L/min by nasal cannula? A. check oxygen concentrator flow rate once a week B. increase flow rate during exercise C. run a flow calibration check once monthly D. do not change flow rate without a physician order

The correct answer is : D Explanation : When instructing a home care patient on the use of their oxygen concentrator, and using oxygen in general, the respiratory therapist must include those things that are appropriate for the patient. In this question no open flames, use of grounded plugs only, and a restriction on changing the flow rate without a physician order are appropriate instructions. It is inappropriate to ask the patient to check or calibrate the concentrator's flow rate once a week. This sort of maintenance should be done by a respiratory therapist.

A 68 year-old female patient has the following pulmonary function values: % of predicted Actual value Fev1/FVC% 54% SVC 85% FEF25-75 75% FEF200-1200 79% DLCO 88% Which of the following could represent the patient's diagnosis? A. mesothelioma B. kyphoscoliosis C. emphysema D. asthma

The correct answer is : D Explanation : When looking at pulmonary function data the primary purpose is to differentiate between an obstructive defect and a restrictive effect. To determine if the patient is obstructive, we must look at flows. To determine if they are restrictive we must look at volumes. In this case flows are obviously reduced with an Fev1/FVC% of 54%. A slow vital capacity shows the patient is not restrictive. When we look at the answers we see two obstructive diseases - asthma and emphysema. To determine which one is correct we must look back at the DLCO. Only emphysema has an abnormal DLCO. This patient has a normal DLCO which means the patient has asthma.

A 60 year-old male patient has the following pulmonary function values: % of predicted Actual Value Fev1/FVC% 58% FVC 81% FEF25-75 79% FEF200-1200 80% DLCO 15 Which of the following could represent the patient's diagnosis? A. pulmonary fibrosis B. sarcoidosis C. chronic bronchitis D. emphysema

The correct answer is : D Explanation : When looking at pulmonary function data the primary purpose is to differentiate between an obstructive defect and a restrictive effect. To determine if the patient is obstructive, we must look at flows. To determine if they are restrictive we must look at volumes. In this case, flows are definitely diminished as seen in the Fev1/FVC% of 58% of predicted. Volumes appear to be normal with an FVC of 81% of predicted. Thus, the patient is obstructive but not restrictive. The answers show two obstructive diseases as possibilities - emphysema and chronic bronchitis. To determine which is correct the DLCO must be examined. A DLCO of less than 80% of predicted is indicative of a patient with emphysema.

A respiratory therapist is transporting a patient in a fixed-wing aircraft. Prior to the flight the patient was stable, but since arriving at altitude, the patient is exhibiting cyanosis, shortness of breath, and anxiety. Which of the following could explain this change? A. increased barometric pressure B. altitude sickness C. decreased FIO2 D. decreased PAO2

The correct answer is : D Explanation : When patients experience an increase in altitude, hypoxemia may occur due to the decrease in barometric pressure. This decrease in pressure first and foremost decreases the oxygen tension in the alveoli or the PAO2.

A respiratory therapist is communicating with a 4-year-old patient to determine the level of pain currently being experienced by the patient. Which of the following methods would be most effective?

Use the Wong-Baker faces and ask the child to point to correct level

While performing endotracheal suctioning with a closed-system suction catheter on a patient who is intubated with a 7.5-mm ET tube, the respiratory therapist notices the suction at the tip of the catheter suddenly stops. Which of the following should the therapist check? A. hospital vacuum system B. patency of the ET tube C. integrity of the suction catheter baggy D. collection canister

The correct answer is : D Explanation : When the collection canister of a suction system becomes full, a valve actuates and interrupts suction. To remedy this, the disposable suction bag or container must be replaced.

An adult patient with asthma is receiving Xopenex by small volume nebulizer T.I.D. at a dosage of 1.25 mg. The patient complains of dizziness, tingling in his fingers, and anxiety with each treatment. The therapist should A. decrease frequency to BID B. switch to Pulmicort 0.5 mg C. increase dose to 1.88 mg D. change dose to 0.63 mg

The correct answer is : D Explanation : When the patient has an adverse reaction to a specific medication, the therapist must modify the therapy. Modifying therapy means to change to a different method or modality that accomplishes the same objective. In this case, the best option is to reduce the dose of Xopenex to 0.63 mg. Xopenex comes in 2 dosages that are appropriate for adults - 1.25 mg and 0.63 mg. If a patient is having adverse reaction at the higher dose, the lower dose may be attempted. Although not offered here, it would also be appropriate to change medications to another bronchodilator such as albuterol or Atrovent.

A therapist would expect to see which of the following capnographic data after providing the first few manual ventilations on a patient who has been apneic for 4 minutes? A. high PetCO2 B. low PaCO2 C. unreadable PetCO2 D. low PetCO2

The correct answer is : D Explanation : When the patient has not been breathing for some time and is placed on an end-tidal CO2 monitor (a capnograph), the initial end-tidal CO2 reading would be low. This is because the patient has not been breathing and therefore CO2 has not crossed the alveolar capillary membrane. Only when ventilation is occuring does the CO2 began to cross that membrane, fill up the alveoli with CO2, and then be shown on an end-tidal CO2 monitor.

A patient receiving 1 L/min oxygen by TTO complains of air hunger. The therapist should FIRST A. flush the trans-tracheal catheter B. connect suction pressure directly to the catheter C. place the patient on 1 L/min nasal cannula D. place the patient on an air-entrainment mask at 28%

The correct answer is : D Explanation : When the patient is receiving oxygen by trans-tracheal oxygen catheter and a chang in modality is required, the flow rate and oxygen percentage required is double that which was administered by the transtracheal device. For instance, in this case the patient was receiving 1 L per minute by transtracheal catheter. This means that by nasal cannula the patient should receive 2 L a minute, twice that which was delivered by the transtracheal oxygen catheter. However, 2 L/min is not offered in these options. The most appropriate oxygen percentageoffered would be 28%, which is equivalent to 2 L/min nasal cannula.

A respiratory therapist is repeating pulmonary function testing on a patient who had initial pulmonary function testing 6 months ago. With the first FVC attempt, the therapist notices the patient was only able to accomplish an FEV1.0 sec of 50% of predicted. Six months prior, the patient was able to reliably accomplish 85% of predicted. The therapist should A. reject the results as they have not met ATS standards B. change the helium analyzer C. report the significant decline as exacerbation of COPD D. repeat the effort, encourage the patient to increase effort

The correct answer is : D Explanation : When the pulmonary status of the patient degrades, it happens slowly. It is not likely that a patient's forced vital capacity would drop from 85% of predicted to 50% of predicted within six months. It is more likely that variation of patient effort is being observed. The patient should be instructed to reattempt the maneuver and apply greater effort.

A COPD patient has a blood gas pH of 7.35, PaCO2 58 mmHg, PaO2 55 mmHg, HCO3- 28 mEq/L. Which of the following is most indicative of COPD? A. hypercapnia B. hypoxemia C. hypercarbia D. metabolic compensation

The correct answer is : D Explanation : When we think of COPD patients, it is easy to think that the elevated CO2 is what defines their disease. While COPD patients are chronic CO2 retainers the real data that indicates they are COPD is the normalized pH. This indicates the chronic nature of their condition. In this question, the most appropriate answer is metabolic compensation because that is the means by which the pH is corrected.

What is needed to determine arterial oxygen content? A. Oxygen index B. P/F ratio C. PvO2 D. Hemoglobin level

The correct answer is : D Explanation : You must know what is required to calculate arterial oxygen content. If you look at the formula you'll realize you need to know arterial saturation, hemoglobin level, and PaO2 only. Venous saturation is not relevant, but would be relevant if we were calculating venous oxygen content.

Which of the following would most likely increase the possibility for nosocomial infection? A. infrequent changing of the ventilator circuit B. use of closed in-line suction catheters C. having an HME inline for 96 hours D. use of multi dose bronchodilator medication

The correct answer is : D Explanation : Of the options given, use of a multi-dose bronchodilator medication would be most likely to increase the possibility or rate of nosocomial infection. This practice has faded in recent years but respiratory therapist commonly carried large bottles of medications with droppers in them to deliver to an individual patient. Because the dropper, which is sterile, could enter the field of the patient's small volume nebulizer, pathogenic organisms could be transmitted. Since this discovery, the field has moved towards unit dose medications in disposable containers. Having an HME in-line for 96 hours is not advisable but would not result in an increase of nosocomial infections, which is an infection received from another patient.

Which of the following would be a good goal to be included in a pulmonary rehabilitation program for a COPD patient? A. reduce infections B. omit ALDs C. restore normal ABG values D. return to normal life

The correct answer is: A Explanation : The purpose of a pulmonary rehabilitation program for a patient with COPD relates to improving their life from their point of view. Legitimate goals include recognizing signs of infection, reducing hospitalizations, increasing their ability to perform activities of daily living, and generally increasing exercise tolerance. Because lung disease cannot be reversed it is unreasonable to think that we can return the patient to normal life or reduce their need for oxygen. It is also unreasonable to achieve normal blood gas or pulmonary function values.

The respiratory therapist is caring for a 28-week-old gestational age infant currently receiving mechanical ventilatory support on the following settings: PIP 30 cm H2O FIO2 0.70 PEEP 5 cm H2O Mandatory rate 40 A chest radiograph shows bilateral haziness. Arterial blood gas analysis shows pH 7.22 PaCO2 67 torr PaO2 44 torr HCO3- 27 mEq/L BE +1 mEq/L The most appropriate recommendation would be to A. Decrease mandatory rate B. Increase FIO2 C. Increase PIP D. Administer Exosurf®

You did not answer this question. The correct answer is : D Explanation : Bilateral haziness observed on the chest x-ray, in conjunction with the fact that the infant is premature, is consistent pulmonary prematurity. Pulmonary prematurity may be treated by administering pulmonary surfactant. Surfactant decreases the surface tension of the alveoli, which allows and promotes explansion and causes improved gas exchange.

A patient receiving mechanical ventilator support is on the following settings and has the following clinical data: Mode SIMV Mandatory rate 14 VT 500 mL FIO2 0.65 PEEP 20 cm H2O Arterial blood gas results pH 7.38 PaCO2 43 torr PaO2 68 torr HCO3- 26 mEq/L BE +2 mEq/L Which of the following should the respiratory therapist recommend? A. increase FIO2 to 1.0 B. decrease PEEP to 18 cm H2O, increase FIO2 to 1.0 C. increase FIO2 to 0.8 D. increase PEEP to 22 cm H2O

You did not answer this question. The correct answer is : D Explanation : Examination of the arterial blood gas reveals adequate ventilation with hypoxemia. To correct the PaO2, FIO2 or PEEP should be increased. When we look at these two parameters we see FIO2 is already at or above 60%. In this case, PEEP should be increased next.

While assisting in the performance of a cardiac stress test, the respiratory therapist notes the following in the medical record after increasing the tension on a stationary bicycle: before increase in workload after increase in workload HR 110/min 130/min BP 132/90 mm Hg 152/92 mm Hg SpO2 97% 95% The respiratory therapist should anticipate

a further increase in workload

While purposefully kinking a bubble-humidified nasal cannula momentarily while in use on a patient receiving oxygen at 2 L/min, the respiratory therapist notices no audible response from the system. Which of the following could be the cause?

a loose reservoir

A 6-minute walk test is used to evaluate

a patient's physical stamina and aerobic endurance.

A patient is being monitored with a PA catheter. The PA waveform shows a missing dicrotic notch. The respiratory therapist should consider

a possible clot in the PA catheter

A 74-kg (166-lb) 5-ft 8-in (173-cm) female is receiving mechanical ventilation by a volume-controlled ventilator in the assist/control mode on the following settings: Mandatory rate 12/min VT 500 mLFIO2 0.5PEEP 5 cmH2O Arterial blood gas results: pH 7.47PaCO2 34 mmHgPO2 80 mmHgHCO3- 23 mEq/LBE -1 mEq/L Which of the following represents the most appropriate action?

add 100 mL deadspace

A patient who has been orally intubated, for a surgical procedure, demonstrates moderate stridor immediately after extubation. Prior to surgery, the anesthesiologist determined the patient's Mallampati classification to be 3. The respiratory therapist should recommend

administering aerosolized racemic epinephrine.

A patient in the intensive care unit (ICU) is receiving mechanical ventilation following a massive heart attack where injury and infarction were sustained. The following data is available: Mode VC, SIMV FIO2 0.50VT 500 mLf 14/minPEEP 5 cm H2O PCWP 12 mm HgCVP 8 mm HgmPAP 20 mm HgC.O. 7.9 L/min ABG pH 7.36PaCO2 49 mm HgPaO2 82 mm HgHCO3- 25 mEq/LBE +1 mEq/L The respiratory therapist should recommend

administration of a loop diuretic.

The following pressure-volume waveform is observed on a 32-year-old patient who is receiving VC A/C ventilation following abdominal surgery. When originally instituted, the waveform demonstrated a small bird's beak pattern. Two hours later, the beak formation has disappeared. VT has stayed consistent at 8 mL/kg. SpO2 remains unchanged at 95%. Which of the following can the respiratory therapist conclude?

alveoli have likely been recruited

A sputum gram stain report indicates the presence of a gram-positive organism (diplococcus) in the sputum. The following data is available: WBC 28,000 cu mm Hb 14.5 g/dL RBC 4.6 g/dL HCT 42% Which of the following medication would be most appropriate?

amoxicillin

A patient with an IBW of 62 kg is receiving VC A/C ventilation with the following settings and corresponding arterial blood gas results: FIO2 0.65 Mandatory rate 15VT 400 mLPEEP 5 cm H2OI:E 1:2 pH 7.36PaCO2 35 torrPaO2 51 torrHCO3- 20 mEq/LBE -5 mEq/L The respiratory therapist should recommend

an increase in PEEP to 10 cm H2O

During a difficult intubation, the emergency room physician requests an endotracheal tube that is smaller than recommended for the patient's height and ideal body weight. As a result, the respiratory therapist should anticipate

an increase in RAW

A respiratory therapist is reviewing the status of a 65-year-old patient who has been treated for obstructive sleep apnea for 3 years. The patient is recently complaining of daytime sleepiness and takes a nap in his car during his lunch break. He is currently receiving nasal CPAP at 10 cm H2O. Recently ordered polysomnography reveals multiple periods of apnea with no chest or abdominal movement. The therapist should recommend

bi-level positive airway pressure

A 5-ft, 2-inch (157-cm), 208-lb (95-kg) male patient has just been diagnosed with obstructive and central sleep apnea. Which of the following will provide most help to the patient?

bi-level therapy with a back up rate

A Bronchogram would be most helpful in evaluating and diagnosing which of the following?

bronchiectasis

A pediatric patient has a non-productive cough. A chest radiograph reveals a foreign body in the right mainstem bronchi. Which of the following should be used to alleviate the problem?

bronchoscope

A patient who is orally intubated requires frequent endotracheal suctioning through an 8.0 ET tube with a 12 Fr. catheter at a suction pressure of 135 mmHg. During suctioning the following ECG tracing can be seen on the monitor. Which of the following should the respiratory therapist recommend? (the rhythm is abnormal)

decrease suction pressure

An 80-kg (176-lb), 5-ft, 9-in (175-cm) adult male is orally intubated with an 8.0-mm endotracheal tube and a 12 Fr. catheter. During suctioning the patient demonstrates sinus tachycardia with frequent PVCs on the ECG monitor. The therapist should do which of the following with future suctioning attempts?

decrease suction time

When auscultating the heart of a newborn, delivered by C-section at 34 weeks-of-gestation, the respiratory therapist observes a murmur. What follow-up exam should be ordered?

echocardiography

To best assure that the blood pressure reading from an in-dwelling arterial catheter is accurate, the respiratory therapist should

ensure the transducer is level with the heart

Which of the following would be most helpful at impeding and/or stopping bleeding occurring within the mainstem bronchus caused by tissue extraction with a bronchoscope?

epinephrine

Under which of the following circumstances should a respiratory therapist consider discontinuing a PEP therapy treatment of a child?

epistaxis and middle ear infection

A respiratory therapist is calibrating a transcutaneous electrode and monitor prior to patient use on a newborn, premature infant. For a CO2 calibration, the respiratory therapist should

expose the electrode to the ambient environment.

Which of the following could be a cause of a low-pressure ventilator alarm A. improved pulmonary compliance B. kinked ET tube C. increased airway resistance D. ARDS

he correct answer is : A Explanation : The solution here is to look for choices that could result in a low pressure or low volume alarm. If someone has improved pulmonary compliance, this could result in lower pressures, thus triggering a low-pressure alarm. Significantly reduced airway resistance would also cause a low-pressure alarm if the low-pressure alarm was set too high or too close to the previous pulmonary pressures. A disconnected ventilator circuit and a deflated ET tube cuff would also result in a low-pressure alarm. Thus, all answers are correct.

A patient is brought to the emergency department (ED) with respiratory distress. The patient has a single-bore tracheostomy tube in place, which was replaced three weeks prior. The patient reports recently coughing up tan, thick secretions through the tube. The medical record from the transferring facility indicates the patient breathes ambient air without supplemental oxygen. Currently, the patient is receiving oxygen by trach collar and a high-flow air entrainment device set at 40%. The respiratory therapist should recommend

heated humidity.

A 75-kg (165-lb) male is receiving mechanical ventilation by a volume-controlled ventilator in the assist/control mode on the following settings: Mandatory rate 14/min VT 450 mLFIO2 0.5PEEP 5 cmH2O ABG results: pH 7.32PaCO2 48 mmHgPO2 76 mmHgHCO3- 23 mEq/LBE -1 mEq/L Which of the following represents the most appropriate action?

increase VT to 550 mL

A patient has the following input/output history and other clinical information: Yesterday Today IN 1900 cc 1600 ccOUT 2200 cc 1900 ccC.O. 5.1 L/min 3.8 L/minBP 110/80 mm Hg 110/80 mm HgCVP 5 mm Hg 5 mm Hg These data would result in which of the following?

increase in SVR

A patient is admitted for monitoring due to a positive result on a Tensilon Test (edrophonium). The patient is assessed every 6 hours. The following bedside data is recorded in the medical record: 12 hrs prior 6 hrs prior Current VC (L) 3.8 2.4 1.6 VT (L) 0.5 0.6 0. 5MIP (cm H2O) -42 -35 -32 The respiratory therapist should recommend

increasing frequency of assessment to every 2 hours.

During a preoperative test of a mechanical ventilator, the respiratory therapist notices the ventilator is not delivering sufficient tidal volume to the test lung compared to the set tidal volume. The following settings are being used: Mandatory rate 20/min VT 800 mLI:E 1:2FIO2 0.30PEEP 5 cm H2OFlow 40 L/min Which of the following is most likely the cause of the problem?

insufficient flow

A patient with COPD, receiving mechanical ventilation, is undergoing early extubation to non-invasive ventilation. Immediately following extubation, the patient is placed on bi-level ventilation with settings IPAP 20 cm H2O, EPAP 5 cm H2O. Thirty minutes later, the patient begins to exhibit moderate inspiratory stridor that advances to a marked condition. The respiratory therapist should

intubate the patient

A patient has undergone a laryngectomy two days prior and currently has a laryngectomy tube in place. In the past hour, the patient has begun to demonstrate marked ventilatory difficulty. An arterial blood gas analysis shows ventilatory failure. In preparation for mechanical ventilation, the respiratory therapist should

intubate the patient though the laryngectomy tube.

Immediately following oral intubation on a patient involved in a motor vehicle crash, auscultation reveals absent breath sounds on the left. Which of following is the most likely reason for this finding?

intubation in the right mainstem bronchus

A patient is receiving volume-controlled ventilation and has a balloon-tipped pulmonary artery catheter in place. The following clinical data is available: 10 am 2 pm CVP (mm Hg) 4 5PCWP (mm Hg) 12 28C(a-v)O2 vol% 5 10 The patient is likely having a problem with the

left heart

An adolescent patient is brought to the emergency room (ER) in a state of panic and respiratory distress. Peak flow measurement is attempted but the patient is unable to perform the maneuver. The patient is only capable of speaking in one-word sentences. The respiratory therapist should consider the situation to be

life threatening.

Patients with large necks who experience frequent dislodging of their tracheostomy tubes need (a)

longer tracheostomy tube

A freshly cleaned and assembled adult ventilator is undergoing pre-operational quality control with an adult circuit. The therapist observes a failure in the delivered and returned volume data. Which of the following is most likely the cause?

loose circuit connection

Which of the following devices is most helpful in performing a safe and proper nasal intubation?

magill forceps

After instructing a patient and demonstrating the proper use of a metered dose inhaler with a valved holding chamber, the patient performs the maneuver by placing the chamber in his mouth, actuating the cannister, and then exhaling prior to performing inhalation. The respiratory therapist should

re-instruct the patient to inhale immediately after actuation of the cannister.

Prior to obtaining an MEP value with a pressure manometer, the respiratory therapist notes the needle is pointing at a positive pressure of -4 cmH2O prior to the maneuver. During the MEP maneuver, the needle reaches 32 cmH2O. The therapist should

record an MEP of 36 cmH2O

A patient is receiving aerosolized Xopenex 1.25 mg every 4 hours. The respiratory therapist notices that the patient's heart rate jumps from 92 to 108 during the last two treatments. The respiratory therapist should recommend

reducing the Xopenex dose to 0.63 mg.

Thirty minutes following the insertion of a tracheostomy tube, the respiratory therapist palpates the skin around the site and notes a crackling sensation. The therapist should recommend which of the following?

removal of the tracheostomy tube and reintubation

A 70-kg (154-lb) male patient is receiving mechanical ventilation. After increasing PEEP, the following data is available: Arterial blood gases reveal pH 7.32 PaCO2 47 mmHgPaO2 80 mmHgHCO3- 26 mEq/LBE +2 mEq/LC.I. 2.5PAP 14 mmHgCVP 4 mmHgPCWP 8 mmHg The respiratory therapist should do which of the following?

remove 100 mL of deadspace

A patient with a fenestrated tracheostomy tube in place is experiencing marked respiratory distress. While attempting to suction the airway with a sterile catheter, the respiratory therapist notices an obstruction in the tube. The therapist should immediately

remove the inner cannula

While ventilating a patient with a VC ventilator, the respiratory therapist notices that the ventilator is not delivering the set tidal volume. The therapist should

remove the ventilator from service, perform preoperative test.

A 10-year-old child does not seem to be performing incentive spirometry well with a flow-oriented device. The patient says the device is boring and difficult to get all three balls floating. The respiratory therapist should

replace the device with a volume-oriented incentive spirometer

A 6-ft, 2-in (188-cm) Caucasian male has inexplicably become tachypneic. Tracheal deviation to the left is noted. A chest radiograph confirms a pneumothorax and chest tube placement is ordered. The respiratory therapist should recommend placement of chest tubes in the

right side, 2nd interspace, mid-clavicular line.

Which of the following should be closely monitored in a patient receiving diuretic medication?

serum electrolytes

The respiratory therapist observes a gradual increase in peak inspiratory pressure over the last 24 hours on a patient who is receiving mechanical ventilation. Additionally, the patient has become febrile and is producing purulent pulmonary secretions. The therapist should recommend which of the following?

sputum gram stain

A respiratory therapist is providing chest physiotherapy and postural drainage on a cystic fibrosis patient with pneumonia in the right lateral segment. The patient was in high fowlers prior to therapy. The patient experiences arrhythmias and dyspnea after 5 minutes of therapy. What should the therapist recommend?

stop therapy, return to high-fowlers, provide oxygen, call the physician

A patient suspected of ARDS is receiving VC mechanical ventilation and has the following ventilatory data: VT 650 mL Mode A/CFIO2 0.60PEEP 12 cm H2OPx(peak) 52 cm H2OPx(plat) 36 cm H2OSpO2 88%OI 32 The respiratory therapist should consider

switching to PC ventilation

Which of the following clinical outcomes is a side effect of inhaled nitric oxide?

systemic hypotension

In preparation for pulmonary function testing, a therapist performs calibration of the PFT machine. After injecting gas from a certified 3.0-L super syringe, the following results are obtained: First Second Third 2.85 L 3.14 L 2.99 L Which of the following can be accurately stated?

the PFT machine is accurate

A patient is in the emergency room (ER) with an oxygen saturation of 86% on a non-rebreathing mask. The respiratory therapist observes paradoxical chest movement on the left side. Trachea is mid-line. Which of the following can the therapist conclude?

the patient has fractured ribs on the left side

Extensive testing has proven no brain activity in a patient who is completely ventilator-dependent. The family decides to discontinue life support. The physician orders extubation and implementation of non-invasive ventilation. For what purpose is NIV being used in this case?

to provide comfort to the patient and family

Which of the following is LEAST likely to promote the spread of nosocomial infection in a hospital?

use of unit dose medication

A 74-year-old male in the intensive care unit has the following arterial blood gas results while receiving volume-controlled ventilation at an FIO2 of 1.0 with PEEP of 10 cm H2O. pH 7.33 PaCO2 50 torrPaO2 220 torrHCO3 - 22 mEq/LBE -2 mEq/L The respiratory therapist should conclude which of the following regarding the patient's condition?

venous admixture


Related study sets

software engineering collection 5

View Set

define each of the types of constitutional powers

View Set

Medical Terminology 2 - The Nervous System

View Set

AWS Academy Cloud Foundations Exam 1 Chapter 1 - 5

View Set